Download as pdf or txt
Download as pdf or txt
You are on page 1of 308

PSIR Crash Course 2021

Model Answers - Test 1


For any issue related to PSIR Test Series, write only at psirtestseries@shubhraranjan.com

Section A

1(a). Decline of political theory.


Once called as the master science , the exponents of new political science like David
Easton,Cobban and Germino began to question the continued relevance of the traditional
political theory around Mid-20th Century. It consequently led to the emergence of the debate on
the ‘decline of the political theory’. While David Easton declared the ‘decline’ of the discipline,
Dahl and Laslett went a step ahead and declared the ‘demise ’ of the discipline.
David Easton, in his work ‘The Decline of Modern Political Theory’ asserted that the political
theory produced speculative theories rather than empirically oriented theories. Thus, lacking
contemporary relevance. The discipline was surviving parasitically on centuries old ideas, failed
to provide new synthesis thereby crushing life out of value theory. According to David Easton,
historicists like Sabine and Dunning weren’t dealing with the contemporary issues such as rise of
fascist tendencies etc. Political scientists have to be sensitive towards social problems and must
construct values . The indifference towards the values led to the decline .
For Easton, both traditionalists and empiricists are to be blamed . If traditionalists reduce the
discipline to speculative discourse , empiricists killed the mental species of the subject. While
fact was needed to build a scientific theory , but excessive fact led to hyper factualism . This
substantially changed the nature of political theory , beyond recognition. Thus, the quest for
fact became a malady for the discipline.
Alfred Cobban attributes the reasons behind the decline to some external factors like huge
military complexes , giant bureaucracy , irresistible increase in state’s intervention inhibited
political thinking in the west . Repressive communists regimes left no scope for critical thinking .
He talks of internal factors like empiricist’s and positivist’s excessive emphasis on value free
approach. This has led to the extinction of political philosophy.

Commenting on the debate, Dante Germino in his ‘Beyond Ideology: The Revival of Political
Theory (1967)’, argued that in most of the 19th and 20th Century, there were two major causes
of the decline of political theory :
A. Rise of positivism leading to craze for science.
B. Ideological reductionism, culminating into Marxism.

However, with the emergence of post-behavioural methodology, political theory is again in


ascendancy. The most important contribution in this regard is of John Rawls ‘A Theory of
Justice’, which has led to a large number of works such as Robert Nozick’s ‘Entitlement Theory’,
C.B. Macpherson’s theory on Democracy. Amartya Sen’s ‘Capability Approach’ etc.
Isaiah Berlin argues that, political theory is neither dead nor in the state of decline . So long
rational curiosity exists, political theory will not disappear.

1(b).Gramsci’s Concept of hegemony.


Gramsci, under the influence of Benedetto Croce, has realized the role of cultural and ideological
factors in determination of power. Gramsci, in his ‘Prison Notebooks’ referred to this ideological
domination as ‘hegemony’, which is an internalized form of domination. Hegemony is a means
by which the capitalist class secures the willing compliance of workers in capitalist societies .The
concepts traces its intellectual roots in the works of Karl Marx ,Engels, Lenin, Benedetto Croce.
In his work "The German Ideology", Marx writes that in every epoch the ideas of the ruling class
are the ruling ideas. This pervasive power of ideas conceals the class contradiction and keeps
the exploited class in false consciousness. Engels coined the phrase ‘false consciousness’ which
keeps workers away from recognizing their oppression. According to Lenin, the power of
bourgeoise ideology is such that proletariat can develop only trade union level consciousness.

Gramsci underlines the role of civil society as the site where the hegemon is produced and
reproduced in the cultural life through media , educational institutions etc.Unlike the political
society, which rules through coercion, the civil society rules through consent.
According to him , bourgeois class rule by making their values as common sense . Intellectuals
play the role in manufacturing consent/hegemony. Hence , he believed that workers would
require two stage revolution - war of position and war of manoeuvre.
War of position is a protracted war where idea is to establish counter hegemony. Like bourgeois
class , working class also needs intellectuals According to him, whenever a new dominant class
emerges, it brings the set of intellectuals. This set of intellectuals is called "organic intellectuals".
Working class also needs to bring their own organic intellectuals which come from their own
class . Primarily the role of intellectuals is to manufacture hegemony.
Gramsci’s insight about how power is constituted in the realm of ideas and knowledge have
contributed to the idea that knowledge is a social construct only, which serves to legitimize the
status quo has inspired. Gramsci is credited for having saved Marxism from economic
determinism by bringing in the concept of hegemony, which gives answers to many questions
left unaddressed by Marx. Therefore, its not without reason that Edward Said argues that
hegemony is an indispensable concept for understanding the cultural life in the west.

1(c).Gandhi’s critique of modern civilization


In his published works MK Gandhi frequently refers the concept of ‘civilisation’. Gandhi divided
this concept into two distinct types: true and false. The former he identified chiefly with western
industrial and technological civilisation that had, in his view, reduced the West to a state of
‘cultural anarchy’. In Hind Swaraj , he appears influenced by Edward Carpenter's essay
‘Civilisation: Its Cause and Cure’, in which Carpenter draws an analogy between civilisation and
disease. He calls modern civilization as "satanic'. He criticized modernity, not because it
originated in West but because of its destructive nature .
For Gandhi, “civilisation is that mode of conduct which points out to man , the path of duty
secured through attaining the mastery over mind and passions ”.True civilization means one
becomes a 'better human being'. In other words, true civilization doesn't degrade man to status
of animal. It doesn't mean, 'satisfaction of wants but minimization of wants'.
In his critique of modern civilization, Gandhi attempt to build 'counterhegemony'. He questions
the legitimacy of civilizing mission and 'white men's burden'. In this way, he didn't favour
excessive mechanization in the context of development of India.
Gandhi’s critique of modern civilization provides the intellectual context of Gandhi’s
nationalism ,critique of colonialism , understanding of the negative consequences of modern
civilization and gives the prospect of more sustainable future.
According to Gandhi, human emancipation is achieved through performance of duties . Real
rights are performance of duties . Gandhi’s blueprint of civilisation can be understood through
his concept of swaraj. Gandhi’s concept of Swaraj is quite different from western notion of
freedom. Swaraj is self rule i.e. self control . Modern civilization neglects self, subvert morality
and promotes physical happiness .Gandhi believed that neglect of ethics (Machiavellism) is
responsible for many horrors of modern civilization like imperialism , fascism . Modern
civilization embodies modern machinery , which places severe hurdle in the realization of self.
Gandhi criticises, railways in particular. He believes that railways have accentuated the evil
nature of modernity. It carried the germs of plague , brought unholy to holy places .Thus , like
Marx , Gandhi is also talking about the alienation of self. Modern state creates such conditions
of life where it is difficult to find the self . Hence , he supported panchayati raj based on
democratic decentralisation.
Gandhi agrees that modern civilization is rational but rationality becomes hideous monster
when the state becomes omnipotent. Gandhi considers modernity as inhospitable to
plurality.The concept of Justice being practiced by modern nation states is just procedural and
not substantive. According to Bhikhu Parekh , Gandhi concludes that those who think modern
civilization is a blessing are intellectually bankrupt. Indian civilization cannot be beaten in terms
of its sustainability, robustness, inclusivity .

1(d) Foucault concept of power.


Michael Foucault has challenged the conventional understanding of power. He moves away
from the juridico-legal conception of power to, what he calls as, ‘disciplinary power’, and later
on as ‘biopower’ and ‘governmentality’.
Foucault provides a micro view of power. For him, power is not limited to any particular
institution rather ‘power is everywhere’ and is dispersed throughout the society in the form of
networks of capillaries. He held that power produces discipline, through the methods of
punishment and reward, which is intended to assure the cohesion of the social body. Thus,
power encompasses the whole society, rather than a relation between the oppressed and
oppressor.
Foucault further argues that Individuals are not the objects of powers, but they’re the locus
where the power and the resistance to it are exerted. For Foucault, power isn’t necessarily
‘coercive’ rather 'productive' as it produces identity and stability in the society. Power is
experienced where there's a creation of identity and not just domination and resistance. At the
core of Foucault’s picture of modern disciplinary society are three primary techniques of control
hierarchical observation, normalizing judgment, and the examination.
Bentham’s Panopticon is, for Foucault, a paradigmatic architectural model of modern
disciplinary power which shows that control can be achieved more by the possibility of internal
monitoring of those controlled than by actual supervision or heavy physical constraints. The
relation of power and knowledge is far closer. Hence , he says knowledge/power.
Foucault’s theory is regarded as an important contribution to political theory by challenging
many traditional assumptions. Noam Chomsky mentions Foucault as someone who wildly
exaggerates the influence of power in scientific discourses.

1(e). Contextualist approach of the interpretation of classics


Contextual Approach focuses on the background of the author, historical- intellectual context of
the text. It attempts to understand a philosophical position with respect to intellectual
movements of its time and earlier times. It highlights that theorists related themselves to the
contexts and had aims and purposes in writing. Thus, it rejects canonical theorists who
approached texts as timeless philosophical works.

The contextual approach is associated with J. G. A. Pocock, Quentin Skinner, Dunn and the other
scholars of Cambridge School.

Quentin Skinner in his article “Meaning and understanding in the history of ideas” recommends
the understanding of linguistic context. Similarly, Pocock believes that concepts do not have
similar meanings across the contexts.

However,the contextual approach has been criticized as scholars have questioned that if text is
to be understood in context, it will undermine the timelessness of classics . Classics transcend
the limits of their contexts(Plamenatz ).

In present times , scholars like Terrence Ball talk about ‘inescapabilty of interpretation’. Derrida
talks about ‘multiple meanings’ in the text and Gadamar mentions the ‘fusion of horizons’.

2(a). Discuss Hannah Arendt’s Theory of Action and its contemporary relevance.
Hannah Arendt is a philosopher who held that politics provides the only guarantee of our sanity,
that political activity alone confers meaning upon life, and that in it the highest form of
happiness is to be found. Arendt is highly influenced by Greek way of life. She writes , "we
understand the political in the sense of the polis." Such view of politics is based on her Theory of
action found in her most ambitious work, The Human Condition.

Labour , work and action are the three elements which constitute Hannah Arendt’s Vita Activa.
Together they represent what it means to be human . She establishes the superiority of vita
activa over vita contemplativa. Labour, work, and action are not merely different forms of
activity but comprise a scale in which each marks the achievement of a progressively higher
level of consciousness.
At the bottom of the scale is labour, an activity in which man enjoys a herdlike level of
consciousness. In labouring he shares with the rest of creation bondage to necessity, being
concerned simply with the maintenance of life as such. "The mark of all labouring," Arendt
writes, is that "it leaves nothing behind, that the result of its effort is almost as quickly
consumed as the effort is spent. If labour produces objects, it produces them only incidentally,
as a means to its own reproduction. The labour of our body is necessitated by its needs is
slavish. Labour , for her is futile , repetitive and ceaseless. There is nothing in it that distinguish
us from either ant or tulip. The individuality is lost in this activity . It represent our mortality. In
this context man is animal laboran. She criticizes John Locke, Adam Smith and Marx for treating
labour as the center of their thought.
Work , on the other hand, produces durable objects, and has the production of such objects as
its primary aim. In work, man emerges for the first time to self-consciousness, since he becomes
aware of his difference, as subject, from the object upon which his efforts are directed. But
because the world in which he moves is a world of things, and not a world of men, he does not
reach the highest level of human development. In so far as he does encounter his fellow men, it
is only in their limited capacity as producers in the context of a market relationship, and never in
their full stature as human beings. Unlike labour, which leaves nothing , work leaves behind
something. In this context , man is Homo Faber i.e. the fabricator/instrumentalist .
Both labour and work are therefore relegated to what Arendt terms the "private realm" of
existence i.e. oikos.
In adopting such view , she rejected the modern hierarchy where economic activity is valued the
most . Her view is similar to Aristotle, who considered that neither labour nor work possessed
sufficient dignity to be termed a way of life (a bios) at all, since they were not "autonomous and
authentically human activities.
The highest level of development is attained only in action, which is "the only activity that goes
on directly between men without the intermediary of things or matter." Action belongs to the
"public realm". Arendt considers that only action is authentically human, and hence superior to
labour and work. She establishes action as the human activity based on her concept of human
essence.

The essential nature of man, then, is to seek immortality, and the attainment of immortality is
only possible when man enters the public realm and moves amongst his peers, for only his
equals are able to judge his actions, and only through their presence as witnesses will the
memory of his deeds survive and assure his immortality.
Like the existentialists, Arendt believes that the source of value and the meaning of existence
are to be found only within action itself, and that it is only in acting that a man defines himself,
by making his essence into a tangible reality in the form of deeds.
There are three assumptions about the human condition.

1. The first is that man is distinguished from the rest of creation by virtue of his desire for
immortality;
2. The second is that this desire can only be satisfied through action, or praxis; and
3. The third is that action must take place before a peer group of spectators who can
transmit the memory of it.
The highest form of action, that is to say, is politics; and politics, for Arendt, means what the
ancient Greeks (or some of them, anyway) took it to mean.
She does not deny that representative institutions have provided a considerable measure of
freedom of speech and thought, but argues that the sort of freedom they have made possible is
worthless, since it "no longer opens the channels for action, for the meaningful exercise of
freedom."
In other words, it leaves men in the private realm, with all the futility that entails. Through
parties and pressure groups the voter can indeed exercise influence over his representatives, but
only with regard to his interest or welfare; and since "interest" is an essentially private matter,
no genuine common good can emerge. What the representative system fails above all to
provide for is the expression of opinion. True opinion can be formed only in a process of open
discussion, and this requires a society in which men possess a public realm where genuine
equality prevails.
Thus,according to her , man is zoon politikon. The distinctiveness of a man is possible only when
he performs action. According to Hannah Arendt, the purpose of life is not to lead a good life
like Aristotle but to be free in existentialist sense . Freedom through action promises the
possibility of transcending ourselves . One important aspect of action is speech . It is in speech ,
we escape our solitude .
According to Bhikhu Parekh , no other philosopher in the history of political thought has dealt
with the analysis of nature and structure of political action. In contemporary times , when the
tendencies of totalitarianism are rising high , Hannah Arendt’s concept of political provides the
basis for the preservation of freedom .

2(b) Elaborate on the Marxist theory of state. Do you think there is not a single theory but
multiple perspectives on state within Marxism. Comment.
The state indeed is a contentious concept, observes Neera Chandoke, as any inquiry into the
state is value laden, so far, no definition of state has been able to capture it in entirely. David
Easton, has spoken about the futility of conceptualization of state as it will lead to only a kind of
“conceptual morass”.
Marx originally treated the modern state as an “epiphenomena” of system of property relations
and the resultant class struggle. David Held mentions that there are two traditions of state in
Marxist. Instrumentalist view, which view state as directly linked to the interests of the
dominant class. Ralph Miliband projects the instrumentalist view.
The second account, where the state retains a degree of power independent of the class. It’s
institutional from and dynamics cannot be inferred directly from the configuration of class
forces i.e. state is relatively autonomous. The Position is propagated by Nicos Poulantzas, Claus
Offe, Theda skcopol.
Marxist perspective is commonly regarded as the class theory of state, evolved from the writings
of Marx, Engels, Gramsci and others. Since Marx considered state epiphenomenal, his work on
state is more fragmented philosophical speculations. The closest Marx ever comes to a
systematic treatment of the state is in his early work “Introduction to the Critique of Hegel's
Philosophy of Right”, where he held that, though state is universal , it is a temporary
phenomenon which ultimately has to be done away with. Marx’s writing are directed to achieve
classless society.One central element of Marxist theorizing about the state is idea of class and
class struggle i.e. pattern of domination and oppression.
In communist Manifesto, he declares the state as executive committee of the bourgeoisie class.
In Marx, contrary to Hegel, state express human alienation. In Critique of Gotha Programme,
Marx suggest that during transition from capitalism to communism, state becomes majoritarian
and more democratic. In “Anti During” ,Engels introduce the notion of “withering away of
state”.

In his classic, State And Revolution, Lenin discussed the dictatorship of Proletariat, which will
have two functions i.e. suppression of bourgeois and establishment of communism.
The second strand representing the relative autonomy theory of state is found in Marx’s The
18th Brumaire of Louis Bonaparte, where Marx suggest that under specific historical context
when economic structure is not controlled by single dominant class, state gains “Relative
autonomy”. There were three dominant classes big landlords, industrial capitalists and finance
capitalist, in the perpetual state of intrigue against one another. Working class played minor
role yet was one of driving force of politics.
In the Asiatic mode of production, he explored the developments in non-European world. In Non-
European world, unlike state in Europe, which has been the instrument of the capitalist class,
state in Asiatic society controlled all classes, appropriating surplus and preventing the rise of
free market.
Structuralist perspective has been revived in the work of Gramsci. He made a clear distinction
between political society representing coercive power and civil society representing power of
consent i.e. structures of legitimation. Thus, the capitalist state, writes Gramsci, in his Prison
Notebooks as overlapping sphere of politics and civil society.
Althusser, deepened the theory of state by holding that ideology plays much stronger role than
Marx’s perception of ideology as false consciousness. Ideology is more pervasive and more
material. Althusser gives ideological state apparatuses and repressive state apparatuses.
Ideological state apparatuses are less centralized, more heterogeneous, have access in the
private rather public realm of existence. Repressive state apparatuses function through violence
and Ideological state apparatuses function through ideology.
Ralph Miliband in his “The state in capitalist society” has emphasized on “instrumentalist
theory” of state in post capitalist societies. He held that the class character of state is
perpetuated by:-

1.Middle and upper class background of state officials


2.Economic power of capitalist class
3.Desire of politician and bureaucrats to continue the existing system on which their own
position depend.
Poulantzas (Political power and social classes) has drawn a clear distinction between the
position of the capitalist class and state power. He emphasized on the structuralist theory. He
has conceded the relative autonomy of the state. He has argued that the class domination is not
automatically translated into state power and the state cannot properly be regarded simply as
instrument of class. The state enhances its legitimacy by invoking authority of ‘the people’. The
state enhances its legitimacy by disassociating itself from state repression.

Hamza Alwi, has developed the theme of Asiatic mode of production by establishing the post-
colonial state as overdeveloped, negotiating on behalf of:-
1.Metropolitian bourgeoisie
2.Indigenous bourgeoisie
3.Indigenous feudal class
Thus, there is not a single approach to the state rather multiple approaches on the state within
Marxism. Bob Jessup identify at least six different approaches involving different theoretical
principles of explanation and political implication. State is institutionally complex multi layered,
historically contingent set of processes and practices.

2(c).Define the salient features of neo-liberalism. Discuss the future of neo- liberalism in
context of Covid crisis.
For years, scholars have been attracted by the sustained popularity of neoliberal doctrine -
small government, low corporate tax, deregulation of labour and product markets, promotion of
financial sector . Even India’s PM Modi gave the mantra of ‘Minimum government , maximum
governance ’ based on the logic of neo-liberalism. British former PM Margret Thatcher,
advocated neo-liberalism by coining the acronym TINA i.e. the only alternative is that there is no
alternative. Neo-liberalism finds its philosophical justification in the works of Hayek , who held
that planning is Road to serfdom; Nozick gave the slogan of Nightwatchman State ; Milton
Freidman considered capitalism as a precondition for freedom and Isaiah Berlin talked about
positive liberty as no liberty.

However, neo-liberalism always had its critics . Nobel laureate Joseph Stiglitz, a social liberal,
has been predicting the death of neoliberalism for more than a decade.During the Great
Recession of 2007-08, in his article, 'The end of neo-liberalism?' , he says "Neoliberal market
fundamentalism was always a political doctrine serving certain interests. It was never supported
by economic theory. Nor, it should now be clear, is it supported by historical experience.
Learning this lesson may be the silver lining in the cloud now hanging over the global economy."
Nobel laureate Paul Krugman , another social liberal, in his book titled 'Arguing with Zombies:
Economics, Politics, and the Fight for a Better Future' , calls neo-liberalism as a 'zombie
idea',which should have been killed .
Raghuram Rajan, former RBI governor and professor of finance at the University of Chicago,
explains in his 2020 book 'The Third Pillar: How markets and the state leave the community
behind' that "Smith and Mill were not rabidly anti-government", that they recognised the role of
government. For example, he writes, Smith was for state providing education and other services
and talked of "perfidy of businessmen" if entrusted with their own regulation (deregulated).
The COVID-19 pandemic has exposed the toxic effects of Neoliberalism, as an economic ideology
of capitalism, that has depleted public services, turned education and healthcare into profit-
driven businesses, hoarded profits at the expense of undervalued and underpaid workers.,
favoured profitability over well-being. The pandemic revealed the full scope of the effects of
neoliberalism . There have been impact on employment, and corporations are asking for bail-
outs; the care burden on women is already massive; the state of emergencies proclaimed
around the world have an effect on freedoms and human rights. Mobility is affected . It is
suggested to use this opportunity to build societies that encourage solidarity, equality, and
caring for the environment and fellow human beings. We can start transforming how we
interact with each other and with the environment and how we respond to crisis so that we
don’t only ‘flatten the curve’ with respect to the spread of COVID-19 but flatten the
consequences of the pandemic.
In this context , there is a renewed interest in the idea of welfare state based on Keynsian
economics .Even , Emmanuel Macron, a former banker turned French President mentioned that
it is time to think the unthinkable . He is not only pointing towards welfare state , but even
socialist state . Even pro business Economist mentions that the state is going to play a
prominent role in economy . Avineri mentions that we are witnessing the death of neo-liberalism
and Rodik suggest that the state is back with vengeance towards capitalism . Number of
authors with left leanings held that the crisis of neo-liberalism was already in making and
pandemic brought it on the surface. However, the neo-liberals and neo-rightists continue to see
statist claim as false propaganda of the left. They have raised the question as to from where the
money will come to pay for the health , education and employment.

To conclude , we can say that , today we are once again at the cross roads

3(a). What does Dworkin want to suggest when he says the right are Trumps? Analyse the
reason given by Dworkin to support his views that government must take rights of individuals
seriously?
“Rights are those conditions of life without which no man can seek in general to be at his best.”
Laski
Discourses on rights are the fundamental aspects of modern political philosophy. Rights are
seen as claims essential for the full realization of the personality of the individual. In his essay
“Taking rights seriously”,Dworkin develops his idea of rights as trumps.
The Core theme of Dworkin is to challenge those who believe that there is a sufficient ground to
curtail the rights of individuals, if such limitation is in the greatest happiness of the greatest
number. Rights are inalienable and therefore government and philosophers should take rights
seriously. Rights are inalienable parts of the personality of man, hence cannot be traded off
between other kinds of goods or moral political values. Rights are trumps in the sense that,
there are some fundamental rights having precedence over other norms including the welfare of
the whole society. His main idea is that there are some moral rights against the government.
Not all constitutional rights are such rights. Constitutional right like freedom of speech, are
rights against the government, in the strong sense i.e. the point to boast our legal system
respects the fundamental rights of the citizens. Thus, government would do wrong in repealing
such rights, even when they are persuaded that the majority would be better off that way.
Certain interests of the individual are so important that it would be wrong for community “to
sacrifice those interest just to secure over all benefit”. Some rights which can be called political
or fundamental mark off and protect these important individual interest. Such rights are trump
over kind of trade off arguments that normally justify political action. Thus, some liberties are so
important that they cannot be disregarded on consequentialist grounds. Even Rawls established
the priority of rights over good.
If the relevance of fundamental rights can be weighed at every new upcoming crisis scenario,
like any other social/ national interest, than balancing is a confusion that threatens to destroy
the concept of individual rights.
Trumps are strong rights, against government and once government defines them as
fundamental, society should bear the cost/burden arising out of it, thus great social cost is
warranted to protect it.
However, Dworkin does not suggest that state is never justified in overriding the right.For
example: - Government may override, the right, to protect the right of others, to prevent
catastrophe or even for major public benefit.
In “Taking rights seriously” he mentions following grounds to limit the rights.Governments
might show values encompassed by the original rights are not really at stake. If rights are held,
then the cost to society could be far beyond the cost paid to grant the original right.
Dworkin, who is a philosopher of jurisprudence aims to give legal positivism found in writings of
H.L.A. Hart and his utilitarian ancestors like Bentham, who believed that rights are properly the
creatures of law.

Dworkins thesis is like Locke who held that individuals can have rights against the state which
existed prior to the rights created by explicit legislation. He justifies his contention that citizens
have rights against the government on the basis of concept of human dignity and political
equality (equal concern).
Dworkin criticizes Rawls for deriving rights by using social contract, whereas Social contract
itself is based on the recognition of rights “to view persons as ends is to view them as possessors
of rights”.
To quote Dworkin, “Individual rights are political trumps held by individuals. Individual have
right even when a collective goal is not a reason to deny them.” Dworkin’s view establishes him
as anti-utilitarian,as individuals have rights and law must recognize them is his central thesis.
He does not accept general right of liberty rather believes that right to equal concern and
respect is the key fundamental rights.
Dworkin develop a theory of judicial reasoning, where he suggest that “hard cases” in which
statutes and precedents are vague have to be decided on the basis of our political institution
which are right based. Dworkin prioritize principle over policy . He even support civil
disobedience to those laws which individual believes violate the rights. Dworkin thesis is that
rights ultimately underlie judicial reasoning. Rights exist independent of legal recognition and
outlines how judge and lawyers should think about it.

3(b) For Marx , freedom means the ability to achieve the totality of human goods.” John
Lewis. Discuss.
The term freedom is notorious for its multiple meanings . It can be argued that its basic and
more generally understood meaning is the classical liberal view which defines freedom
negatively as absence of constraints .In contrast , Marx saw freedom as ability to exercise a
conscious rational control not only of his natural environment , but also social environments .
The social environment in the capitalist societies is not conducive for human emancipation. It is
possible only in such societies where humans can achieve the totality of human goods. The
totality of human goods is based on the notion that man is by nature social and creative . Man
performs work not just for the satisfaction of his basic necessities , but for realization of his true
self. Capitalist societies alienates man not only from the process of production, product of
labour , society and ultimately from oneself . Thus the freedom in capitalist societies is an
utopian idea . Though , there has been alienation in all societies scarred by class divisions ,
capitalism present alienation in much greater form. Alienation in capitalism does not flow from
technological advancements ,rather from capitalist social relations. Hence, only communist
society can end alienation and help in realization of totality of human goods. In his text On
Jewish Question, Marx started distinguishing between his conception of freedom and the liberal
conception. His concept of freedom associated with alienation emerges more deeply in his
Economic and Philosophical Manuscripts . Marx believed that political emancipation is
insufficient . The liberal conception that man needs protection from fellow human beings is a
flawed conception. For Marx , real freedom is to be found positively in our relations with other
humans. It is to be found in human community and not in isolation. The regime of liberal rights
found in capitalist societies is insufficient to realise freedom.
Thus, while the liberal view is based on the centrality of the individual and his freedom of choice,
the Marxists would see the notion of liberty based on the liberal notion of individual and society
as conditions of unfreedom. For Marxists, the individual is not separated from other individuals
in society by boundaries of autonomous spaces for the free exercise of choice. They are rather
bound together in mutual dependence. The notion of individuality is likewise transformed into a
notion of rich individuality, which emphasizes the social embeddedness of the individual, the
idea that individuals can reach a state of creative excellence and develop their capacities only in
a society which seeks the development of all its members. For the Marxists, therefore, freedom
lies in the development of creative individuality, and cannot be achieved in a capitalist society
where individuals are separated by boundaries of self-interest and where they can only imagine
themselves to be free when in reality they are bound by structures of exploitation. It is only in a
society, which is free from the selfish promotion of private interests that a state of freedom can
exist. Freedom, thus, cannot be achieved in a capitalist society.
Marxists do not view the individual as an atomistic and isolated unit. The natural need of co-
existence brings an individual into relations with others and therefore, a civil and political
society comes into being. The socio economic conditions prevailing in society define the freedom
to self-determine. Karl Marx says that the capitalist mode of production is an exploitative
system as it deprives an individual of his freedom. He bases his theory on historical materialism
saying that the economy has been the foundation of every society. He historically traces the
development of society where earlier in ancient society, people were free and equal and they
had access to goods to satisfy their basic needs. But as the society moved further and they
acquired means of production like machinery, tools, resources, industries; a division was
created. The society that came after ancient society was a feudalistic society where there were
landlords who possessed lands and the landless serfs. After the feudal society, came the
industrial society where the division was between the industry and capital owning bourgeoisie
and the poor, wage earning proletariat. Marx says that an individual cannot enjoy freedom and
self-determine his actions in feudal and capitalist society, because he does not have access to
resources and feels alienated. Marx says that an individual gets alienated from his society, from
family, from his environment and also from the product he makes, as he does not earn enough
to buy the very same product. A capitalist sells the product at a higher rate appropriating the
surplus value and giving lower wages to the proletariat. Marx says that an individual can attain
freedom only if he revolts against the capitalist system of production. He gave an international
call to the workers and said that ‘workers of all lands unite together; you have nothing to lose
only worlds to gain’. So the fourth phase of society in which the workers will rule in politics will
be the socialist phase where the exploitation will end and everyone will get according to his
work. To achieve further equal and free society, workers will give away their power to create a
communist society where no one will rule others and distribution of resources will be according
to need.
Marx says that an individual can realize himself truly only in a communist society. Neo-Marxists
such as Herbert Marcuse in his work, One Dimensional Man: Studies in the Ideology of Advanced
Industrial Society blamed the consumerist society for depriving individual of their real freedom.
He says that the mass media creates a false desire for trivial material goods, thus, diverting
them from the genuine need of freedom. He says that an individual should be made aware of
alienation to realize freedom. C.B. Macpherson in his work Democratic Theory: Essays in
Retrieval says that capitalism emphasizes more on extractive power rather than developmental
power. The capitalist system focuses on what profit can be derived/extracted out of an
individual, rather than how the individual develops so that he can realize his freedom and
determine his course of actions. He adds that society can achieve its freedom when
developmental power gets developed and extractive power becomes nil. Antonio Gramsci says
that coercive structures of society like the police, the army and the judiciary have given way to
hegemonic structures like civil society, schools and literature. Domination and exploitation in
society happens through control of the mind where the working class identifies its interests with
the ruling class. So, Gramsci says that the exploited class can attain freedom when they
produce counter hegemony to the ideas of ruling class
3(c). What are the moral foundations of democracy . Analyse the major debates in the field of
normative democratic theory.
“My notion of democracy is that under it the weakest shall have the same opportunities as the
strongest..” Mahatma Gandhi.
Bernard Crick pointed out that , though democracy is a widely used term and has become
synonymous with virtuous government , democracy continues to be the most promiscuous of all
political terms , thus essentially contested.
Though generally understood as a form of government , yet , the normative theories of
democracy emphasized on the substantive aspects of it i.e. the moral foundations of democracy.
The moral foundations of democracy are based on the recognition of the sovereignty of the
individual . Democratic society is a society of equals based on shared values of dignity , freedom,
equality , justice and fraternity . For scholars like Dworkin , human dignity is a foundational
value based on the recognition of the intrinsic worth of humans .Freedom/liberty assumes that
man ought to be the master of his life and hence collective decision making must include his
vote, affirming right to self government to protect liberty. Equality , another cardinal value of
democracy respecting the equal right of all citizens to develop their potentialities and the
expectations that state show equal concern for all. The idea of fraternity represent that the
members of political community share common goals , aspirations and commitments . Dignity is
the heart of demo racy because it is based on the view that all people hold special value
irrespective of their caste , class , gender. The normative democratic theory is based on the
above moral foundations. The normative democratic theory can be discussed with reference to
Macpherson’s work of democracy, discussed in his books Democratic Theory: Essays in Retrieval;
The Life and Times of Liberal Democracy. Macpherson has categorized theories of democracies
into two types.
A. Classical theories
B. Modern theories
Classical theories are normative in nature based on the value of liberty. He mentions two models
in classical theories :

A. The protective model represented by Locke and Bentham , who look at democracy as a
means to protect the liberties
B. The developmental model is represented by J.S. Mill look at democracy as a form of
government which gives opportunity for development of human potentialities.
The modern theories are essentially empirical , overlook the normative dimensions, emphasise
on the procedural aspect of democracy . If elitists like max weber and Schumpeter defines
democracy as electoral system , pluralists like Robert Dahl emphasise on politics of pressure
groups. Macpherson’s own egalitarian model of democracy aiming to provide the scope for
creative freedom comes under the normative theory. Macpherson criticizes the theory of
possessive individualism .
In Democratic Theory: Essays in Retrieval, Macpherson modifies, extends, and clarifies the
concepts of a person's power arguing that a twentieth-century liberal-democratic theory can be
based on an adequate concept of human powers and capacities . He argues that the neo-
classical liberalisms fall short of accomplishing the goal of eliminating extractive powers and
maximizing developmental powers.

Macpherson suggests that the liberal theory of property should be, and can be, fundamentally
revised in order to accommodate new democratic demands. He establishes the need for a
theory of democracy that steers clear of the disabling central defect of current liberal-
democratic theory, while recovering the humanistic values that liberal democracy has always
claimed.

4(a). “While utilitarianism treats some individuals only as a means towards the ends of others
, justice as fairness considers persons as ends and not as means .”- Rawls. In light of the above
statement explain the salient features of Rawls theory of justice.
“A Theory of justice is powerful, deep, subtle wide-ranging systematic work in moral
philosophy…Political scholars must either work within Rawls theory or explain why not.”- Nozick
John Rawls, considered as the greatest political philosopher of the 20 th century, has criticized
Utilitarianism and established egalitarianism as the basis of justice in philosophy of liberalism
.He intended to provide a viable alternative to utilitarianism.
Utilitarianism is a moral philosophy of which, Bentham is considered as the father. He aimed to
establish moral philosophy on rational basis. He tried to make ethics as an analytical science
which can be proven by logical and mathematical principles (felicific calculus).
Philosophy of utilitarianism has two components -welfare and consequentialism .
Welfare (happiness) of the individual is the sole good, the only thing valuable for its own sake.
Consequentialism holds that actions should be judged only from the value of its outcome i.e.
excess of pleasure over pain.
Utilitarianism assumes:

1) Values can be measured


2) Aim of public policy should be to maximize happiness i.e. greatest happiness of greatest
number ( Introduction to the principles of morals and legislation, Bentham)
Therefore, Institutions should be arranged so as to produce the greatest balance of happiness
over unhappiness affected by these institutions. This moral principle is used to evaluate social-
political institutions.
Rawls has criticized the very fundamentals of utilitarianism. Utilitarianism expects institutions to
achieve greatest happiness and only then it is just. Rawls suggest that utilitarianism is an
objectionable creed . For example:- If slavery produces greatest amount of happiness, it will be
seen by utilitarian as just but our intuition tells it is unjust. The basic moral principle of
utilitarianism is inappropriate as a standard, because it will require us to consider such system
just. Rawls believe that in the ‘original position’, behind the ‘veil of ignorance’, man as a rational
actor would choose ‘justice as fairness’. According to Rawls, the principles of justice should be in
reflexive equilibrium with our intuitions.
To quote Rawls justice as fairness is based, on the view that each person possess inviolability
founded on justice. Thus, Rawls revies the tradition of Emmanuel Kant. The 1st principle in Rawls
theory of justice ensures that each person has same claim to fully adequate scheme of basic
liberties, compatible with the same scheme of liberties for all. The 2nd principle suggests that
there should be equality of opportunities. Since people differ in their talents, second principle
also requires that social institutions be arranged so that inequalities of wealth and income
works to the greatest advantage of the worst off in the best possible way.

Rawls also critiques utilitarianism as it tries to maximize the social outcome as a whole, fails to
take into account the distribution of utility. Rawls held that while utilitarianism is individualistic
theory par excellence it ignored the distinctions that exist between people. Though, it talk about
‘greatest happiness of the greatest number’ it ignores the interest of the least advantaged. He
suggests that greater gains of some should compensate lesser loss of other, violation of liberty
of few should be made right by the goods shared by many. If utilitarianism treats man as a
means, justice as fairness treats man as an end in itself. In justice as fairness, basic liberties and
rights are secured and not subjected to political calculus. Utilitarianism is a consequentialist
theory as it does not specify good independently from maximization of pleasure. Justice as
fairness is more congruent with our common sense convictions and hence closer to intuitions.

Rawls held that even after revision Mill continued to see right as maximisation of good without
sufficient guarantees of seeking equal liberties for all. Mill also fails to explain how distributive
ideal could be subsumed under an aggregative one. What happens when maximisation of total
happiness leads to extreme inequalities.
According to Dworkin, recognition of individual distinctiveness enables to argue a case for equal
rights making it clearly a right based theory which was long overdue in Anglo American
tradition.
4(b).Explain Machiavelli’s concept of virtue and its relevance in the field of state craft.
The relationship between virtue, fortuna and free will is the most interesting philosophical issue
posed by Machiavelli in his book “The Prince”. Machiavelli’s description of:
Fortuna- All those circumstances which human beings cannot control, having bearing on his
success or failure.
Virtue- Human energy or action that stands in opposition to fortune.

Though, he did not exclude the idea of goodness or virtuous behavior, yet he does not
necessarily include it. It is drive, talent and ability of a prince, directed towards achievement of
certain goals.

Machiavelli’s concept of virtue can be best explained only in context of his ideas on fortuna. He
compares fortuna to an impetuous river, which when angry turns plains into lakes and destroys
trees, buildings, but it does not mean that its destructions are beyond human control. She shows
her power where virtue and wisdom are not prepared to resist her, directs her fury, where she
knows no dykes/ embankments ready to hold her. Thus, Machiavelli wants Prince to inculcate
virtue both at individual level and collective level.
Machiavelli explains the role of fate with the example of Cesare Borgia, strong, effective ruler,
yet ended up falling from grace, losing glory due to the role of fortuna. Virtue for Machiavelli
differs from classical Greek and Roman notion of virtue i.e. moral action of being just, honest,
wise. Virtue in Machiavelli is a morally and ethically neutral.
Through virtue, ruler brings glory for himself and his principality. Virtue in Machiavelli’s sense
are qualities of boldness, steadfastness, cunningness. He suggest that ruler has to bold like lion
and cunning like fox. Prudence, accompanied with critical insight, foresight, intuitive grasp of
situation and not the moral worth of an action is the characteristics feature of Machiavelli’s
virtue. Virtue is seizing the moment by prudence, efficiency and flexibility because nothing is
permanent in the sphere of Politics.
Machiavelli’s discussed virtue at individual level in his book ‘The prince’ and collective virtue in
‘The Discourses’. In Republic, citizens as a whole should develop civic virtue to govern their
collective life. Virtue requires Prince to have knowledge of history and empirical circumstances
.Machiavelli also compared Fortuna with women embracing brave men, thus Prince should deal
with it boldly. He suggest that Prince should appears virtuous, but virtuous for sake of virtue
can be detrimental. Prince should not avoid vices like cruelty, dishonestly if it serves raison
d'etat. Virtue has to be pursued not for its sake but because it can act as a mean to achieve the
aims of state and not in terms of any intrinsic moral value.
Machiavelli’s views on Virtue are undoubtedly most inspiring aspect of his state craft. His
suggestion that instead of yielding before fortune one had to act decisively and boldly makes
‘The Prince’, undoubtedly one of the best books ever written in the state craft.
According to Skinner, Machiavelli’s discussion of fortune and virtue to be based in the context of
Italian humanism, setting as side Christian perception of divine necessity with very little room
for maneuver, whereas Machiavelli believes that through his creative power and assertiveness
man would be able to control destiny against the flux unleased by Fortuna.

4(c). Critically examine the relevance of Kautilya’s views on inter state relations in present
times.
In his essay “Politics as vocation”, Max Weber characterizes Kautilya’s Arthashatra as a classic
text of state craft. The characteristic feature of classics is that they have meaning beyond the
historical context of its origin, remains inspiring and thought provoking across time.

George Modelski in his article ‘Kautilya: Foreign policy and international system in the Ancient
Hindu World’, published in American Political Science Review, writes, “Arthashastra is the finest,
fullest and the most cognently reasoned Sanskrit treatise on state craft”.

The Arthashastra encapsulales the science dealing with state craft in both internal and external
spheres. It is seen as foundational text for modern day international relations.

Kautilya, describes Arthashastra as treatise on science of Politics for acquisition and protection
of Earth. He is the earliest proponent of realism. His view on interstate relations can be
discussed as following:-

1. Shadgunya Siddhant:- “He who sees the six measures of policy as being interdependent
in this manner, plays, as he pleases , with kings tied by chains of his intellect” - Kautilya.

As per Kautilya, there are six measures of foreign policy i.e. peace, war, expedition, halt
seeking protection and dual policy.  Sandhi, Vigraha, yaan, Aasana samashrya and
Dvaidhbhav.

Kautliya mentions, when in decline, make peace with enemy and when prospering make
war, when depleted in power take shelter.

2. Four Upayas: Teachings of saam, daam, Danda ,Bheda have resonance in modern day’s
diplomacy.
3. Mandala Siddhant: - “Neighbor is a natural enemy and neighbor’s neighbor is a natural
friend”, is the foundational thought of his Mandala theory.
The theory is based on geographical assumption that immediate neighbor is most likely
to be enemy, and immediate neighbor’s neighbor is most likely a friend with respect to
vijigishu. Third and fifth constituents are friendly elements, whereas second, fourth and
sixth are unfriendly elements.

Mandala is circles of friends and foes embracing 12 kings in vicinity . Circle is dynamic shows the
maxim of international politics that there is no permanent friend and enemy.His advices king
should strive to expand, as war is Shatriya Dharma. He also recognizes the importance of
neutral and buffer states in strategic planning.
On alliances he believe that “A king should not hesitate in breaking alliance if appears
advantageous”.
Thus, Kautilyas views on interstate relations provide earliest insights in geopolitics, realpolitik,
smart power, alliances, balance of power, deterrence and compel lance.

Though his concepts acts as a barrier against the idea of integration, but unfortunately his view
provide a perspective on interstate relations in present times especially in south Asia. Kautilya’s
ideas are evident in the working of international politics today in one way or the other .
Kautilya’s Arthashastra is a treatise rooted in Indian subcontinent, based on wisdom of ages. It
is true that some aspects of international politics like globalization, eco interdependence is not a
part of it. Yet, Arthshastra enriches one’s knowledge of warfare, diplomacy and politics.
Kautilya’s insight is timeless. Present times weigh heavily towards Kautilyan wisdom . For
examples, India coming out of ‘hesitations of past’, entering into QUAD, a Quasi alliance led by
USA, against China to maintain global balance of power.

Medha Bisht in her Book “Kautilya’s Arthashastra: Philosophy of strategy” mentions that
Arthashastra as a text on grand strategy, fills the gap in otherwise Eurocentric discipline.
Section B

5(a) Overlapping consensus

Overlapping consensus is a term coined by John Rawls in his book Political Liberalism. The term
refers to how supporters of different comprehensive normative doctrines—that entail
apparently inconsistent conceptions of justice—can agree on particular principles of justice that
underwrite a political community's basic social institutions. Comprehensive doctrines can
include systems of religion, political ideology, or morality.
Rawls explains that an overlapping consensus on principles of justice can occur despite
"considerable differences in citizens' conceptions of justice provided that these conceptions lead
to similar political judgements." The groups are able to achieve this consensus in part by
refraining from political/public disputes over fundamental (metaphysical) arguments regarding
religion and philosophy. Rawls elaborates that the existence of an overlapping consensus on
conceptions of justice among major social groups holding differing—yet reasonable—
comprehensive doctrines is a necessary and distinctive characteristic of political liberalism.
Rawls also explains that the overlapping consensus on principles of justice is itself a moral
conception and is supported by moral reasoning—although the fundamental grounds of this
support may differ for each of the various groups holding disparate comprehensive doctrines,
and these lines of reasoning may also differ from the public reasons provided for supporting the
principles. These latter features distinguish his idea of an overlapping consensus from a mere
modus vivendi, which is a strategic agreement entered into for pragmatic purposes, and
therefore potentially unprincipled and unstable. The overlapping consensus could in sum be said
to "depend, in effect, on there being a morally significant core of commitments common to the
'reasonable' fragment of each of the main comprehensive doctrines in the community".

5(b) Overdeveloped state .


Hamza Alvi, explains the case of the governability crisis of Pakistan and Bangladesh using the
concept of Over Developed State .
Overdeveloped State is an explanation offered to the ineffectiveness of the functioning of post-
colonial States that operate with the similar structure of bureaucratic governance before and
after their independence. Despite political liberation, the transformative approach did not
percolate down the administrative structure of the States after they emerged free from the
colonial yoke. The colonial powers of western Europe maintained double standards in designing
the polity of the colonized States. When their respective States recognized political life, liberty
and economic freedom of its citizens and in total the State was like a pet for its people, on the
contrast the States were over centralized with unquestionable power in the colonized States.
Such an arrangement was congenial to the colonizers to have unfretted siphoning of wealth
from their colonies. This massive exploitation was effectively engineered by the bureaucracy
that continued the colonial legacy even after independence with no or little change in the
attitude of the political class that replaced the colonial masters.

When the State gets increasingly modernized when the society and its economy are lagging
behind without modernization the State and the society grossly mismatched each other. This
creates a divide between the people and the State’s apparatus. What is good for people are not
decided by them in a representative democracy but by an influential few. In the case of Pakistan
and Bangladesh, Hamza Alvi coins the word ‘military bureaucracy oligarchy’ that is similar to
the license raj of India and over-centralization as a continuation of colonial legacy that has been
constantly addressed by the government of India.

5(c) Pluralist critique of Austinian theory of Sovereignty.


Sovereignty is a relatively modern political concept implying supreme authority within a
territory. The term ‘sovereignty’ originates from the Latin word ‘superanus’, meaning supreme.
The concept was an outcome of a set of complex socio-political and economic processes
spanning over two centuries in the history of Europe. Interestingly, since then the nature of
sovereignty continued to evolve corresponding to the changing historical settings, making it a
dynamic and multi-dimensional concept.
The French political philosopher Jean Bodin attempted to define sovereignty for the first time.
Sovereignty is “supreme power over citizens and subjects, unrestrained by law.”
John Austin, in his book Lectures on Jurisprudence, defines Sovereign as a “If a determinate
human superior not in the habit of obedience to a like superior receives habitual obedience from
the bulk of a given society, that determinate superior is sovereign in that society and that
society (including the superior) is a society political and independent.” . Law is the command of
sovereign

Sovereignty is a distinctive authority vested in a single person or a body . It is original, absolute,


indivisible, inalienable and unlimited.
Pluralist Theory of Sovereignty emerged in the early 20th century as a mark of protest against
the growing incidents of state interferences into the rights and liberties of individual in the West
and also as a critique of the monistic theory of sovereignty which effectively provided a
validation to these incidents. Harold J Laski, one of the leading pluralist theoreticians in his A
Grammar of Politics (1925) argued: “Because society is federal, authority must be federal also.
... The state is only one among many forms of human associations.” Laski even claimed: “It
would be of lasting benefit to political science if the whole concept of sovereignty were
surrendered.” Major pluralists include AV Gierke, FW Maitland, E Barker, JN Figgis, HJ Laski,
McIver, GDH Cole, Krabbe, Lindsay, Duguit . Some basic arguments forwarded by the Pluralists
are that the state is one among various socio-economic associations formed by men to meet
their wants. Man needs the state as well as the other associations. The associations are formed
spontaneously and they do not depend on the state. The state is not able to effectively conduct
the affairs of these associations. The monistic theory is absurd as it is not based on sociological
ideas.
From the perspective of social structure, Laski suggest that the group is real in the same sense
that the state is real . Society is associational and the state cannot be the alternative to the
associations. Authority must be proportional to the action. Men must be guided by conscience,
not by power only.
From the perspective of the concept of law, pluralists believe that Law is not just the command
of sovereign, but a kind of social code of conduct as well. As a social organization, the state is
also subjected to the social norms. The authority to formulate laws must be shared among the
state and other organizations in the society. Laski argues that the full implications of
sovereignty in international context are a license to wreck civilization.

However, pluralists are criticized for being inconsistent because they do not abolish state , state
assumes the role of keystone of the social architecture because of the unique function it
performs.

5(d) Contribution of Buddhist tradition to the political thought


Politics in India was not considered an independent and autonomous discipline. It deals with the
larger questions of human life, mainly religious and philosophical in nature. The salient features
of the Buddhist political thought can be discussed as :
In contrast to Brahminical literature, we have a different version of the origin of kingship in
Buddhist literature. The divine origin theory is not accepted. We have innumerable instances in
the Jatakas of elections for the royal office. Some Jatakas contain description of the king ,
elected by council of elders. Hence, the king is called as Mahajana-Sammata, meaning one who
is accepted by all. Generally Kshatriyas of good family were chosen, but caste was not a bar to
this election to royal office.
If the king proved tyrannical, the remedy was naturally a popular revolt.

Features of king-ship includes:


1. Kingship arose out of a contract between the subjects and the one chosen by them.
2. The sovereign rights of the king were limited to the protection of subjects and
punishment of wrong-doers and he was bound by the law.
Prof. Rhys Davids is of the opinion that the Buddhist Sangha was founded upon democratic
principles. According to him, the Buddhist Sangha “was a kind of republic in which all
proceedings were settled by resolutions agreed upon in regular meetings.
The Dighanikaya challenges the vedic dogma of divine creation of the social order based on
varnas.
Since the Buddhists did not believe in the caste system, they denied that one of the duties of the
king was to maintain Varnashrama dharma. They further denied the restriction of kingship to
the kshatriya varna. They did not believe in the sanctity that surrounded the person of the king.
In the Buddhist literature, danda does not have a central role. Dharma is to be upheld in more
positive ways. The principle of righteousness is different from the Brahminical conception of
dharma. It is closer to the Western conception of virtue. Buddhists too consider the primary
purpose of the state to safeguard the social order. But this order is understood more in moral
terms and dharma must be the standard for all the king’s activities. Righteousness is an ethical
doctrine as well as a mental discipline.
In Buddhism, there is a total application of the principle of righteousness in guiding both internal
as well as foreign polices. The king is bound by same set of ethical principles as are his subjects.
The principle of political righteousness is extended to include the concept of world ruler or
chakravartin. Even , in the sphere of foreign relations, the chakravarti’s conquest of the kingdom
is achieved not by force but by righteousness. The principles of righteousness means; right
views, right intention, right speech, right actions, right livelihood, right effort, right mindedness
etc.
Thus , Buddhist tradition comes near to the socratic tradition.

5(e). Personal is political


The second-wave feminist Carol Hanish is credited with the slogan “the personal is the political.”
This remains one of the most celebrated phrases from an ideology that boldly recasts the focus
of traditional ideologies towards the private sphere. This well-known phrase encapsulates the
various attempts made by feminists of all persuasions to raise awareness of how the private
realm serves a very deliberate political aim.
Feminism is at heart an ideology that seeks to politicise the private realm. In doing so, it
attempts to highlight the subordinate status of women and thereby challenge the traditional
dominance of the male within the home and the family. Feminists have also sought to emphasis
the political importance of lifestyle choices made by women (notably reproductive rights within
the United States).
The redefinition of that which we should consider political is one of the lasting legacies of the
feminist movement. Hanish should therefore be viewed as part of a wider movement for social
change and one that has undoubtedly broadened our consideration of that which falls under the
realm of political ideas. It may also have served to empower women from the confines of
cultural and social expectations of femininity (particularly in the context of motherhood and the
domestic realm).
Adopting the phrase ‘the personal is the political’ is part of the broader aim amongst feminists
to break down the traditional dichotomy between the public sphere and the private sphere. This
is based upon an understanding that gender expectations can both reflect and reinforce the
power imbalance between men and women. As such, that which is traditionally considered to be
in the private realm should warrant the same degree of attention as other more conventional
considerations. It is only by doing so that females can escape the stifling confines of that
behaviour which is expected of them.
Susan Moller Okin in her book ‘Justice, Gender and Family’, while giving reference to the maxim
“personal is political”, highlight that the contemporary theories of justice ignored feminist view
and suffer in both coherence and relevance . Gender should be important component of theory
of justice. She defines gender as institutionalized differences between sexes , chief among them
is the division of labour in family in which woman have role of homemaker and men that of
bread winners. These roles do not arise out of biological necessities , but by patterns of
socialisation. Confronting gender roles is crucial because of the different roles , there is unequal
distribution of power prestige, opportunities for development and even self esteem .
Catherine Mackinnon shows that how personal is political has shaped the laws made by state.
She has analysed the rape laws and laws related to pornography, insufficient to provide justice
to women.

6(a). “Who so ever therefore out of state of nature unite into a community must be
understood to give up all the powers to the ends for which they unite into the society.” Locke .
Elucidate
John Locke is known as father of liberalism . He has given the theory of natural rights ,
government by consent and toleration .
If Hobbes has seen the troubled phase of British history and gives the pessimistic view, Locke has
seen the stable period and so gives ‘balanced view’. Locke takes the view of enlightened self
interest. In man reasons and passions are present in balanced form. In the words of Locke,
“reason in man guides him, not to harm the other in his life, liberty, health and property.” Thus,
man is rational enough to understand that if he respect the life, liberty, property of other
person, other person will also respect his rights.
Unlike Hobbes, in whom, the state of nature is the state of war because man does not have
sufficient reason, the state of nature in Locke is the state where peace, mutual assistance and
goodwill prevail. In the state of nature, people have natural rights. They are able to enjoy the
natural rights because of the presence of reason. i.e. the natural law .

Though peace and goodwill prevails in state of nature, yet there were certain inconveniences. If
man has reason, man also has passions. It may happen that passions dominate over his reason .
Hence, man requires insurance against such a scenario. Since, man can lead a good life even
without state, there is no need for giving absolute powers to the state.
Locke also mentions certain inconveniences in the state of nature. Absence of common authority
to make law, execute law and adjudicate law. In the state of nature, common authority was
absent hence everyone was interpreting the natural law according to his own preference. To
remove these inconveniences, and to ensure that peace and goodwill continues to prevail, man
enters into the contract. The main purpose behind the contract is the creation of common
authority. This common authority is government. People agree to transfer three of their natural
rights to the government.
1. Right to make law.
2. Right to execute law.
3. Right to adjudicate law.

Thus, emerged the common authority with three organs - Legislature, Executive, and Judiciary
Hence, according to Locke, Who so ever therefore out of state of nature unite into a community
must be understood to give up all the powers to the ends for which they unite into the
society.Thus , Locke gives the concept of government by consent .
Since government is not a necessity, Locke gives only limited rights. Power of government is
limited by the natural rights i.e. Right to life, liberty and property. Thus, government cannot
take away these rights. It cannot make any such law which deprive a person from his life, liberty
and property. Government has only delegated powers. It means government is a trust and
people are trustees.
Locke even gives right to revolution i.e. peaceful change under the following conditions :
1. If executive does not allow legislature to function.
2. If executive does not implement the laws passed by legislature.
3. If executive puts people under the rule of foreign prince or government.
4. Elections and the ways of elections are altered without consent.
5. When government establish its arbitrary will in place of laws.

Thus, to end the inconvenience of the state of nature and to be able to enjoy the natural rights
of life, liberty and property , all will have to transfer their right to make , execute and adjudicate
law.

6(b).Explain the difference between equality of opportunity and equality of outcomes.


Explicate the relationship between liberty ,equality and justice.
Equality of opportunity is a state of fairness, in which , individuals should succeed or fail based
on their own efforts and not extraneous circumstances . The idea is to remove arbitrariness . It is
opposed to nepotism. .The concept is applicable in areas of public life such
as employment and education. Equality of opportunity is central to the concept of meritocracy.
Equality of opportunity can be categorized into :
A. Formal equality of opportunity , which is procedural and points towards lack of direct
discrimination.
B. Substantive equality of opportunity. It requires that society should fair as well, and not
just meritocratic.
Substantive equality is often difficult to address, often measured by parameters like social
mobility and Gatsby curve.
According to Milton Freidman formal equality of opportunity should not be interpreted as the
survival of the fittest , but , that positions are opened for the talented and they should not face
obstacles like birth, colour, religion , sex like found in medieval ages.
Scholars like John Rawls , Dworkin , Amartya Sen recognize that it is difficult to achieve formal
equality of opportunity in the absence of level playing field in the society. Hence , Rawls
proposed “fair equality of opportunity”. Equality of opportunity is fair only when it results into
the maximum advantage to the least advantaged. Dworkin recommends strongly to make
people equal at the initial level by giving extra clamshells to those who suffer from brute luck
.Hence, he recommends ambition sensitive and endowment insensitive auction. Amartya sen
recommends the equality of capabilities and not just equality of sources. Bhikhu Parekh’s in his
book, ‘Rethinking Multiculturalism ’, writes that all citizens should enjoy equal opportunities to
acquire the capacities and skills needed to function in society and to pursue their self chosen
goals . Equality of opportunity will include cultural rights which are insured by politics of
recognition.
Affirmative action policies are based on the substantive idea of equality of opportunity .The so
called fair equality of opportunity has been criticized by neo-rightists and neo-liberal scholars .
For example – Hayek held that, not only equality of opportunity , but the entire concept of social
justice is a ‘mirage’. Nozick , argued against equality of opportunity on the ground of violation
of right to property .

Equality of outcomes is often contrasted with equality of opportunity . The equality of outcomes
is more controversial because it is seen that for the sake of equality of outcomes , there can be a
compromise with equality of opportunity and may adversely impact merit. Equality of outcomes
is often associated with Marxists and socialists scholars . In Marx’s communism , there is perfect
equality with the abolition of classes . For Marxists , to achieve results , elimination of material
inequalities is essential. Equality of outcome is projected as deeper , broader, result oriented ,
redistributive justice . It is based on the realization that equality of opportunity may not be
enough to treat inequalities. However, the equality of outcome is seen to stifle individuality and
abilities .
Equality of outcome is strongly opposed by the conservatives and libertarian . To quote Milton
Friedman , equality of outcome would necessarily and unfortunately increase the coercion by
government . Striving for equality of outcome leaves most of the people without equality and
without opportunity. However , socialists look at equality of outcome as a positive good.

Liberty, equality and justice are the prominent values in normative political theory. Not only
their relationship is contested, their meanings are also contested .Their relationship can be
discussed only from the perspective of different schools of thoughts. The meaning of liberty and
equality shapes the conception of justice . There is a major disagreement over the relationship
between liberty and equality. Scholars like Alexis de Tocqueville , Hayek, Nozick consider them to
be anti-thetical . Whereas scholars like Laski, T.H. Green , Rawls, Dworkin and Amartya Sen look
at these concepts as complimentary . For example , any attempt to increase economic equality
dilutes liberty and goes against entitlement . However , for Rawls , liberty and equality can be
reconciled within the framework of Justice as fairness.To conclude , it can be said the
relationship between these three normative concepts remains contested. If Neo-liberals give
primacy to liberty , social liberals gives primacy to equality . To quote Isaiah Berlin , values are
inherently pluralistic and incommensurable .

6(c) Explain the relevance of Bhikhu Parekh’s approach to multiculturalism in present times.

According to Bhikhu Parekh, multiculturalism is best understood neither as a political doctrine


with a programmatic content nor a philosophical school with a distinct theory of man’s place in
the world but as a perspective on or a way of viewing human life. Its central insights are three
.First, human beings are culturally embedded in the sense that they grow up and live within a
culturally structured world.
Second, different cultures represent different systems of meaning and visions of the good life.
Third, every culture is internally plural and reflects a continuing conversation between its
different traditions and strands of thought.
From a multiculturalist perspective, no political doctrine or ideology can represent the full truth
of human life. Each of them – be it liberalism, conservatism, socialism or nationalism – is
embedded in a particular culture, represents a particular vision of the good life, and is
necessarily narrow and partial. Liberalism, for example, is an inspiring political doctrine
stressing such great values as human dignity, autonomy, liberty, critical thought and equality.
However, they can be defined in several different ways, of which the liberal is only one and not
always the most coherent.

From a multiculturalist perspective the good society cherishes the diversity of and encourages a
creative dialogue between its different cultures and their moral visions. Such a society not only
respects its members’ rights to their culture and increases their range of choices but also
cultivates their powers of self-criticism, self-determination, imagination, intellectual and moral
sympathy, and contributes to their development and well-being.
A multicultural society should not repeat the mistake of its monocultural counterpart by
requiring that all its communities should become multicultural. Indeed, it is precisely because it
cherishes cultural plurality that it accommodates those that do not share its dominant cultural
ethos.
A multicultural society cannot be stable and last long without developing a common sense of
belonging among its citizens.
In a multicultural society different communities have different needs, and some might be
structurally disadvantaged or lack the skill and the confidence to participate in the mainstream
society and avail of its opportunities. Both justice and the need to foster a common sense of
belonging then require such measures as group-differentiated rights, culturally differentiated
applications of laws and policies, state support for minority institutions, and a judicious
programme of affirmative action.
Although equal citizenship is essential to fostering a common sense of belonging, it is not
enough. Citizenship is about status and rights; belonging is about acceptance, feeling welcome,
a sense of identification.

The multiculturalism represented by Bhikhu Parekh represent post colonial perspective. His
thesis emerges out of the criticism of liberals who do not take culture seriously enough . He
criticizes Rawls for being deeply inhospitable to cultural plurality . Rawls political liberalism
limits political discourse . Even Will Kymlicka goes for very thin approach of multiculturalism.
Kymlicka’s suggestion that national minorities to be given special rights subject to the condition
they adhere to certain liberal parameters, fails to take cultural diversity seriously enough. He
calls Rawls and Kymlicka as minimal universalists .Bhikhu Parekh also criticizes Joseph Raz who
makes cultural autonomy to minorities in public realm conditional that they develop some
notion of personal autonomy of their members . Bhikhu Parekh’s Theory takes pluralism more
deeply according to him , rights of cultures are primary rights.
According to Thomas Pantham , Bhikhu Parekh highlighted the flaw of liberalism which continue
to remain under the ghost of universalism .

7(a).Individualism and absolutism of the state were the two sides of the same coin in
Hobbes’s theory. Comment. Discuss Hobbes’s concept of political obligation.
Hobbes is known as the first modern thinker in the history of western philosophy . He is the first
one to give the complete theory of sovereignty . According to him , the state of nature has been
the state of war because of extremely individualistic nature of man . Hence , the utilitarian man
comes together to enter into the contract. They have transferred all of their rights to the state in
order to come out of the state of nature . People have transferred all of their rights except right
to self preservation . Once state comes into existence , no one can claim liberty except and to
the extent permitted by the sovereign . To quote Hobbes, liberty is where law is silent .
According to him , man has only two options , either to live under the absolute state or under
absolute anarchy. If Hobbes is known as greatest of all absolutist for giving absolute power to
the state . He is also known as greatest of individualists too. According to Wolin , individualism
and absolutism are the two sides of the same coin in Hobbes’s theory . According to
Macpherson , he is individualistic in his assumptions and absolutists in his conclusions .
According to Dunn, Hobbes represent a combination of liberal and illiberal features. If Leviathan
has absolute powers , individual is not submerged into an anonymous mass. According to Sabine
, the two themes are interlaced to provide the political order , to eliminate the state of war .
Political obligation is a core area of political philosophy , the objective is to explain the rationale
behind man’s obedience to the state . Hobbes’s political obligation can be discussed as
following.

Hobbes offers various arguments as to why people should obey the state .
A. Purely prudential consideration that if they disobey, they would be punished .
B. He gives moral considerations in the sense that they must honour the contract provided
others are also doing so .
C. Political considerations that sovereign is there duly authorized representative who is
acting on their behalf .
Leo Strauss, held that Hobbes’s obligation is physical because Leviathan has over whelming
power and subjects obey due to fear of punishment .
For Taylor , Hobbesian obligation is linked to psychological nature of man guided by pursuit of
self interest.
According to Oakeshott, Hobbes obligation does not arise out of purely individualistic nature or
self interest. It is also guided by the moral obligation which emerged due to consent .
Warrender points towards the motive to live in peace , contrary to natural passions.
Macpherson who considers Hobbes as scholar of possessive individualism believes that Hobbes
is trying to present political obligation as moral to prevent the acts of rebellion

7(b) Discuss the factors responsible for the rise of behavioral approach to politics. Critically
evaluate the contribution of behavioralism towards the discipline of political science.
The process of development of new science of Politics, which has now come to be known as
modern Political Science, began with the coming of Behavioural revolution. .Behaviouralism
seeks to study politics as an aspects of human behaviour in a framework of reference common
to other social sciences and prescribes the use of empirical research, mathematical-statistical-
quantification techniques of data collection and analysis with the purpose of building a scientific
theory political behavior.
Almond and Powell have observed, “Behavioural approach is the study of the actual behaviour
of the incumbents or political roles, rather than of the content of legal rules or ideological
patterns”.
Robert Dahl defines it as, “an attempt to make empirical component of political science more
scientific. It aims at stating all the phenomena of government in terms of observed and
observable behaviour of men........”
Though the roots of political behaviouralism stretch back in the work of such European scholars
as Max Weber and Graham Wallas, its phenomenal post 1945, development has been primarily
the work of Americans.

Behaviouralism came as a protest against the formalistic legal-institutionalism of the traditional


political science. In the words of Dahl, “Historically speaking behavioural approach was a
protest movement within political science..... emerged as “a strong sense of dissatisfaction with
the achievements of conventional political science” . The legal and institutional approaches
were rejected as parochial, formal configurative, static and inadequate for the study of politics.
The real picture of political life could not be obtained through the traditional approach. Those
were not given any importance in government’s policy-making. Traditional approach lacked
realistic study.
The happenings during Second World War inspired the political thinkers for new researches and
concepts. It became necessary to concentrate on political organizations and on the behaviour of
persons working there, rather than institutions and their organizations, to understand
completely the complexities of political life.
Scientific study methods and realistic approach had already been used in other social sciences.
So inspired by them, Political Science also started using scientific and new methods of study.
Increasing use of new methods, techniques, instruments, research methods, statistical
approaches, etc. made study of Political Science more realistic and helped behaviouralism to
grow.
Behaviouralism has had revolutionary impact on Political Science. Its emphasis has been to
understand man in society and bring about unity in social sciences. Scholars have learnt new
modes of analysis and methods of data collection. There is a tendency of cross fertilisation or
exchange of knowledge among scholars belonging to various disciplines. It has given birth to
many behavioural sciences, and has transformed the format of traditional disciplines. It has
broadened the scope of Political Science itself.
Apart from academic purposes, behaviouralism has a great practical utility, particularly, to bring
about social change. It intends to develop modern political theory in a manner that it may be
utilised for the purposes of ‘manipulation’ and ‘policy formulation’. Due its interdisciplinary
approach capacity and competence of social scientists has also increased, so is their
responsibility.
Behavioralism also influenced international relations, though it did not achieve the same
dominance in this area that it enjoyed in domestic and comparative politics. Behaviouralists
gathered much quantitative data and became one of the leading sources for scholars studying
the causes and effects of war and international tension.
Behavioralism also established itself in studies of judicial and bureaucratic systems.
However, the approach has come under criticism from both conservatives and radicals for the
purported value-neutrality. Conservatives see the distinction between values and facts as a way
of undermining the possibility of political philosophy. Leo Strauss believes behaviouralism
dismisses "the task of ethical recommendation" because behaviouralists believe "truth or falsity
of values cannot be established scientifically and are beyond the scope of legitimate inquiry."
Christian Bay believed behaviouralism was a pseudopolitical science and that it did not
represent "genuine" political research. Bay objected to empirical consideration taking
precedence over normative and moral examination of politics.
Behaviouralism , as an approach has been criticized for "naive scientism". Additionally, radical
critics believe that the separation of fact from value makes the empirical study of politics
impossible.
In order to overcome the drawbacks in the behavioural approach , post-behaviouralism
emerged. It is a future oriented approach aimed at solving the social problems of both present
and future. In this way by interlinking the empirical and normative study of political science to
include all realities of politics social change. Post behaviouralism is instrumental in a revival of
political science by making it more relevant to the society.
Thus , the situational factors related to the decline of political theory and the intellectual factors
recognizing the decline and the consequent neglect in international conferences fused the new
life in the discipline towards more reliable theories.

7(c). Discuss the salient features of different waves of feminism . What do you understand by
intersectionality in feminism.

The term feminism describes political, cultural, and economic movements that aim to establish
equal rights and legal protections for women. Over time, feminist activists have campaigned for
issues such as women’s legal rights, , property, and voting; body integrity and autonomy;
abortion and reproductive rights, including contraception and prenatal care; protection from
domestic violence, sexual harassment, and rape; workplace rights, including maternity leave
and equal pay; and against all forms of discrimination women encounter.
Feminist history can be divided into three waves.
The first wave, occurring in the 19th and early 20th century, was mainly concerned with
women’s right to vote. The second wave, at its height in the 1960s and 1970s, refers to the
women’s liberation movement for equal legal and social rights. The third wave, beginning in the
1990s, refers to a continuation of, and a reaction to, second-wave feminism.
The First-wave feminism promoted equal contract and property rights for women, opposing
ownership of married women by their husbands. By the late 19th century, feminist activism was
primarily focused on the right to vote. American first-wave feminism ended with passage of the
19th Amendment to the US Constitution in 1919, granting women voting rights. The
philosophical basis of liberal feminism lies in the principle of ‘individualism’ i.e. all individuals are
entitled to equal treatment, regardless of their sex, race, culture etc. According to Mary
Wollstonecraft’s ‘Vindication of Rights of Women’ women should be entitled to the same rights
and privileges as men on the ground that they are all ‘Human beings’. She claimed that the
distinction of sex would become unimportant in political and social life if women gained access
to education and other rights.
Second-wave feminism of the 1960s-1980s focused on issues of equality and discrimination. The
second-wave slogan, “The Personal is Political,” identified women’s cultural and political
inequalities as inextricably linked and encouraged women to understand how their personal
lives reflected sexist power structures. Betty Friedan was a key player in second-wave feminism.
In 1963, her book The Feminine Mystique criticized the idea that women could find fulfillment
only through childrearing and homemaking. Her work ignited the contemporary women’s
movement in 1963 and as a result permanently transformed the social fabric of the United
States and countries around the world. Friedan hypothesizes that women are victims of false
beliefs requiring them to find identity in their lives through husbands and children. This causes
women to lose their own identities in that of their family.
Third-wave feminism began in the early 1990s, responding to perceived failures of the second
wave and to the backlash against second-wave initiatives. This ideology seeks to challenge the
definitions of femininity that grew out of the ideas of the second-wave, arguing that the second-
wave over-emphasized experiences of upper middle-class white women. The third-wave sees
women’s lives as intersectional, demonstrating how race, ethnicity, class, religion, gender, and
nationality are all significant factors when discussing feminism. It examines issues related to
women’s lives on an international basis.
Intersectionality is an analytical framework for understanding how aspects of a person's social
and political identities combine to create different modes of discrimination and privilege. The
term was conceptualized by Kimberlé Williams Crenshaw. According to her, interconnected
nature of social categorization such as race, class and gender create interdependent and
overlapping set of disadvantages.
Intersectionality broadens the lens of the first and second waves of feminism, which largely
focused on the experiences of women who were both white and middle-class, to include the
different experiences of women of color, women who are poor, immigrant women, and other
groups. Intersectional feminism aims to separate itself from white feminism by acknowledging
women's different experiences and identities. Intersectionality makes feminist movement more
inclusive and allows women of all races , religions, identities , orientations for their voices to be
heard because all of us might experience discrimination and inequalities differently.
Intersectionality opposes analytical systems that treat each oppressive factor in isolation, as if
the discrimination against black women could be explained away as only a simple sum of the
discrimination against black men and the discrimination against white women.
However, intersectionality has been criticized for it's tendency to reduce individuals to specific
demographic factors, its use as an ideological tool against other feminist theories, and its
association with antisemitism. Critics have characterized the framework as ambiguous and
lacking defined goals.Critics say the focus on subjective experiences can lead to contradictions
and the inability to identify common causes of oppression.

8(a).“Socialism is like a hat that has lost its shape because everyone wears it.” C.E.M Joad.
Comment. Discuss the relevance of socialism in 21st century.
In his Introduction to modern political theory, Joad observed that socialism is like a hat that has
lost its shape , because every one wears it. He tries to show that the term has been variously
defined and understood . In popular parlance , socialism is seen as an economic system where
the means of production are under social ownership rather than private ownership . There are
many varieties of socialism . For example- The two broad categories are :

1. Evolutionary Socialism – It denotes a peaceful and a piecemeal approach to social


transformation .
2. Revolutionary socialism – It denotes the sudden violent and comprehensive over throw of
the system .
Some of the prominent varieties of evolutionary socialism are revisionism of Edward Berstein ,
Fabianism of Sydney Webb, Guild socialism of G.D.H Cole. The Marxists tradition comes under
revolutionary . Even the syndicalism represented by Ferdinand Pelloutier comes under violent
category .
With the emergence of WTO, neo-liberalism became the hegemonic ideology, if liberalism
became end of history, socialism became ‘dustbin of history’. However very soon, the negative
impacts of neo-liberal model of economic growth have been realized. Thomas Piketty, who is
also called as modern Marx has demonstrated the huge inequalities which have emerged
throughout the world in his book THE CAPITAL IN 21st CENTURY.
We have seen the rise of civil society movements against the neo-liberal economic policies. Thus
at present, we do not see socialism to be adopted by the countries in their economic policies but
socialism is gaining relevance as a movement known as ‘anti-capitalist movement’. There are
numerous protest movements influenced by left ideology, challenging neo-liberalism at present.

Socialism always remained an utopia. Socialism’s biggest relevance lies in making capitalism
humane. To ensure capitalism remains humane, socialism will always be relevant. e.g. The
biggest achievement of Marx can be considered as emergence of welfare state.
8(b) Elaborate on Aurobindo’s theory of nationalism. To what extent it would be appropriate
to call him ‘Prophet of Indian nationalism’
There are three schools of nationalism – Liberal, Marxist and cultural . Liberals believe nation is
a result of certain historical developments. Marxists believe that nation is an invented tradition .
The cultural nationalists believe that nation is natural .Aurobindo Ghosh belonged to the school
of cultural nationalism. Aurobindo countered the view of colonial state that India is nothing
than a geographical expression and found response of early nationalists like Surendra Nath
Bannerji , who held that India is a nation in making . Aurobindo tried to establish that India is a
nation from eternity . Emergence of India as a nation is inevitable and is in the interest of the
world . Aurobindo's concept of nation was deeply influenced by Vivekanand and Bankim
Chandra . To quote Aurobindo , “Nation is not just a piece of land nor a mass of human beings.
It is neither a figure of speech nor the creation of mind. It is something more than a
geographical unit .” Thus, his concept of nation is profound and very different from the
commonly held view of India as a nation.
He glorified lndia as a Mother Goddess, and appealed the youth that the liberation of the
motherland is the most urgent duty .

He considered the nationalist movement sparked off by the partition of Bengal as a religious
mission. For him, "nationalism is a religion by which people try to realise God in their nation,
and their fellow country men".

The concept of nationalism which dominated his thought and activity in the early phase was just
a stepping stone to move in the direction of the unity of humankind. This unity of humankind
was regarded by him as a part of nature's eventual scheme and as the inevitable goal of human
development.

It is argued that to regard nationalism as an instrument of spiritual perfection is too idealistic


and visionary for the common person. To associate religion with politics, though in the name of
spirituality, is a dangerous proposition in a multicultural plural society like India. It is argued
that in the ultimate analysis, this exercise has resulted in increasing the strife between the
Hindus and the Muslims, which finally resulted in the partition of the country.

His concept of nationalism and human unity were based on his understanding of the Hindu
Sanatana Dharma, which to him meant an open and universal philosophy of life. His concept of
nationalism clearly indicates his spiritual approach to politics. Not merely his theory of
nationalism, but his political philosophy in its totality has spiritual overtones.

By advocating complete freedom from the foreign domination as the final goal of the freedom
struggle he brought a change in the texture of our national movement. It was quite a radical
stand taken by him when the moderates were spearheading the national movement.
Aurobindo was the first political thinker who recognised the need of giving a broad base to the
national movement. With his advocacy of radical methods to attain the goal of full freedom for
Indian nation, he created a favourable atmosphere for revolutionary spirit among the political
activists and by participating in it as their leader he set an example of being both, a theoretician
as well as practitioner and a good organizer which is a rare combination of the qualities of the
prophet and the leader. Thus , Aurobindo is truly the ‘Prophet of Indian nationalism’

8(c) What do you understand by legitimation crisis. Discuss the ways in which states try to
gain legitimacy .
Political legitimacy is a virtue of political institutions. It is the right and acceptance of an
authority, usually a governing law or a regime. It is considered as a basic condition for
governing. With respect to political theory, a state is perceived as being legitimate when its
citizens treat it as properly holding and exercising political power.
Detailed at greater length in The Social Contract, Rousseau insists that government legitimacy is
dependent upon the "general will" of its members. According to Weber, a political regime is
legitimate when the citizens have faith in that system. Weber provides three main sources of
legitimate rule: traditional, rational-legal, and charismatic .
In his book, ‘Legitimation Crisis’, Jurgen Habermas examines the deep tensions and the
legitimation problems faced by post capitalist states. Habermas argues that Western societies
have succeeded to some extent in stabilizing the economic fluctuations associated with
capitalism, but this has created a new range of crisis tendencies which are expressed in other
spheres. States intervene in economic life and attempt to regulate markets, but they find
themselves confronted by increasing and often conflicting demands. As individuals become
increasingly disillusioned, the state is faced with the possibility of a mass withdrawal of loyalty
or support i.e. 'legitimation crisis'. It refers to a decline in the confidence of administrative
functions, institutions, or leadership. With a legitimation crisis, an institution or organization
does not have the administrative capabilities to maintain or establish structures effective in
achieving their end goals.
Habermas, a neo-Marxist scholar, argues that within a liberal democratic set-up there are ‘crisis
tendencies’ which challenge the stability of such regimes by undermining its legitimacy to
govern. The core of this argument is the tension or incompatibility between capitalist economy
and a democratic political setup. The democratic forces make it necessary for the government to
respond to popular demands, sanctioned by a number of political, social and other rights. In
order to meet the popular demands, government pursue such policies which threatens the long-
term survival of capitalist order. ‘
On the other hand, if the popular demands exceed the limit within which the government has
the capacity and will to meet them there emerges a wide gap between the ability of the
government to deliver and the expectations of the people. According to Anthony King, this
problem is one of the ‘Government Overload’.
In a capitalist system, there’s a limit to which the political system can be used to meet the
popular demands. In this context, Habermas argues that the capitalist society is based on the
fundamental contradiction of social production versus private appropriation. This suggest that
liberal democracies cannot permanently satisfy both popular demands for social security &
welfare rights on one hand and requirements of a market economy based upon private profit.
Thus, emerges a situation when, in an attempt to balance the democratic pressures and risk of
economic collapse, government find it difficult to maintain its legitimacy. It is this gap that
manifests itself in the form of ‘legitimation crisis’ in the form of steady accumulation of public
discontent, popular protests and widespread disillusionment.
States maintain legitimacy through numerous ways including the effectiveness of public
institutions in their performance of various functions, such as service delivery, taxation and
social protection systems; and their degree of representation and accountability.
States broaden their Citizen engagement to understand people's needs. It involves
acknowledging citizens as individuals with different interests, skills, needs and ambitions.
Governments strengthens their legitimacy by bringing in transparency in the way of functioning
and the citizen's voice in decision-making over and above the obvious democratic benefits.State
project its competence and fairness as government must be seen as treating all people equally
and impartially, without favoritism or discrimination.
Elections are the most important mechanism to seek legitimacy. Governments seek fresh
mandates periodically . Level of participation helps in gauging the degree of support the state
enjoys.For example – the percentage of participation in polling in areas with separatist
tendencies is closely watched and is seen as an evidence of legitimacy.
Similarly, inclusive legislations, effective execution and independent adjudication is another way
states can maintain their legitimacy.
PSIR Crash Course 2021
Model Answers - Test 2

Section A
Answer the following in about 150 words each
1(a). Contribution of Chipko movement in the evolution of India’s environmental
movement.
Chipko Movement, born in March 1973 in Reni village, under the leadership of Gaura Devi,
symbolise the beginning of India’s environmental movement. The movement for the first
time catapulted the environment into political discourse and has shaped the understanding
of environmentalism in India and the world. A major impact of the Chipko movement was
that it prompted the Union government to amend the Indian Forest Act, 1927, and
introduced the Forest Conservation Act 1980, which says forest land cannot be used for non-
forest purpose. Same year, in a historical order, commercial green felling was banned in
forests above the 1,000 metre altitude. It led to many other similar movements like Appiko
in Karnataka, Silent Valley in Kerala , North 24 Parganas in west Bengal . The impact of
Chipko movement is not limited to India and the global south , even Japan had similar
movement in Mt. Takao. The movement caught the imaginations of groups concerned with
the environment in countries such as Switzerland, France, Mexico, Denmark, Australia,
Canada and Malaysia. The biggest impact of the movement is the explosion of the myth that
poors are the part of the problem rather than the participant in the solutions . The
movement has inspired eco-feminism in India and worldwide. The impact of Chipko is
palpable across the valley. A study by the Space Applications Centre of the ISRO ,
Ahmedabad, in August 1994 shows that between 1972 and 1991, at least 5,113 ha of forest
has been raised outside the reserve forest boundary and in the vicinity of villages, in the
watersheds of Nagolgad, Amritganga (Balkhila), Menagad and Kalpaganga and other
watershed of the Alaknanda. Tehri now witnesses a second wave of the movement under
Beej Bachao Andolan (BBA) and Uttarakhand Jan Jagriti Sansthan (UJJS). In Dehradun,
Chipko activists have set up Himalaya Action Research Centre that trains farmers in organic
farming and micro-enterprises.
The Chipko movement can also be seen as a peasant and women’s movement in the
evolution of India’s social movements. Belief in non-violence, cooperation and self-help are
the basic axioms of the Sarvodaya Philosophy helped the Chipko movement moving forward.
Chipko movement can also be seen from the perspective of Marxian ideology which see the
destruction of environment as profit maximisation by the bourgeoise class. Chipko can be
seen as a cultural response of the people's love for their environment, particular to Indian
ethos. Despite its many successes, Shekhar Pathak, a historian with in-depth knowledge of
Uttarakhand and founder of People’s Association for Himalaya Area Research, claims that
the movement has failed in providing sustainable livelihood . However, he thinks, the
movement is far from over.
To conclude, a movement must never cease. Chipko still survives and the impact philosophies
of the movement has spread beyond Uttarakhand hills, and has its global footprints not only
in social movements , but even in social sciences.

1(b). GST is a mantra for a cooperative federalism or fiscal centralization ? Comment.


Cooperative federalism, also known as marble-cake federalism, is defined as a flexible
relationship between the federal and state governments in which both work together on a
variety of issues and programs. Thus , it differs from dual federalism where the two levels of
governments work like two watertight compartments .
GST, which was introduced by 101st amendment act , 2016 is a significant step in the field of
indirect tax reforms in India. By amalgamating a large number of Central and State taxes
into a single tax, GST aims at mitigating the ill effects of cascading or double taxation in a
major way and pave the way for a common national market.
According to Yamini Aiyar (‘End of the road for cooperative federalism’), once projected as
the poster child of the new era of cooperative federalism has become a new battleground for
centre and states fiscal disputes . Giving way to a widening trust deficit. It is important to
locate the GST debate within the broader framework of political economy of centre state
fiscal relations . States in India have routinely complained that successive union governments
tried to encroach the functions of state government. There has been issue over GST
compensation and the demand to exempt essential Covid Supplies from states .
According to Louis Tillin ,the dynamics of Indian politics has shifted towards deepening of
centralization since 2014 onwards. India has witnessed unparallel consensus in July , 2017
when GST was ushered in . States willingly gave up fiscal autonomy for promise of economic
efficiency . Centre demonstrated a genuine federal spirit promising the compensation cess.
According to economist Rathin Roy, India has been going through the silent fiscal crisis . He
gives following reasons :
A. Continuation of centrally sponsored schemes
B. Actual transfer to the states has been between 30 -35 % despite accepting the
recommendation of the 14th Finance commission for transferring 42%.
C. Union government resorted to tactics like delaying payments and transferring GST
cess to its own coffers.
D. Pandemic played havoc and widened the trust deficit.
1(c) Critically evaluate the role of revolutionaries in Indian freedom struggle.
British Government’s repression and frustration caused by the failure of the moderate
politics ultimately resulted in revolutionary movements. At the dawn of the 20th century the
school of militant nationalists, represented by leaders like Tilak, Bipin Chandra Pal,
,LalaLajpat Rai, found a favourable political climate. They were influenced by the methods of
the Irish terrorists and the Russian Nihilists, revolutionaries that force must be stopped by
force.

Revolutionaries could inspire people to join the movement through several newspapers like
The Sandhya and the Yugantar in Bengal and the Kal in Maharashtra. Many secret societies
like the Anushilan Samiti came into existence .

Revolutionaries operated from foreign lands as well . In London the lead was taken by
Shyamji Krishnavarma, V.D, Savarkar, and Lala Har Dayal, while in Europe Madam Cama
and Ajit Singh were the prominent leaders. The Ghadar Party was pledged to wage
revolutionary war against the British in India as soon as the First World War broke out in
1914.
The distinctive political aspects of the revolutionaries was that great sacrifices and sufferings
were needed . Their speeches, writings, and political work were full of boldness and self-
confidence and they considered no personal sacrifice too great for the good of their country.
They denied that India could progress under the “benevolent guidance” and control of the
English.

Significant is the fact that liberation of women received a great stimulus from the rise of the
revolutionary national movement in the 20th century. Women like Bhikhaji Cama, Kalpana
Datta, Pritilata Waddedar played an active and important role in the revolutionary struggle
for freedom.
The revolutionary movement took a socialist turn and, in 1928, under the leadership of
Chandra Shekhar Azad, the Hindustan Socialist Republican Association widened its struggle
against any such system, native or foreign , ‘which made exploitation of man by man
possible ’
However, due to demand of extreme patriotism and its methods of violence , the
revolutionary movement could not become as broad as Gandhian . Large sections of women
and elderly could not afford to go for such extreme and violent methods . The support for
violence too was somewhere in conflict with cultural ethos of India. The use of force gave an
excuse as well as justification for British to unleash force on its part as well. The violent
movements were easy to be crushed as a counter hegemony for violent methods could not
be built in international press and civil society, the way it was seen in case of Gandhian non
violent movements .
They became immensely popular because of their heroism. Though the revolutionary
movement is said to have failed in formal sense and Gandhian method is credited for
winning India’s freedom . Nevertheless , Revolutionaries made a valuable contribution to the
growth of nationalism in India and appealed to the patriotism of masses.

1(d).Parliamentary committees form the soul of Parliamentary democracy. Comment


The importance of Parliamentary committees can be gauged by Woodrow Wilson's
statement in 1885: “...it is not far from the truth to say that Congress in session is Congress
on public exhibition, whilst Congress in its committee rooms is Congress at work.”

Parliamentary standing committees are permanent committees appointed or elected by the


House or nominated by the Speaker/Chairman. They present their report to the Houses;
thereby assist the working of the Parliament in its various activities. Some of them are Public
Accounts Committee, Estimates Committee, Committee on Public Sector Undertakings,
Departmental Standing Committees etc.

The Parliamentary Committees strengthens the ability of the Parliament to scrutinize


government policies and make the government accountable. Parliamentary Committees
enhances the role of the Opposition whose members are part of these Committees. It
increases engagement with relevant stakeholders as the Committees act as links between
the Parliament and the people on the one hand, and the administration and the Parliament
on the other.

Parliamentary Committees work in non-partisan way. Their meetings are held behind closed
doors and members are not bound by party whips, which allow them to have meaningful
exchange of views. They help in financial prudence as they ensure economy and efficiency in
public expenditure.

However, despite their significance, only 25% of the bills introduced were referred to the
Committees in the 16th Lok Sabha, as compared to 71% and 60% in the 15th and 14th Lok
Sabha respectively (PRS).

The House rules do not mandates that all bills introduced in the House should be referred to
the standing committee as a matter of course - Although it's impractical to send all bill to the
standing committees. However the chair must use his or her independent judgement on case
by case basis. There has been delay in the present government, towards the constitution of
standing committees . Government’s preference for Ordinance route , further undermine the
role of parliamentary scrutiny.

Therefore , it is imperative to adopt the recommendations of the National Commission to


Review the Working of the Constitution, 2002 of referring all bills to the Committees, longer
tenure for its members and strengthening the Committees with adequate research support.
The committee process in India can be made more effective and transparent by live-
streaming.MPs should have longer tenure in committees so that they could build up their
expertise in subject areas.
Parliamentary committees are the brain of Parliament which ensures that the Parliament
becomes the intellectual compass for good governance in the country.

1(e). Urban local governance in India is perpetually afflicted with lack of financial
autonomy . Elaborate.
According to the 1901 census, the urban population was 11.4%, which increased to 28.53%
by the 2001 census, and is currently 34% according to The World Bank. According to a survey
by UN, in 2030 40.76% of country's population is expected to reside in urban areas.
Urban governance is a young concept evolved in UNDP and UN-Habitat in the last two
decades . UNDP defines urban governance as the exercise of political economic and
administrative authority to manage cities at all levels . The international Urban Governance
Index(UGI) maps governance on 5 parameters – effectiveness of services , equity ,
participation , accountability , security . UGI flags off a crucial aspect of urban governance :
financial and empowerment . In India, city governments tend to be financially dependent on
state governments which restrict their functioning , reduces accountability to the citizens
and make them susceptible to the executive . Studies recommend that cities should evolve
their independent sources of finance for more efficiency . India’s taxation structure leaves
little for local governments , GST regime has further increased the dependency on the state
governments.
74th amendment act not only give constitutional status to the municipalities , also provide
constitutional guarantees for their economic empowerment . Article 243X entrusts state
governments to impose taxes , duties , tolls and fees . It allows state governments to assign
revenue from specific taxes to ULGs. Article 243Y gives state finance commission , the task of
reviewing and recommending devolution of tax revenues and grants in aid . Article 280 has
been amended giving additional functions to Finance Commission to make recommendations
to augment the consolidated funds of a state to supplement the resources of municipalities .
The study commissioned by 15th finance commission gives following trends .
A. The municipal revenue to GDP ratio has been stagnant at around 1 % of GDP
between 2007-08 and 2017-18. IN the same period the ratio for south Africa is 6%.,
Brazil 7.4% and UK 13.9%.
B. The share of municipality’s own revenues has declined from 55% in 2007-08 to 43%
in 2017-18 .
C. In India, property tax is the only major tax base for municipalities , whereas in the
world , they have access to wider taxes . For example – tax on urban land use , real
estate tax . Even there is an insufficient growth in the property tax.
D. The intergovernmental transfers , though increased remains low . For example – In
Norway , municipalites get 6%; in UK 9.9 % , whereas in India , it is just 0.45%.
E. The municipal borrowing is very low . Only 1% of the need of urban bodies is met by
municipal bonds as opposed to 10% in USA. The municipal bonds have failed to grow
in India , primarily because of poor credit ratings .
The Economic Survey 2017-18 points out that urban local governments generate about 44%
of their revenue from own sources. It points out that lack of sufficient independent resources
pose challenge for fiscal federalism. Such situation has been termed as “low equilibrium
trap” .

Considering the criticality of urban governance , the 15th Finance Commission has taken
bold, imaginative and far-sighted measures. There are five specific recommendations.

1. Substantive increase in funds to cities: Rs 1.55 lakh crore over a five-year period from
FY21-22 to FY25-26, is a 78 percent increase over grants during the 14th FC period.
2. Efforts made to mainstream Metropolitan Governance for the first time since 74th
CAA, 1992, comprising 100 percent outcome funding of Rs 38,000 crore for 50
million-plus urban agglomerations (UAs), with an aggregate population of 150
million, tied to performance indicators on air quality, water and sanitation; shared
municipal services a fitting complement. With 100 percent funding linked to
outcome, larger cities are expected to accomplish the suggested reforms.
3. Game-changer entry conditions pertaining to publishing of audited annual accounts
and notification of floor rates for property tax applicable to every municipality, to
avail any grant from FC. Municipalities in India have not managed to publish their
audited annual accounts in a credible, timely manner for far too long. For the first
time, every municipality in India will need to publicly disclose online both audited and
unaudited annual accounts in a time-bound manner. This is a mandatory condition to
access any grant under 15th FC.
4. Deadline of March 2024 set for states to strengthen State Finance Commissions and
table action taken reports in state legislatures. Most states have not nurtured them
into credible institutions. Only fifteen states have set up the fifth or the sixth SFCs and
several states have still not moved beyond the second or third SFC. By stipulating a
deadline to state governments, the 15th FC has once again tried to conclude on a
crucial reform agenda of fiscal decentralisation in India.
5. All service level benchmarks i.e. operational or performance information on what
they actually achieved in terms of citizen outcomes or functional performance.
As per NITI Aayog’s ‘Strategy for New India@75’, Indian ULBs have huge scope to improve
their financial autonomy and capacity to raise resources. Ultimately in a democracy, higher
levels of funding and greater transparency would need to be complemented by greater civic
engagement and demand for accountability.

Large-scale systemic change does not always happen through a big bang.The 15th Finance
Commission has essentially built on a range of progressive measures with a touch of
boldness and ambition. However, without political will at the union and state level it would
not be achievable. Some of the suggestion to improve the municipal finance include
strengthening municipal bonds, property tax reform (use of GIS for extending the coverage
and frequent re-evaluations ), compensation of losses induced by GST , capacity building in
terms of financial planning and accounting , innovative financing like venture capital etc.
2(a). Governor as an appointee of centre becomes a potent weapon in centre’s armory and
often fail to fulfill its designated role as the first citizen of the state over which he reins but
does not rule .Comment .
In August , 1983, the CM of Karnataka, R.K Hegde in a seminar on centre and state relations
held that even governors have become the glorified servants of the union, later tabled a
white paper in the assembly on office of Governor which was published as appendix in the
report of Sarkaria Commission revealing that governors are hand picked . However , Sarkaria
Commission emphasised on the crucial role of the governors in India’s constitutional
schemes by calling the post as the “linchpin of the constitutional apparatus of a state”.
According to Ambedkar, the primary duty of the Governor was to ensure the discharge of
“good, efficient, honest administration”. However, the office of Governor remained a potent
weapon in the armoury of an overbearing Centre in the quasi-federal structure of India.
In the constitutional framework of India’s cooperative federalism with parliamentary form of
government, governor has dual role.

1. As the constitutional head of the state executive


2. As a bridge between union and state governments

While explaining the constitutional status , the SC in Hargovind Pant vs Raghukul Tilak and
othrs 1979, the constitutional bench with unanimity held that governor is not the employee
of the union rather the constitutional office of dignity. However, it also mentions that in
respect of some specified functions , constitution requires him to perform as an agent of
union and he is accountable to the President. In 1970, a committee of governors was
appointed by president consisting of Bhagwan Sahay, Gopala Reddy, S. S. Dhawan to study
and formulate the norms with respect to the role of governor. The committee held that , in
no sense , governor is an agent of president . Even under Article 356, there is nothing which
makes governor as an agent , though president can entrust some powers which he assumes
under Article 356, on governor.
The controversies emerge because of the allegation of the misuse of the institution of
governor by the ruling party at the centre . In place of the linchpin , governor becomes the
“long arm of the union with velvet gloves”.
The union governments , irrespective of the political parties have been alleged of misusing
the institution for the party interest , even at the cost of national interest. The institution of
governor is used to undermine the parliamentary form of government at the state
level.(Punchhi Commission). Union gets leverage on the institution of governor because of :

1. The manner of appointment


2. The manner of removal

Constitution does not make any elaborate provision , and in a way gives free hand to the
union executive as a “matter of faith”.
Some of the recent controversies with respect to the actions of the governors and the rising
tussle with the elected state government can be mentioned as following:

1. Former of Puducherry L.G. Kiran Bedi , at several moments , has been at odds with
former Chief Minister V. Narayansamy .
2. Similar clashes were seen between Chief Minister Arvind Kejriwal and former L.G. of
Delhi Najeeb Jung
3. There have been other instances in States such as West Bengal and Kerala, where
Governors have been accused of running spats with the Chief Ministers rather than
acting as a friend, philosopher and guide .

Above examples can be considered as recent one , but irrespective of the political party at
the helm at centre , has tried to misuse its power. Even Dr. Rajendra Prasad, the first
president, observed that the party politics has undermined the dignity of the office and it is
essential that the people of state should have full confidence in Supreme , non-partisan
institution like that of governor.
The first ARC (1968) observed that the politically inclined appointments of the governors
undermined the institution of governor.
Soli Sorabjee , the eminent jurist in his book “The Governor: Sage or Saboteur”, has held that
“there is no other institution that has suffered so much erosion . Despite erosion , governor
continues to be largely on uncorrected path.”
It is to be noted that governor has a critical role in the smooth functioning of India’s
cooperative federalism . Hence , it is essential that necessary reforms as suggested by
Sarkaria Commission with respect to the appointment and the removal of the governors
should be taken up at earliest.
A political and legal consensus is the need of the hour so that governors can act as a
guardian of the constitutional governance and the institutional efficacy of the system
remains.
In the 1974 case of Shamsher Singh vs the State of Punjab, Justice Krishna Iyer ruled that
governors exercise formal constitutional powers only upon and in accordance with the
advice of their Ministers save in a few well-known exceptional situations.

In B.P. Singhal vs Union of India, a Constitution Bench of the Supreme Court laid down the
following principles for the removal of Governors: “ The power of removal should be used
only if there is material to demonstrate misbehaviour, impropriety or incapacity.” The court
further held: “We have… already rejected the contention that the Governor should be in sync
with the ideologies of the Union Government. Therefore, a Governor cannot be removed on
the ground that he is not sync or refuses to act as an agent of the party in power at the
Centre.
Four substantial commissions have made various recommendations in this regard: the first
Administrative Reforms Commission Report of 1969, the Sarkaria Commission Report of
1984, the National Commission to Review the Working of the Constitution of 1998, and the
Punchhi Commission Report of 2010. Broadly speaking, these reports speak of appointing
non-political Governors through independent broad-based mechanisms, securing the
consent of the concerned Chief Minister to the appointment, and giving Governors a fixed,
non-renewable tenure.

2(b)The real purpose behind the Constitution (69th Amendment) Act, 1991, is to establish
a democratic setup and representative form of government. Analyse this statement in the
light of recent National Capital Territory of Delhi (Amendment) Act, 2021.
Though technically a Union Territory, Delhi was treated as a special case, being the capital of
the country, and given a special constitutional status by parliament through the Constitution
(Sixty Ninth Amendment) Act 1991 . Article 239AA was added , which contains general
provisions relating to the administration of Union territories. This article provided for an
assembly, fully elected, and a council of ministers responsible to the assembly. It conferred
on the assembly the power to legislate on all matters in the state list as well as the
concurrent list except land, police and public order.

The purpose behind granting a special status to Delhi has been explained by the Supreme
Court in GNCTD Vs. Union of India is to establish a democratic set up and representative
from of government wherein the majority has a right to embody their opinions in laws and
policies pertaining to the NCT of Delhi subject to the limitations imposed by the constitution.

However, there has been one sticking point in the act , that in case of a difference between
the LG and the council of ministers on any matter, the matter shall be referred to the
president by the LG for his decision and pending such decision the LG can take any action on
the matter as he thinks fit. It became the single source of federal conflicts between the LG
and the elected government of Delhi. The Supreme Court, however, ruled that LG cannot
refer a matter mechanically or mindlessly and has to make all attempts to resolve the
differences .

The recently passed GNCT of Delhi (Amendment) Act 2021 is being debated on various
parameters . It amends the Sections 21, 24, 33 and 44 of the 1991 Act.

It states that the “government” in the National Capital Territory of Delhi meant the
Lieutenant-Governor of Delhi. It gives discretionary powers to the L-G even in matters where
the Legislative Assembly of Delhi is empowered to make laws. It bars the Assembly or its
committees from making rules to take up matters concerning day-to-day administration, or
to conduct inquiries in relation to administrative decisions.

The 2021 amendment provides that the rules of the assembly shall not be inconsistent with
the rules of Lok Sabha. However , constitution experts argue that it is the privilege inherent
in every legislature to conduct its own proceedings as per the rules made by it. Framing the
rules to conduct its proceedings is thus a part of the privilege each house of a legislature
enjoys. Each legislative house is independent of the other. So, the Delhi assembly is an
independent legislative house and the Lok Sabha should not control it. Parliament has no
power to legislate and take away the inherent right of a legislature to frame rules for
conducting its proceedings.

Another provision of the recent amendment has very serious consequences. It says that the
Delhi assembly shall not make rules to enable itself or its committees to consider matters of
day-to-day administration. No rule shall be made by the assembly to conduct inquiries in
relation to administrative decisions and if such a rule already exists, it will become void .
Every democratic legislature has the inherent right to scrutinise the decisions taken by the
executive. Executive accountability is the essence of the parliamentary system of
government, which is a part of the basic structure of the constitution. Parliament has no
competence to take away this right through some legislation.

Another provision in the amendment requires the state government to obtain the opinion of
the LG on their decisions before executive action is taken on those decisions. Thus, the
elected government cannot take any action unless it obtains the LG’s opinion , even if it is an
urgent decision.

Thus, the Government of National Capital Territory of Delhi (Amendment) Act, 2021, has
provisions that enhances the powers of the lieutenant governor (LG) of Delhi and further
weakens the elected government and emasculate the assembly.

The Supreme Court held that Lieutenant Governor of Delhi had no independent decision
making powers and was bound to follow the "aid and advice" of Delhi chief minister headed
council of ministers of the Government of Delhi on all matters except those pertaining to
police ,public order and land.

It can be said that the amendment is in conflict with the decision of the Supreme Court that
the elected government of Delhi can take all decisions within its jurisdiction and execute
them without obtaining the concurrence of the LG. only, in case of a difference of opinion on
a matter between the LG and the government, the former should make all efforts to resolve
it and only in extreme cases should she or refer the matter to the president for a decision.

The amendment has been challenged in Delhi High Court in Neeraj Sharma vs UOI, 2021.
The experts believe that the amendments violate the basic structure of the constitution as
well as the privileges of the Delhi assembly. It remains to be seen whether the amendment
can stand judicial scrutiny.

2(c).What are the constitutional provisions related to freedom of speech and expression in
India. Analyse some recent controversies related to the status of freedom of speech and
expression in India.
“I disapprove of what you say, but I will defend to the death your right to say it.”― S.G.
Tallentyre, The Friends of Voltaire.

Freedom of speech is a fundamental human right that supports the freedom of an individual
or a community to articulate their opinions and ideas without fear. The right to freedom of
expression is recognized as a human right under Article 19 of the Universal Declaration of
Human Rights (UDHR) and under Article 19 of the International Covenant on Civil and
Political Rights (ICCPR). In the United States, freedom of speech and expression is strongly
protected from government restrictions by the First Amendment to the United States
Constitution, many state constitutions, and state and federal laws.

Being a truly liberal and democratic document, the Indian Constitution too grants Right to
freedom of speech and expression under Article 19(1)(a).

The Preamble explicitly mentions ‘liberty of thought and expression’ principle thereby
ensuring freedom of speech and expression to Indian citizens.

However, the constitution also ensures that right to free expression is not equated or
confused with a license to make accusations unfounded and are irresponsible. Therefore, the
right to speech and expression is not an absolute right, and the State can enforce reasonable
restrictions under Article 19(2) of the Constitution. Every constraint on the practice of the
right referred to in Article 19(1)(a) which does not come within the four corners of Article
19(2) might not be applicable. The State can enforce reasonable restrictions on the exercise
of the right to freedom of speech and expression based on grounds of defamation, Contempt
of court, decency or morality, security of the state, friendly relations with other states,
sedition, public Order and incitement to an offence.

Yet, it is important to note that these restrictions can be imposed only by law enacted by
parliament and the restrictions have to be reasonable.

One of the recent controversies related to curtailment of the freedom of speech is charging a
person with sedition. In June , 2021, while hearing a case involving sedition charges on two
Telugu news channels, the apex court called for renewed scrutiny of the “ambit and
parameters” of the laws governing sedition. In Vinod Dua Case, the court observed that
“Every journalist is entitled to protection under the Kedar Nath Singh judgment ”. In 1962
Kedar Nath Judgement, a constitution bench of the Supreme Court considered the
constitutionality of sedition in Section 124A of IPC and held that a citizen has a right to say or
write whatever he likes about the Government, or its measures, by way of criticism or
comment, so long as he does not incite people to violence against the Government
established by law or with the intention of creating public disorder.”
In a democratic age no institution should be beyond the reach of honest criticism. The courts
are no exception. The criticism of courts, within permissible limits, should not be taken to
lower the authority of the courts or to scandalise them. Yet, there have been numerous
instances to restrict freedom of speech through contempt of court provision.
Hate speech constitutes a criminal charge under Section 153A, which is the offence of
promoting communal disharmony or feelings of hatred between different religious, racial,
language or regional groups or castes or communities. These laws have often been misused
to victimise artists, journalists, and activists on the pretext of causing communal
disharmony. For instance, activists like Harsh Mander have been wrongly accused of
delivering hate speech. On the other hand, politicians of the ruling party who openly called
for shooting peaceful anti-Citizen Amendment Act protesters have not been arrested or tried
under the prevailing hate speech laws.
India has been listed under countries considered ‘most dangerous places’ in the world for
journalists, according to World Press Freedom Index,2021 published by Reporters Without
Borders’s. Governments in past have also used “iron hand” to silence dissent and criticism. In
Hathras case , the media’s freedom was seen being restricted by prohibiting the media from
reporting anything about the events. Siddique Kappan, a journalist,on his way to Hathras
has been booked under provisions of the Unlawful Activities (Prevention) Act (UAPA) and
sedition.

District or state-wide internet shutdowns are becoming a tool to stifle freedom of expression
through prior restraint. Shutdowns are effected through blanket orders under the guise of
preventing breach of peace. In most cases, they are deployed when the authorities
apprehend that people may exercise their fundamental right to freedom of expression to
organise a peaceful protest that is critical of the State. An internet shutdown is a highly
disproportionate response, since it affects everyone who uses the internet for professional
reasons, for communicating with family or friends, for access to education, medical facilities
and so on. In Anuradha Bhasin’s case, the Supreme Court in January 2020 while deciding the
legitimacy of internet shutdowns as well as physical lockdowns in Jammu and Kashmir
stopped short of declaring access to internet as a fundamental right, but declared that “the
right to freedom of speech and expression under Article 19(1)(a) and the right to carry on
any trade or business under 19(1)(g), using the medium of internet is constitutionally
protected.” The Kerala high court has recognised that access to the internet is essential for
not only exercise of freedom of speech but also the right to education. The UN Human Rights
Council had declared that right to access the internet is a fundamental freedom.

There is no doubt that the fundamental right to free speech is extremely important for any
civilised democracy to survive. While it is important for citizens to exercise our fundamental
rights within reasonable limits laid down by law, there is a greater obligation on the
establishment to ensure that the laws are not twisted, misused or abused in such a manner
that citizens are deprived of fundamental rights that impact the liberty of an individual. It is
to be realised that in any democracy, there are bound to be different points of view. These
must be respected – otherwise the fabric of our society might disintegrate, and fraternity,
one of the key words in the preamble to our Constitution might just become another dead
idea.

3(a).The anti-defection law is often flouted in spirit, if not in letter. Comment. Discuss the
impact of 52nd amendment act in addressing the problems of defections India .
The provision of anti-defection was added in the Tenth Schedule of the Constitution through
the 52nd CAA, 1985 by Rajiv Gandhi’s government with the specific intent of 'combating the
evil of political defections'. According to the Act, a member of Parliament or a State
Assembly belonging to any political party, shall be disqualified if he or she gives up the
membership of the party or votes or abstains from voting in the House contrary to the whip
issued by the party without prior permission.

However, with time it has become a law whose unintended consequences outweigh its
purpose. Irrespective of the stern provisions in the anti-defection law for disqualification,
legislators have been circumventing it with impunity .Some recent examples are defection in
Rajasthan, Gujarat,Madhya Pradesh, Goa.

The anti-defection law is often flouted in spirit, if not in letter. Legislatures switch sides using
lacuna in the anti-defection law. This can be best exemplified in a case of Rajasthan. The six
Bahujan Samaj Party MLAs joined the ruling Congress. They did not incur any disqualification
because more than two-thirds, in this instance all the MLAs, joined the Congress. According
to the provisions of the Tenth Schedule, unless two-thirds of the party defects, the members
are disqualified.
Law’s spirit has been compromised due to the twin issues of the Speaker’s inertia and
legislators’ resignations. There have been cases where a party issues a whip, yet some
members do not turn up for the no-confidence motion, the government falls and such
candidates then contest from the party they covertly supported . This amounts to cheating of
the people’s mandate.

According to Gautam Bhatia, “The anti-defection law has resulted in destruction of


democratic process. Members are switching sides not because of ideological compulsions
but because of political blackmail.”

In Kihota Hollohon vs. Zachilhu (1992) Case, while upholding the validity of the law, the court
held that decisions of disqualification shall be open to judicial review.The court also
highlighted the need to limit disqualifications in the cases where legislators vote against the
directions of Party. The court held that “such provisions should be limited to only those
voting that are crucial to the existence of the government and to matters integral to the
electoral programme of the party.
The anti-defection law has thus been called as “a paper tiger”, which has “been under
scrutiny for ineffective implementation”. It is time Parliament should rethink on whether
disqualification petitions ought to be entrusted to a Speaker as a quasi-judicial authority
when such a Speaker continues to belong to a political party either de jure or de facto.
Legal experts want the anti-defection law to be re-examined as it is not equipped in its
present form to prevent rampant misuse of its provisions.
As for the anti-defection law, suggestions have been made to bar legislators who resign mid-
term from contesting elections during the same term.
It’s important that voters must punish the legislators who defect for narrow political
interests, because it has been observed that voters are not punishing turncoats and they get
re-elected. It is also suggested that such legislators should not be allowed to contest in the
same term.

The main issue is that laws are drafted presuming there will be bona fide on the part of the
legislators. The challenge is to get the main political party to respect the rule of law and
respect constitutional propriety. The law cannot ensure everything. No law is perfect. There
could be a problem with the anti-defection law, but a greater problem lies in lack of
constitutional propriety and respect for the rule of law, and the spirit of the anti-defection
law.

3(b). The Constitution does not envisage courts to be silent spectators when constitutional
rights of citizens are infringed. Explain in context of recent activism showed by judiciary
with respect to handling of the covid crisis.
The Indian Constitution of 1950 may be viewed as the first truly transformational
constitution. It aimed not only to restructure the state, but also to transform deeply
entrenched social institutions based on caste and patriarchy by emphasizing transcendent
constitutional values such as the principle of equality. It provides for an independent and
impartial judiciary as the sine qua non of the democratic system.

In democratic countries , citizens enjoy certain rights, which are protected by judicial system
. Their violation, even by the State, is not allowed by the courts. The Founding Fathers wisely
felt that a right without proper remedy was totally meaningless and, this is why, the
Supreme Court and High Courts have been empowered to act as the vigilant protector of
such rights. They offer a guaranteed remedy whenever such a right is, in any way,
transgressed upon. In this matter, the Supreme Court and the High Courts enjoy concurrent
jurisdiction, because an affected person can proceed to either , under Article 32 and Article
226 respectively. Article 32 provides a guaranteed remedy for the enforcement of these
rights and this remedial right is itself made a fundamental right by being included in Part III
(Ramesh Thapar v. Madras, 1950). The Supreme Court is thus constituted as the protector
and guarantor of fundamental rights and it cannot refuse to entertain application seeking
protection against the infringements of such rights.

Further, under Article 142, the Supreme Court in the exercise of its jurisdiction may pass such
decree or make such order as is necessary for doing complete justice in any cause or matter
pending before it. Judiciary has used this power for upholding citizens’ rights and
implementing constitutional principles when the executive and legislature fails to do so. For
example – In Union Carbide Case,1989, the Supreme Court invoked Article 142 to provide
relief awarding the compensation of to victims affected in the Bhopal Gas Tragedy. In
Vishakha v State of Rajasthan case, the Court laid down the guidelines to protect a woman
from sexual harassment at its workplace.
Through its actions, the Supreme Court has created a legitimate expectation among citizens
that it would come to their rescue if the Executive falters.

Judicial intervention in response to the Union government’s less than satisfactory response
to the covid crisis is yet another instance of judicial endeavour towards protection of rights.
The Supreme Court ordered forming a 12-member national task force for the effective and
transparent allocation of medical oxygen to the States and Union Territories “on a scientific,
rational and equitable basis” along with audits by sub-groups within each State and UT . The
Court also mandated to review and suggest measures for ensuring the availability of
essential drugs and remedial measures to meet future emergencies during the pandemic.

The Supreme Court has intervened to alleviate the plight of millions of migrant workers
stranded in various parts of India due to lockdown . The court took suo motu cognisance of
the matter.

The Supreme Court ruled that the Centre must pay compensation to the family of those who
succumbed to Covid-19 and directed the National Disaster Management Authority (NDMA)
to create minimum standards for relief so that some amount of compensation can be paid.
The Supreme Court, in Shashank Deo Sudhi v. Union of India, 2021, directed that COVID-19
testing, whether in approved government or private laboratories, should be free of cost.

In another order on the same day in Jerryl Banait v. Union of India, the same Bench issued
directions for availability of PPE kits and medical supplies to medical professionals, and
directed that security be provided to medical staff while conducting screening.
However, the Court has also been criticised for rejecting the relief to migrant workers,
stating that it is best “if left to the wisdom of the Parliament”.

However, at times , this proactiveness has been seen as judicial overreach and breach of
separation of powers principle . Katju. J in 2007 had warned that courts should not pass
orders that are “not capable of enforcement".

There have been instances when judiciary was also criticised for shrugging of its
responsibility as the protector of fundamental rights . One such occasion was ADM, Jabalpur
v. Shivakant Shukla , 1976 , when the supreme court ruled that habeas corpus was virtually
not available even in respect of proven mala fide orders of detention. This judgment was
rendered at a time when there was neither war nor rebellion.

As long as the court does not usurp executive’s role, action to mitigate a crisis is welcome.
Judicial activism should be a ‘medicine ’ but not the daily bread.

3(c). A combination of executive failure and breakdown of the federal compact has led to
judicial governance . Comment.
The constitution of India has established a fairly powerful judiciary.Assertion of this power
has been referred to as judicial activism. Justice. Verma defined Judicial activism as the
active process of implementation of the rule of law, essential for the preservation of a
functional democracy. It is said that India’s judiciary is witnessing the fall of ‘literal legalism’
and the advent of age of judicial activism, where Court is seen as a political institution that
can solve social disagreement. According to Pratap Bhanu Mehta , we are witnessing
‘judicial co-governance’.
The prime factor behind this is a combination of executive failure and breakdown of the
federal compact. Judiciary’s activism during the first and second wave of Covid Pandemic is a
case in point . Under Article 226 of the Constitution , judicial interference was sought by
private entities and individual citizens production and allocation of medical oxygen and the
urgent requirement of basic medical infrastructure and vaccination . Subsequently, the
Supreme Court (SC), under Article 32 took suo motu cognisance of the pandemic.

he breakdown of the ‘congress system’ and Nehruvian Consensus has resulted into instances
of failure of executive & legislature. After the recognition of doctrine of Due Process of law in
Maeneka Gandhi Case, 1978 and the Doctrine of Basic Structure in Keshavanand Bharati
Case, 1973, the judiciary has been further empowered towards judidical governance .
The basic structure of the Constitution holds federalism as essential to upholding our
constitutional democracy.

The federal disputes have been recognized as the original jurisdiction of the Supreme Court
under Article 131 . Thus, in federal disputes like interstate river water sharing disputes, the
Supreme Court exercised original jurisdiction till 1956 when the parliament under Article 262
excluded the subject out of the original jurisdiction of the apex court. Howver, inspite of that
the SC has been exercising jurisdiction over such federal disputes by accepting special leave
petition under Article 136.

Beside factors related to executive failure and breakdown of the federal compact , there are
some other factors like emergence of Public Interest Litigation (PIL), which relaxed the test
of locus standi, has introduced a new dimension to judiciary's involvement in public
administration. There have been some judges who have often taken for themselves role of
social reformers like in the Vishakha and others v State of Rajasthan. Even constitutional
scheme provides avenues for judicial activism like through Article 13, Article 32 , Article 226,
the role of Supreme court as the final interpreter of constitution as provided for in Article
141.

While judicial activism has emerged in the spirit of the law and with the demand of the
changing times,it is advisable that it should not be the trend . Judicial restraint that
encourages judges to limit the exercise of their own power should be the normal course of
action.

4(a).The progress with regard to decentralisation and panchayats having modest degree
of autonomy is slow due to numerous systemic bottlenecks and institutional challenges.
Discuss.
The effort towards democratic decentralization got a boost after 73rd and 74th amendments.
The idea was to design governance on the basis of ‘principle of subsidiarity’ i.e. what can be
done at a particular level, should be done at that level and not at a higher level.

The two Constitution Amendments initiated a process with standardised features such as
elections every five years; reservations for historically marginalised communities and
women; the creation of participatory institutions; the establishment of State Finance
Commissions (SFCs), a counterpart of the Finance Commission at the sub-national level; the
creation of District Planning Committees (DPCs); and so on.

Except for some success stories like Kerala ,the third tier is often seen as a systemic failure.
Thus, the 73rd & 74th Amendment Act is referred to as a compromised document and is
considered as ‘Half- Baked Cake’. There are inherent weakness in the institution of
democratic decentralization. As a compulsory provision, it creates the institutional
structures, but powers & functions have been left with the state government. In addition, the
provisions, on which states have objections, have been made voluntary and left on states will
& capacity. Though uniform institutions have been created, yet activity mapping has not
been done and thus, there’re variation among states. Gram Sabha doesn’t have any
significant powers & functions as it hasn’t been provided by the Acts.

There has been no perceptible hand-holding and support by the States to foster
decentralized governance. This is evident from action of states like postponing elections or
the failure to constitute SFCs and DPCs etc. There has been no institutional decentralization
except in Kerala. The roles and responsibilities of local governments remain ill-defined
despite activity mapping in several States. States control funds, functions and functionaries,
making autonomous governance almost impossible.

Most States continue to create parallel bodies ,often consisting of ministers and senior
bureaucrats, that hinders in the functional domain of local governments. For example,
Haryana has created a Rural Development Agency, presided over by the Chief Minister, to
enter into the functional domain of panchayats. There is no mandate to create a DPC tasked
to draft a district development plan that takes into account spatial planning, environmental
conservation, rural-urban integration, etc. In States like Gujarat, the DPC has not been
constituted.

There is huge financial crunch at LSG level .Increasing allocations to (now suspended )
Members of Parliament Local Area Development Scheme, or MPLADS, and State-level
counterparts, MLALADS, has further resulted into diversion of local development funds which
should have gone to the LSG bodies. Even after 3 decades, local government expenditure as
a percentage of total public sector expenditure comprising Union, State and local
governments is just above 7% as compared to 24% in Europe, 27% in North America and 55%
in Denmark. The own source revenue of local governments as a share of total public sector
own source revenue is only a little over 2% and if disaggregated, the Panchayat share is a
negligible 0.3%. The Economic Survey 2017-18 describes such situation in terms of “low
equilibrium trap” , posing challenge for fiscal federalism. The functioning of state finance
commission has not been satisfactory.
Despite the reservation of seats for Adivasis, Dalits and women, these categories remain on
the periphery, often as victims of atrocities and caste oppression rather than as active
agents of social change.

This has failed to create the required political environment in the country necessary for the
sufficient democratic decentralization . The 2nd ARC argues that LSG is the lowest tier of the
government, but not the least important. It recommends proper implementation of the
‘Principle of subsidiarity’ and devolution of funds, functions and functionary to the local
governments.

With panchayats emerging as front warriors in the collective fight against COVID-19
pandemic,it has been established that the institution is of critical importance in mobilising
resources, managing intricate tasks and shouldering responsibilities that no other institution
can replace. Both Centre and States should not forget to ensure their legitimate
constitutional status and a substantive transformation.

4(b).The need for a Uniform Civil Code flows from the very definition of secularism .
Comment.
Secularism is most commonly defined as the separation of religion from civic affairs and the
state, and may be broadened to a similar position concerning the need to remove or
minimalize the role of religion in any public sphere. The way secularism is understood in most
general sense can be traced back to the history of Europe. The philosophical foundation of
European / western model was laid by Machiavelli, who called for need of separation between
church and state. In practical sense, the idea of secularism is associated with French
Revolution. It called for strict official neutrality in religious matters. However, the term has a
broad range of meanings . There are other distinct traditions of secularism other than French,
like Turkish , Anglo-American , Indian. There has been lot of debate on the nature of Indian
secularism. There has been a question mark on the manner in which secularism is practiced in
India. Unlike western notion of secularism, India partially separates religion and state.

At one hand , the 42nd amendment added the word 'secular' in the preamble, making India's
commitment to secularism more explicit. India does not have official state religion, under
Article 27, No person shall be compelled to pay any taxes, the proceeds of which are
specifically appropriated in payment of expenses for the promotion or maintenance of any
particular religion or religious denomination. Article 28 says that no religious instruction shall
be provided in any educational institution wholly maintained out of State funds. At the same
time ,Indian constitution gives enough avenues for state's interference in religious matters.
The application of personal law varies on the basis of individual's religion. For example -
Muslim Personal law is based on Shariat. Indian constitution permits financial support for
religious educational institutes. The Islamic Central Wakf Council and many Hindu temples are
administered by the government. The 7th Schedule of Indian Constitution places religious
institutions, charities and trusts into concurrent list on which both union and state
governments can make laws.
Although Article 44 lays down that the state shall endeavour to secure a Uniform Civil
Code(UCC) for the citizens throughout the territory of India. Being a directive principle and as
defined in Article 37, is not justiciable . However in many judgment the supreme court has
emphasized on the need for a Uniform Civil Code.In Shah Bano Case in 1985, the court
observed that a common Civil Code will help the cause of national integration by removing
disparate loyalties to laws which have conflicting ideologies. Similar view was expressed in
the 1995 Sarla Mudgal Case. Goa is, at present, the only state in India with a uniform civil
code. Recently, the SC has put the spotlight back on the debate in Jose Paulo Coutinho vs
Maria Luiza Valentina Pereira, 2019 by describing Goa as a “shining example” with a
Uniform Civil Code and said that the founders of the Constitution had “hoped and expected”
a UCC for India but the government had made no attempt yet.

The Law Commission(2018), has suggested that the best way forward is to reform the
personal laws i.e. preserve the diversity of personal laws but at the same time ensure that
they are not discriminatory across genders and do not violate the fundamental rights as
enshrined in the constitution of India. The legislature should first consider guaranteeing
equality within communities between men and women.

4(c). Autonomy of civil society is critical for proper functioning of Indian democracy .
Critically analyse.
The origins of the term ‘Civil Society’ can be traced back to Aristotle who defines ‘civil
society’ as more or less formalized institutions which form an autonomous social sphere .
Hannah Arendt has focussed on the capacity of action in public.
Sunil Khilnani, on the other hand considers the concept of civil society as ‘the idea of the late
twentieth century’. The concept was reinvented in the 1980s in Eastern Europe and Latin
America, and incorporated into discourse on international development in the 1990s .
Civil societies are as those associations, which affirm openness of entry and exist, and stand
by universalist criteria of citizenship. Neerja Jayal argues that civil societies do not exclude
persons on the basis of ethnicity, class, or religious persuasion. Examples of well-known civil
society organizations include Amnesty International, the International Trade Union
Confederation, the World Wide Fund for Nature (WWF), Greenpeace and the Danish Refugee
Council (DRC). Civil societies have contributed to democratic pluralism.

Popular mobilisation within the Indian civil society was evident already in the colonial period
. Indian National Congress (INC) became the main source of civil society activity in early 20th
century British India.

In post independence India, the civil society has played important role in central to the
function of democracy . For example – Post National emergency imposed by Indira Gandhi
government , there was substantial increase of activities within traditional social movements
such as peasants, workers and students, as well as amongst the so called “new social
movements”, including environmental groups and women’s organisations.
The World Bank identifies the following functions for civil societies in a democracy :
A. Representation.
B. Advocacy and technical inputs.
C. Capacity building.
D. Service delivery.
E. Social functions.

Today, India boasts an active and vibrant civil society, with over three million civil society
organisations (CSOs) and social movements fostering socio-economic, political and cultural
development, and playing a major role in promoting, protecting and strengthening human
rights. The major roles of civil society in India includes promoting participatory government
and democracy, implementing government flagship programmes, promoting informal
spaces for dialogue and voluntarism and protesting against anti-people policies. Civil society
promotes trust, choice, and the virtues of democracy . Thus, it leads to strengthening the
government of policies’ effectiveness such as human right violation, anti-corruption, and
good governance.

Although , Civil society has a central role in promoting democracy and social welfare under
liberal constitutionalist regimes. However, the civil society’s role remains questionable and
debatable. However, according to Lee Kuan Yew, the idea of promoting civil society and
democracy is discipline more than democracy. The democracy’s exuberance leads to
disorderly conduct and indiscipline. So, it is inimical to development . If we go by
governments narrative , India too has many obstructive elements working under the garb of
CSO with the sole aim of impeding the development . They are often funded by states with
whom India has adversarial relations like Pakistan , China. On the other hand , it is being
alleged that the civic space is increasingly being contested. The governments have been
using such tactics as restrictive legislation to deny CSOs their right to register, and in some
cases suspends or withdraws CSO permits to operate. Some CSOs have been prevented from
receiving funding from external sources, and some have had their bank accounts suspended,
stopping them from accessing funds to carry out their activities. Unlike the developed
societies , the CSOs in developing countries , needs the support of government . In her book
,‘The Role of Civil Society in Shaping India's Development Partnerships’,Shagun Gupta argues
that although consultations with civil society have been initiated by the government as well
as the UN, its recommendations have not been taken seriously .

It is important to remember that in a truly plural democracy, civil societies have their share
of responsibilities. However, as mentioned in the Preamble of Indian Constitution ,
democracy is as important as the sovereignty , unity and Integrity of India. Government
needs to be diligent and Civil society organisations with dubious records and funding should
not be allowed to function with nation’s territory .
Section B

5(a).Define constitutional morality and its linkages with transformative constitutionalism .


Since , Indian constitution is a transformative constitution , constitutional morality is seen
as the guiding spirit to achieve transformation in the society. Of late the use of
constitutional morality has become much more significant while interpreting the
constitution . The Supreme Court has applied different facets of this progressive and
transformative doctrine in large number of cases. The phrase "constitutional morality"
existed in Indian constitutional context since the times of Ambedkar . Ambedkar, the father
of the constitution, is perhaps the first who tried to define the intent of the phrase
"constitutional morality". While addressing the constituent assembly in 1948 , he invoked
George Grote to explain the constitutional morality and its significance to the country
where democracy is just the top soil . Grote described constitutional morality as a
"paramount reverence for the forms of the constitution". On similar lines , Ambedkar
defines constitutional morality as a foundational value of any constitutional form of
government. Constitutional morality is the only way to realize objectives of the
constitution in country like India with inequality and inequity. Twice it was used in passing
reference in Keshavanand Bharati and S.P. Gupta judgments. In Manoj Narula Vs. Union of
India case , the Supreme Court held that constitutional morality is to ensure the Rule of
Law. In the historic Government of NCT vs. Union of India , the court observed that the
constitutional morality is not just the forms and procedure of the constitution , but
provides an enabling framework that allows a society the possibility of self renewal . In
Navtez Singh Johar Case pertaining to section 377 of IPC, 1860 , the Supreme court held
that ," Constitutional morality cannot be altered at the altar of social morality "

However, the doctrine of constitutional morality has been called as dangerous because of its
subjectivity (Manu Singhvi in his book "The Trenches"). It is seen as another controversial
doctrine like that of "Basic Structure". Attorney general of India, K.K. Venugopal and Solicitor
General of India ,Tushar Mehta also considered it as dangerous doctrine which will give
judiciary an opportunity to emerge as the Third Chamber.

Indian constitution is without doubt a transformative constitution . It transformed the legal


relationship between individual and the state. The subjects of colonial rule became the
citizens of the Republic. This was a tectonic shift in the philosophy of governance in India.
The constitution sought a thorough going reconstruction of state, culture and society itself.
The Supreme Court in number of cases has clarified the transformative nature of Indian
constitution .For example - The Supreme Court took the note of transformative constitution
in its 2014 NALSA judgment and called Indian constitution , as a living organism . Similarly
in the Navtej Singh Johar Case , the court explicitly mentioned that the purpose of having a
constitution is to transform the society for better and the objective is transformative
constitutionalism ".

It is to be noted that the transformative power of the constitution is not just a means to
correct historical wrongs but a key to more equitable future . Transformation is a
permanent ideal i.e. constitution is a living document .

Thus constitutional morality provides the basis to transformative constitution .


Constitutional morality is important for constitution to be effective and protect Indian
democracy from being a "Tyranny of majority."

5(b). Compare liberal perspectives on Indian national movement with that of Marxist.
The colonialist view on Indian nation and nationalism disregarded the possibility of India
ever becoming a nation. It, moreover, considered the national movement as a movement of
narrow self-interest led by a microscopic English-educated minority. The liberals did not deny
the role of modern Western ideas in making India a nation state . For S.N. Banerjee, India
was a nation in making. They believed that under the influence of modernization, India will
emerge as a nation and considered the British rule in India as a ‘blessing in disguise’. They
held it as a providential mission capable of eradicating the misrules of the past and a ‘sine
qua non’ of India’s progress as a civilized nation. The virtues of law & order, liberty & justice,
which the Britishers introduced in India were inherently superior to their traditional polity
which they defined as ‘Asiatic despotism’. Such belief was criticized by cultural nationalists
like Aurobindo Ghosh. For him , India is neither a geographical unit nor the figment of
people’s imagination. India is not a nation in making, rather India was always a nation and is
like a spirit, which is immortal due to the strength of God in it.

The Marxist historians argued against both, colonialist and liberal perspectives . Their
analysis of the national movement was based on an understanding of the role of economic
factors and classes in the making of the nation as well as a movement. According to them,
although the national movement was an expression of the basic antagonism between the
Indian people and imperialist government, it was a movement either directly influenced by
bourgeoisie or indirectly working in the direction of capitalist development. Thus, even
though various classes and groups were involved in the movement, it ultimately served the
fundamental interests of the Indian bourgeois classes. Marx considered nationalism as an
ideology of upper-class indigenous bourgeoisie.
For him 1857 revolt was a revolt by feudal elements. Benedict Anderson, a Marxist,
considers nation as an invented concept to serve the needs bourgeoise class. Indian Marxist
scholars like R.C. Palme Dutt and A. R. Desai have criticized the class character of Indian
nationalism calling INM as a movement of Indian elite exclusively. The peasantry and other
lower-class people & their grievances were ignored by the nationalist elite.
MN Roy , in his , 'India in transition’, calls congress as a bourgeoise party and Gandhi as a
bourgeoise leader. However, this conventional Marxist narrow class approach was criticized
by later set of Marxist scholars . S.N. Mukherjee called Indian nationalism as a complex
process with multiple layers, which cannot be understood by a reductionist class analysis.
Sumit Sarkar argues that congress had no conscious or planned strategy against masses.
Bipin Chandra considers the freedom struggle led by the congress as the most spectacular
mass movement.
Both of these approaches, even at their best, are nothing more than sectional at least
segmented in their nature. One cannot grasp the entire gamut of the issues involved by using
anyone of them singularly.

5(c).Discuss the role of pressure groups in Indian politics with reference to the
effectiveness of farmer’s organization.
Pressure groups are associations of people formed to articulate the interests of the members
through pressure tactics, to influence government’s decisions and policies. As such they are
often called 'private associations formed to influence public policy'. Finer defines pressure
groups as “the invisible empires”. PGs have been given greater importance in scholars of
pluralist democracy. Robert Dahl , for example , has analysed the role of PGs in USA . J.S Mill
and Alex De Tocquille have also appreciated the role of PGs in developing a democratic
political culture.

IN context of India, according to Prof. Rajni Kothari, pressure groups have acted as agents of
modernisation and have provided the reservoir of leadership. Kothari refers to the politics of
pressure groups as 'non-party political process' .India does not present a strong example of
pressure group politics. It is held that in India we see a state dominated pluralism.This is so
because , India is considered as an over-developed state, state itself is most powerful
interest group.In developing countries like India, institutional pressure groups have been
more effective. The Neo pluralist scholars like Eric Nordlinger have emphasised that in
societies like India, state itself is a powerful pressure group. Being a prismatic society and an
overdeveloped post colonial state, pressure groups have remained peripheral. To some
extent pressure groups of farmers like All India Kisan Sabha, Bharatiya Kisan Union, etc have
been effective at state level. The rise of peasants groups in India has been mainly due to
abolition of Zamindari

System, implementation of Panchayati Raj, land reform measures, Green Revolution


Movement. They gained power since 1960s. In 1936, the All India Kisan Sabha was
established and after 1942 the Communist Party of India acquired control over it.
Different political parties have got their own peasant organisations. Even though there are
some important All India Kisan Associations like All India Kisan Congress, All India Kisan
Kamgar Sammelan, Akhil Bharatiya Kisan Sangh, peasant groups have been mainly
organised on territorial basis. Their demands relate to procurement prices of agricultural
products, fertiliser subsidy, tenancy rights, electricity charges, etc. The Bharatiya Kisan Party
(BKP) in Western U.P. is considered the most significant pressure group.

According to Sudhai Pai, in post Green Revolution period, the rich farmer organisations
acted as a pressure group to extract benefits from the state.These new Bullock capitalists
(Rudolph&Rudolph) became the new power holders.
The farmers’ organizations have used methods like offering support to the political parties
during the elections , staging protests

The overall impact has been tremendous for landless labourers and tillers of the soil. The
farmers’ organizations succeeded in many places increasing the wage rates for agriculture
labourers and securing a due share for poor peasants. Pressure has been exerted by
organized agrarian lobbies to persuade the government to improve the socio-economic
position of the farmers. Hence varied land reforms measures have been adopted since
independence like the abolition of Zamindari system, tenancy, reforms, ceiling of land
holdings, setting up of co-operative farms etc. These organisations have pressed
governments to pursue the better realisation of prices for farm produces through policies
like MSP and agricultural export policies .

However, the overall impact of these groups have been below expectations when compared
to other big economies. The overall political voice and influence of these groups have been
less. The organisations have limited territorial reach because of a fractured mandate,
political party’s support, geography and various other factors. These organizations have
become a tool for the vote bank politics and because of lack of a truly national organization
since the last couple of decades, has rendered these organisations ineffective in their
persuasiveness. The interplay of language, caste factor, weak financial positions, etc. have
been greatly responsible for non-emergence of national-level pressure groups. Unlike the
farmers pressure groups in the developed countries of the West, where these are invariably
organised to safeguard economic, social, cultural interests, etc. of farmers, in India, these
groups are organised around regional and ethnic issues. They have been partially successful
and helped the farmers to gain some concessions . Since agriculture still remains the major
economic activity in India, any government cannot totally ignore farmers needs. Structural
reforms are long overdue in the sector. The governments of Madhya Pradesh, Chhattisgarh
and Rajasthan waived loans off immediately after coming to power but failed to follow up it
with any real structural reforms in the agriculture sector of the states. Farmers pressure
groups are now considered as an indispensable and helpful element of the democratic
process and politics through consultation. It is very essential for the government to consult
these farmers’ organised groups at the time of policy formulation and implementation.

5(d).The AFSPA debate reveals the biggest dilemma before India’s human right
movement.Comment . Examine the effectiveness of National Human Right Commission.

AFSPA was enacted in 1958 and was first implemented in the Northeast, and then in Punjab
to bring under control what the government of India considered ‘disturbed’ areas which is
declared by notification under Section 3 of the AFSPA.

However, the law has opened up the biggest dilemma before India’s human right movement.
At one hand the law is an operational necessity for Indian security forces for handling the
extremist and terrorist tendencies , especially in frontier states . On the other hand AFSPA
has been criticised for threatening the fundamental rights and undermine rule of law. There
have been multiple excesses by armed forces resulting in violation of human rights including
custodial deaths, rapes etc. It is suggested that being an exceptional power, it should be
used in emergency, but AFSPA has been in application since its beginning, particularly in
Nagaland. The Supreme court (SC) has ruled that keeping AFSPA in operation indefinitely
defeats its purpose of being an emergency provision. The AFSPA has been blamed to foster
an environment of impunity in “disturbed areas”.Thus , the dilemma is whether AFSPA is
enforced or revoked , both ways , there seems to be violation of fundamental rights ,
especially right to life .

In 2004, Justice B P Jeevan Reddy Committee recommended that AFSPA should be repealed
and appropriate provisions should be inserted in the Unlawful Activities (Prevention) Act,
1967. The 5th report of the Second Administrative Reforms Commission on public order has
also recommended the repeal . Recently SC ruled that every death caused by armed forces in
a disturbed area involving either a common person or an insurgent must be thoroughly
enquired. It must be ensured that the execution of AFSPA is within the framework of
Constitution, upholding Rule of Law and Right to Life.

The NHRC released specific guidelines in 1997 to act as safeguards in instances of deaths due
to encounters. These guidelines have since been amended twice owing to experiences of
States who were either not following the guidelines in their “true spirit” or dismissing them
altogether as being merely recommendatory, leading to a compromised procedural safety
net against illegal extra-judicial killings. The NHRC is being seen as a “toothless tiger” due to
its statutory limitations, lack of staff . It is alleged that NHRC has been mechanically closing
cases without any substantive enquiry . There is increasing reliance on the magisterial
enquiry reports , which , by NHRC’s own admission , were of “poor quality” . The NHRC has
the power of a civil court. It can conduct investigations into any allegation of human rights
violation, summon any person during the course of the investigation and reach conclusions
based on it. But this is not so in the case of the armed forces, where on receiving a complaint
or while taking suo motu cognisance of a violation, the Commission can only ask for a report
from the concerned department and make recommendations based on it. The Commission
has limited powers over the defence forces. This becomes a major handicap for the
Commission in states under AFSPA or the Armed Forces (Special Powers) Act – such as
Jammu and Kashmir and Manipur – where allegations of violations are common. In
September 2016, Central government , opposed the NHRC’s offer to probe over 1500 cases
of alleged extra-judicial killings in Manipur . The NHRC has been of the view that the AFSPA
should be repealed
Though , NHRC has faced its own share of problems and obstacles. Nonetheless, whether it
has done enough till now or whether it is prepared to step up once provided with more
“teeth to bite” are issues which it must genuinely introspect at the earliest. Instead of
bemoaning its lack of powers, NHRC has to play a more proactive and transformative role for
the advancement of human rights in the country.
5(e). The deeper crisis in Indian agriculture can be overcome through overhauling of land
relations. Comment.
According to the 2011-12 agricultural census , about 4.9% of farmers control 32% of India’s
farmland. 56.4% of rural households, own no land. Only 12.9% of land marked for takeover
from landlords was taken over by December 2015. Even less was given to poor farmers. All
these data becomes important because 47.1% Indians , still depend on agriculture and
contribute 15% to GDP. Growth within the agricultural sector has been low and volatile. In
the last two decades, agricultural growth has dwindled from 8.6% in 2010-11 to -0.2% in
2014-15 and 0.8% in 2015-16.

Over the years, increasing fragmentation of agricultural landholdings has contributed to


decreasing viability of agriculture for the majority of small and marginal farmers in the
country. The number of marginal landholdings (less than one hectare) have increased to 93
million in 2011 and the average landholding decreased to 1.08 hectares.
At the foundation of the present crisis in India is the skewed nature of capitalist development
undertaken in India immediately after Independence and the failure of land reforms in most
Indian states, except in the states of Jammu & Kashmir, Kerala, West Bengal and Tripura.
West Bengal has taken possession of 93.6% of land declared surplus statewide and accounts
for more than half (54.2%) of India’s land-reform beneficiaries. In most of states land
reforms ended up being an agenda on paper which never turned into a reality.Effectively, the
rich landlord class continued to maintain their hold on their landholdings and enjoy the perks
of the developmental policies introduced by the government. The interests of the landless
peasantry and small and marginal tenants were not protected. Millions of tenants found
themselves evicted. Partial agricultural reforms, brought about in nexus with the rural,
landed elite meant that the inequalities and exploitation prevailing within the previous
structure continued in post-independent India. Deeply inegalitarian and caste-based land
relations have prevailed .
After Independence, the government brought about some improvement in agricultural
production through state-led capitalism. During this period, the government made
investments in expansion of irrigation, power, science and technology, transport,
communication, storage and credit facilities etc. The government also introduced subsidies
for agricultural inputs and a public procurement system ensuring MSP to protect the
interests of farmers. These benefits were also reaped by big landlords and rich farmers.
Therefore , these developmental efforts did not alter the reality of rural poverty and
unemployment.
The Green Revolution, too, benefitted those with greater ownership of land, resources and
accessibility of agricultural credit, enabled the rich and landed rural peasantry to reap
disproportionate gains.
In early 1990s when neoliberal economic reforms and structural adjustment programmes
were introduced . It translated into deflationary fiscal policies, reduction in public investment
on agriculture, including rural infrastructure, irrigation, agricultural subsidies and
agricultural research. A decline in rural and agricultural spending adversely affected the
marginal farmers . Unfortunately, opening up of international trade coincided with a fall in
international prices of non-food grain crops like cotton and oilseeds. At the same time, the
protection provided by the government to the peasantry, in the form of agricultural
subsidies, and MSP, weakened. Bank credit to crop lending went down and the small and
marginal farmers in particular, have been pushed towards private money-lenders .
The resistance to the three farm laws reflect this deeper rot in the agrarian situation of the
country. It requires more far-reaching reforms, starting from an overhaul of pre-capitalist
land relations and relations of production. As many as 95.1% of Indian farmers are called
“marginal, small and semi-medium”, meaning they own up to 2.47, 4.94 and 9.88 acres of
land, respectively. Countries like South Korea , Taiwan and Japan implemented sweeping
land reforms before transforming agriculture and moving their people to manufacturing .
India will have to revisit its approach to land reforms .

6(a).“Raj opened new horizons both intellectually and institutionally in a society that had
stood still in a conservative and hierarchical mold for centuries.”- André Béteille. Critically
evaluate .

Former PM Manmohan Sigh , in 2005 Oxford lecture, made an assessment of the British
legacy and talked about the positive contribution of the Raj. He asserted that India’s
experience with Britain had its beneficial consequences too. India’s notions of the rule of law,
of a Constitutional government, of a free press, or a professional civil service, of modern
universities and research laboratories, have all been fashioned in the crucible where an age-
old civilization of India met the dominant Empire of the day. Indian judiciary, the legal
system, the bureaucracy, and the police are all great institutions, derived from British-Indian
administration, and they have served the country exceedingly well.
Historian Ramachandra Guha has pointed out, “of all relations between the former colonies
and erstwhile empire, India’s connection is the least acrimonious and ever enduring.” Both
India and Pakistan adopted the imperial law codes as well as the civil service with few
changes after independence. The governments have even boasted that their own civil service
is “the successor to the finest civil service in the world.”

Thus, the Indian sociologist André Béteille acknowledges that the institutions established by
the Raj opened new horizons both intellectually and institutionally in a society that had
stood still in a conservative and hierarchical structure for centuries. These open and secular
institutions allowed Indians to question the age-old restrictions of gender and caste. Cricket,
liberalism, the rule of law, incipient democracy, the English language, professional civil
service, remains to be seen as a positive contribution to Indian society . The welding together
of hundreds of little states that has enabled India to become the vast united country that it is
today, is also credited to British. The excellence of Gothic architecture as is seen in high court
buildings , the government buildings, the splendors of New Delhi and the railways is also
seen as important contributions. It is claimed that Indians learned to use British ways and
institutions to transform their lives and to create their new nations.
Historian C. A. Bayly argues that Britain helped liberalism take root in India by
institutionalizing it through schools and colleges, newspapers and colonial law courts, and
thereby converted an entire generation of Indians to a way of thinking about their own
future that led to today’s Indian democracy.

However, there are many, usually Indian nationalists and British intellectuals of the Left, who
contest this view. Britain’s legacy, they claimed, was exploitation, oppression, and division.
The British governed India by employing the Roman policy of divide et impera—“divide and
rule.” For them, the British rule was ephemeral and ultimately irrelevant, something which
would regarded as an unimportant interlude in India’s history.
Shashi Tharoor, in his book , ‘An Era Of Darkness: The British Empire In India’, argues that
colonial rule not only impoverished India, it also enfeebled it and Britain owes an apology to
India . It refutes British claims of superiority, questions the benefits of British rule and
ridicules the claim that the British rule was a divine dispensation, which civilized the natives.
Far from being a pioneer in free trade, the East India Company was a private monopoly with
state backing. Tharoor highlights the manner in which the much-cited “rule of law" was
actually rule by laws, which the British made and which had different standards for the
British and the Indians. Communities were segregated and marginalized by calling them
criminal tribes, and poignantly highlights the colonial-era laws that persist in India, which
have far outlived their purpose (such as the sedition law) and which should never have been
enacted (such as the laws that criminalize same-sex relationships).
The British did not do much to teach Indians how to practice democracy. It would be truer to
say that the colonial subjects taught themselves how to imitate their colonialists. The
academic Sunil Khilnani writes , democracy was not a gift of the departing British , but the
concept was already pioneered in Mahajanpada of Vaishali and in Buddhist traditions . As
for liberalism, one could not claim that the British taught it to the Indians, but in the 19th
century the British took certain decisions, to further their colonial ends, on matters of law,
education and the English language which made it almost inevitable that an Indian version
of it would be adopted on the subcontinent.

6(b).Regionalism is rooted in India’s cultural and linguistic diversity . Ever since


independence , the cultural politics , be it in the name of language or nativism of son of
soil have deepened religious identities . Comment.
Regionalism is a political ideology expressed in territorial sense. Since,regional movements
can be seen as ‘sub-national’ movements. Hence, they can threaten the territorial integrity
of the state.
According to Paul R Brass, regionalism has been a long-term trend whereas nationalism has
been a short term trend in Indian politics. Considering the size and diversity, regionalism is
natural in country like India
In India, there has been a range of regional aspirations. Range starts from autonomy
movements to secessionist movements.
Gurharpal Singh categorised the movement into two categories – Emerging from core (like in
Maharashtra, Andhra Pradesh) and Emerging from peripheries (like Nagaland).
Government of India is not much concerned about the movements in core region.
Indian constitution provides the basic framework to accommodate regional demands. The
adoption of asymmetrical model of federalism is one such provision.
Though there is no declared policy, yet govt. has followed the following approach to deal
with regionalism –

1. Government is ready to resolve the problem through dialogue


2. However, no talks with terror
3. If the secessionist groups use force, government will use bigger force
4. Government is ready for radical solutions , but within the framework of the
constitution
5. Government is ready to offer the political and economic packages of devolution

The phenomenon of regionalism has been explained from different perspectives like by
Modernisation Theory ( ‘Rudolph and Rudolph’ ); Theory Of Uneven Development in case of
Jharkhand, Telangana, Vidarbha etc. Myron Weiner has explained it as Son of Soil
movements ,as seen in case of Maharashtra( Shiva Sena, MNS), AGP in Assam .
However, the regional movements are not necessarily bad. According to Dr Prerna Singh(
Book: “How solidarity works for welfare?” ) , regional movements can address the
development deficit. She gives the example of South Indian state asserting strong regional
identities and hence, have been better in terms of development. On the other hand, UP &
Bihar lack any regional identity, failed to develop the movement for better bargain.
Regionalism has led the movements such as those for Vidarbha, Telangana, Bodoland,
Gurkhaland. Separatism, as in Kashmir, gets mixed with religious identities and become
further divisive . Secessionist demands on the basis of religion has its roots in the two nation
theory and subsequent partition on the basis of religion. Such fissiparous tendencies can only
be combated if we pull ourselves back from the brink of fragmentation caused by religion,
caste, class and regionalism. As far as the question of regional aspirations challenging
India’s unity and integrity is concerned. The recommendations of the Sarkaria Commission
and Punchi Commissions regarding democratic decentralisation, can be the best way
forward.
6(c). The legacy of 1991 reforms has, on the whole, been unambiguously positive. Discuss.
What should be the components of next phase of reforms.

The then finance minister Manmohan Singh quoted Victor Hugo as “No power on earth can
stop an idea whose time has come,” while presenting the Union Budget in 1991. He was
referring to idea of economic liberalisation in India.
The liberalisation was aimed at ending the licence-permit raj by decreasing the government
intervention in the business, thereby pushing economic growth through reforms. The policy
opened up the country to global economy. It discouraged public sector monopoly and paved
the way for competition in the market.
The 1991 reforms were based on Neo-classical ideas of economics, with the underlying
assumptions that a free-market removes politics from the economy; leads to an efficient
allocation of resources and efficient growth over time. The Reform focused on attracting NRI
deposits and FDI and liberalising capital markets. According to Varshney , it marked a
fundamental transformation of India’s economic strategy.

The legacy of these reforms has, on the whole, been unambiguously positive. The three
decades have seen Indian GDP increase almost eight-fold, with the economy generating
enough savings and investment to ensure an annual average growth rate of almost 5%. The
most positive effect of delicensing and trade liberalisation is evident in the telecom,
electronics and fast moving consumer good sectors where improvements have benefitted
society at large.
Once admonished for its “Hindu rate of growth” – cliché for low rate of economic growth –
post-reforms, India remains one of the fastest growing major economy in the world. Foreign
direct investment inflow has increased drastically. Before 1991, foreign investment was
negligible. India received $81.72 billion foreign direct investment (FDI) in Financial Year of
21. India’s dismal state of forex reserves that forced the government to bring in economic
reforms. Now, three decades later, forex reserves are at a record high. In 1991, it stood at
just $5.8 billion, currently it has surged to all-time high of $609. As a percentage of GDP the
external debt has declined from 38% in 1991-92 to 21% in 2021. Similarly , the per capita
income rose from Rs 6,270 in 1991 to Rs 86,659.
However as former RBI governor Raghuram Rajan acknowledged, with this number we are
nowhere near ending poverty , there is a lot of things to do. We are still a relatively poor
economy .Economic reform is a continuing process and not a one-time action.
But structural problems remain in the economy. Three decades of deregulation has provided
little boost to manufacturing. With grossly inadequate job growth, stuttering manufacturing
and poor performance in health, education, sanitation and urban planning, India needs a
atleast a ‘minimum state’
The post-reform period shows the gradual decline in the agriculture sector’s contribution to
the Indian economy. India’s traditional occupation, agriculture now contributes only about
19% to the GDP, down from 29 percent in 1991.The sector involves 60% percent of labour .
This clearly shows underproductivity , lack of efficiency and disguised unemployment . The
services sector has taken the lead role in propelling the economy at the global stage,
contributing around 53 percent to the national economy. In the meanwhile, the industrial
sector has undergone marginal growth only and contributes only 17 percent of GDP.
Thus , future reforms need to fill the gap left open due to the execution of 1991 reforms .
Agricultural growth is required for equitable distribution of wealth as around 60 % of the
population is still dependent on the sector. Similar, push is required in labour intensive
manufacturing as it will have multiplier effect and the gains would be spilled over to allied
sectors . Moreover, future reforms need to be sustainable in nature in line with the SDG
Goals , especially GOAL 8(Decent Work and Economic Growth), GOAL 9(Industry, Innovation
and Infrastructure).
There is a need of renewed push on land reforms as it is an economic factor of production
that has become a highly politicised and emotive subject. Poorly-conceived and shabbily-
executed snatching of land has killed the credibility of land acquisition.

Consolidation of Labour laws in to 4 labour codes is a step in right direction . However,


proper implementation and timely adjudication will be required for developing confidence
among Indian labours and labour unions .

Further reforms are required in taxation regime . Goods and services tax (GST) has been a
big bang step in simplifying the tax system. However, there is a need of ironing out of
implementation issue arising primarily due to federal incoherence . With reference to Direct
tax too , the current tax infrastructure needs a reorganisation. Due the complexity of tax
laws and a rent-seeking tax bureaucracy, the case for staying out of the tax network and
evading taxes is strong. The tax base can be widened through a single and simple law, more
predictability and objectivity , by reducing the cost of compliance and by reducing the
ambiguity that facilitates tax avoidance.

7(a). Interstate (River) Water Disputes (ISWDs) are rooted in constitutional


complexities, historico-geographical, institutional ambiguities and electoral
politics.Elaborate.
Interstate River Water Disputes (ISWDs) are a continuing challenge to federal water
governance in India. The governance of interstate rivers is mired in conflict due to hostile
hydro-politics, conflictual federalism, and the ambiguity around dispute resolution. They are
rooted in the constitutional complexities, contentious political federalism, and identity-based
electoral political dynamics of India .
India has 25 major river basins, most rivers flowing across states, thereby necessitating a
coordinated approach between the states, with adequate involvement of the Centre.
However, interstate rivers in India have become sites of contestations, fueled by conflicting
perceptions of property rights, flawed economic instruments for food security, the lack of an
integrated ecosystems approach, and the prevalence of reductionist hydrology for water
resource development. Such conflicts have persisted since the inception of the Indian
republic, however in recent years, increasing water scarcity, a rapid rise in urban and rural
demands for freshwater, and contentious political dynamics have further exacerbated the
problem.
There are three fundamental structural ambiguities that currently affect the system: federal-
jurisdictional, historico-geographical, and institutional. These ambiguities are interrelated
and have shaped the constitutional design, institutional response, and political interaction
on ISWDs in India.
Before independence, the governmental power was highly centralized . For any dispute
between the provinces, the decision of the Secretary of State was final and binding. In
independent India, legislative powers concerning water were distributed between the Centre
and the states to ensure optimum utilization while balancing the interests of the states.
Schedule 7 of the Constitution distinguishes between the use of water within a state and the
purpose of regulating interstate waters. It gives the Union Parliament the power to
formulate laws and mechanisms for regulating interstate rivers (Union List: Entry 56 ), while
the states retain autonomy regarding water utilisation (State List: Entry 17). Thus,creating
federal-jurisdictional ambiguity.
Independence witnessed the fusion of 571 disjointed states to form the Union of India. While
the states were initially organised on the basis of political and historical considerations, the
States Reorganisation Act (1956) finally resulted in 14 states and six union territories.
Thereafter, the boundaries of Indian states have continued to evolve based on cultural and
political factors, with little regard for the historical and ecological dynamics of these
regions.It complicates the existing jurisdictional and resource-sharing agreements and
eventually become sources of interstate political contestation, leading to historico-
geographical ambiguity.
Under Article 131, the Supreme Court has original jurisdiction over federal disputes like inter
state river water disputes . The tribunals lack such authority, thus failing to efficiently
enforce an award, especially in disputes that get amplified due to political
overtones.Moreover , the Supreme Court’s role undermines that of the tribunals as
adjudicators of ISWDs, despite the latter being established for the implementation of binding
awards and their decision granted the same force as an order of the Supreme Court. While
Article 262 deters the highest judiciary from adjudicating ISWDs, Article 136 empowers it to
hear appeals against the tribunals and ensure the implementation of the tribunal. Thus, the
apex court remains the adjudicatory body along with the tribunals, creating an institutional
ambiguity regarding which body is the ultimate adjudicatory power on ISWDs in India. The
involvement of the Supreme Court in ISWDs, has been called as a dysfunction of “systemic
proportion.” The apex court has limited the role of the tribunals to quantification and
allocation of water between riparian states, and its own role is to be an interpreter of the
awards and the water-sharing agreements. The Supreme Court has repeatedly taken on the
responsibility of administrative tasks, upsetting the balance of power between the judiciary,
executive and legislature.

According to Fali S. Nariman, the tribunals were conceived as an innovative experiment in


1956, but they have failed in their endeavor. He identifies the two-layered scheme of
adjudication by a tribunal as the reason for the inordinate delay. The first is arriving at a
final decision rendered under Section 5(2) of the ISWD Act, 1956, and the second is the scope
for questioning that decision through Section 5(3). The parties to the litigation are permitted
to seek explanation or guidance from the tribunal on all points, which triggers an endless
cycle. Nariman further notes various operational characteristics of the tribunals as
problematic, since they do not adhere to any established system. For instance, the sittings
are not routine, the functioning is outside the regular court system, and day-to-day or week-
to-week hearings are few and far in between. Despite the 2002 Amendment necessitating a
time-bound completion of proceedings, tribunals continued to operate in a laidback manner.
There are numerous challenges in the judicial-legal adjudication of ISWDs. Focus should be
on political negotiation and mediation in building a consensus involving the concerned states
and the Centre within the Indian federal structure. Discussion regarding dispute resolution
must shift from the domain of “technocratic should” to “democratic could.” . ISWDs are not
simply conflicts over vital resources but inextricably linked to the complex dynamics of
ethnic, communal and regional identarian sentiments as well as redistribution and
entitlement conflicts—sentiments that drove the politics of territorial state-making and
subsequent developmental politics. Such emotive issues of identity, coupled with the demand
for resources, have triggered major mass-based political mobilisation in the country.
Therefore, while an issue as vital as this must ideally be de-politicised, it will be difficult to
sustain such a solution in the long-term.
Consensus on river governance must be simultaneously achieved in two levels: federal
consensus i.e. at the institutional level and electoral consensus. It can be achieved by the
“positive politicisation” of the issue, which can only happen when the electorate is sensitized
regarding the tangible economic and ecological costs of prolonged disputes. The political
discourse of regional identity and culture must be unraveled by bringing to public notice the
developmental hindrances, economic losses, and environmental degradation resulting from
a lack of a solution to the dispute.. The focus should be on strengthening the existing and
evolving institutional mechanisms, and accommodating political sensitivities to find a long-
term and mutually amicable path for the governance of interstate river water.

7(b). Discuss the mandate of finance commission in India’s cooperative federalism . What
have been the controversies with respect to the terms of reference of 15 th finance
commission. To what extent its recommendations address the concerns of the state
governments
Granville Austin , the most recognized authority on Indian constitution has rightly explained
Indian model as cooperative federalism . Unlike dual federalism , cooperative federalism is
based on interdependence rather than independence . The conditions of partition, the
secessionist trends, regional imbalance and the need for nation building led to the adoption
of cooperative federalism.

One of the most critical aspect of federal relations is economic considering the huge vertical
and horizontal imbalance existing in India as well as the constitutional commitment to five a
life of dignity to large number of poors . The Finance Commission is a constitutional & Quasi-
judicial body constituted by the President under Art 280 of the Indian Constitution. The
constitution envisages FC as the ‘balancing wheel’ of the fiscal federalism in India. It is the
duty of the Commission to make recommendations regarding the principles which should
govern the grants-in-aid of the revenues of the States out of the Consolidated Fund of
India.The FC also suggests the measures needed to augment the Consolidated Fund of a
State to supplement the resources of the Panchayats and municipalities .

According to Y.V. Reddy, 15th Finance commission represent shift towards centralization.
The reasons to criticise the 15th Finance Commission by some states are

A. Use of the 2011 population data rather than 1971 for fiscal devolution . This has been
opposed by some southern states who performed well in population control and
invested in family planning programmes.
B. The 15th FC has reviewed the award of its predecessor which increased the
devolution of taxes from 32% to 42% to states and recommends a devolution of 41 %.
C. Creation of a separate fund for defence modernisation, the provision of performance-
based incentives and grants in line with the Centre’s priorities, and imposition of
conditionalities on state borrowings.

While there is much apprehension its recommendations . According to central government,


the Finance Commission has taken a balanced approach . The Commission has kept the
states’ share in the divisible tax pool at 41 per cent. The difference of 1 per cent with the
previous commission’s award is due to the change in the status of Jammu and Kashmir from
a state to a Union territory.
However, considering the Centre’s practice of levying cesses and surcharges to garner
resources , this amounts to around 31.2 per cent of gross tax revenues. The finance
commission has allayed the concerns of the states by giving only 12.5 per cent weight to
demographic performance.
Another contentious issue was the possibility of setting up a separate fund for the
modernisation of defence and internal security. There were apprehensions that sequestering
a large amount from the gross tax revenues would further reduce the divisible tax pool that
is shared with states. Here, the commission has tried to strike a fine balance. While it has
recommended setting up such a fund, only part of the proceeds are expected to come from
the Consolidated Fund of India. The balance is to accrue from other sources.
However, it is said that taking 2011 population basis will allocate more resources to the
populous and poorer States, which need additional funds for providing education, health and
other services to the people, which own resources of these poorer States may not allow.
Thus , 15th Finance Commission , in its final recommendations has tried to allay the concerns
of the state governments . However, it is up to the Centre to ensure that States do not feel
short-changed from the new fiscal framework, given their frayed ties over GST compensation
dues.

7(c).Once regarded as the ‘global gold standard’, Election Commission of India is said to
have witnessed the institutional decay in recent times. Discuss.
Elections are vital for the well-being of India’s democracy, especially given the relative
weakness of what are meant to be ‘non-electoral’ checks and democratic forums. An
incremental rise in the level of electoral participation and contestation has helped Indian
democracy become far more representative.
As Sunil Khilnani has argued, elections in India carry ‘the entire society’s aspirations’ to the
extent that as the ‘sole bridge between state and society’ they have come to ‘stand for
democracy itself.’
With a huge number of parties in contention against each other in reasonably ‘free and fair’
elections, with massive people’s participation , one sixth of the world electorate, spread
across a vast and widely varying terrain, India’s Election commission has done phenomenally
well in conducting regular elections.

The ECI is a highly trusted institution entrusted with the task of ‘superintendence, direction
and control of elections’. The source of the ECI’s power stems from the Constitution of India (
Article 324) and the Representation of Peoples Act .
The EC of India has been the role model for the entire democratic world. Hillary Clinton
rightly stated that the Indian election is the gold standard.

In their book ,‘Election Commission of India: Institutionalising Democratic Uncertainties’,


Ujjwal Kumar Singh and Anupama Roy argue that last three decades, in particular, have
posed new challenges with the massification of electoral democracy. Significantly, this
period also witnessed the ECI in ‘an activist’ phase as Indian politics underwent a process of
transition and uncertainty. Short-lived coalition governments tumbled one after another,
anti-incumbency became the buzzword in election studies and new claimants to political
power violated established democratic norms of electoral conduct with impunity. The
reckless use of empty populism, sectarian or divisive ethnic politics, political violence and
money power posed a great threat to the well-being of democracy even as it ‘widened and
deepened’ with the surge in participation of the socially and economically marginal,
including women and minorities. It was during this period, referred to by Yogendra Yadav as
the ‘Third Electoral System’, when ‘unbridled mobilisation’ unleashed by mass politics
seemed to ‘exceed institutionalisation’, that the ECI under the watch of Commissioners like
T.N. Seshan, M.S. Gill and James M. Lyngdoh showed great authority and tenacity in
restoring the democratic balance. ECI saw a transition from merely being an election-
conducting institution to a ‘referee/regulatory institution’. The ECI’s regulatory tasks have
involved not only determining the electoral rules but also periodically ‘innovating,
strengthening and reinforcing them.’ With the help of ‘authoritative judicial interpretations’
as in PUCL & Anr. vs Union of India and Anr. 2003 of Article 324, the ECI sought to empower
itself by becoming an autonomous and visible body. The Supreme Court held in Mohinder
Singh Gill vs Chief Election Commissioner,1978 , that Article 324 contains plenary powers to
ensure free and fair elections and these are vested in the ECI which can take all necessary
steps to achieve this constitutional object. All subsequent decisions of the Supreme Court
reaffirmed Gill’s decision and thus the ECI was fortified by these court decisions in taking
tough measures. The ECI has not only been instrumental in institutionalising a Model Code of
Conduct which political parties have agreed to but also exercising extraordinary powers
regarding the posting and deployment of the bureaucracy during ‘election time.’ The ECI has
succeeded in providing a ‘procedural certainty to ensure the democratic uncertainty of
electoral outcomes, and electoral integrity to assure the deliberative content of election.’
However, according to Pratap Bhanu Mehta, as has happened to many other
constitutional/statutory bodies in recent times including the Supreme Court (SC) of India and
the Comptroller and Auditor General of India (CAG), the neutrality of the ECI has come under
a scanner. Controversies have started plaguing the institution. The role of CECs like the
move by CEC Gopalaswami to remove then EC Navin Chawla and more recently, the refusal
to note minority decisions by EC Ashok Lavasa leading to his recusal are two incidents that
illustrate the internal bickering within the institution and the proclivity of the party in power
to take advantage of it. The last few years, in particular, have seen a further subversion of
the autonomy and integrity of the ECI, leading to several questions about its neutrality. The
opposition has raised concerns about ECI decisions regarding the use of electronic voting
machines (EVMs) and not allowing a Verifiable Paper Audit Trail (VVPAT) machine in all
polling booths. The role of the ECI has also come under scrutiny in conflict areas like Jammu
and Kashmir and the north-eastern states. The continued influence of money power and
hate speeches by politicians are areas where the ECI has been seen as helpless, unable to
take tough action. ECI was unable to ensure political parties adhered to Covid protocols
during the recent campaigning in West Bengal , Tamil Nadu . The Madras High Court orally
remarked that the ECI is “singularly responsible for the second wave of Covid-19” and it
“should be put up for murder charges”. However, the power of the ECI to conduct elections
during this pandemic must be seen through the prism of the Disaster Management Act,
2005, under which appropriate guidelines are issued by the GOI/state governments
It is being argued that ECI, so important for the well-being of its democracy, has become
more procedural than substantive in essence. There have been several committees like the
Goswami Committee which have suggested reforms to weed out criminals, role of money
and hate-mongering from the electoral process. However, it may be slightly unfair to criticise
the ECI alone for its inability to effect these reforms as political parties have not cooperated.
Its important to understand that the Election Commission’s image for fairness, efficiency,
and credibility must be preserved at all costs. It’s a matter of record that the Election
Commission is rated as one of the most trusted institutions. This trust should sustain in times
to come.
8(a). Reservation introduces a form of identity politics that makes caste visible, whereas
the goal ought to be the eradication of caste .Comment. Discuss the politics of
reservations in India .
“Reservation debate generates more heat than light.”- Ramchandra Guha

Identity politics can be defined as politics in which groups of people having a particular
racial, religious, ethnic, social, or cultural identity tend to promote their own specific
interests or concerns . It emerged as an emancipatory mode of political action and thinking
based on the shared experience of injustice by particular groups. Identity politics , though
situated in historical injustices , should have a view of the future. The purpose of organising
under an identity in the present is to overcome a historical injustice, with a view of the
identity no longer being required in the future when the objectives outlined by the present
political project have been achieved.
In India, the notion of identity politics is rooted in the violent history of Partition and the
demand for separate electorates for religious and caste identities. Since then, whenever a
marginalised community has demanded space in politics, it has largely been seen as divisive,
and as a threat to the equality and the unity of the nation. Over the years, many scholars
have problematised the idea of “identity politics,” and more specifically discussed how it can
limit the very emancipatory politics that it seeks to achieve. Identity politics is an inevitability
in democracies. As Asghar Ali Engineer wrote, the issue is never simply the assertion of a
caste, religious or regional identity by itself, but how identity functions as “an instrument” to
access material gains in a power set-up.

The accusation of “identity politics” has resurfaced whenever the matter of caste-based
reservations have come up, most noticeably in relation to the Mandal Commission
recommendations.

According to , Kancha Ilaiah , reservations should therefore need to be read as attempts to


unfreeze the order of things to get the wheel of social mobility turning.

Reservations have been considered as the strongest form affirmative action . However, it is
also an issue of high politics in India . The policy aimed to make India a casteless society.
However, opposite has been observed. Caste has become more salient feature of Indian
Society . Though , the Supreme court in NEET case ruled that reservation is not a
fundamental right. Yet , there have been a race to the bottom . Marathas in Maharashtra,
Patidars in Gujarat, Jats in Haryana, Kapus in Andhra Pradesh have been demanding quotas
in jobs and higher education. Recently , in Jaishri Laxman Rao Patil Vs Chief Minister of
Maharshtra, 2020, 5-judge Constitution Bench, unanimously declared reservation benefits to
the Maratha community in excess of 50%, as unconstitutional.

However the basis of reservation policy in India is changing . The recent 103rd constitutional
amendment providing for reservation for economically weaker sections (EWS) is an
expression of this view. Thus , we see manifestation of reservation demand in other forms .
Thus it can be concluded that reservation, far from helping to reduce the power of caste,
retains and strengthens caste divisions. Reservation introduces a form of identity politics
that makes caste visible, when the goal was the eradication of caste.
Prof. Satish Deshpandey considers reservation necessary to bring the marginalized sections
into the mainstream. Similarly , Yogendra Yadav agrees on the necessity of reservation but
at the same time acknowledge that the reservation policy in India has reached a dead end
and raises more questions than providing answers.

All of this highlights that it is incumbent upon the state to act and remedy the pernicious
influence of caste. The political project needs to broaden and look at various ways in which
caste is reproduced and hierarchy is entrenched. Reservation is one component of this
broader project. For reservation to be truly transformative, it must be accompanied by a
strong commitment to overcome the segregation of the caste system. Only then will Indians
be able to live Ambedkar’s vision of fraternity, or, as he called it, democracy.

8(b).NITI Ayog has emerged as the premier policy ‘Think Tank’ of the Government of India
fostering the spirit of cooperative federalism. Substantiate.
The National Institution for Transforming India (NITI Aayog), was formed via a resolution of
the Union Cabinet on January 1, 2015, with, broadening cooperative federalism as one of its
objectives . The Planning Commission, NITI Aayog’s predecessor , was set up in 1950 as an
agency to promote cooperative federalism , but got transformed into an agency of
bargaining federalism . It ended up as a dispenser of patronage. The Neo Liberal economic
order made planning commission an anachronism . For long the need for policy making
institution has been felt which resulted into the establishment of NITI Aayog . It is expected
to act as a premier think tank of government of India with special focus on promoting the
spirit of cooperative federalism reflecting the vision of Team India .

Ever since its inception , NITI Aayog has been taken a series of initiatives . For example - In
consultation with states , NITI Aayog has taken some initiatives for reforming agriculture
through Model Land Leasing Law, Reforms of the Agricultural Produce Marketing Committee
Act. NITI Aayog has developed the first ever ‘Agriculture Marketing and Farmer Friendly
Reforms Index’ to sensitise states about the need to undertake reforms in the three key areas
of Agriculture Market Reforms, Land Lease Reforms and Forestry on Private Land.
NITI Aayog has taken up an federally inclusive approach for reforming Medical Education,
recommended scrapping of the Medical Council of India and replace it with a National
Medical Commission (NMC).
Similarly, for initiatives on Digital Payments, NITI Aayog constituted a Committee of Chief
Ministers on Digital Payments in 2016 with the then CM of Andhra Pradesh, Chandrababu
Naidu, as the Convener to promote transparency, financial inclusion and a healthy financial
ecosystem nationwide.

As an example of competitive and cooperative federalism, NITI has come out with indices to
measure incremental annual outcomes in critical social sectors like health, education and
water with a view to nudge the states into competing with each other for better outcomes,
while at the same time sharing best practices & innovations to help each other .
In 2016 , NITI Aayog constituted Sub-Group of Chief Ministers on Rationalization of Centrally
Sponsored Schemes, on whose recommendations the rationalization of the existing CSSs into
28 umbrella schemes. Similarly , there have been sub groups of Chief Ministers on Swachh
Bharat Abhiyan and on Skill Development, Constituted by NITI Aayog to foster the spirit of
cooperative federalism.
However, experts believe that there is a strong case to rethink the role of the NITI Aayog,
especially against the emerging backdrop of new dimensions in cooperative federalism .
The former Reserve Bank of India governor and head of the 14th Finance Commission Y.V.
Reddy points out that the NITI Aayog has been confined to writing reports on the Indian
economy, preparing draft laws for the Union government, evaluating central schemes and
holding conferences with state government officials. The scope and the remit of the NITI
Aayog has been expanded and its stature reduced. This has resulted in a vacuum of
institutional and procedural arrangements for the interactions between the Union and
States. That vacuum has been unfortunately occupied by the ministries in the Union
government.
Vijay Kelkar, chairman of the 13th Finance Commission, argues that a restructured NITI
Aayog should be given a funding role so that it can help deal with the development
experience between states. Unlike the micromanagement of the Planning Commission, the
transfers to states should be conditional and based on a formula rather than discretion.
Thus , there is lot of scope for re-imagining its role to address the institutional vaccum to
foster cooperative federalism . NITI Aayog has to evolve further to become a meaningful
institution , especially when the number of centre and state disputes on financial fronts are
increasing and there is no dedicated platform at par with erstwhile NDC, for the consultation
with the states . Though we have interstate council , yet this institution has also not been
used in the manner conceptualised by the constituent Assembly.
8(c).The study of electoral politics and political behaviour in India reflect the centrality of
distributive politics , role of ethnicity , deinstitutionalisation of political parties and the
subnational dynamics . Discuss.
The study of electoral behaviour is a prominent area of research under behavioural
revolution. The study of electoral politics and political behaviour is seen as one of the most
interesting in the field of comparative politics considering the uniqueness of the Indian
experiment of democracy. To quote Pratap Bhanu Mehta, nowhere ever such a large number
of electorate, largely uneducated and poor , got the right to vote in a single stroke . The
study of Indian electoral politics is too complex considering the size and heterogeneity .
Some of the prominent worked in the field of electoral politics and political behaviour
includes Pranav Roy, David Butler , Ashok Lahiri’s study ‘A compendium of Indian elections ’;
Mukulika Bannerji’s “How India Votes”.
Indian elections reflecting the centrality of distributive politics, role of ethnicity ,
deinstitutionalisation of political parties and the subnational dynamics as following .
Distributive politics :
The distributive politics of Indian state can be seen in the form of reservations and allocation
of welfare measures and freebies . Reservation is so central to Indian politics is that Indian
politics is described through the phrase of Mandalisation . Time and again , political parties
make reservation as core electoral issue .
Role of ethnicity
According to Myron Weiner, ethnicity in India can be understood in terms of caste , religion ,
tribe an language . According to Atul Kohli , it is natural for prismatic society like India that
elites will resort to ethnic mobilisation . Ethnic mobilisation and specifically the role of
religion is evident in Indian elections where different ethnic groups vote en bloc.
Deinstitutionalisation of political parties
According to Prof. Yogendra Yadav , Indian political parties reflect institutionalisation as well
as deinstitutionalisation . If on one hand , their reach is increasing on the other hand the
disconnect with the voters is also growing . As suggested by Zoya Hasan , the
deinstitutionalization of Indian parties has reduced them to the singular role as electoral
machines . The phenomenon of coalition politics , defections can be attributed to the
ongoing deinstitutionalisation .
Subnational dynamics
Paul Brass calls regionalism as the long term trend of Indian politics , whereas nationalism is
a short term trend . Regionalism has been a prominent factor especially since the creation of
linguistic states . Sub national dynamics/regional politics represent itself in the form of
various types of demands like greater autonomy , smaller states and even secessionism .
Thus , Indian politics is an interplay of varied factors.
PSIR Crash Course 2021
Model Answers - Test 3
For any issue related to PSIR Test Series, write only at psirtestseries@shubhraranjan.com

Section A
Answer the following in about 150 words each

1(a). Discuss the limitations of political sociology approach to comparative politics.

Political Sociology approach to Comparative Politics is one of the oldest approaches. It


analyses the interaction between politics and society. It primarily aims to understand the
impact of sociological factors on the functioning of Political system. In basic term, political
sociology explores to understand the process of interaction between government and
society, decision making authorities and conflicting social forces and interests. According
to Sartori, a real political sociology is a cross disciplinary break through seeking enlarged
models. It is the study of interactions and relationships between politics and civilization;
between a political system and its social, economic and cultural environment.
Aristotle is known as the father of Political Sociology. His theory of Revolution is one of the
earliest works in Political sociology where he tried to understand the social conditions that
give to the rise of Revolutions. Machiavelli has also used political sociology. According to
him, the nature of society shapes the desirable nature of the political system. If people are
virtuous, then republican form of government is preferable. Otherwise, Monarchy is suitable
if citizens are corrupt.
Political Sociology was further developed by Karl Marx. According to him, we cannot
understand politics/ nature of state without understanding the mode of production and class
relations. He does not consider that state has autonomy. Max Weber is considered as the
father of Non-Marxist School of Political Sociology. The most important contribution of
Weber is the concept of Authority and Legitimacy. According to him, the basis of authority
and legitimacy is shaped by the nature of society.
Later on elitist and pluralist scholars developed the school of Political sociology like Robert
Michell’s, C. Wright Mills, Robert Dahl. There is an enormous literature on Indian politics by
Political sociologists like Rajni Kothari, Andre Beteille, Cristophe Jaffrelot, Paul Brass where
they have studied the impact of caste, religion and ethnicity on Indian politics.
Political Sociology is not simply an approach, it has emerged as a hybrid discipline of politics
and sociology. Scholars utilize analytical and scientific methods in combination to produce
the research. There is a greater enthusiasm among sociologists towards political sociology.
Similar enthusiasm does not exist among political analysts. Some political scholars believe
that the approach subordinates the study of politics to the study of society. It shows as if
political system lacks autonomy and is responding to sociological variables. It appears as if
society is shaping politics and not vice versa. Theda Skocpol called for bringing the state
back in.

1(b). Compare new social movements with old social movements.


Social Movements are large, often informal groupings of people who come together against
perceived oppression or unrealised social political economic or cultural demands at the core
they are not the spaces for debates rather there is a sustain sequence of opposition and
contestations. For much of the 20th Century, social movements were class based such as
working class movements, peasant movements or anti-colonial movements.
Social movements can be classified into old and new social movements.
The labour movements campaigning for improving the situation of workers where the
earliest examples the other examples includes national movements struggling to come out of
multinational empires. It was also common for Fascist and right wing authoritarian groups
to be seen as movements rather than conventional political parties. Since 1960’s there is an
emergence of “new social movements for example environmental movement, peace
movement and so on.”
Differences between Old and New Social Movement

Old social movements where the movements of oppressed classes whereas the new social
movements attract the educated and relatively affluent classes.

The orientation of old social movement was addressing the bread and butter issues whereas
new social movements are post materialistic in orientation and concerned with quality of life
issues.

The old social movements had little in common and seldom worked in tandem, new social
movements subscribed to common if not always clearly defined ideology. They are linked to
ideology of new left. Since they subscribed to common ideology there is a huge overlap in
their membership & sympathies.
Old social movements tend to have organisational structures which were closed whereas
new social movements go for decentralisation and participatory approach. They go for more
innovative and theatrical form of protest politics for example Occupy Wallstreet Movement.
The emergency of movements of various types have let to the diverse theorization of
movements for example the experience of totalitarianism led Eric Fromm and Hannah
Arendt see movements in negative sense. From there perspective the movements are
attempts by alienated individuals to achieve security and identity through fanatical
commitments to a cause or a leader. Zald and McCarthy view the movements as the work of
rational actors using unconventional means and resources available to them. Laclau and
Mouffe suggest that the power in post industrial societies is greatly dispersed giving rise to
new form of politics that is “democratic pluralism” in place of class based politics of the old.
Conclusion:
Social movements offer new centers of power resisting entrenched power structures
including democracy. However the impact of social movement is difficult to study.

1(c). Compare the nature of pressure group politics between developed and developing
countries.
A pressure group is a group of people who are organized actively for promoting and
defending their common interest. It is called so, as it attempts to bring a change in public
policy by exerting pressure on the government. It acts as a liaison between the government
and its members.
The pressure groups are also called interest groups or vested groups. They are different
from the political parties, as they neither contest elections nor try to capture political
power. They are concerned with specific programmes and issues and their activities are
confined to the protection and promotion of the interests of their members by influencing
the government. V.O.Key suggest the most striking feature of American politics is the
extent to which pressure groups supplements the role of the political parties. Herman Finer
suggest interest groups form a new axiom where political parties are weak.
The pressure groups influence the policy-making and policy implementation in the
government through legal and legitimate methods like lobbying, correspondence, publicity,
propagandizing, petitioning, public debating, maintaining contacts with their legislators
and so forth.
There are different types of Pressure groups. One of the classification has been given by
Almond and Powell which classifies the P/G into four categories.
a) Institutional- which are part of government like the pressure groups of IAS officers,
army officers. They are present in the system and thus, have significant power to
influence. They are one of the most powerful pressure groups especially in developing
countries. For example: the pressure groups of civil servants in India play a crucial
role in policy making- both internal as well as external. Similarly, the Army in
Pakistan too plays the role of a Pressure group when a democratic leader is at the
helm.
b) Associational- It is usually an organization or collective group driven by Interest. The
organization functions on the basis of common interest among members rather than
other community relations. Countries which are more developed and where ethnic
identities don’t play a big role will have primacy of associational Pressure groups. For
example: in US, the most powerful pressure groups are associational in nature like
those of corporates.
Some examples of Associational Interest Groups in India are Bengal Chamber of
Commerce and Industry, Indian Chamber of Commerce, Trade Unions such as AITUC
(All India Trade Union Congress), Teachers Associations, Students Associations such
as National Students Union of India (NSUI) etc.
c) Non-Associational- These are the kinship and lineage groups and ethnic, regional,
status and class groups that articulate interests on the basis of individuals, family
and religious heads. These pressure groups are more common in developing
communities. For example: caste based groups. They are more prominent in
developing societies.
d) Anomic- These groups come into existence through a spontaneous breakthrough into
the political system from the society such as riots, demonstrations, assassinations
and the like. These groups show community action but can’t be classified in the
above three- crowd behavior. For example: student strike or dharnas. They are more
common in developing societies.

Jean Blondel gives another classification of Pressure groups. He argues that as the
society shifts from developing to developed, the nature of Pressure groups too change
from non-associational/communal to associational Pressure groups. The community or
non-associational interest groups are formed to promote community interests. The social
relations are in the back of their formation. They put pressure on the government to seek
state protection and assistance.
The associational groups have two sub-categories (a) protective groups and (b)
developmental groups. The protective groups try to protect the interests of their
members like those of trade unions and associations of traders or professionals. They,
thus, have more or less homogeneous clientele.
The developmental groups, on the other hand, have membership or large cross-sections
of community. The promotional groups may include group for disarmament, or the
Greens seeking promotion of environmental security. As the society starts having surplus
and democracy starts strengthening in the state, the orientation of the Pressure groups
changes from protection to development.

In conclusion, Pressure groups are now considered as an indispensable and helpful


element of the democratic process in both developing and developed countries. The
society has become highly complex and individuals cannot pursue their interests on their
own. They need the support of other fellow beings in order to gain greater bargaining
power; this gives rise to pressure groups based on common interests.
Democratic politics has to be politics through consultation, through negotiation and
some amount of bargaining is also involved. Thus, it is very essential for the government
to consult these organised groups at the time of policy formulation and implementation.

1(d).To what extent it would be appropriate to call system’s approach as status quoist.
The System’s approach is the study of inter-related variables forming one system, a unit, a
whole which is composed of a set of elements standing in interaction. This approach
assumes that the system consists of discernible, regular and internally consistent patterns,
each interacting with another, and giving, on the whole, the picture of a self-regulating
order.
System’s approach in political science was given by David Easton in his book: A system
analysis of Political life. It was the result of inter-disciplinary movement that was taking
place under Behaviouralism. David Easton replaced the study of state with the study of
political system. State is a legal term and thus, is a static concept. David Easton prefers the
term Political system as it is dynamic and comprehensive. It gives the idea of environment in
which the state is placed.
Specific Features of Political Systems :

Political system performs a specific function of “authoritative allocation of values”. In other


words, it is the supreme decision making body in a defined territory.
How Political System functions :
Political system functions like a machine. Its task is conversion of input into output. Input is
the demands and supports for the demand. Every demand will not enter the political system
but demands with substantial support will enter. The output is in the form of policies and
decisions. The policies interact with the environment, re-enter the system through the
feedback loop. Feedback is important for the survival of the political system.
Critical analysis: Systems Approach as status quoist

The Marxists believe that the model is based on the working of political systems in liberal
democracies. The model is not suitable for understanding political changes like the
phenomenon of revolution which may change the political system itself or alter some of its
features. Thus, system’s approach is status quoist i.e. the approach works on the belief that
liberal political systems are ideal and all systems should be like that. It does not reflect on
the contradictions in the political system. Thus, Systems approach is considered as Status
Quoist. The other criticism of systems approach include - it is a a very primitive type of
conceptual framework. It may be useful at the initial level of the research for collection and
organization of the data but it cannot provide deeper critical analysis. The traditionalists
believe that behavioralists introduced unnecessary jargons.
1(e). Examine the changing trends in the nature of state in advanced industrial societies
since the beginning of 21st century.

Once Political scientists ignored the study of state but later on realized that the study of
state is the core concern of Political science. Theda Skocpol called for bringing the state
back in.

The developed countries, which together constitute the 'Global North' are called as
'Advanced Industrial states'. Most of them have been imperial powers in the past (like UK)
and continue to have dominance on Politics, economics and Cultural-ideological spheres.

However, advanced Industrial societies cannot be treated as one uniform and homogenous
category. There are a lot of variations in reality. One can compare advanced industrial
societies from developing societies at a very general level.

Changing trend of states in Advanced industrial societies:


The state is a historical institution emerging in 15th- 16th Century Europe. The treaty of
Westphalia (1648) is taken to have formalized the modern notion of statehood.
As a historical institution, the state has undergone variety of transformations.

a) Early 19th Century: Rise of nationalism led to the creation of 'nation-state', replacing
'city-states'.

b) Upto 1930’s: the nature of state was characterized as “Laissez faire” or “minimal
state”.

c) 1930’s to 1970’s:The character of state transformed into 'welfare state'. In this sense,
the ability to deliver prosperity and protect citizens from social deprivation became the
principal source of legitimacy in most states.

d) 1980’s to 1990’s: This phase was characterized by a general trend towards


'neoliberalism' and progressive 'hollowing out' of the state.

e) From 1990’s to the start of 21st century: Since the phase represented globalisation
we see the weakening of state and the rise of supra national institutions for example EU.
f) Since 2008 financial crisis: These states aren’t only facing economic but also political
crisis also, which is reflected in rise of 'neo-rightists' in USA and Europe. However there is a
growth of “statism”.

Happymon Jacob: In the context of covid 19 Crisis mention that the states has failed in its
attempt to effectively counter the pandemic, yet the state has returned with more power,
legitimacy and surveillance technologies. The state which was losing its influence to global
economic forces will return as the last refuge of the people in the coming age of mass
disruption.
2a. Traditional form of globalization is in partial retreat and new agendas are morphing
into the new forms of globalization. Analyse the statement in the light of recently
concluded G7 Summit.
Some of the traditional forms of globalisation (the free movement of goods, money,
people, etc) are in partial retreat, but globalisation is morphing as new agendas come into
focus. These now drive action on controlling climate change, taxation of global companies,
tackling terrorism, sharing vaccines and the like. Cross-border problems in a more
integrated world are forcing countries to come together, even as elements of traditional
globalisation lose traction.
Global trade, for instance, has begun growing slower than global GDP. This significantly
reverses a long-term trend. In only one of the last seven years did the volume of merchandise
trade grew faster than the world economy. In 2019, global trade shrank in absolute terms
for the first time in a decade, and did so again in 2020 because of the pandemic.
Protectionist walls have been going up in several countries, including India.
Another aspect of Traditional form of Globalisation: free movement of people has also been
affected. Europe and North America account for over half the total of world migrants, and
their number has shrunk, though admittedly by a small margin. Brexit and Donald Trump’s
policies have signalled the reversal of a 70-year trend of immigration regimes getting
progressively more liberal. Some West Asian countries, too, have begun tightening visa
policies.

However, one aspect of traditional Globalisation continue unabated, like the cross-border
movement of capital. Also, the impact of new technologies that led to Thomas Friedman’s
“Flat World” thesis also persists. But new agendas are shaping new forms of globalisation
like

 New International Corporate Taxation Regime: It stipulates a minimum rate of tax


to be paid in the country where revenue is generated. The primary beneficiaries of
the new regime, when it comes into effect, will be the wealthy countries.
 Climate Change: Although developing countries like India are an enthusiastic
implementer of the Paris Agreement of 2015, they get no assistance (financial or
technical) to switch to new technologies and give up old ones like coal-based energy.
At the same time, the countries responsible for much of the historical emission of
carbon gases get a free pass.
 Cross-Border Terrorism: The return of Taliban in Afghanistan show the failure of the
continued efforts of International communities with respect to transnational
Terrorism.
 Sharing Vaccines: Despite Covax initiative, developed countries have indulged in
Vaccine Nationalism which basically entails reserving large quantity of vaccines for
their own population rather than equitable distribution of the same.
 Social Media: The giant tech companies that dominate the field have had a free run
but have increasingly come up against sovereign state power, including in India. They
have demanded exclusive rights which have challenged not only the sovereignty of
nation states but also democratic principles like Free and fair elections. There is a
need for setting global rules for global business.

The 2021 G7 summit was held from 11 to 13 June 2021 in Cornwall, England, during the
United Kingdom's tenure of the presidency of the Group of Seven (G7), an inter-
governmental political forum of seven advanced nations. Participants included leaders
of the G7 member states plus representatives of the European Union. The president of
the European Commission has been a permanent participant at all meetings since 1981.
Boris Johnson, Prime Minister of the United Kingdom, invited leaders from India, South
Korea, South Africa and Australia.

Topics of discussion included developing a response to the COVID-19 pandemic and


climate change. British Prime Minister Boris Johnson pledged to call for the G7 to work
on a global approach to pandemics to ensure an equal global distribution of COVID-19
vaccines and to prevent future pandemics. He proposed a five-point plan to prevent
future pandemics, which includes a worldwide network of zoonotic research hubs,
developing global manufacturing capacity for treatments and vaccines, the design of a
global early warning system, the agreement of global protocols for a future health
emergency and the reduction of trade barriers.
Another topic of discussion was international co-ordination on economic policies.
United States Secretary of the Treasury Janet Yellen has said there would be a revival in
American multilateralism, focusing on continued economic support. Negotiations over
reforming the corporate taxation of multinationals were also carried out, with the United
States agreeing to a deadline of the summer to make progress on the issue after
threatening to impose tariffs on European countries in retaliation for their new digital
sales tax.

The Cornwall Consensus was circulated ahead of the G7 leaders meeting in Cornwall. It
is the title of an advisory memo written by a committee of academics and policymakers
from each of the seven countries of G7. It sets out an “ambitious agenda to build
forward better from the pandemic”. The document contains ideas, such as “greater
equity and solidarity in global health responses”. It also makes more detailed proposals,
such as the creation of “a ‘Data and Technology Board’ akin to the Financial Stability
Board” to oversee the global internet and a “CERN (European Organization for Nuclear
Research) for climate technology”. The memo suggests that the recent G7 corporate tax
accord should herald a new phase of western collaboration along new ideological lines.
In Conclusion, the 2021, G7 summit did help in shaping the evolving agendas of new forms
of Globalisation. There was a push for global coordination on minimum corporate taxation
and climate change efforts. However, it is important to note that if the G-7 wants to truly
exercise more leadership, it will have to convince the world that all its new principles,
resilience, inclusion, global public goods, are not simply ruses to serve only the interests of
the developed world. Or that important gestures like providing vaccines are not just one-off
interventions in the aftermath of a crisis.

2b. Is Realist Approach the best method to understand International Relations? Examine
this in the context of Classical Realism.
In the discipline of International Relations (IR), realism is a school of thought that
emphasises the competitive and conflictual side of international relations. Realism’s roots
are often said to be found in some of humankind’s earliest historical writings, particularly
Thucydides’ history of the Peloponnesian War, which raged between 431 and 404 BCE.
Fundamentals of Realism:
The first assumption of realism is that the nation-state (usually abbreviated to ‘state’) is the
principle actor in international relations. Other bodies exist, such as individuals and
organisations, but their power is limited.
Second, the state is a unitary actor. National interests, especially in times of war, lead the
state to speak and act with one voice.

Third, decision-makers are rational actors in the sense that rational decision-making leads
to the pursuit of the national interest.

Finally, states live in a context of anarchy – that is, in the absence of anyone being in charge
internationally. The often-used analogy of there being ‘no one to call’ in an international
emergency helps to underline this point.

As realism frequently draws on examples from the past, there is a great deal of emphasis on
the idea that humans are essentially held hostage to repetitive patterns of behaviour
determined by their nature. Central to that assumption is the view that human beings are
egoistic and desire power. Realists believe that our selfishness, our appetite for power and
our inability to trust others leads to predictable outcomes. Perhaps this is why war has been
so common throughout recorded history. Since individuals are organised into states, human
nature impacts on state behaviour.
In that respect, Niccolò Machiavelli focused on how the basic human characteristics
influence the security of the state.

Classical Realism as a self-conscious movement in the study of international relations


emerged during the mid-20th century and was inspired by the British political scientist and
historian E.H. Carr. Carr attacked what he perceived as the dangerous and deluded
“idealism” of liberal internationalists and, in particular, their belief in the possibility of
progress through the construction of international institutions such as the League of Nations.
He focused instead on the perennial role of power and self-interest in determining state
behaviour. The outbreak of World War II converted many scholars to that pessimistic vision.
In the aftermath of the Second World War, Hans Morgenthau (1948) sought to develop a
comprehensive international theory as he believed that politics, like society in general, is
governed by laws that have roots in human nature. His concern was to clarify the
relationship between interests and morality in international politics, and his work drew
heavily on the insights of historical figures such as Thucydides and Machiavelli.

In contrast to more optimistically minded idealists who expected international tensions to be


resolved through open negotiations marked by goodwill, Morgenthau set out an approach
that emphasised power over morality. Indeed, morality was portrayed as some- thing that
should be avoided in policymaking. In Morgenthau’s account, every political action is
directed towards keeping, increasing or demonstrating power. The thinking is that policies
based on morality or idealism can lead to weakness – and possibly the destruction or
domination of a state by a competitor. In this sense pursuing the national interest is ‘amoral’
– meaning that it is not subject to calculations of morality.
Numerous other contributors to realist theory emerged in the decade or so after World War
II, including Arnold Wolfers, George F. Kennan, Robert Strausz-Hupé, Henry Kissinger, and
the theologian Reinhold Niebuhr.
During the 1960s and 1970s, the Classical Realist approach started being challenged.
Following the behavioral revolution scholars began to place a new emphasis on creating a
more empirical methodology for analyzing international relations. Neo-realist scholars
criticised how Classical realist scholars had created methodologies which lacked the
standards of proof to be considered scientific theories. Classical realists had emphasized
human nature as the primary form of explaining the international system; Neo-realists
emphasized the international structure instead. Kenneth Waltz’s Theory of International
Politics was a critical text in this debate as it argued that international anarchy was a core
element of international politics.
In Theory of International Politics (1979), Kenneth Waltz modernised IR theory by moving
realism away from its unprovable (albeit persuasive) assumptions about human nature.
His theoretical contribution is termed ‘neorealism’ or ‘structural realism’ because he
emphasised the notion of ‘structure’ in his explanation. Rather than a state’s decisions and
actions being based on human nature, they are arrived at via a simple formula. First, all
states are constrained by existing in an international anarchic system (this is the structure).
Second, any course of action they pursue is based on their relative power when measured
against other states. So, Waltz offered a version of realism that recommended that
theorists examine the characteristics of the international system for answers rather than
delve into flaws in human nature. In doing so, he sparked a new era in IR theory that
attempted to use social scientific methods rather than political theory (or philosophical)
methods. After this era Classical Realist doctrines became less prominent in favour of Neo-
realism.
As it is designed to explain repetition and a timeless pattern of behaviour, realism was not
able to predict or explain a major recent transformation of the international system: the
end of the Cold War between the United States of America (US) and the Soviet Union in
1991. When the Cold War ended, international politics underwent rapid change that pointed
to a new era of limited competition between states and abundant opportunities for
cooperation. This transformation prompted the emergence of an optimistic vision of world
politics that discarded realism as ‘old thinking’.
Realists are also accused of focusing too much on the state as a solid unit, ultimately
overlooking other actors and forces within the state and also ignoring international issues
not directly connected to the survival of the state. For example, the Cold War ended
because ordinary citizens in Soviet-controlled nations in Eastern Europe decided to rebel
against existing power structures. This rebellion swept from one country to another within
the Soviet Union’s vast empire, resulting in its gradual collapse between 1989 and 1991.
Realism’s toolbox did not and does not account for such events: the actions of ordinary
citizens (or international organisations, for that matter) have no major part in its
calculations. This is due to the state-centred nature of the thinking that realism is built upon.
It views states as solid pool balls bouncing around a table – never stopping to look inside
each pool ball to see what it comprises and why it moves the way it does.
Despite these criticisms, realism remains central within the field of IR theory, with most other
theories concerned (at least in part) with critiquing it. For that reason, it would be
inappropriate to write that Realism is not an important Theory of IR but it is certainly not
the best approach to the field.

2c. Examine the World Systems Approach as developed by Immanuel Wallerstein.


World systems theory is a response to the criticisms of Dependency Theory. It was
developed by Immanuel Wallerstein (1979) who suggests that the way a country is
integrated into the capitalist world system determines how economic development takes
place in that country.
World Systems Theory, like dependency theory, suggests that wealthy countries benefit from
other countries and exploit those countries’ citizens.
Wallerstein accepts the fact ex-colonies are not doomed to be forever trapped in a state of
dependency; it is possible for them to climb the economic ladder of development, as many of
them have done. However, he also believes that the global capitalism system still requires
some countries, or at least regions within countries to be poor so they can be exploited by
the wealthy at the top. Wallerstein’s theory has four underlying principles:

 One must look at the world system as a whole, rather than just at individual
countries. Dependency Theory tended to argue that countries are poor because
they used to be exploited by other countries. However focusing on countries (or
governments/ nation states) is the wrong level of analysis – government today
have declined in power, whereas Corporations are more powerful than ever. Global
Corporations, and global capital, transcend national boundaries, and nation states
(even wealthy ones) are relatively powerless to control them, thus in order to
understand why countries are rich or poor, we should be looking at global economic
institutions and corporations rather than countries. Global Economic Institutions
form what Wallerstein calls a Modern World System, and all countries, rich and poor
alike are caught up in it.
 Wallerstein believes that the Modern World System is characterised by an
international division of labour consisting of a structured set of relations between
three types of capitalist zone:
o The core, or developed countries control world wages and monopolise the
production of manufactured goods. Core countries (e.g., U.S., Japan and
Germany) are dominant, capitalist countries characterized by high levels of
industrialization and urbanization. They are capital intensive, have high
wages and high technology production patterns and lower amounts of labour
exploitation and coercion. They own most of the world’s capital and
technology and have great control over world trade and economic
agreements. They are also the cultural centres which attract artists and
intellectuals.
o The semi-peripheral zone includes countries like South Africa or Brazil which
resemble the core in terms of their urban centres but also have areas of rural
poverty which resemble the peripheral countries. The core contracts work out
to these countries.
o Finally, there are the peripheral countries at the bottom, mainly in Africa,
which provide labour and raw materials such as cash crops to the core and
semi periphery. These are also the emerging markets in which the core
attempts to market their manufactured goods.
 Countries can be upwardly or downwardly mobile in the world system. This is one
of the key differences between World System’s Theory and Dependency Theory.
Many countries, such as the BRIC nations have moved up from being peripheral
countries to semi-peripheral countries.
 The Modern World System is dynamic – core countries are constantly evolving new
ways of extracting profit from poorer countries and regions. Examples of new ways of
extracting profit from poor countries include Unfair Trade rules, monopoly of
Western Corporations etc.

Critical Evaluation:
Wallerstein can also be criticised in the same way Dependency Theorists can be criticised
– there are more causes of underdevelopment than just Capitalism – Such as cultural
factors, corruption and ethnic conflict. Wallerstein puts too much emphasis of
economics and the dominance of Capitalism – There are other ways people can be
exploited and oppressed – such as tyrannical religious regimes for example. Also, there
are some areas are still not included in the World System – some tribal peoples in South
America and Bhutan for example remain relatively unaffected by global capitalism.
Finally, Wallerstein’s concepts of Core, Semi-Periphery and Periphery are vague and this
means his theory is difficult to test in practise. The general criticism of Marxist theory of
international politics is that it is based on the experiences of western countries and hence
euro centric.

3a. “Beijing is marking the 100th anniversary of the communist party of China in a big way.
However, it masks uncertainty about China’s future.” Do you agree? Give arguments in
support of your point of view.
On 1st July, the Communist Party of China celebrated its 100th anniversary with a huge
celebration in Beijing. The highlight of the event was President Xi Jinping’s speech that
indicated China’s supremacy in the International sphere. He said: “The Chinese people will
never allow any foreign forces to bully, oppress or enslave us.”

The Chinese Communist Party is the de-facto sole governing party in China since 1949. The
CCP was founded in 1921, with the help of the Far Eastern Bureau of the Communist Party of
the Soviet Union and Far Eastern Secretariat of the Communist International. The party grew
quickly, and by 1949 it had driven the Kuomintang (KMT)'s Nationalist Government from
mainland China to Taiwan after the Chinese Civil War, leading to the establishment of the
People's Republic of China on 1 October 1949. It controls the country's armed forces, the
People's Liberation Army (PLA).

Although, the party has been in power for 100 years, there is still uncertainty about China’s
future.

 President Xi has pegged much of his economic development strategy on


transforming China into a world-class innovative nation. There is no doubt that
China has the edge on innovation as it is already innovating in a variety of
technologies and fields and it will continue to do so. The uncertainty is: Can China
become a fully innovative nation without substantially liberalizing its deeply
autocratic political system? To date, there has not been a single instance of a non-
democratic country becoming a multi-sector innovative nation.
 Rigidity and Repression of Xi’s rule- Many people in China today—both in and
outside the party—are not content with the renewed rigidity and repression of Xi’s
rule or the scrapping of term limits on his own positions. A state that runs on rigid
discipline must constantly enforce it—lest normal social and institutional desires for
greater autonomy and freedoms emerge to challenge the dictatorial regime.
The alternative is to give up some control and return to a more tolerant political
system characteristic of all of Xi’s predecessors dating back to Deng Xiaoping. Yet Xi
and his regime show no desire whatsoever to do so. Their vision of China’s future is
one dominated by an all-powerful party-state controlling all domains of domestic
activity. Only time will tell whether this is a compatible model for ruling 1.4 billion
people in an era of globalization and interdependence.
 Nature of China’s nationalism: The real question is whether it will continue to
nurture and perpetuate the insecure and aggrieved national identity that derives
from what the CCP ritualistically refers to as its “century of shame and humiliation”—
or, conversely, whether the Chinese government will speak and act with much
greater confidence and without the sharp-edged defensiveness so evident in its “wolf
warrior” public diplomacy today.

Conclusion: These three big questions—innovation, the party, and nationalism—are among
the most important to consider when contemplating China’s future trajectory. Nor are they
disassociated from each other. A strong and unified party-state, infused with aggrieved
nationalism, and emboldened by technological innovation (including military) is the track
China is now on under Xi—and it should be considerable cause of global concern.

3b. What should be New Delhi’s policy towards democratic struggles in neighbouring
countries? Discuss in context of recent developments in Myanmar.
A matter of great concern for India since Independence has been the future of Burma, a
country with which it shares land and maritime boundaries. Three years before
Independence, India’s ambassador K M Panikkar had underlined the strategic significance of
Burma in the following words: “The defence of Burma is, in fact, the defence of India and it
is India’s primary concern, no less than Burma’s, to see that its frontiers remain inviolate.
In fact, no responsibility should be considered too heavy for India when it comes to the
question of defending Burma.”

In the early years of Independence, Jawaharlal Nehru played a significant role in bolstering
Burma politically and militarily. In fact, like Indonesia, Burma and its leadership were very
close to New Delhi. Professor Werner Levi, the distinguished political scientist, even
remarked that Burma was India’s satellite.
Soon after its independence, Burma was plunged into a civil war. The assassination of Aung
San was followed by armed revolts led by the communists, Kachins and Karens. Even the
security of Rangoon was threatened by rebel forces. Indian concern was naturally sharpened
with the emergence of the People’s Republic of China in 1949, which shared common
borders with both India and Burma. Nehru persuaded the Commonwealth countries to
provide military and economic aid to Burma. Indian assistance encompassed both military
and economic assistance and also the bolstering of the U Nu regime. It aimed to have a
friendly, non-aligned and non-communist buffer between India and China. It was also to
prevent the destabilisation of northeast India, where the Nagas, Mizos, and Meities straddle
the India-Burma border.

India’s principled stance continued for many years. But during the Rajiv Gandhi era, with J
N Dixit as the foreign secretary, New Delhi began to get closer to the military regime. The
change was dictated by strategic considerations. The rebels in the Northeast—Nagas and
Mizos in particular—were supported by China with arms and they used Burmese territory to
go to China. The military brass felt that if we got the Burmese army’s support, we would be
able to contain the threat posed by the rebels. There had been several occasions when the
Indian Army, in hot pursuit, entered Burmese territory to carry out their fight against the
rebels. The result was the betrayal of the democratic forces in Myanmar.
Myanmar is now sitting on the top of a volcano. According to a recent report of the
International Crisis Group, Myanmar “is edging towards a state collapse”. A coup d'état in
Myanmar began on the morning of 1 February 2021, when democratically elected members
of the country's ruling party, the National League for Democracy (NLD), were deposed by the
Tatmadaw—Myanmar's military.
The Tatmadaw proclaimed a year-long state of emergency and declared power had been
transferred to Commander-in-Chief of Defence Services Min Aung Hlaing. It declared the
results of the November 2020 general election invalid and stated its intent to hold a new
election at the end of the state of emergency.The coup d'état occurred the day before the
Parliament of Myanmar was due to swear in the members elected at the 2020 election,
thereby preventing this from occurring. President Win Myint and State Counsellor Aung San
Suu Kyi were detained.
On 7th March 2021, the Armed Forces Day, the military held a massive parade of troops and
weapons. All neighbouring countries—China, India, Bangladesh, Thailand and Laos—as
also Pakistan, Russia, and Vietnam sent their representatives to witness the parade. In its
official statements, the GoI has stated it would work for the restoration of democracy in
Myanmar. But it sounds hollow in the context of India’s participation in the military parade
on March 6, which implies providing legitimacy to the military establishment.
The need of the hour is for New Delhi to mobilize international support for the restoration
of democracy, raise the issue in the UN Security Council, General Assembly and Human
Rights Council; work for an arms embargo to the military; impose economic sanctions on
military-owned business interests etc.
Myanmar is an important element for the success of India’s ‘Neighbourhood First’, ‘Act
East’, and ‘Indo-Pacific’ policies. It was from Naypyitaw that Prime Minister Modi launched
India’s Act East policy in 2014. India’s initial response to the February 1 coup expressing
“deep concern” was moderate. After the military coup in Myanmar in February, General
Naravane at an event underlined that a "series of operations under Operation Sunrise with
Myanmar Army has witnessed growing cooperation and synergy between the soldiers on
ground with reasonable operational dividends"
India did not condemn Myanmar's military over coup but it stepped up efforts to seal the
border and push back fleeing dissidents. On March 18, the Union home ministry wrote to the
state governments of Mizoram, Nagaland, Arunachal Pradesh and Manipur asking them to
take appropriate measures in stemming inflow of refugees.
Thus, the case study of Myanmar creates a huge challenge for India with respect to the
democratic struggles taking place in neighboring countries. As suggested by democratic
peace theorist, democracies form a zone of peace. However, the theory may not work in
South Asian context considering the fragile nature of these democracies. It is to be noted
that the democracies on the periphery of India are too divisive and politicians often use anti-
India card to show their nationalistic approach. The increasing interest of China is creating
bigger challenges for India as India doesn’t have big pocket like China.

In the context of South Asia it will be much better if we follow the fundamental principles of
Panchsheel that is noninterference in the domestic affairs. It is to be noticed that India can’t
ignore insurgencies’ in its neighborhood which require India to remain engaged with to so
ever in power. However their can’t be one size fit for all formula.

3c. Though the concept of hegemony is frequently employed in international politics, yet it
suffers from conceptual and theoretical ambiguities. Comment What America’s
withdrawal from Afghanistan tells about the status of US hegemony?
The notion of hegemony is especially difficult to enumerate both in concrete political terms
and in a less tangible philosophical manner. It is the political, economic, or military
predominance or control of one state over others. The concept of Hegemony was primarily
discussed by Antonia Gramsci who argued that: the supremacy of a social group or class
manifests itself in two different ways: domination/ coercion and intellectual and moral
leadership. This later type of supremacy constitutes Hegemony.

In international relations, hegemony refers to the ability of an actor with overwhelming


capability to shape the international system through both coercive and non-coercive means.
Ian Clark argues that the hegemony debate revolves largely around two principal meanings:
domination and leadership.
There are five basic dimensions of hegemony:

o Military: The hegemon has the strongest military in the world, considerably stronger
than any of its rivals. Its military alliance system is significantly stronger than any
rival military blocs.
o Economic: The hegemon has the biggest and most technologically advanced
economy in the world. It is a major trading partner of most of the nations of the
world, including most of the major powers.
o Political: The hegemon has array of political allies, and friendly relations with most
nations and major powers.
o Institutional: The hegemon, working with its associates, makes most of the rules that
govern global political and economic relations. The hegemon, along with its allies,
usually controls most of the international institutions. Thus, most of the policies of
the international institutions favour the hegemon and its partners.
o Ideological: The hegemon mainly determines the terms of discourse in international
relations. Marx wrote, "The ruling ideas of any age are the ideas of the ruling class."
Currently, the predominant ideas about globalization are the ideas of hegemon.

Theoretical and Conceptual Ambiguities:


In Realism, Hegemony is defined in terms of first, overwhelming power, and second, and the
ability to use this power to dominate others. They equate it with overwhelming material
power. A Hegemon is identified as the state that possesses vastly superior material
capabilities including military, economic and, sometimes, diplomatic or soft power. John
Mearsheimer, for example, defines a hegemon as a “state that is so powerful that it
dominates all the other states in the system”. He adds, “no other state has the military
wherewithal to put up a serious fight against it.” Barry Posen, for example, argues that the
United States’ command of the commons – command of the sea, space, and air – provides
the military foundation of US hegemony.

Realists label the most powerful state in the international system as the hegemon. The latter
is identified as the state that possesses vastly superior material capabilities including
military, and economic. Power, according to this view, is synonymous with capabilities, and
the capabilities of a state represent nothing more than the sum total of a number of loosely
identified national attributes including “size of population and territory, resource
endowment, economic capability, military strength, political stability and competence.”

This understanding ignores the importance of additional factors such as the capacity to
exercise power based on material capabilities, and ‘soft power’ or ideological power,
meaning the capability to change others’ behaviour by influencing their belief system,
their way of thinking, and even their rationality.
In Liberalism, scholars accentuate the particular type of leadership that is exercised by the
hegemon. Liberals do not completely discount the importance of preponderant material
power, but they argue that this alone is insufficient for understanding the concept of
hegemony. Liberal theorists are, for the most part, interested in the mechanisms and
processes through which hegemony is exercised.
In his influential work, After Hegemony, liberal theorist Robert Keohane made a distinction
between the “basic force model” and the “force activation model” of hegemony. He defined
the basic force model of hegemony as the possession of unrivalled tangible measures of
power, and the force activation model as encompassing both abundant power capabilities
and the will to exercise leadership.
The neo-Gramscian approach to hegemony also accepts the view that hegemony is about
more than just raw material power and domination. For Robert Cox, one of the leading neo-
Gramscians, “dominance by a powerful state may be a necessary but not a sufficient
condition of hegemony.” Cox combines material power, ideas, and institutions into a
comprehensive theory of hegemony.

Establishing hegemony has been one of the objectives of US foreign policy. Hegemony on the
world ensures peace and prosperity in USA. However since, the end of cold war, when US
leaders declared the establishment of unipolar world, we see constant challenges to US
hegemony. At present the 2 major challenges to US hegemony are:
1. The Islamic world
2. China

Samuel P Huntington had warned about the nexus between China and Islamic world as a big
challenge. The recent withdrawal of US from Afghanistan poses a big question about US
hegemony.
The speedy withdrawal of US troops from Afghanistan has been matched by the swift
advance of the Taliban across the nation. While the US has confirmed that 90% of the
withdrawal is done, the Taliban has claimed that it is in control of 85% of Afghanistan
territory. For the civilian leadership of the United States, the untimely drawdown of the
combat mission in Afghanistan, denounced by its own generals in the Pentagon has led it to
face questions regarding US Hegemony in global politics.
For a campaign that cost the US $685.6 billion and 2,259 casualties, the current state of
affairs in Afghanistan highlight the futility of the same. The combat mission has ended at a
time when the Taliban and its affiliated factions–Hezb-e-Islami and the Haqqani Network–
still remain deeply entrenched in the country, and genuine rapprochement with Pakistan is
yet to be seen.

In conclusion, there is a risk that Afghanistan might become yet another Iraq, crippled by a
premature withdrawal, rendering the vast expenditure in lives and treasure wasted. The
ability of the U.S. to project itself as normative influencer, a first among equals, with a stake
in maintaining order and systemic integrity globally is under dire threat by a looming crisis of
legitimacy and the questioning of its moral fortitude. The widespread unrest in Afghanistan
has challenged the US authority on upholding some measure of order in the country. In front
of the global audience, the failure in Afghanistan has cracked the image of US as a global
hegemon. The growing nexus between Taliban and China should put US on the front foot.
4a.To what extent it can be said that climate is the most important global concern of the
year 2021. Do you think that India has enough soft power to bring together the various
conflicting parties together?
In December 2020, United Nations Secretary General Antonio Guterres said in his State of
the Planet speech that: “ Covid and climate have brought us to a threshold… we have
come to a moment of truth…2021 will certainly be a crunch year for tackling climate
change. It is a “make or break” moment for the issue.”
From unprecedented wildfires across the US to the extraordinary heat of Siberia, the impacts
of climate change were felt in every corner of the world in 2020. Five crucial reasons can be
cited which makes climate change as the most important global concern in 2021:

a) CO2 levels: The amount of CO2 in the atmosphere reached record levels in 2020,
hitting 417 parts per million in May.
b) Record Heat: The past decade was the hottest on record. The year 2020 was more
than 1.2C hotter than the average year in the 19th Century.
c) Arctic Ice: Nowhere is that increase in heat more keenly felt than in the Arctic. In June
2020, the temperature reached 38C in eastern Siberia, the hottest ever recorded
within the Arctic Circle.
d) Permafrost: Across the northern hemisphere, permafrost – the ground that remains
frozen year-round for two or more years – is warming rapidly.
e) Forests: Since 1990 the world has lost 178 million hectares of forest (690,000 square
miles) – an area the size of Libya.

To discuss these issues and come up with a solution, In November 2021, world leaders will
be gathering in Glasgow for the successor to the landmark Paris meeting of 2015.
Under the terms of the Paris deal, countries promised to come back every five years and
raise their carbon-cutting ambitions. That was due to happen in Glasgow in November
2020. The pandemic put a halt to that and the conference was bumped forward to this year.
So, Glasgow 2021 gives us a forum at which those carbon cuts can be ratcheted up. The
programme aims to drive forward climate ambition and action against key issues. With
Climate change and action in focus, other themes ‘like science, innovation and inclusivity
will run throughout the programme. The stress, however, will remain on ‘the need to
mitigate climate change, adapt to its impacts, and mobilize public and private finance’.
Among the countries that will be attending the Glasgow meeting, India does have a
considerable sway due to its soft power. India’s plan of action is to convince other
developing nations to join hands to oppose plans by the European Union and the U.S. to
penalize imports of carbon-intensive goods to curb emissions. India will instead seek more
action from European nations and the U.S., which have not kept their commitments on
reducing emissions.
India would push for more action from developed nations at the Conference of Parties or
COP26 in November. The then Environment Minister Prakash Javadekar said. “We are
paying, we are suffering from climate change which was caused by the reckless emissions for
hundreds of years by the developed world.”

4b.Explain Cornwall Consensus . To what degree does it differs from the Washington
Consensus in a significant manner? Do you think a new international economic order is
unfolding?
Three decades ago, the British economist John Williamson coined the phrase “Washington
consensus” to describe a collection of free-market, pro-globalisation ideas that American
leaders (among others) were promoting around the world. In 2021, The Cornwall
Consensus was circulated ahead of the G7 leaders meeting in Cornwall. It is the title of an
advisory memo written by a committee of academics and policymakers from each of the
seven countries of G7. It sets out an “ambitious agenda to build forward better from the
pandemic”. The document contains ideas, such as “greater equity and solidarity in global
health responses”. It also makes more detailed proposals, such as the creation of “a ‘Data
and Technology Board’ akin to the Financial Stability Board” to oversee the global internet
and a “CERN (European Organization for Nuclear Research) for climate technology”. The
memo suggests that the recent G7 corporate tax accord should herald a new phase of
western collaboration along new ideological lines.

This 1.5-page memo wrote, "The G7 should lead on agreeing a new consensus and restore
public trust in a rules-based, free, fair and open economic system. This new consensus
would make sure that economic growth is green and inclusive, and makes us resilient
against environmental, economic and geopolitical risks."
The Group of Seven (G7) is an inter-governmental political forum consisting of Canada,
France, Germany, Italy, Japan, the United Kingdom and the United States. Its members are
the world's largest IMF advanced economies and wealthiest liberal democracies. The G7
(excluding Canada) met for the first time in 1975 and since then, have met annually. India,
South Korea and Australia were invited to attend the G7 summit as participating guests.

The Washington Consensus is a set of ten economic policy prescriptions considered to


constitute the "standard" reform package promoted for crisis-wracked developing
countries by Washington, D.C.-based institutions such as the International Monetary Fund
(IMF), World Bank and United States Department of the Treasury. The prescriptions
encompassed free-market promoting policies in such areas as macroeconomic
stabilization, economic opening with respect to both trade and investment, and the
expansion of market forces within the domestic economy.
The consensus as originally stated by Williamson included ten broad sets of relatively
specific policy recommendations:
 Fiscal policy discipline, with avoidance of large fiscal deficits relative to GDP;
 Redirection of public spending from subsidies ("especially indiscriminate subsidies")
toward broad-based provision of key pro-growth, pro-poor services like primary
education, primary health care and infrastructure investment;
 Tax reform, broadening the tax base and adopting moderate marginal tax rates;
 Interest rates that are market determined and positive (but moderate) in real terms;
 Competitive exchange rates;
 Trade liberalization: liberalization of imports, with particular emphasis on elimination
of quantitative restrictions (licensing, etc.); any trade protection to be provided by
low and relatively uniform tariffs;
 Liberalization of inward foreign direct investment;
 Privatization of state enterprises;
 Deregulation: abolition of regulations that impede market entry or restrict
competition, except for those justified on safety, environmental and consumer
protection grounds, and prudential oversight of financial institutions;
 Legal security for property rights.

The widespread adoption by governments of the Washington Consensus was to a large


degree a reaction to the macroeconomic crisis that hit much of Latin America, and
some other developing regions, during the 1980s. The crisis had multiple origins: the
drastic rise in the price of imported oil following the emergence of OPEC, mounting
levels of external debt, the rise in US (and hence international) interest rates, and—
consequent to the foregoing problems—loss of access to additional foreign credit.

However, the consensus resulted in socio-economic exclusion and weakened trade unions
in Latin America, resulting with unrest in the region. Countries who followed the
consensus initially alleviated high inflation and excessive regulation, though economic
growth and poverty relief was insignificant. The consensus resulted with a shrinking
middle class in Latin America that prompted dissatisfaction of neoliberalism, a turn to
the political left and populist leaders by the late-1990s, with economists saying that the
failure of the consensus established support for Hugo Chávez in Venezuela, Evo Morales
in Bolivia and Rafael Correa in Ecuador.
As of the 2000s, several Latin American countries were led by socialist or other left wing
governments, some of which—including Argentina and Venezuela—have adopted
policies contrary to the Washington Consensus policies. Washington consensus based
policies where adopted globally but resulted into increasing inequalities.
The global economic landscape has undergone profound changes since the 2000s. After
the 2008 financial crisis, the world economy entered a “new normal,” which was further
challenged by the onset of covid pandemic in 2019. There is a scope for a new
international economic order to emerge due to the following:
First, western leaders today fear political pitchforks. Thirty years ago, political figures such
as Margaret Thatcher and Ronald Reagan took it for granted that free-market globalisation
would benefit everyone. Today’s leaders fret that free-market fruits are so unevenly
distributed that it is sparking a popular backlash. “Inclusion” is one of the new buzzwords.

Second, G7 leaders also now acknowledge that globalisation and free-market competition
create vulnerabilities as well as efficiencies. Previously, they hoped that individual corporate
incentives would create an optimised cross-border supply system. Now they know that
global supply chains are threatened by a collective action problem.
Third, G7 debate is haunted by a fear of China. Beijing is not mentioned by name in the
Cornwall consensus memo. But there are multiple calls to diversify global supply chains, not
just for advanced technology, but for medical equipment and minerals too. Belatedly,
western governments have accepted that it was a terrible strategic mistake to allow global
chip production to be concentrated in the hub of Taiwan.

Fourth, there is a subtle, but nonetheless profound, reset under way of the relationship
between business and government. In the Washington consensus companies were regarded
as independent actors competing with one another, without state involvement. Now all the
talk is of “partnership” between government and business.
To conclude, the Cornwall Consensus might be the final trigger to kick-start the evolving
international economic order. After all, what is driving this ideological shift is not just Covid-
19, but also the rise of China, the threat of climate change and the evaporation of the
western hubris around free-market ideas that followed the collapse of the Soviet Union.

4c. In spite of being the dominant form of government in the contemporary world,
Democracy faces foundational challenges in both advanced Industrial societies and
developing societies? Comment.
Democracy, in simple words, can be summed up as a form of government in which the rulers
are elected by the people. The rulers elected by the people take all the major decisions. The
elections to choose a ruler offer a choice and fair opportunity to the people to change the
current rulers. This choice and opportunity is available to all the people on an equal basis
and the exercise of this choice leads to a government limited by basic rules of the
constitution and citizens’ rights.
In political science, the waves of democracy are major surges of democracy which have
occurred in history. The concept was popularized by Samuel P. Huntington, a political
scientist at Harvard University in his article published in the Journal of Democracy and
further expounded in his 1991 book “The Third Wave: Democratization in the Late
Twentieth Century.” Democratization waves have been linked to sudden shifts in the
distribution of power among the great powers, which creates openings and incentives to
introduce sweeping domestic reforms.
Huntington describes three waves:

 The first "slow" wave of the 19th century- The First wave of democracy, 1828–1926
began in the early 19th century when suffrage was granted to the majority of white
males in the United States. Then came France, Britain, Canada, Australia, Italy and
Argentina, and a few others before 1900. At its peak, after the breakup of the
Russian, German, Austrian and Ottoman empires in 1918, the first wave saw 29
democracies in the world. Reversal began in 1922, when Benito Mussolini rose to
power in Italy. The collapse primarily hit newly formed democracies, which could not
stand against the aggressions rise of expansionist communist, fascist and militaristic
authoritarian or totalitarian movements which systematically rejected democracy.
 A second wave after World War II- The Second wave began following the Allied
victory in World War II, and crested nearly 20 years later in 1962 with 36 recognised
democracies in the world. The Second wave ebbed as well at this point, and the total
number dropped to 30 democracies between 1962 and the mid-1970s.
 A third wave beginning in the mid-1970s in South Europe, followed by Latin America
and Asia- The Third wave began in 1974 Carnation Revolution in Portugal and
Spanish transition to democracy in late 1970s, with the historic democratic
transitions in Latin America in the 1980s, Asia Pacific countries (Philippines, South
Korea, and Taiwan) from 1986 to 1988, Eastern Europe after the collapse of the
Soviet Union, and sub-Saharan Africa beginning in 1989.

Several scholars have associated the Arab Spring with the Fourth wave of Democracy.
However, a few months after the apparent beginning of the transition, most of the Arab
countries reverted back to tradition non-democratic institutions, causing an inevitable pull-
back. The only country that actually made a transition was Tunisia that successfully
consolidated into a democratic state.
Today, democracy is the dominant form of government in the contemporary world in both
industrial as well as developing societies. It does not face a serious challenger or rival.
However, the promise of democracy is far from realised anywhere in the world. In other
words, democracy does not have a challenger, but that does not mean that it does not face
any challenges
Different countries face different kinds of challenges. These are:

 Transitional challenge: At least one fourth of the globe is still not under democratic
government. The challenge for democracy in these parts of the world is very stark.
These countries face the foundational challenge of making the transition to
democracy and then instituting democratic government. This involves bringing
down the existing non-democratic regime, keeping military away from controlling
government and establishing a sovereign and functional state.
For example: The Arab Spring protests across Middle East tried to usher in
Democracy in various countries by challenging the non-democratic institutions as
well as military rule, but the lack of political consensus led to a stalemate in the
protests and subsequently, traditional institutions continued to rule like in Bahrain.
 Challenge of Expansion: Most of the established democracies face the challenge of
expansion. This involves applying the basic principle of democratic government
across all the regions, different social groups and various institutions. Ensuring
greater power to local governments, extension of federal principle to all the units
of the federation, inclusion of women and minority groups, etc., falls under this
challenge. This also means that less and less decisions should remain outside the
arena of democratic control. Most countries including India and other democracies
like the US face this challenge.
 Deepening of Democracy: This is faced by every democracy in one form or another.
This involves strengthening of the institutions and practices of democracy. This
should happen in such a way that people can realise their expectations of democracy.
But ordinary people have different expectations from democracy in different
societies. Therefore, this challenge takes different meanings and paths in different
parts of the world. In general terms, it usually means strengthening those institutions
that help people’s participation and control. This requires an attempt to bring down
the control and influence of the rich and powerful people in making governmental
decision.

In Conclusion, No country is a perfect democracy, despite it being the dominant form of


government in the contemporary world. In every society, democracy faces several
foundational challenges—whether internal or external. These challenges can be successfully
met by political reforms especially by addressing the political culture in different societies.
There is no one-solution-fits-all solution, instead each society has to look within its own
traditional structures to strengthen the democratic values and institutions of the society.
Section B

5(a) Mention the major assumptions of the theory of balance of power.


Balance of Power is an important pillar of realist approach to international politics . It talks
about the maintenance of such a just equilibrium between the members of the family of
nations as should prevent any one of them from becoming sufficiently strong to impose its
will upon the rest. However, this equilibrium is subject to constant, ceaseless change while it
seeks to maintain a balance . In a balance of power system, the big states or powerful states
are the players. The small states or less powerful states are either spectators or the victims
of the game. National Interest is the basis of balance of power. The period of 1815-1914 is
considered as the golden age of Balance of Power. It was regarded as a nearly fundamental
law of international relations, until it broke down due the outbreak of First World War in
1914.
Like any other theory, the BoP theory is also based on certain assumptions . Two assumptions
are of central relevance. First, the international system is considered to be anarchic, with no
system-wide authority being formally enforced on its agents. Since, states do not have a
world government to resort to in a situation of danger, they can only try to increase their
capabilities relative to one another through either internal efforts of self-strengthening, or
external efforts of alignment and realignment with other states. Second, states are the key
actors in the international system. States are the unitary actors in the international system.
Moreover , BoP also assumes that the relative power position of states can be measured
with a degree of accuracy. “Balance” will either deter the threatening state from launching
an attack or permit the victim to avoid defeat if an attack should occur. The foreign policy
decisions are made on the sole basis of ‘power’ considerations.

According to Waltz, balance of power occurs when any state attempt to gain hegemony , other
states attempt counter coalitions . The rise of nuclear weapons , bipolar and unipolar world ,
the rise of non-state actors and the growth of complex interdependence has created challenge
for the concept of Balance of power.

5(b). Discuss the limitations of collective security system.


Unlike realists' state centric approach to security , liberals have proposed the idea of
‘international security’, as distinct from ‘national security’ in the sense that it is based on
belief that aggression can be best resisted by united action . This is defined as collective
security .

Under a collective security arrangement, aggression against any one state is considered an
aggressor against all other states. It is based on the idea of ‘one for all and All for one’.
Collective security is the liberal alternative to balance of power .
Under League of Nations, the collective security mechanism could not be operationalized .
Countries kept on following the policy of appeasement towards fascist powers. Thus,it is
argued that Collective security is based on false assumptions that states prefer international
peace over their national interest. Except for two occasions - 1950 Korean Crisis and Gulf
War I of 1989, the collective security under UN remained paralyzed because of ‘east west
conflict’. Countries continue to give primacy to the national interest over justice, peace or
order.

The concept of Collective Security is based upon certain idealistic assumptions which make
its operationalization difficult. For example:
1. It assumes that there can be a complete international understanding regarding the
nature of all threats or aggressions against international peace and security.
2. It is assumed that all nations could and would come forward to name the aggressor
and to take up collective security actions against the aggressor.

The fact is all nations are not active in international relations. Nor can all the
nations be expected to join a collective security action. At times it is not possible to
identify the Aggressor because, often the aggressor acts in the name of self-defence
and justifies its aggression as a defensive action.

3. Collective Security is self-negating in so far as it first denounces war or aggression as


an illegal activity and then indirectly accepts that wars and aggressions are bound to
remain present in international relations.
4. The concept of Collective Security makes it an international obligation of all the
nations to pool their resources and undertake collective action in the event of an
aggression. Many nations often prefer to remain away from war. It makes Collective
Security war an international obligation and wrongly assumes that all nations are
willing to participate in such a war.
5. The Collective Security system is difficult to operationalize in the absence of a
permanent forces. It is only after a decision of the Security Council to take military
action against an aggressor is taken that the constitution of a collective security
military force in initiated. This process is slow and difficult.
6. States don’t have an equal say in arriving at collective security decisions. In actual
operation, it fails to work on the principle of equality. Powerful states always
dominate collective security decisions and actions.

The concept has been defined as Eurocentric rather universalist. The post -colonial theorists
consider it as being inherently imperialist and culturally monistic. However, despite these
points of criticism, collective Security constitutes a modern device of crisis management.

5(c). Role of non-governmental organisations in global politics.Comment.


The liberal theory of international politics, especially the Cobweb model and Complex
interdependence models have recognised the importance of the role of transnational actors
like NGOs.However, the role of NGOs is still a matter of debate. Liberals recognize their
positive role. Even UN charter recognize the role of NGOs as consultative bodies attached to
ECOSOC. (Art 72 of UN charter). However, realists do not give importance to NGOs and
continue to believe that international politics is state centric.
According to Marxists, role of NGOs is not new. Along with East India Company came
Christian missionaries. The role of missionaries was to build the soft power. NGOs and MNCs
work in close nexus. MNCs channelize the corporate responsibility fund to these NGOs. Along
with MNCs, NGOs have been accused of attempting regime change.

While NGOs have many rights at the U.N., they do not have the right to vote in any of the
principal organs. However, NGOs do enhance democracy in global governance. It should be
acknowledged that NGOs from Amnesty International and Oxfam to Greenpeace and Save
the Children are now key players in global politics. It is because of the role of NGOs that we
have moved from intergovernmental relations, until 1990s towards global governance
model. At times , NGOs are seen by governments as threat to their security . For example –
Russia banned USAID , India also highlighted the negative roles of NGOs like Greenpeace.

5(d)What are the salient features of the school of social constructivism with respect to
global politics.
Constructivism’s arrival in IR is often associated with the end of the Cold War, an event that
the traditional theories such as realism and liberalism failed to account for.
Constructivism postulates that actors like leaders and influential citizens continually shape the
very nature of international relations through their actions and interactions.
Alexander Wendt argues that the social construction gives meaning to the material
structure. For constructivists, reality is always under construction, which opens the prospect
for change. In other words, meanings are not fixed but can change over time depending on
the ideas and beliefs that actors hold.
Constructivists argue that agency and structure are mutually constituted, which implies that
structures influence agency and that agency influences structures. They talk about
intersubjective structures .

While realists have argued that the anarchic structure of the international system determines
the behaviour of states. Constructivists, on the other hand, argue that ‘anarchy is what states
make of it’ . This means that anarchy can be interpreted in different ways depending on the
meaning that actors assign to it.
Social norms are also central to constructivism. These are generally defined as ‘a standard
of appropriate behaviour for actors with a given identity’ (Katzenstein).
Thus, for social constructivists school, the international system does not exist on its own . It
exists only as an inter-subjective awareness among people. It is a human invention or
creation of a purely intellectual and ideational kind. The system will change when there is
a change in the thoughts and ideas that enter into the existence of International Relations.
The intellect is the main guiding principle behind the conception, organization and use of such
cases in alliances, armed forces etc. The physical assets lose the meaning and utility without
the intellectual component. Nations, nationalism and national identities are social
constructions of time and place.

Constructivism’s ideas, when applied theoretically, has significant implications for how we can
understand the world. Constructivists offer alternative explanations and insights for events
occurring in the social world. Their focus on ideational factors shows that reality is not fixed,
but rather subject to change.

5(e)Compare Billiard Ball model of international politics with the Cob-web model.
To study international politics traditionally, the international system has been divided into a
collection of states, viewed as independent and autonomous entities. This state-centric
approach has often been illustrated through the so-called ‘billiard ball model’, which
dominated thinking about international relations in the 1950s and later, and was particularly
associated with realist theory. This suggested that states, like billiard balls, are
impermeable and self-contained units, which influence each other through external
pressure. Sovereign states interacting within the state-system are thus seen to behave like a
collection of billiard balls moving over the table and colliding with each other. In this view,
interactions between and amongst states, or ‘collisions’, are linked, in most cases to military
and security matters, reflecting the assumption that power and survival are the primary
concerns of the state. International politics is thus orientated mainly around issues of war
and peace, with diplomacy and possibly military action being the principal forms of state
interaction. The billiard ball model of world politics has two key implications. It suggests a
clear distinction between domestic politics and international politics.Sovereignty is the hard
shell of the billiard ball that divides the ‘outside’ from the ‘inside’. Secondly , it implies that
patterns of conflict and cooperation within the international system are largely determined
by the distribution of power among states i.e. not all billiard balls are the same size.
The billiard ball model has nevertheless come under pressure as a result of recent trends
and developments.There has been a substantial growth in cross-border, or transnational,
flows and transactions – movements of people, good, money, information and ideas. State
borders have become increasingly ‘porous’, and, as a result, the conventional
domestic/international, or ‘inside/outside’, divide is increasingly difficult to sustain.The
relations among states have come to be characterized by growing interdependence and
interconnectedness. States are forced to work together, relying on collective efforts and
energies. For Keohane and Nye, such a web of relationships has created a condition of
‘complex interdependence’, in which states are drawn into cooperation and integration by
forces such as closer trading and other economic relationships. This is illustrated by what has
been called the ‘cobweb model’ of world politics . In cobweb model, people are connected
with each other through multiple threats or channels. This model is seen as a more realistic
reflection of the "real world" than the billiard ball model.

6(a)Even after 75 years of the founding of UN , human rights look like an elusive dream.
Discuss the major limitations of global human rights regimes.
Human rights are rights inherent to all human beings, regardless of race, sex, nationality,
ethnicity, language, religion, or any other status.
Features of Human Rights

1. People have rights simply because they are human -


2. Human rights are universal -
3. Human rights treat all people as equal -
4. Belong primarily to individuals -
5. Human rights encompass the fundamental principles of humanity
6. The promotion and protection of human rights is not limited to national boundaries
but rather stipulates certain ideals that apply the world over
The Universal Declaration of Human Rights (UDHR), adopted by the UN General Assembly in
1948, was the first legal document to set out the fundamental human rights to be universally
protected. The UDHR continues to be the foundation of all international human rights law.
United Nations (UN) remains the central global institution for developing international norms
and legitimizing efforts to implement them. The primary mechanisms include UNSC action, the
UN Human Rights Council(UNHRC), , various rapporteurs, special representatives, and working
groups, War crimes tribunals, the International Criminal Court (ICC).

However, even after seven decades of existence of UDHR, Research like the 2019 Rule of Law
index shows that threats to human rights exist in ⅔ of the surveyed 113 countries. States,
especially in the south, themselves are involved in human rights violations either directly by
committing such violations intentionally or by failing to prevent such violations.
The United Nations Office of the High Commissioner for Human Rights gives a handful of
examples where Civil and political rights are being violated through genocide, torture, and
arbitrary arrests. The economic, social, and cultural rights gets violated in instances like
eviction of people by force from their homes, discrimination at work based on traits like ,
caste, race, gender, sexual orientation, insufficient minimum wages etc. There are other
instances of human rights violations due to Corruption, which compromises States’ ability to
fulfil it’s obligation to protect the human rights of individuals within it’s jurisdictions; due to
human trafficking; refugee crisis due climate change and armed conflicts; due to gender
inequality, due to discrimantion against LGBTQ+ community etc.
Due to institutional as well as resource limitations, enforcement remains inconsistent and
varies geographically. The institutionalization and implementation of different rights is
progressing at varying rates. The scope of human rights is ever widening. From Freedom from
slavery and torture to Right to clean environment , increasing issues are being asserted as
human rights. Thus , the organisations appear to lose focus. Furthermore, nations continue to
dispute the importance of civil and political versus economic, social, and cultural rights.
National governments sometimes resist adhering to international norms they perceive as
contradicting local cultural or social values.

The role of these human rights regimes is limited to publicizing abuses rather than applying
punitive measures. The global human rights governance architecture lacks binding clauses to
ensure that action matches rhetoric.

In the long run, multilateral institutions should premise their declaratory, diplomatic, and aid
policies on human rights parameters as the foundation. As rightly said by Kofi Annan, Former
Secretary-General, United Nations, “We all know what the problems are, and we all know
what we have promised to achieve. What is needed now is not more declarations or promises,
but action to fulfil the promises already made.”

6(b)The NPT which became effective in 1970 has achieved much in last 50 years. But the
final goal of complete disarmament does not seem to be on the horizon. Comment. Discuss
the policy options available with the international community to deal with the nuclear
crisis that may emerge from Iran?
The Treaty on the Non-Proliferation of Nuclear Weapons, commonly known as the Non-
Proliferation Treaty or NPT, is an international treaty whose objective is to prevent the
spread of nuclear weapons and weapons technology, to promote cooperation in the
peaceful uses of nuclear energy, and to further the goal of achieving nuclear disarmament
and general and complete disarmament. The treaty institutionalised the non-proliferation
norm by de-legitimising ‘proliferation’ (production and transfer) of nuclear weapons, fissile
materials and related technology by the non-nuclear weapon states (NNWS) while the
recognised five nuclear-weapon states (NWS) — namely the US, Russia, the UK, France and
China, can continue to possess nuclear weapons.

The NPT succeeded in its initial task of preventing the proliferation of nuclear weapons
among potential countries with sufficient industrial capabilities like Canada, West Germany,
Sweden, Switzerland, Japan, South Korea, and Australia . The NPT, over the years, also
succeeded in enticing the second tier countries, thus bringing the treaty to a near universal
adherence. NPT has been successful in promoting peaceful use of nuclear energy in the past
50 years , helped by the IAEA works in the field. However, in the last three decades, the NPT
started to show its limits. India and Pakistan went nuclear and North Korea has claimed to
have withdrawn from the treaty and started to build its own nuclear weapons. Clandestine
nuclear weapons programs had been proceeding in Iraq, Libya, and South Africa. NPT faces
another challenge from the Middle East. Even, IAEA safeguards proved insufficient in
uncovering nuclear weapon programs in a timely fashion. The experts believe that the
nuclear proliferation was prevented, not solely through the legal obligation of the NPT, but it
took US diplomatic pressure and persuasion to do so.

In the area of promoting nuclear disarmament, the NPT cannot claim to be very successful.
Even after 50 years of its existence, the achievement of the goal set in Article VI of the treaty
has a long way to go. As the disappointment of non-nuclear weapon states (NNWSs) about
the slow progress of nuclear disarmament intensifies, the adherence of those states to the
treaty obligation is under threat of being weakened.
The peaceful use of nuclear energy has been animated over the past 50 years, helped by the
IAEA works in the field. However, analysts believe that it was more driven by commercial
motivation . However, recent nuclear accidents have proved to be a reason for reluctance
towards this end and the safety of nuclear operations remains a supreme challenge.

The Nuclear crisis emerging out of Iran’s nuclear ambitions is another challenge to NPT and
put a question mark on its efficacy. Iran is a party to the NPT since 1970 but was found in
non-compliance with its NPT with respect to reporting of nuclear material imported to Iran;
reporting of the subsequent processing and use of imported nuclear material; declaring of
facilities and other locations where nuclear material had been stored and processed. The UN
Security Council in 2006, passed a resolution demanding that Iran suspend its enrichment.
Instead, Iran resumed its enrichment program at its Natanz facility .
Iran argues that it has a legal right to enrich uranium for peaceful purposes under the NPT,
and it had constantly complied with its obligations under the NPT and the Statute of the
IAEA. Iran states that its enrichment program has been part of its civilian nuclear energy
program, which is allowed under Article IV of the NPT.
In 2015, Iran negotiated Joint Comprehensive Plan of Action with the P5+1, a group of
countries that consisted of the five permanent members of the UN Security Council (China,
France, Russia, the United Kingdom, and the United States) plus Germany. The deal allowed
lifting sanctions on Iran in exchange for increased verification by the IAEA. However, On 8
May 2018, President Donald Trump withdrew the United States from the JCPOA and
reimposed sanctions on Iran. The Biden administration has not been able to make any
significant breakthrough on this matter. So, the issues remains intact.

Policy options available :


a) If Iran acquires nuclear weapons it will lead to new arms race in West Asia spiraling
to other regions. It can also result into the non-state actors acquiring nuclear
weapons. Hence, International community has to work together to bring US and Iran
on the table of diplomacy. There are different countries which can act as the
mediator including India.
b) There is a need to create the public pressure to stigmatise the acquiring of nuclear
weapons.
c) We should work together to achieve the time-bound program of comprehensive
nuclear disarmament.
d) Universal No-First Use agreement can address the security dilemma of non-nuclear
weapon states.
e) Conclusion of CTBT, FMCT can also act as a part of the solution

6(c)“Terrorism is the biggest problem confronting the world”- PM Modi. Comment . Do


you agree that the U.N. has ·failed to contain transnational terrorism?
The Sustainable Development Goals 16 aims to promote just, peaceful and inclusive
societies. It includes a target ‘to significantly reduce all forms of violence and related death
rates everywhere’ by 2030. However, terrorism appears to be the biggest global concerns
which may come in the way of achieving SDG 16.
Depending on the objectives , the nature of terrorism also differs. The major types of
terrorist operations commonly identified globally include:

 Ethno-Nationalist Terrorism , motivated by ethno-nationalist and separatist


aspirations . Examples - Tamil Nationalist groups in Sri Lanka and insurgent groups in
North East India .
 Religious Terrorism, motivated largely by religious imperatives
 Ideology Oriented Terrorism - Left-wing and Right-wing terrorism.
 State-sponsored Terrorism to obtain certain clearly defined foreign policy objectives. .
Countries like Iran, Iraq, Sudan, Libya North Korea have been engaged in sponsorship
of political violence of different nature in their ‘enemy’ countries.
 Narco-terrorism
 Nuclear terrorism
 Cyber-terrorism
 Suicide Terrorism - The assassination of Shri Rajiv Gandhi by LTTE.
According to Global Terrorism Database, 2019 there has been more than 20 thousands
death due to terrorism. Not all the damages done can be quantified.

A global concern of such magnitude requires strong intervention from United Nations.
The UN’s counter-terrorism work in recent years includes the development and promotion of
a Global Counter-Terrorism Strategy and efforts to counter violent extremism, a set of
international conventions, and far-reaching Security Council resolutions imposing
counterterrorism obligations on member states. UN has been striving for Capacity building
activities to help countries meet these obligations; and imposing Security Council-mandated
sanctions.

Starting in 1963, many international conventions have been negotiated under the UN’s
auspices criminalizing specific acts of terrorism. Some of these conventions, such as the 1999
Convention on Terrorist Financing, enjoy near-universal membership. However, none of the
treaties contains a monitoring and follow-up regime. The treaty framework constitutes a
necessary but insufficient condition for effective counter-terrorism.

Unfortunately, efforts to adopt an all-encompassing comprehensive counter-terrorism


convention have eluded the UN. This is because member states have been unable to agree
on a definition of terrorism, in particular on the questions of whether the definition should
include so called “state terrorism” and whether people under foreign occupation should
retain the right of violent resistance.

In the aftermath of 9/11 , Resolution 1368, adopted on 12 September 2001, established an


important precedent by invoking—for the first time—the right of self-defense against
terrorist attacks under Article 51 of the UN Charter, providing an international seal of legal
approval to the subsequent US invasion of Afghanistan. The Council unanimously adopted
the Resolution 1540, which requires all UN member states to take legislative and regulatory
steps to prevent terrorism.

While it is true that the UN’s operational counter-terrorism activities have faced severe
shortfalls and limitations, the UN has proven a useful venue for establishing the broad
normative and cooperative frameworks for collective counter-terrorism action. However, the
UN’s norm development has proven too weak to offset the negative effects of
counterproductive counter-terrorism policies by Member States that ultimately exacerbate
the terrorist threat.

Richard Barrett, the former head of the UN expert panel monitoring implementation of
sanctions against Al Qaida and the Taliban, concluded that “the U.N. is too political, too
uncoordinated, too focused on process rather than outcomes and follow-up, and too far
removed from the people who actually deal with the problems of terrorism on the ground
to make much of an impact, or even to appear relevant. “

7a) World Politics is a sum of regionalized geo-political developments. Elaborate. Do you


agree that cross regional linkages bring different dynamics to the regionalization of
politics?
Despite the revolution in communication technology and resultant shrinking of the 'global'
realm, regional geopolitics has always remained an important subset of global/international
politics, partly because of geo-cultural contiguities and close and shared historical
experiences.
Regionalism, even when it is parochially perceived, emphasises a will of togetherness and a
desire to overcome ethno-cultural prejudices in quest of the larger goal of working with
synergy to ensure mutual advantage and prosperity for a region as a whole. Some of the
prominent theories of regionalization are functionalism, sociological liberalism, and
economic inter-dependence. Though, primarily an economic phenomenon but cannot be
separated from the geo-political and geo-economic aspects for example: EU though seen as
a trade bloc was also a platform to limit expansion of communism. Though world politics
primarily a regional politics and the most challenging aspect of foreign policy is managing
the periphery yet it would be wrong to say World Politics is a sum of regionalized geo-
political developments alone.
With advancement of transport and communication technology and increasing economic
interdependence, we see the rise of cross- regional linkages also. Cross-regional linkages
bring in another dynamic to regionalisation of politics and mark it as a defining feature of
the same. A strong sense of regionalism should not be seen as opposite or in conflict with the
imagining of the world as one in global politics. With the increasing pace of information
technology (IT) revolution and increased use of AI, regionalized geo-political developments
still impact others in varying levels. It is impossible for any region or group of countries to
conduct their foreign policy in an insulated manner. Thus, many states are members of
different regional groupings and blocks that come together to maximise their security and
raise their economic status like BRICS.
Some of the examples of cross-regional groupings include BIMSTEC (Bay of Bengal Initiative
for Multi-Sectoral Technical and Economic Cooperation), Quad, APEC (Asia Pacific Economic
Forum) and SCO (Shanghai Cooperation Organisation) indicate the huge potential of cross-
regional linkages considering the global dimension of the problems faced by the
International community today.
In conclusion, regionalization of politics and cross-regional linkages are not isolated events in
the world politics. Instead they are the building blocks/ reality of the same.

7b) Discuss the primary role and the purpose of the UN. Discuss the attempts by
International community through the platform of UN to help in the post pandemic
recovery.
The United Nations (UN) is an international organization founded in 1945. It is currently
made up of 193 Member States. Its mission and work is guided by the purposes and
principles contained in its founding Charter and implemented by its various organs and
specialised agencies. Its activities include maintaining international peace and security,
protecting human rights, delivering humanitarian aid, promoting sustainable development
and upholding international law.
The UN was formed following the devastating World War II, with the aim of preventing
future global-scale conflicts. It was a successor to the ineffective League of Nations. The
representatives of 50 governments met in San Francisco on 25 April 1945, to draft what
would become the UN Charter. The Charter was adopted on 25 June 1945 and came into
effect on 24 October 1945.
Under the United Nations Charter, the role and purpose of the UN are:

 to maintain international peace and security in accordance with the principles and
purposes of the UN Charter;
 to investigate any dispute or situation which might lead to international friction;
 to recommend methods of adjusting such disputes or the terms of settlement;
 protection of human rights
 promotion of development

From the outset of the pandemic, the United Nations system mobilized early and
comprehensively. It led on the global health response, provided life-saving humanitarian
assistance to the most vulnerable, established instruments for rapid responses to the socio-
economic impact and laid out a broad policy agenda for action on all fronts. It also provided
logistics, common services and operational support to governments and other partners
around the world on the front lines of the pandemic, as they mounted national responses to
this new virus and unprecedented global challenge.
Now, as the pandemic is slowly becoming neutralized due to increased spread of vaccination
across the world, UN has stepped up efforts in the post-pandemic recovery as well. Measure
have been taken to fund these plans. This has been done through a three-pronged approach:
 A large-scale, coordinated and comprehensive health response, guided by the World
Health Organization (WHO) and its Strategic Preparedness and Response Plan,
which aims to mobilize all sectors and communities in the response, control and
suppression of the transmission of the virus, reduce mortality by providing care for
those affected, and develop safe and effective vaccines and therapeutics that can be
delivered at scale and that are accessible based on need.

A world where COVID-19 is no longer a threat to humanity requires the most massive public
health effort in history, that recognizes universal access to health as a critical global public
good. Part of this response is a new global collaboration – the Access to COVID-19 Tools
(ACT) Accelerator – the aim of which is to accelerate development, production, and equitable
access to COVID-19 tests, treatments, and vaccines. The UN has also provided international
coordination and operational support at the global, regional and country level, and
supported the scaling up of country preparedness and response operations.

 A wide-ranging effort to safeguard lives and livelihoods by addressing the


devastating near-term socio-economic, humanitarian and human rights aspects of
the crisis with attention to those hit hardest. The focus is on saving lives, keeping vital
services accessible, households afloat, businesses solvent, supply chains functioning,
institutions strong, public services delivering and human rights at the forefront. This
is achieved through immediate humanitarian support to the hardest-hit population in
the most vulnerable 63 countries with life-saving assistance through a Global
Humanitarian Response Plan (GHRP), as well as support to more than 120 countries
for an immediate socio-economic response guided by the UN development system
framework.

At global level, it includes the policy agenda contained in the series of policy briefs, as well as
strong advocacy for support to developing countries, including a debt standstill, debt
restructuring and greater support through the international financial institutions. Preventing
and responding to the increased levels of violence against women and girls is also a critical
feature.
 A transformative recovery process that leads to a better post-COVID-19 world by
addressing underlying fragilities and identifying opportunities for transformative
change towards more just, equal and resilient societies and economies. Emerging
from this crisis is an opportunity to address the climate crisis, inequalities, exclusion,
gaps in social protection systems and the many other injustices that have been
exposed and exacerbated.

The UN Secretary-General António Guterres addressing a Special Meeting of the UN


Economic and Social Council (ECOSOC) said, “With the COVID-19 pandemic exposing social
and economic inequalities rooted in racism, discrimination and xenophobia, recovery must
lead to more inclusive societies.”
Though UN took initiatives but the real will to cooperate has been missing. There is no
question over weakening of globalization and multilateralism. In the absence of political will
above policies will remain on paper.

7c) Discuss EU’s current approach towards US-China geo-political and geo-economic
tensions
Great power competition brings major issues, and few opportunities to the EU. Since 2016
the EU’s trust in the US and China –both indispensable partners– has eroded rapidly.
Meanwhile, pressure from Washington and Beijing has dramatically increased, in fields
ranging from defence to technology via trade. As the US and China find themselves locked in
spiraling tensions, European institutions seek to buttress their autonomy but face
considerable challenges in doing so.
A transatlantic bond in need of a ‘reset’
The US has been and remains the EU’s closest ally and security provider. While transatlantic
ties have never been immune to criticism, frustration and divergences, recent developments
have cast doubt on the nature and sustainability of this privileged relationship.
While at the administrative and technical level, transatlantic cooperation remains strong,
the quality of the political dialogue has dramatically eroded. This had an major impact on
the European debate over the Union’s ‘strategic autonomy’ as well as on its internal
cohesion over European security and its international outlook. Official declarations of
European leaders in favour of a stronger common defence agenda have grown more
numerous and more straightforward as relations with the US Administration apparently
deteriorated.
The EU’s growing frustration with China
From the beginning of diplomatic ties in 1975, EU-PRC relations have gradually expanded
into a ‘comprehensive strategic partnership’ comprising not only an annual summit and two
high-level strategic dialogues. Since 2016, due to the conjunction of at least three factors in
the bilateral relationship, the EU has grown more critical of China.
First, the perception within the EU of China as an unfair competitor has increased
significantly.
Secondly, European assessments of political developments in China proper converged on the
negative side of the spectrum. Under President Xi Jinping there is a growing concern over the
future of Hongong and increasing violation of human rights in Xinjiang and Tibet besides
hard-headed foreign policy

Thirdly, EU officials started to perceive Chinese diplomacy as undermining European unity.


Most visible was China’s participation to a sub-regional policy platform, best known as the
‘17+1’ format, with Central and Central-Eastern European countries, including 13 EU
member states.
Need for Strategic Autonomy

Russian aggression in Ukraine, the war in Syria, instability in Libya, the constant threat of
terrorism and the rise of new challenges such as climate change and cyber-attacks
constituted as many reasons for European countries and institutions to ponder about their
common foreign and security policy. The spiraling rivalry between China and the US further
fueled the process. Meanwhile, discussions over Europe’s ‘strategic autonomy’, and the
means to attain it, have multiplied.
Way Ahead
For the time being, the EU has adopted a reactive policy approach to the emerging
geopolitical tensions between the US and China and is trying to gain the balance as both are
indispensable trading partners for the EU. Brussels simply cannot sidetrack one, even if for
the benefit (or by pressure) of the other. The EU has proved willing to adopt a tough line
against unilateral tariffs from the US or market-distorting measures by China, if need be, and
to be more considerate about political and strategic issues in trade-related affairs. We
should not forget EU’s new strategic approach towards India.
8a. The decades long conflict between Israel and Palestine is rooted in the competing
claims to the Holy Land, disputed borders and issues concerning Palestinian refugees.
Elucidate.
The Israeli-Palestinian conflict is rooted in a century-long territorial dispute over the Holy
Land, a Middle Eastern region with great religious and historical significance to Christians,
Jews, and Muslims. Since the creation of Israel, the conflict is continuing, giving rise to many
other conflicts and rightly called as the “mother of all conflicts.” It has made Middle East one
of the most unstable regions.
The core issues of the conflict are:

Borders: The notion of having two separate states, one Israeli and the other Palestinian,
commonly referred to as the two-state solution, has had significant international support for
decades. It would establish a Palestinian state that includes most of the West Bank—with
land swaps to compensate it for Israel’s absorption of some Jewish settlements there—and
Gaza, which Israel unilaterally withdrew from in 2005. Most international diplomacy
promoting a two-state solution favors Israel’s reverting to a version of its pre-1967 borders,
but there is no consensus on how doing so could account for Palestinians within those
borders and Jewish Israelis living beyond them.
Jerusalem. The disputed city straddles the border of Israel and the West Bank. Israel has
annexed the whole city as its capital; the Palestinians claim East Jerusalem for the capital of
their state. A two-state solution would require a Palestinian capital in East Jerusalem.
Refugees. The wars in 1948 and 1967 created some one million Palestinian refugees. The
survivors and their descendants, mostly living in Jordan, Lebanon, and Syria, claim the right
to return to Israel, as supported by a 1948 UN General Assembly resolution. Debate
continues on whether survivors’ descendants should also be considered refugees with that
right. Israel sees the right to return as a threat to its existence as a Jewish state, and believes
the refugees should go to the Palestinian state that would be created as part of a two-state
solution.
Security. Israel views some Palestinian militant groups as existential threats,
particularly Hamas, the Islamist organization that governs Gaza and has vowed to destroy
Israel. Its suicide bombings and rocket attacks usually target Israeli civilians. Israel wants
these groups to disarm and the Palestinian state to be demilitarized, but accepts that
Palestinians should have a strong police force. The Palestinians seek an end to Israel’s
military occupation and want full control over their own security, but accept limitations on
their arms. Israel wants to maintain the ability to act in Palestinian territory against threats
to its security.
Mutual recognition. Each side seeks recognition of its state by the other, as well as the
international community. Most Israeli Jews want to see Israel recognized as a Jewish state,
while Palestinians want Israel to acknowledge their forced displacement under the Nakba.
Thus, the multiple issues add complexity to the problem. However, these issues should not
compel us to ignore the geo-political ambitions of the major powers behind the continuation
of the dispute.
Conclusion:
The outlook for Israeli-Palestinian negotiations is bleak. It’s highly unlikely that the
Palestinians will receive a package of concessions from Israel that is more favorable than
those that the Palestinians have rejected in the past. It is now suggested that the prospects
for a two-state solution will continue to fade, especially if the Israeli government annexes
settlements in the West Bank and the Jordan Valley, actions it agreed to suspend as part of
the Abraham Accord with the UAE. And many analysts like Steven A. Cook say the recent
Arab-Israeli normalization deals, as well as the 2021 Israel-Hamas conflict, have reduced the
likelihood of new talks between Israel and the PA. Meanwhile, political divisions between
Hamas and the PA remain a challenge to any future negotiated settlement.

8b. Differentiate between multipolarity and multilateralism. Discuss the future of


multilateralism in present context .
Multipolarity is a distribution of power in which more than two nation-states have nearly
equal amounts of military, cultural, and economic influence. On the other hand,
multilateralism refers to an alliance of multiple countries pursuing a common goal.
During the Cold War, the world was divided into two major power blocs, leading analysts to
call it a ‘bipolar’ world. With the disintegration of the Soviet Union and end of the Cold War,
the world witnessed a brief unipolar Pax-Americana moment, when analysts believed that
the United States of America (USA) had emerged as the sole dominant power at the global
level. However, with the emergence of China, Japan, Germany, India, Russia, Brazil, South
Africa, etc., as alternate power centres in different continents of the world, ‘multipolarity’, as
a concept, received wider scholarly attention. Today's world is thus conceived as multipolar,
multi-civilizational and multiplex, where no single country has the wherewithal to dominate
the affairs of the world to the disadvantage of others.
On the other hand, Multilateralism in the form of membership in international institutions,
serves to bind powerful nations, discourage unilateralism, and gives small powers a voice
and influence that they could not otherwise exercise. For a small power to influence a great
power, the Lilliputian strategy of small countries banding together to collectively bind a
larger one can be effective. Similarly, multilateralism may allow one great power to
influence another great power. In 1990, Robert Keohane defined multilateralism as "the
practice of coordinating national policies in groups of three or more states."
Multilateralism is largely a post-World War II element of U.S. foreign policy. After World War
I, President Woodrow Wilson proposed a truly multilateral organization — The League of
Nations — to prevent another such war. However, it is only after World War II, that U.S.
becomes involved in a flurry of multilateral arrangements. For Example: The World Bank and
the International Monetary Fund in 1944, The United Nations in 1945, The World Health
Organization (WHO) in 1948, North Atlantic Treaty Organization (NATO) in 1949 etc.
With the end of the Cold War, the multilateral system gained a new momentum as global
exchanges accelerated and more inclusive, multi-stakeholder approaches allowed for the
participation of new actors, including civil society organisations and private businesses. This
is also when the notion of “global governance” was first coined and started to gain traction,
as reflected in mechanisms such as the Commission on Global Governance created in 1992.

In 2002, Joseph S. Nye (Jr.) in his article Unilateralism v/s Multilateralism made an
interesting observation that “Multilateralism is a matter of degree. No country can rule out
unilateral action in cases that involve its very survival.” His argument highlights that
Multilateralism even as an approach to Peace in an anarchic world as discussed by Woodrow
Wilson, countries will always keep their national interests first.

More recently, however, it seems that multilateralism has entered a state of crisis or decline.
The postcolonial backlash against the predominance of the West and the rising tide of
populism has been challenging the liberal values underpinning the multilateral system.

Pursuing an aggressive “America first” policy, the Trump administration had relinquished the
traditional role of the US as herald of the international liberal order and withdrawn from the
Paris climate convention, the Iran nuclear deal, and the Trans-Pacific Partnership. China,
aiming to bend the multilateral system in its favour, is setting up parallel governance
structures such as the Shanghai Cooperation Organisation or the China Development Bank.
The EU, a traditional champion of multilateralism, is internally divided and losing influence
on the international scene. The G8 has shrunk to the G7, and a trade war is looming between
the US and China.
Concurrently, the UN and its manifold agencies have been losing their lustre, criticised for
their lack of efficiency, institutional sclerosis and ideological infighting. The WTO has failed
to conclude the negotiations of the Doha Agenda started in 2001, as bilateralism and
protectionism are resurging worldwide, and its dispute settlement system has stalled.
Multilateral efforts to address climate change have made symbolic progress at best.

In 2020, Shashi Tharoor said, “Tharoor said, "It's a perilous time for multilateralism because
all its great achievements since 1945 are being undermined by increasingly perverse
unilateral state behaviour.”

There is, however, a different, more optimistic story than the bleak picture of multilateralism
in crisis. It has been argued that far from being on the verge of collapse, the multilateral
system was merely undergoing a profound mutation. It now consists of a multilayered
system of regimes, agreements, networks, and initiatives. No longer the sole prerogative of
states, it is driven by a multitude of actors (IOs, corporations, civil society organisations,
advocacy groups, etc).
Prime Minister Narendra Modi in the 20th Summit of Shanghai Cooperation Organization
(SCO) Council of Heads of State called for “reformed multilateralism” that reflects global
realities of the present times and discussed topics such as expectations of all stakeholders,
contemporary challenges and human welfare.

8c. Why South China sea is called a ‘Geo-political hotspot’? Trace the evolution of the
dispute. Discuss the international laws on freedom of navigation and competing maritime
claims in light of the above dispute ?
South China Sea is an arm of western Pacific Ocean in Southeast Asia. This sea holds
tremendous strategic importance for its location as it is the connecting link between the
Indian Ocean and the Pacific Ocean. According to the United Nations Conference on Trade
And Development (UNCTAD) one-third of the global shipping passes through it, carrying
trillions of trade which makes it a significant geopolitical water body. According to the
Department of Environment and Natural Resources, Philippines, this sea has one-third of the
entire world’s marine biodiversity and contains lucrative fisheries providing food security to
the Southeast Asian nations. South China Sea is believed to have huge oil and gas
reserves beneath its seabed.

The region is turning to be a geopolitical hotspot due to the conflicting interests of the
United States (US) and China. China has territorial claims in this region and is not willing to
take a back seat. The US, however wants these waters to be free for navigation.

This conflict must be understood in the backdrop of International Tribunal for the Law of the
Sea (ITLOS) which came into existence following the entry into force of the United Nations
Convention on the Law of the Sea (UNCLOS), on 16 November 1994. UNCLOS, also known
as Law of the Sea Convention or the Law of the Sea Treaty is the international
agreement that resulted out of the third United Nations Conference on the Law of the Sea. It
defines the rights and responsibilities of nations with respect to their use of the world's
oceans, establishing guidelines for businesses, the environment, and the management of
marine natural resources.

Timeline of the Dispute:

 1994 – The Convention on the Law of the Sea went into effect. The United States
called this treaty the “Law of the Sea Convention.” In the same year, China occupied
Mischief Reef, located some 250 miles from the Philippine coast. Occupation was
made in the middle of an energy resources race in the Spratlys, where China lacked a
presence while the other countries were starting their oil exploration businesses.
Mischief Reef marked the first time when China had a military confrontation with the
Philippines, an ally of the United States.
 1997 – Beijing shared the first rendering of its “Nine-dash Line” extending roughly
1,118 miles from Hainan Island to waters off equatorial Borneo under China’s
historical claim of having it in the past.

 2002 – ASEAN and China signed the Declaration on the Conduct of Parties in the
South China Sea.

 2011-China, Brunei, Malaysia, the Philippines, Taiwan and Vietnam agreed a set of
preliminary guidelines on the implementation of the DOC (Declaration of Conduct of
Parties in the South China Sea) which would help resolve disputes

 2013 – The Philippines challenged China’s claims of historic rights and other actions
in an arbitration case under the Law of the Sea Convention.

 2016 – The Arbitration Tribunal ruled in favor of the Philippines and rejected China’s
maritime claims that go beyond the entitlements set out in the Convention.

Time and again US administration has asserted South China Sea as its core national interest
talked about observing rule based international order, maintaining the freedom of
navigation and warned against Island activities.
President Trump's administration increased the freedom of navigation operations in the
South China Sea region as a part of their dynamic force employment strategy. As of May
2019, the United States had conducted four freedom of navigation operations in the Spratlys
during early 2019. On 14 July 2020, US Secretary of State Mike Pompeo declared China's
claims and coercions of in parts of the South China Sea “completely unlawful”. Formation of
Quad should also be seen as a part of wider strategy.
Way forward:

The region will remain a geopolitical hotspot for great power rivalry between both USA and
China. The US wants China to maintain status quo in the SCS. This will help to bolster the US
performance in global governance by maintaining peace in the region. The disputes will
sustain until both sides sit together along with other regional actors to device a mechanism
for maintaining stability. The claimants of the SCS must resolve the conflict to promote
regional integration, stability and international peace.
PSIR Crash Course 2021
Model Answers - Test 4
For any issue related to PSIR Test Series, write only at psirtestseries@shubhraranjan.com

Section A

1(a) Trace the roots of ideological vision of India’s present foreign policy.
An effective foreign policy has at the very least a guiding vision , focussed execution and
continuous reassessment of tactics to support the vision. The underpinnings of the present
government’s foreign policy can be found in the election manifesto of BJP in 2014 – “The
vision is to fundamentally reboot and re-orient the foreign policy goals, content and
process in the manner that locates India’s global strategic engagements in a new
paradigm and on a wider canvas, not limited to diplomacy , but securing our economic ,
cultural , scientific and security interests.” Thus , the foreign policy of the present
government takes a holistic view .

The ideological vision of India’s present foreign policy is expressed by the concept of
‘Panchamrit’, the five cornerstones of the foreign policy as
1. " samman" (dignity),
2. "samvad" (dialogue),
3. " samriddhi" (shared prosperity),
4. "suraksha" (regional and global security) and
5. " sanskriti evam sabhayata" (cultural and civilisational links).
India's foreign policy so far has drawn from the doctrine of Panchsheel, enunciated by
Jawaharlal Nehru. Its five principles were mutual respect for each other's territorial
integrity and sovereignty, mutual non-aggression, mutual non-interference in each other's
internal affairs, equality and cooperation for mutual benefit and peaceful co-
existence.Panchsheel was formally codified into a treaty that was signed in 1954 between
India and China. While Panchsheel approached foreign relations largely in a spirit of
pacifism, Panchamrit is founded on an assumption that India is second to none in the global
arena and can hold its own against others. The present government is rebuilding the
partnerships across the board and across the new frontiers in our foreign relations. India’s
current engagement with most of the countries has been characterised by independence of
thought (geo-political dehyphenation) and action, and self-confidence that comes from our
heritage and draws strength from it.

1
It is the result of new approach that there is an optimism with respect to India as many
countries recognise it as the net security provider , anchor of the global economy and as a
leader in advancing peace and prosperity across the world. Prime Minister Modi's strong
belief in the shared future of our neighbourhood - 'Together we grow' - has resulted in
several concrete measures like SAGAR

Thus , the foreign policy of the present government trace its roots in the works of ancient
India’s master strategist Kautilya rather than the previous dispensations which took
inspiration from Buddhism . However, foreign policies of a country do reflect both,
continuity and change . Panchamrit and Panchsheel cannot be regarded as mutually
exclusive.

1(b) Define Strategic Culture. Does India have a strategic culture?


The concept of strategic culture as given by Ken Booth in 1990s, refers to a nation’s
traditions, values, attitudes, patterns of behaviour, habits, symbols, achievements and
particular ways of adapting to the environment and solving problems with respect to the
threat or use of force.

The strategic culture of a nation derives from its history, geography and political culture,
and it represents the aggregation of the attitudes and patterns of behaviour of the most
influential voices; these may be, depending on the nation, the political elite, the military
establishment and/or public opinion. It helps shape but does not determine how a nation
interacts with others in the security field. Strategic culture also helps shape behaviour on
such issues as the use of force in international politics, sensitivity to external dangers,
civil-military relations and strategic doctrine.
In 1992, George Tanham wrote an article: ‘Indian Strategic Culture’. In this article he said
that although India has great ambitions, it has never clearly articulated them. He further
argued that India’s values, cultural diversity, recent historical legacy and non-alignment as
a state policy, stymied the proliferation of strategic thought and could result in sub-
optimal responses to challenges faced on multiple fronts, particularly in areas related to
national security. However, this narrative faced considerable flak from Indian strategic
thinkers like K Subrahmanyam, who criticised it for its myopic and jaundiced view of higher
national security decision making.
Rejecting the arguments of George Tanham, Shiv Shankar Menon (former India's NSA )
made a powerful case arguing that India not only has a strategic culture of its own, but has
practiced it consistently in its bilateral relations and multilateral arena. Menon argued
that "since independence India has been an independent actor on the international stage
with a role, a diplomatic style, and a unique and recognisable personality all her own.”
Underlining that India has practised "the most frugal diplomacy, with a small band of
professional diplomats with minimum means to deliver all and more that much larger,
better equipped and well- funded foreign services do,” Menon argued that despite these

2
limitations Indian diplomacy has "resulted in India making quiet but substantive
contributions to the world in many ways.”
Dr. S. Jaishankar in his book , “The India Way” , in the chapter “Krishna’s choice :
The strategic culture of a rising power”, talks about the rich strategic culture of India
with reference to Mahabharata. Unlike Arthashastra , which has a clinical approach to
statecraft, he find Mahabharat as a vivid distillation of India’s statecraft. Bhagwat
Geeta mentions about the importance of courage in implementing policies .
Mahabharat talks about tactical compromises , Balance of Power , diplomacy ,
alliances. Mahabharat is a tale of ethics as well as power.
However, it is important to note that India strategic culture is rarely talked about or
discussed in the international community. The primary reason behind this is the lack of
focus on blending Theory with Practice in IR by Indian academicians. The IR studies in
India focused more on narrative and discourse rather than about the process itself. For
example: India’s relation with China has always been studied through lens of Realism or
Balance of Power but that narrative fails to explain the growing trade between the nations.
Also, there is a lack of interaction between the world of Academia and the government in
India which results in an incomplete blending of ideas when it comes to India’s take on the
International politics. On one hand, policies are formulated by the government agencies
without the input of academicians who have studied the issue for years and have a lot to
add to the same. On the other, the academicians churn out a lot of written work without the
input of actual practitioners.

These issues have led the International community to believe that India doesn’t have a
strategic culture. However, the reality is that far from that. In the words of Shiv Shankar
Menon, “My view is that this is an impossibility for a self- conscious culture and
civilisation such as ours, with our heritage and sense of our own importance and role. Just
as saying one is apolitical is itself a political choice, saying that India has no strategic
culture is only to say that it is different from the strategic cultures one is used to.”

1(c) There are no easy options for India as far as the complexity of the Israel-Palestinian
conflict is concerned. Critically analyse in context of India’s approach with respect to the
recent crisis.
India’s policy on the longest running conflict in the world has gone from being
unequivocally pro-Palestine for the first four decades, to a tense balancing act with its
three-decade-old friendly ties with Israel.
The balancing began with India’s decision to normalise ties with Israel in 1992, which came
against the backdrop of the break-up of the Soviet Union, and massive shifts in the
geopolitics of West Asia on account of the first Gulf War in 1990. That year, the Palestinian
Liberation Organisation (PLO) lost much of its clout in the Arab world by siding with Iraq
and Saddam Hussein in the occupation of Kuwait.
The relationship with Palestine was almost an article of faith in Indian foreign policy for over
four decades. For two-and-a-half decades from 1992, the India-Israel relationship continued

3
to grow, mostly through defence deals, and in sectors such as science and technology and
agriculture. But India never acknowledged the relationship fully.
It was during the government under Prime Minister Narendra Modi decided to take full
ownership of the relationship with Israel. The most important indication is the standalone
visit of PM Modi to Israel, first ever visit by Indian PM.
Recently, India strived to maintain balance between India’s historic ties with Palestine and
its improving relations with Israel by making a carefully drafted statement on the recent
outbreak of the violence in the month of May 2021. In the UNSC ‘open debate’ on the
ongoing violence in Israel, India expressed concern over the violence in Jerusalem and the
“possible eviction process” of Palestinian families in Sheikh Jarrah and warned against
“attempts to unilaterally change the status quo” in Jerusalem. The statement also reiterated
India’s “strong support for the just Palestinian cause and its unwavering commitment to the
two-state solution”. But India was careful not to upset Israel’s sensitivities. There was a
direct condemnation of the rocket attacks from Gaza but no direct reference to the
disproportionate bombing Israel has been carrying out on the Gaza Strip. India also did not
make any reference to the status of Jerusalem or the future borders of the two states, in
line with a recent change in its policy.
In conclusion, in recent years, India has been delicately balancing its relations with both
Israel and Palestine. It has already de-hyphenated its approach towards the conflict by
establishing independent strong relations with both the parties concerned. C. Raja Mohan
in his article ‘Making a case for Indo-Abrahamic Accords’ has argued that the growing
importance of India in the West Asia region can provide it with means of being a facilitator
for the conflict. He argued that India under Prime Minister Modi has been able to balance
close bilateral ties with Arab countries such as UAE and Saudi Arabia along with Israel. This
in turn provides a golden opportunity to “to India’s engagement with the extended
neighbourhood to the west.”
However, there are no easy options for India , as New Delhi has vital interests with the Arab
World on one hand and Israel and US on the other. As the faultlines in the region are
becoming more prominent , it may be difficult for India to navigate through the choppy
waters.

1(d) Describe the Challenges emerging out of EU’s new legislation “Fit for 55”, for India.

In May 2021, European lawmakers set the bloc’s climate goals in stone, legally committing
to reduce net greenhouse emissions 55% below 1990 levels by 2030, and to reach net zero
by 2050. On July 14, the European Commission introduced a set of 13 proposals— “Fit for
55″—that legislators will debate and possibly approve over the next two years. They range
from a total phaseout of gas-fuelled cars, to tighter caps on emissions from power plants,
vehicles, and farms, to new support for low-carbon aviation fuels, to a carbon tax on certain

4
imports, to new measures to increase carbon uptake by forests. Overall, the package
constitutes the world’s most detailed and ambitious set of climate change policies.
These proposals – some new and others revisions of existing laws are being heralded as a
game-changer.
The most controversial of these proposals is the carbon border adjustment mechanism
(CBAM), which would be the world’s first, requiring importers of certain goods to pay a tax
to access the bloc. The exact rules remain undecided, but the tariff would likely cover steel,
fertilizer, cement, aluminium, and electric power.
It would essentially extend the boundaries of the bloc’s existing cap-and-trade market—
which imposes an internal carbon price on those sectors—to its global trading partners.
Lower-emissions versions of those products—for example, steel manufactured with green
hydrogen—would face a lower penalty.
The idea behind the adjustment is to give European manufacturers that are bearing the
costs of legally mandated decarbonization a leg up over foreign competitors that are able to
produce products in a lower-cost, higher-emissions way like in countries such as China and
India.

This plan has already raised the hackles of officials in China and India, over concern that
manufacturers there could find themselves shut out of lucrative European markets. India is
Europe’s third-largest trading partner, and it does not have its own carbon tax or cap. A
UNCTAD study predicts that India will lose $1-1.7 billion in exports of energy-intensive
products such as steel and aluminium. This is a major hit especially when India’s total trade
with the EU was $74 billion in 2020.
In conclusion, CBAM is likely to become a main point of contention at the COP26 climate
summit in November 2021 and India needs to strongly oppose the same on grounds that it
is unfair to developing nations as a whole. As Minister Prakash Javadekar said “We are
paying, we are suffering from climate change which was caused by the reckless emissions
for hundreds of years by the developed world.”

1(e) Discuss the role of India Council of World Affair (ICWA) in formulation of India’s
foreign policy.
The Indian Council of World Affairs was established in 1943 by a group of Indian intellectuals
as a think tank. In 1955, the ICWA sponsored the establishment of the Indian School of
International Studies, currently a part of the JNU. By an Act of Parliament in 2001, the Indian
Council of World Affairs has been declared an institution of national importance. The Vice
President of India is the ex-officio President of ICWA.

It is devoted exclusively for the study of international relations and foreign affairs. Historic
international conferences like 'Asian Relations Conference' in 1947 under the leadership of
freedom fighter Sarojini Naidu and 'United Nations and the New World Order' in 1994 have

5
been held by this think tank in which world renowned dignitaries addressed huge gatherings
of intellectuals.
The ICWA was set up with a goal – that of encouraging the development of original
perspectives relating to international relations and area studies from a distinctively Indian
vantage point. From its very inception, ICWA has been holding periodical lectures,
meetings, and seminars to realize its goal. Besides Indian and foreign scholars and
statesmen, visiting dignitaries of many foreign countries addressed the meetings of the
Council over the years like Pt. Jawaharlal Nehru, Indira Gandhi, Henry Kissinger, Oscar
Fischer, Margaret Thatcher, Kurt Waldheim, V. K. Krishna Menon, Roy Jenkins, Morarji
Desai, Chandra Shekhar, President Ortega of Nicaragua, President Sam Nujoma of Namibia,
His Holiness Dalai Lama, Antonio Guterres and many others.
ICWA undertakes a lot of activities with a view to fulfil its goal of creating an India centric
understanding of IR. In this light, it published India Quarterly and Foreign Affairs Reports. It
is also the nodal agency for Track-II diplomacy funded by the Ministry of External Affairs.
The Africa Centre undertakes a variety of programmes for rekindling public interest in
Africa. The Secretariat of Council for Security Cooperation in the Asia Pacific (CSCAP) India
Committee has also been relocated in the premises of ICWA.
In this light, one can argue that ICWA is an extension of the Ministry of External Affairs and
therefore, hold an important sway in India’s foreign policy. It is the place where
academicians/ theorists meet the practitioners and influence their thoughts and processes.

2(a) India is a rising power that seeks to protect its widening interests and advance its
influence in international affairs. Comment. Is India’s foreign policy capacity
commensurate with it delivering on its greater ambitions?
India has been recognized as a rising power – on a trajectory to become a great power with
ambitions of being amongst the most powerful and influential states in the international
system. Jim O’Neill, the former head of Goldman Sachs, famously coined the term BRIC to
refer to four rising powers including India.

India’s trajectory towards becoming a great power can be described by the size of its
economy and its military capabilities. In 2019, Indian Gross Domestic Product (GDP) in
nominal terms was USD 2.6 trillion (IMF), making it the third largest economy in PPP terms.
Its military consists of 1.4 million active personnel and is the third largest after China and
the United States. In 2014, Indian Prime Minister Narendra Modi rightly described India’s
goal as taking a “leading role” in global affairs.
However, former Indian Foreign Secretary and National Security Advisor Shiv Shankar
Menon, argues that “India has serious capacity issues in the implementation of foreign
policy and lacks the institutional depth to see policy through.” According to him, India
shows boldness in policy conception, but caution in implementation. He traces
implementation problems to centralization of decision-making, lack of “capability” in the
foreign ministry, weak “institutionalization of foreign policy implementation”, and serious

6
“capacity issues”. India does not have adequate diplomatic manpower, diplomatic presence
, think tank culture . There is little participation of the Parliament in the formulation of
foreign policies. India does not have deep pockets like China . India’s work culture needs to
be changed as the impression is – ‘India promises , China delivers’.
On the other hand, in 2019, Kanti Bajpai has argued that “foreign policy capacity in India is
relatively strong at the level of individual competence and relatively weak at the
organizational and systemic levels.”
The balance sheet for India’s foreign policy after seven decades presents a mixed picture. A
changing world is clearly a more actionable one for an aspiring great power like India .The
existing gaps in India’s foreign policy capacity, mainly between conceptualization and
implementation, should be abridged effectively in order to match New Delhi’s rising
ambitions.

2(b) The shared geography has always made Afghanistan an important subject of mutual
interest for India and Iran. Comment.

In July 2021, External Affairs Minister S. Jaishankar stopped in Tehran, on his way to
Moscow amidst the ongoing crisis in Afghanistan. The shared interests of Iran and India in
Afghanistan have become a strong point of mutual interest and primarily emerges from the
geographical factors. Afghanistan is a mountainous landlocked country at the crossroads of
Central and South Asia. In the given geo-political context and considering India’s ever
acrimonious relationship with Pakistan , Afghanistan can be accessed by India only through
Iran.
Recently, Iran’s new President Ebrahim Raisi backed India’s role in the establishment of
peace and security in Afghanistan and highlighted the need for a joint plan to elevate
relations between Tehran and New Delhi.
As the Taliban returns to power, once again with the Pakistan Army’s support, India and Iran
need each other even more than before. If India and Iran worked closely with Russia against
the Taliban in the 1990s, Moscow now seems eager to embrace the Taliban. For Delhi and
Tehran, the contradictions with the Taliban are real.
India sees its ties with Iran as facilitating its pursuit of Indian national interests in
Afghanistan and, more broadly, in Central Asia and the Caspian Basin. Traditionally , in
Afghanistan, both New Delhi and Tehran have opposed the Sunni-dominated Taliban regime
and have a strong interest in the success of efforts to stabilize Afghanistan.
However, with respect to recent resurgence of Taliban in Afghanistan ,analysts are of the
view that India should have no expectations of complete unanimity of views with Tehran.
Sharing a long border with Afghanistan, Iran may be eager to keep its channels of
communication open with the Taliban. According.
To conclude, both have a common interest in ensuring a legitimate and peaceful
rearrangement of the current order in Afghanistan. They also have a stake in preventing

7
Afghanistan from becoming a safe haven for international terrorism. New Delhi and Tehran
can pool their resources in shoring up the current government against the Taliban’s
offensive backed by Pakistan.

2(c) India’s broad spectrum relationship with Israel has transformed into a strategic
partnership. Substantiate.

India’s broad-spectrum relationship with Israel transformed into a strategic partnership in


2017, a quarter century after the establishment of full diplomatic ties. India and Israel have
successfully steered the relationship forward, despite the baggage of fraught and convulsive
neighbourhoods.
The India-Israel relationship is often described as made up of “diamonds and
defense.”However, the broad spectrum of the relationship encapsulates a wide range of
engagements including trade&commerce, defence, agriculture , space technology, water
harvesting etc. From US$ 200 million in 1992 (comprising primarily trade in diamonds),
bilateral merchandise trade stood at US$ 5.65 billion (excluding defence) in 2018-19, with
the balance of trade being in India’s favour by US$ 1.8 billion. India is Israel's third largest
trade partner in Asia and seventh largest globally. Indian software companies, notably TCS,
Infosys, Tech Mahindra and Wipro, are beginning to expand their presence in the Israeli
market.
Indian investments in Israel (April 2000-June 2017) totalled USD 122.4 million .
India has benefited from Israeli expertise and technologies in horticulture mechanization,
protected cultivation, nursery management, micro- irrigation and post-harvest management
.
India is Israel's largest buyer of arms and the two countries have elevated their ties to a
strategic partnership. In February 2014, India and Israel signed three important agreements
on Mutual Legal Assistance in Criminal Matters, Cooperation in Homeland Security, and
Protection of Classified Material. India buys drones, LRSAM,MRSAM,Phalcom
AEW,Torpedoes, Comprehensive Integrated border Management Systems, Sniffer dogs etc
from Israel.
India-Israel cooperation in S&T is overseen by the Joint Committe on S&T, established under
the S&T Cooperation Agreement signed in 1993. During the visit of PM Modi in July 2017,
an MoU for establishing India-Israel Industrial R&D and Innovation Fund (I4F) was signed.
In popular Israeli perception, India is an attractive, alternative tourist destination. The
Government of Israel also offers short term summer scholarships for Indian students. India
offers ICCR scholarships to Israelis every year for various courses in Indian institutions.
There are approximately 95,000 Jews of Indian-origin in Israel , who are all Israeli passport
holders. There are about 14,000 Indian citizens in Israel, working as care-givers ,diamond
traders, IT professionals and students.

8
India and Israel look to increase their bilateral trade to $20 billion over the next five years
from the current level, with a focus on areas like urban infrastructure, digital technologies,
and agriculture, water management and counter-terrorism.

3.(a) Though Geo-politics and Geo-strategic interest take precedence, science, technology
and trade remain central to the strong Indo-US ties. Comment.
The changing relations between India and USA has been seen as two ‘estranged
democracies’ becoming ‘engaged ’. India-U.S. bilateral relations have developed into a
"global strategic partnership", based on shared democratic values and increasing
convergence of interests on bilateral, regional and global issues. The bilateral relationship
has been aptly described as “ChaleinSaathSaath: Forward Together We Go”, or
"SanjhaPrayas, Sab ka Vikas" (Shared Effort, Progress for All) adopted during the first two
summits of Prime Minister Modi and President Obama in September 2014 and January 2015
respectively. The summit level joint statement issued in June 2016 called the India-U.S.
relationship an “Enduring Global Partners in the 21st Century”.
While the India-United States (US) strategic partnership started in the second term of the
Bill Clinton administration, the defining moment in the relationship was the civil nuclear
agreement that the George W Bush administration championed and signed with India. Ties
got a further impetus with Barack Obama declaring support for a permanent seat in the
United Nations Security Council and the Joe Biden administration’s push to further the
strategic agenda between the two nations.
The geo-strategic and geo-political factors like rise of a ‘new’ China, more aggressive and
assertive internationally, pushed the US to give its relationship with India strategic
importance. The bipartisan support in the US Congress, the influential India-American
community, and the role of US business leaders who see India as the next big production
centre and market are other factors that played a role. For its part, New Delhi wants to grow
economically, militarily and geopolitically, and thinks that a stable and strategic relationship
with the US is in India’s interest.
Despite geopolitical and geostrategic interests taking precedence, other aspects of the
bilateral relations are slowly becoming important. Science and technology, and trade still
remains the focus in the India-US relationship. Both countries support cutting edge research
and are building public-private partnerships that promote science and technology-based
innovation and entrepreneurship. Over the past year, two bilateral endowments – the Indo-
U.S. Science and Technology Forum (IUSSTF) and U.S.-India Science and Technology
Endowment Fund – have provided roughly $3 million to fund hundreds of exchange visits
and support technology commercialization through U.S.-India joint ventures.
In 2019-20, the bilateral trade between the USA and India stood at USD 88.75 billion. The
USA is one of the few countries with which India has a trade surplus. India's trade surplus
with the USA increased to USD 17.42 billion in 2019-20 from USD 16.86 billion in 2018-19.
For the USA, India was the sixth largest supplier of services imports. India's large market,

9
economic growth, and progress towards development make it an essential market for USA
exporters.
However, a robust trade policy needs to be in place. Successive US administrations have
wanted to push a trade pact with India , which is more suitable to American interests.
However, India has been reluctant due to its domestic compulsions. In critical sectors such
as agriculture, reform in India will take time to overcome tariff and non-tariff barriers. For
the US, scientific collaborations and selling technology to boost productivity for Indian
farmers should take precedence over pushing dairy and other farms produce to the Indian
market. India also needs to open up its services, finance, legal, health care and
pharmaceutical sectors to US companies.
Today, the US goods and services trade with India is less than $150 billion compared to
nearly $630 billion with China. The indicates that there is a huge potential to boost bilateral
relations between the countries especially on account of increasing anti-China sentiment in
both the nations. Thus, the importance multi-faceted relations take precedence especially
when it moves beyond geo-strategic and geo-political relations. As technology continues to
take big strides in the world, innovation and trade will continue to grow leaps and bounds
between the two nations.

3(b). “Formulation of a foreign policy is a complex process mediated by the participation


of various stake holders”. Discuss in context of India.
Decision-making in foreign policy, is the most complex process, which needs multiple
specialized agencies, individuals and institutions to deal with range of issues of different
subjects. In context of India ,though, the Ministry of External Affairs is primarily responsible
to deal with the matters related to foreign affairs, agencies like, the Cabinet, the Prime
Minister’s Office, Ministry of Defence, the National Security Council, the Strategic Policy
Group and the Parliament play an important role in the formulation of India’s external
policy. They analyse the foreign policy issues and take decisions to address these issues
Ministry of External Affairs (MEA) is the primary institution of foreign policy decision-
making in India Since its formation, it has been successfully dealing with a spectrum of
activities related to foreign affairs of India. Though a variety of agencies and actors have
been involved in foreign policy decision making process, it is the MEA which takes the final
call on any of the issues and determine the course of foreign policy officially. The Policy
Planning and Research Division undertake research activities and is responsible for the
planning and formulation of India’s foreign and strategic policy. The division awards
research projects to the academia and think-tanks and organise seminars and conferences
on various issues of international affairs.

In most of the Parliamentary systems of the governments like India, the important decisions
on issues of national and international affairs are decided by the cabinet. Cabinet, being the
top most decision-making body of the government, determines the course of external
relations of the country by giving necessary directions. However due to the increasing

10
burden of the central cabinet, it unable to pay special attention to the significance national
politics and several specialized cabinet committees have been setup to assist the cabinet in
policy formulations.
The PMO, in the Parliamentary system like India, is one of the most significant institutions of
foreign policy decision-making. Though the management of the external affairs comes under
the Ministry of External Affairs, the decision of the Prime Minister remains the deciding
factor on any policy formulation. All important matters on foreign policy and national
security need the Prime Minister’s approval before implementation.

After the formation of the National Security Council (NSC) and with the appointment of the
National Security Advisor, the role of the PMO has increased . However, the role of the
PMO and the degree of his influence is very much contingent on his personality, leadership
quality and the stability of his position within the political system.
The three agencies - the National Security Council(NSC), Strategic Policy Group (SPG),
National Security Advisory Board (NSAB) and the Joint Intelligence Committee (JIC) have
emerged as major stake holders in the shaping of India’s foreign and security policies. It also
assists the government to formulate other significant policies that directly or indirectly
affects India’s national Interest, such as policies on economic and energy security.
Since, defending national territory and sovereignty is the main priority of the country’s
foreign policy. Hence, Ministry of Defence (MoD) has been one of the important
stakeholders of the Indian’s foreign policy decision-making machinery. Apart from the
Defence Minister, the Defence Secretary, the three Chiefs of the Staff and Secretary of the
Defence Production Division also participate in the decision-making process as the members
of Strategic Policy Group.

Parliament plays a significant role in foreign policy making in India as it enjoys complete
authority in making laws, for both domestic and foreign affairs, of this country. The
domestic policies too influence the foreign policies significantly. Article 246 of the Indian
Constitution which distributes powers between the Union Government and the states
authorize the Parliament to legislate on all aspects of external affairs of this country.
In conclusion, by analyzing the role played by the various stake holders in determining the
foreign policy of India, one can reflect on the complexity of forming a policy in the same.
Although, the above mentioned stake holders have well-defined role in foreign policy
formulation, but sometimes, their power and position overlap leading to a conflict of
interests. In such cases, the decisions by the PMO stands final on foreign policy
formulations.

3(C) With US withdrawal from Afghanistan, there is strong case for India to revive SAARC.
Critically Analyse.
US military withdrawal from Afghanistan has fuelled fears of intensification of trends of
ethnic, cultural and religious rivalries in Asia especially in South Asia. General Scott Miller’s

11
warning that “civil war in Afghanistan is certainly a path that can be visualised”, appears to
be materialising . Afghanistan has been a venue for two great wars. The first in 1979 when
Soviet forces invaded Afghanistan and second in 2001, when the US military went into
Afghanistan to “disrupt the terrorist base of operations,” and “attack the military capability
of the Taliban regime.” Two decades after the US invasion, UN reports suggest that “the
Taliban and Al Qaeda remain closely aligned and show no indication of breaking ties.”

In this worsening geo-political background , regionalism can be a confidence-building


measure to produce a common resolve to navigate common challenges. South Asian
Association for Regional Cooperation (SAARC), established in 1985, with the goal of
contributing “to mutual trust”, can be one such regional forums . However, despite
providing a framework for cooperation amongst South Asian nations, SAARC has remained
sidelined and dormant since its 18th summit of 2014 in Kathmandu.
In March 2020, Indian Prime Minister Narendra Modi seized the Covid-19 crisis and utilised
SAARC’s seal to convene a video conference of SAARC leaders. They underscored the need
for cooperation on a regional basis for fighting the pandemic. If the proceedings had not
taken place under the SAARC banner, leaders from the eight countries would not have come
together so readily on such short notice. Such capacity to bring member-states together
shows the potential power of SAARC.

In conclusion, as the largest regional cooperation organisation, SAARC’s importance in


stabilising and effectively transforming the region is becoming increasingly self-evident.
Allowing SAARC to become dysfunctional and irrelevant greatly distorts our ability to
address the realities and mounting challenges facing SAARC nations. The failure of South
Asian nations to act in accord in Afghanistan will plunge South Asia into a perilous theatre of
discord and escalating tensions with jihadi militias at the forefront, placing the entire region
in turmoil. SAARC is needed as institutional scaffolding to allow for the diplomacy and
coordination that is needed between member-states in order to adequately address the
numerous threats and challenges the region is facing from the threat of the rise of Taliban
2.0 in Afghanistan.

4(a) Perceptions, Perimeters, Partnerships and Power shape India-China relations.


Elucidate.
Kanti Bajpai in his book, “India Versus China: Why They Are Not Friends,” writes that “India
and China are not friends for four key reasons: deep seated differences over their
perceptions of each other, their territorial perimeters, their strategic partnerships and,
most importantly, the asymmetry of power.”

India and China’s mutual perceptions have changed pretty significantly over the centuries.
China looked up to Buddhist India during the medieval period. A millennium later, India held
Imperial China in great regard. The situation changed from the 19th century onwards.
China’s respect for India declined, whilst India began to see it through British eyes.

12
In recent years, the bilateral perceptions have deteriorated further. On one hand, China
from its position of strength does not view India as a hegemonic rival. It does not see the
need to accommodate India on various issues. On the other hand, India, as a rising power
feels the need to push-back Chinese ambition in order to grow as a regional power. India
feels that succumbing to Chinese pressure would lead to erosion of strategic autonomy and
thus, continues to project China as a looming enemy in the periphery.

In this context, one can analyse the power asymmetry between the two nations. When it
comes to soft power, China using its economic might has been able to influence several
countries in various regions. India on the other hand is lagging far behind in terms of
investments or economic reach. However, even in this battle, all is not lost for India, as it
has been able to create a sense of good will amongst many countries in the world through
diplomatic aid and other means. Nonetheless, in a battle between economic might and
good will, one can safely bet which one would trump the cards.
When it comes to hard power, there is a clear disparity which is obvious. Although India has
the larger army, China has three-and-a-half times more aircraft, three times more
submarines and twice as many nuclear weapons. But it’s the difference in indigenous
defence production and China’s ability to produce new technologies that gives Beijing the
decisive edge.

The disparity in hard power is obvious when it comes to Chinese aggression of India’s
borders especially in Arunachal Pradesh and Ladakh. The lack of agreement on the border
lines have continued to make Perimeter as a matter of concern for both the countries.

This brings us to the last aspect: Partnerships. When it comes to India-Chinese relations,
bilateral trade is one of the biggest features. Despite tensions in Galwan and Ladakh regions,
in 2021, Bilateral trade between India and China grew over 62 per cent. The total bilateral
trade between the two countries increased to $57.48 billion in the first half of 2021.
In conclusion, the four aspects of Perception, Power, Partnership and Perimeter govern the
relation between India and China. The growing disparity between the two countries in these
parameters has led to a very limited prospect of a lasting rapprochement.

4(b). Discuss the evolution of India’s policy on foreign aid.


Since Independence, India has had an interesting relationship with accepting foreign aid. Till
about 2003, India needed aid but could not dictate terms with the donor. Unsurprisingly,
this resulted in a lot of tied aid. For example: When US gave PL-480 assistance, it made it
clear that it was political in nature and based on quid pro- quo.
India is still a big recipient of foreign aid. It is just that more and more of this is now
multilateral, and project-specific assistance meant for infrastructure and social spending.

In January 2001, following the earthquake in Bhuj, the then Prime Minister Atal Bihari
Vajpayee issued an earnest international appeal for help which scores of countries

13
responded to. Interestingly, this was the last time a formal appeal to the international
community for help went out from the Government of India.
By June 2003, however, things changed. The then Finance minister Jaswant Singh, in his
Budget speech, made a policy statement in which he made clear that India would no longer
accept tied aid. One reason was behind this decision was that the donors often commented
on what could be deemed internal affairs of India. The other reason was purely functional.
There were 22-odd countries which gave small (relatively) amounts of assistance and it was
becoming difficult, if not impractical, to coordinate all of it. So, in 2003, India requested all
these countries to route their assistance through non-governmental organisations (NGOs),
the United Nations (UN) and multilateral institutions. These continue to be preferred even
today.
India also took a strategic decision in 2003 that bilateral aid will be accepted only from the
United Kingdom, the US, Russia, Germany, Japan and the European Union. It was also
decided that debt owed to India by heavily indebted poor countries, including Ghana,
Mozambique and Tanzania will be cancelled outright. India also repaid debts amounting to
$1.6 billion belonging to several countries.
In 2004 ,the Indian government officially stopped accepting foreign aid for disasters.
Therefore, during the Indian Ocean tsunami, the government not only released $115 million
for the National Contingency Fund, but also refused any humanitarian assistance from
Japan, the US and Israel. Former Prime Minister (PM) Manmohan Singh said, “We feel that
we can cope with the situation on our own.”

In March 2008, a position paper prepared by Department Of Economic Affairs set out the
fundamental parameters of India’s policy on receiving foreign aid. The paper included the
following important observations.

 India was no longer reliant on external assistance or foreign aid for financing its plan
outlays or for gross capital formation.
 The economy was strong enough to move away from tied aid.
 The government would not accept aid in areas where it had substantial control.
 All countries could provide bilateral assistance directly to autonomous institutions,
NGOs and UN agencies, among others.
 Shift in sectoral focus. While, in the early years, it was about agriculture and food
aid, it now shifted to infrastructure and social sectors such as health and education.
 Foreign aid was skewed in favour of states such as Gujarat, Andhra Pradesh and
West Bengal but states like Bihar hardly got any assistance.
 In terms of composition, loans dominated and there were hardly any grants.
 There was preference for multilateral sources rather than bilateral sources of
funding.

These principles still constitute the blueprint for India’s policy regarding foreign aid. The
policy has a fair amount of flexibility and confers discretion to the central government. This

14
is reflected in the National Disaster Management Policy of 2015. If the national
government of another country voluntarily offers assistance as a goodwill gesture in
solidarity with disaster victims, the Centre may accept the offer. This policy was again tested
in 2018 when floods ravaged Kerala and the United Arab Emirates (UAE) offered bilateral
assistance.

In 2021, amidst the disastrous second wave of Covid pandemic, India began accepting
assistance from foreign countries. Many scholars argued that this was a policy shift in India’s
acceptance of foreign aid. However, this is not true and the action was in line with the fine
details of Indian policy on foreign aid . It can be argued, therefore, that there has been no
fundamental change in the government’s policy when it comes to accepting foreign aid.

3(C). Explore the place of central Asia in India’s Extended Neighbourhood Policy
Central Asian Region (CAR) is considered to be the part of India’s ‘extended neighbourhood’.
Central Asia of today consists of five nations namely- Kazakhstan, Kirghizstan, Tajikistan,
Turkmenistan and Uzbekistan. All these five nations became independent after the collapse
of USSR in 1991.

The key elements which outline India’s Central Asia policy today, constitutes Political,
Economic and Strategic Cooperation. Political cooperation involves building strong political
ties through high-level engagements and interaction between leaders at bilateral and
multilateral levels. Prime Minister Narendra Modi’s visit to all five Central Asian countries
last year in July 2015, can be seen as a part of this political cooperation.
The cooperation becomes even more vital when one looks into India’s need for oil and
energy. India looks at Central Asian region as a provider of its needs for natural resources.
For that to happen, India needs to have strong political ties with these nations. India wants
to make up for its low prioritization of this region following the collapse of erstwhile USSR,
which these Central Asian nations were a part of.

The point which is noteworthy is that political engagements can provide a solid base for
developing good economic relations. India being one of the largest agrarian economies is
looking forward to cooperate in production of profitable crops with value addition in the
crop quality. This would be beneficial for Central Asian region.
Some international relations experts say that “compulsion drives policy” and this phrase
looks perfectly apt for India’s involvement with the region. Indian companies now are in
search for new markets where they can expand their business, market experts are of
opinion that Central Asia has a huge untapped potential.
Dr. Meena Singh Roy from IDSA (Institute for Defence Studies and Analysis), opines that
“After 1991, India has constantly worked towards building strong political ties with all
central Asian countries. However, economic and trade linkages with the region have been
the most unsatisfactory part of its overall fruitful political relations. While Central Asia
continues to be of great strategic significance to India, its engagement with the region has
been moderate as compared to China and USA.”

15
India needs to both refine and redefine its policies with respect to the region because the
geo-politics of this resource rich region is changing very fast. Keeping aloof from the region
would prove very costly to India’s ambition of gaining a high seat among the great powers
of the international system. India has already committed the mistake of keeping itself
away from the region by not prioritizing it in the last two decades after the collapse of
USSR in 1991.

India’s Connect Central Asia Policy has a forward looking orientation which means it looks
for a better future for India-Central Asia relations and at the same time promoting India’s
Geo-strategic as well as Geo-Economic interests in the region.

16
Section B
5(a) India need to re-examine it’s Tibet policy. Comment.
The importance of the Tibetan plateau to Asia in general and India in particular, is
threefold: geopolitical, cultural and environmental.
Tibet’s strategic location made it a natural buffer between India and China, but in 1954 India
signed an agreement with China, agreeing to trading terms on what it called the “Tibet
region of China”. With this India lost its natural advantage and since then the official Indian
policy is that the Dalai Lama is a spiritual leader, and the Tibetan community in India is not
allowed to undertake any political activity.
There is now a recognition across capitals that post-Dalai Lama politics of Tibet will emerge
as a key geopolitical fault line and thus the Indian position needs to re-examined .
Suhasini Haider gives convincing arguments in favour of an overhaul in India’s Tibet policy.
According to her, over the past few decades, the approach of Chinese government has also
changed in many ways. Beijing is pouring investment in infrastructure projects, pouring in
Han or mainland Chinese, in an effort to Sinicize the population. In its White Paper on Tibet
, the Chinese government has made it clear that it intends to tighten its controls in Tibet.
Moreover, Chinese are aggressively building dams on the upper riparian areas of the
Brahmaputra, and constructing Tibetan villages along the LAC. Post the Galwan clashes ,
China has begun to raise Tibetan Militia groups to counter the Tibetan Special Frontier
Force, which the Indian Army trains .
The question over the future of the Tibetan community in India also needs a rethink . The
government doesn’t give citizenship to Tibetans born in India after the cut-off year of 1987,
leaving the youth of the community living in segregated communes in different parts of
India and with no rights. The larger question is over the succession to the 86-year-old Dalai
Lama, who has been not only the spiritual leader, and the leader of the Gelugpa sect of
Tibetan Buddhism, but the political leader of the community as well. If China announces its
own Dalai Lama, who gains legitimacy. It might be a complex political situation for India. The
young and restive Tibetan population that resides here, may start looking outside of India
for its leadership and command structure .
All these factors points towards a major recalibration which is required towards India’s Tibet
policy .The government needs a proactive policy that takes into account these new realities.

5(b).Myanmar as the lynch-pin of India’s Act East Policy. Discuss.


India’s ‘Act East’ policy(AEP) is a diplomatic initiative to promote economic, strategic and
cultural relations with the vast Asia-Pacific region at different levels. The country’s eastward
drive since 1992 has underscored the importance of this region in its contemporary
international relations. The policy has often been depicted as ‘the cornerstone’ of India’s
engagement towards east and south east.

17
Myanmar is the only member-state of ASEAN that has both land and maritime borders with
India. In the backdrop of Act East Policy , Myanmar is a nation of considerable importance as
a partner capable to cooperate for the security and development of our Northeast and as a
strategic ‘buffer state’ in the context of China. It is an attractive market as well as an
economic partner.

Myanmar is a key pillar to achieve objectives of the Act East Policy in promoting economic
cooperation, cultural ties, and developing a strategic relationship with countries in the Asia-
Pacific region through continuous engagement at regional, bilateral, and multilateral levels.
Myanmar and India share a strong cultural ties. In his book India-Myanmar Relations:
Changing contours Rajiv Bhatia argues India’s relations with Myanmar may be explained
primarily in terms of ethnicity and religion. The Buddhist connection has been the most
compelling and lasting bond between the two countries.
Act East involves development of North-Eastern Indian states , as one of its goals .Projects
like Kaladan and Trilateral Highway , which are based in Myanmar have contributed to this
end. Myanmar acts as a gateway to explore the alternative to traditional business partners
in Pacific and South East Asian region
Being the only country that sits at the intersection of India’s “Neighborhood First” policy and
its “Act East” policy, Myanmar is an essential element in India’s practice of regional
diplomacy and serves as a land bridge to connect South Asia and Southeast Asia.
According to Pralash Nanda (Rediscovering Asia: Evolution of India’s Look East Policy), the
Act East policy has recognized the centrality of Northeast to the achievement of India’s
national interests and objectives in this region and beyond , and Myanmar’s strategic
location makes it a potential powerhouse for India.

5(c). Enumerate the achievements in India-Latin America relations.


Latin America and the Caribbean (LAC) is a region comprising of three principal sub-regions,
viz. the South American continent, Central or Meso-America from Mexico to Panama and
the Caribbean.Until recently LAC was considered a ‘forgotten continent’ for India but it is
slowly emerging as a ‘Next Big Thing’. In July 2014, just one month after his election, Prime
Minister Modi headed to Brazil to attend the annual BRICS summit.

Latin America is becoming increasingly important to India, particularly in the economic


arena. With two-way commerce slightly below the US$ 40 billion-mark, current trade is
indeed still the business of just a handful of products. However, in the last decade, trade has
grown twenty-fold and bilateral investment is on the rise. Both regions rely on cooperation
programs in different areas, but opportunities are still growing. Agriculture, aerospace
cooperation and renewable energy are some examples of the sectors in which effective
cooperation could benefit all participants.
Latin America has also emerged as a key contributor to India’s energy security. India now
imports 20% of its crude oil from Brazil, Columbia, Mexico and Venezuela by some
estimates. India’s private sector has also invested billions of dollars into Latin America,

18
sharply increasing trade flows and expanding India’s global brand. The country’s leading
companies have been gaining presence in the region. India’s steel giant, Jindal Steel &
Power, has invested $2.3 billion in an iron ore mine in Bolivia, the largest foreign direct
investment project in Bolivian history. In Trinidad and Tobago, Essar Steel is in the process
of constructing a 2.5 million ton steel plant. More than one hundred Indian companies have
invested over $12 billion in Latin America across a wide variety of industries, including
mining, metals, agriculture, petrochemicals, pharmaceuticals, plastics and plane parts.
India also constitutes one of the largest suppliers of Information Technology (IT) services to
Latin America Of the fourteen Indian companies operating in Argentina, half are focused on
IT-related services. According to some estimates, over 35,000 Latin Americans are now
employed at Indian IT companies operating in the region. New Delhi is also actively
promoting official policies intended to further expand Indian IT services in Latin America.

Although India has steadily expanded its footprint there over the past several years, it is still
dwarfed by China’s immense presence in the region. As a result, India’s approach to Latin
America is considered more measured and cautious. Although some regard India’s approach
as too timid, others view it as strategic and calculated.

5(d) Explain the importance of NAM 2.0.

Non-Alignment 2.0 is an attempt to identify the basic principles that should guide India’s
foreign and strategic policy over the next decade. The phrase “non-aligned” was first used
by V K Krishna Menon at the United Nations General Assembly in 1953 and by Pandit
Nehru in 1956. However, the strategy that lay behind the phrase had been spelt out by
Nehru much earlier, first in Constituent Assembly debates and later in Parliament. The very
sense of India, with its history and civilisational attributes, he said, demands the pursuit of
an independent foreign policy. Decisions relating to India’s vital interests should not be
externally determined. Maintaining and, if possible, expanding the country’s strategic
autonomy is a continuing objective.

The objectives of NAM 2.0 are three-fold: to lay out the opportunities that India enjoys in
the international sphere; to identify the challenges and threats it is likely to confront; and
to define the broad perspective and approach that India should adopt as it works to
enhance its strategic autonomy in global circumstances. The essence of NAM 2.0 has not
changed much from the logic of non-alignment, but they are to be applied in a vastly
transformed international landscape and at a time when India itself is being transformed.
India must continue to seek strategic autonomy but through a more contemporary
reinterpretation and application of the principles of non-alignment. That is what “Non-
Alignment 2.0” seeks to do. In a situation where the world is no longer bifurcated between
two dominant powers, nonalignment today will require managing complicated coalitions
and opportunities in an environment that is not structurally settled.

While none of these may be new, their enunciation in an integrated framework is valuable
in building a comprehensive understanding among policy makers, directly interested

19
participants, and the general public. The context of NAM2.0, thus sets the ground for an
extensive public debate on an Indian grand strategy that can best be described as a work in
progress.

5(e) Write a short note on dynamics of India’s vaccine diplomacy.


For over two decades, India has acquired the reputation of being the “pharmacy of the
world”. In early 2021, India-driven by its ‘Neighbourhood First’ policy and in its
understanding of its role as the ‘net security provider’ of the region-began providing COVID-
19 vaccines on a priority basis to its immediate neighbours. In keeping with its historical role
as a provider of affordable medicines, India has taken two significant initiatives to overcome
the COVID-pandemic. The first is to make vaccines widely available, which is in response to
the growing evidence of the criticality of making COVID-19 vaccines accessible to all. The
second initiative is a joint proposal tabled along with South Africa in the World Trade
Organization (WTO), which seeks temporary waiver from the implementation and
enforcement of four forms of intellectual property rights (IPRs). The objective is to ensure
that COVID-19 vaccines, medicines and other medical products are freed from the
encumbrances of these IPRs, thus making them affordable. The twin initiatives underline a
key message for the global community: in pandemic times, medical products must be
treated as global public goods.
India lifted its ban on exports of the drug HCQ, when there was demand for it, sending
medical teams to countries in the neighbourhood, and then for its massive Vaccine Maitri
programme, under which India exported COVID-19 vaccines to over 95 countries worldwide.
India is also at the forefront of multilateral diplomacy efforts, including the Quad plan for
production of one billion vaccines to be offered to South East Asian countries.
India’s efforts are in sharp contrast to the United States, which has through the use of
executive orders and its Defense Production Act, made domestic production and use of
COVID-19 vaccines and pharma supplies its priority, refusing exports at present. PM Modi,
while speaking at the Raisina Dialogue, said that India has "walked the talk" on the global
COVID-19 effort, pointing to other countries that haven’t.

However, the vaccine diplomacy came under severe criticism during the second wave in
India . Firstly, for not anticipating the crisis and sending out the wrong message
internationally that India had defeated the coronavirus. Secondly, for starting the Vaccine
Maitri programme at exactly the same time as the domestic vaccination programme began,
without properly estimating the need or the urgency of vaccinating the whole population.
Nevertheless, India stood up as a net security provider, as far as health security is
concerned. India proved to be a good global citizen.

20
6(a) One of the signs of India’s growing centrality in Indo-Pacific security architecture is its
burgeoning engagement with the key western countries. Illustrate.
The Indo-Pacific is an integrated theatre that combines the Indian Ocean and the Pacific
Ocean, and the land masses that surround them. Its geographical expanse is still undefined
but it is said to range from the coast of East Africa, across the Indian Ocean, to the
Western Pacific, including countries like Japan and Australia. It is both a strategic as well as
an economic domain comprising important sea-lines of communication that connect the
littorals of the two oceans. Since it is primarily a maritime space, the Indo-Pacific is
associated with maritime security and cooperation. The Indo-Pacific region is a vast
maritime zone where the interests of many players are engaged: India, Japan, France, and
the United States, as well as medium and smaller powers like Australia, Indonesia, and
South Africa; there are stakeholders from beyond the region, too. During a visit to India in
2007, the former Prime Minister of Japan Shinzo Abe talked about the “confluence of the
two seas,” meaning the Indian and Pacific oceans, and the creation of a “broader Asia”.
In the ‘Contest for the Indo-Pacific: Why China Won’t Map the Future’, Rory Medcalf writes
that “The Indo-Pacific, is unified by the quest to balance, dilute and absorb Chinese power.
It is both a region and an idea.”
India’s definition of the Indo-Pacific region stretches from the western coast of North
America to the eastern shores of Africa. This region is being called ''the geopolitical centre
of the world.''

The vast Indo-Pacific region comprises at least 38 countries, shares 44 percent of the world
surface area, is home to more than 64 percent of the world’s population, and accounts for
62 percent of the global GDP with more than 50 percent of global trade traversing through
its waters. The region is highly heterogeneous with countries at different levels of
development connected by a common thread of ‘the ocean’.
In 2019, at the East Asia Summit in Bangkok, India announced the Indo-Pacific Oceans’
Initiative (IPOI) to support the building of a rules-based regional architecture centred on
seven pillars — maritime security; maritime ecology; maritime resources; capacity building
and resource sharing; disaster risk reduction and management; science, technology and
academic cooperation; trade, connectivity and maritime transport. IPOI is anchored on
India’s ‘Act East’ (focusing on the Eastern Indian Ocean and the Western Pacific) and ‘Act
West’ (focusing on the Western Indian Ocean) policies.
India has also built partnerships and collaborated with likeminded countries in the region
that have shared values and common goals — from the Pacific Islands to the archipelagos of
the western Indian Ocean and off the eastern coast of Africa; to networks such as the
Quadrilateral Security Dialogue (QUAD) with the US, Japan and Australia, the Supply Chain
Resilience Initiative (SCRI) with Japan and Australia as participants, and the India-Japan-US,
India-France-Australia and the India-Indonesia-Australia trilateral arrangements.
India and the U.S. have a common interest in promoting security and prosperity in the Indo-
Pacific region and their ability to work together has a positive impact in effectively

21
addressing regional and global challenges. The two countries are defence partners. The US
designated India as ''major Defence Partner'' in 2016 and signed the LEMOA (Logistics
Exchange Memorandum Agreement). Through it, America and India can now use each
other's bases for repair and replenishment of supplies. In October 2020, India and the US
signed the BECA (Basic Exchange and Cooperation Agreement) to boost bilateral defence
ties.Both India and the United States have a common challenge, China.

Similarly India is at the heart of the UK’s Indo-Pacific ‘tilt’. New Delhi has come to the
realisation that, alone, the US is incapable of constraining China’s most damaging
tendencies and so increasingly recognises the value of partnerships with mid-sized
democratic powers in building a resilient order in the Indo-Pacific. Compared to its ties with
Japan, Australia and France, the UK remains a relatively untapped strategic partner for
India—a fact that is driving New Delhi to invest more political capital into the relationship.
India and France too explored ways to strengthen cooperation in the Indo-Pacific, including
under a trilateral mechanism with Australia to address emerging challenges in the maritime
and space domains.
India is actively engaging Japan,the first countries to ideate the notion of an “open and
free” Indo-Pacific region(FOIP). India is considered an indispensable partner in Japan’s FOIP
vision owing to their shared goals; to begin with, the two are Asian maritime powers: Japan
in the Pacific Ocean, and India in the Indian Ocean. The two countries therefore are
strengthening cooperation in areas such as maritime security, connectivity, and HADR; they
are also two of the four stakeholders in the Quadrilateral Security Dialogue forum or Quad

Similarly, India will work with Germany on a “convergent vision” of a free, open and
inclusive Indo-Pacific region as part of enhanced strategic engagement between the two.
Germany became the second European country to issue guidelines for the Indo-Pacific.

The Australia-India Indo-Pacific Oceans Initiative Partnership (AIIPOIP) will help shape
maritime cooperation in the Indo-Pacific to support an open, inclusive, resilient, prosperous
and rules-based maritime order. Australia perceives India as a natural partner in shaping the
future of the Indo-Pacific and the elevation of bilateral ties to a comprehensive strategic
partnership has boosted cooperation in areas including critical minerals and defence.

Thus , even though it is still an evolving concept, most analysts see it as an idea that
captures the shift in power and influence from the West to the East. Prime Minister
Narendra Modi aptly articulated India’s Indo-Pacific concept as the SAGAR doctrine —
‘Security and Growth for All in the Region’, an aspiration that depends on ensuring
prosperity for all stakeholder nations, guided by norms and governed by rules, with freedom
of navigation.

22
6(b).How the recent moves of India to engage with Pacific island nations fit into its larger
geo-political scheme of things?

India’s interaction with the region goes back to the colonial era, in the early 19th century
when Indian workers were taken to the region as indentured plantation labourers, a large
number of them settled there. In the post-colonial period, the Pacific region did not figure
prominently in India’s strategic thinking. However, as India looks beyond its immediate
region, with strategic ambitions in the wider Indo-Pacific region, India’s approach towards
the South Pacific is gradually changing. In recent years, New Delhi has been reaching out to
these small island states, highlighting the government’s willingness for greater engagement
particularly in the light of the reinvigorated Act East Policy.

This change can be attributed to various geopolitical, economic and strategic factors. Geo-
strategically, the Pacific Island Countries(PICs) are a part of the larger Indo-Pacific region. As
the global focus shifts to the Indo-Pacific, the South Pacific sub-region lying at the
crossroads of strategically significant maritime trade routes, comprising small island
countries with large EEZs is also attracting attention from regional and extra-regional
powers.

India has been actively involved in establishing deep economic, political, and strategic
relationships with the PIC. India’s interaction with the PICs still largely driven by the
presence of sizeable Indian Diaspora in the region; nearly 40 percent of Fiji’s population is of
Indian origin. In terms of institutional engagements, India participates in the Pacific Island
Forum (PIF) as one of the key dialogue partners of the Forum. The most important
development in facilitating India’s interaction with the PICs in recent years has been the
formation of action-oriented Forum for India and Pacific Islands Cooperation (FIPIC).The
FIPIC is a multinational grouping for enhancing cooperation between India and the 14 Pacific
Island Nations. In 2014, India offered a grant-in-aid of US$125,000 soon after FIPIC was
established.

Recognizing climate change as a major issue of concern in the region, PM Modi announced a
Special Adaptation Fund of US$1million to provide technical assistance and training for
capacity building to the PICs. To enhance connectivity, a Pan Pacific Islands Project for E-
network was announced. India and the PICs have been actively working together during the
UN Climate Change Negotiations, and in 2017, held the India-Pacific Island Sustainable
Development Conference in Suva. India’s efforts in ensuring greater collaboration with the
PIC in matters pertaining to climate change, marine resources, fisheries, and the blue
economy have enabled it to establish greater areas of convergence with the PIC.

To augment trade relations between India and the Pacific Islands, setting up of a Trade
Office in India at the Federation of Indian Chambers of Commerce and Industry (FICCI) was
announced. Possibilities of joint research in the field of traditional medicine and healthcare
were also proposed to be explored. In September 2017, India launched Climate Early
Warning Systems in seven PICs. India has regularly provided assistance in these counties to

23
deal with consequences of frequent cyclones like the relief and rehabilitation grant was
provided when Tropical Cyclone Hola hit Vanuatu in 2018.

Other than this India has also played a key role in establishing space technical applications
centers, space detection capacities to map out water, fisheries, forest resources, weather,
and climate change. India plans to expand its strategic footprint in the South Pacific region.

The second summit of the FIPIC Forum was hosted by India at Jaipur, Rajasthan. A significant
increase in India’s grant-in-aid to the PICs from US$125,000 to US$200,000 was announced.
During the latest meeting in September 2019, PM Modi announced US$12 million grant
(US$ 1 million to each) towards the implementation of high impact developmental project in
the area of their choice.

These recent moves fits perfectly into India’s larger geo-political scheme of things because
the PICs with their resource-rich EEZs can be attractive sources of natural resources like LNG
and hydrocarbons to fuel India’s growing economy and can also provide new markets. The
concept of the Indo-Pacific region provides India with the perfect opportunity to maximise
its friendly relations with its Pacific partners and establish stronger foundations. The region
and its island states provide multiple economic, political, and strategic opportunities that
India could use to its advantage.

6(c) India’s Look East Policy is actually a broader foreign policy approach reconciling
multiple factors and determinants, where China is the most important input. Comment
Initially an economic policy aimed at developing regional trade and attracting foreign direct
investment from the capital-rich Asian economies to finance India’s economic reforms, the
Look East policy grew rapidly into a comprehensive strategy with political and military
dimensions concerning the entire Asia-Pacific region. The Look East policy was perhaps the
most important foreign policy initiative of India in the immediate post-Cold War period.
Officially launched in 1992 during the tenure of Prime Minister Narasimha Rao, it was an
attempt to reconnect with Asia as part of India’s economic globalization.
In the cumulative development of its economic, political and strategic dimensions, the Look
East policy has become India’s favourite instrument to mitigate its China concerns and
prevent the emergence of a China-led regional order. India’s look east policy emerged
gradually through phases. The first phase was ASEAN cantered and focused primarily on
trade and investment linkages. The second phase was characterised by enlargement of
geographic and thematic scope i.e an expansion of “East” from Southeast Asia to East Asia
and Australia and a shift to wider economic and security issues. These phases were
accompanied by a phase of institutionalization in which India gradually joined existing Asian
institutions.

Various imperatives or determinants of India’s LEP are:

24
Economic: The Look East policy resulted in the early 1990s from the awareness that New
Delhi had to look beyond the confines of its traditionally isolationist policy. The initial focus
of the LEP was therefore to attract foreign direct investment (FDI) from the most advanced
economies of Asia. India’s economic engagement with ASEAN thus became central to the
LEP. In 1993 and 1994, Rao himself led economic missions to Thailand, Vietnam, and
Singapore. India also intended to reap the potential benefits of globalization through
greater integration within an increasingly dynamic Asia. India feared to be left behind China
as it had already embarked upon Deng Xiaoping’s ‘Four Modernizations’ in 1979. However,
India held a comparative advantage vis-à-vis China in terms of a huge, reasonably skilled,
and cheap labor force as well as, in some sectors, a solid technological base. Above all, it
combined these advantages with a very large domestic market.
Political: The broad policy objectives defined for the LEP in the 1990s were three-fold: first,
India intended to institutionalize linkages with ASEAN and its affiliates; second, it intended
to strengthen bilateral relationships with member states of ASEAN; third, India sought to
carve out a suitable diplomatic status for itself in Asia and, in doing so, to prevent Southeast
Asia from falling under the influence of any one major power. A relatively isolated country
at the end of the Cold War, India intended to re-emerge as a major Asian player and to
participate in a rule-based regional system to limit the impact of a balance of power
favourable to China. The convergence of interests grew with regional partners as China’s
rise increased regional fears of a potential China threat.
Military: The evolution of the China factor in India’s foreign policy was critical and complex
strategic determinant of the LEP. Even the strictly economic dimensions of the LEP had
strategic motivations partly related to China. For India, the northeast of the Indian Ocean
represented a key strategic buffer against potential threats emanating from or through the
Southeast Asian archipelago. The ability to control the sea lanes of communication that
enter the Pacific Ocean through the Strait of Malacca was supposed to provide India with
considerable strategic leverage in dealing with its rival powers like China. Balancing China
was therefore a main strategic goal since the inception of the LEP.

Ideational: The determination to recover its status as a leading Asian nation and the desire
to preserve its strategic autonomy as much as possible, shaped India’s engagements
through LEP. Christophe Jaffrelot has convincingly argued that India’s Look East policy can
be traced to the Asianist ideal of the national movement- Asian solidarity.

Thus, LEP was a major break in India’s strategic thinking and China was a major input into it.
This argument is supported by Frederic Grare, author of India Turns East-International
engagement and Us-China Rivalry, who says that managing China’s rise was an Indian
concern long before it became an American concern. This concern is well reflected in all
dimensions of LEP

25
7(a).To what extent it would be correct to say that India doesn’t have a well defined
neighbourhood policy. Analyse in context of recent developments in India’s bilateral
relationship with it’s South Asian neighbours.
India as a geographical entity has a unique character. It shares its boundaries with nations
greatly varying in their size, resources and strength. The South Asian region, which is home
to eight countries, and the Indian Ocean region comes under the broad geographic expanse
of India’s neighbourhood. Moreover, there are ideas such as “extended neighbourhood”
linking India with other regions which do not necessarily share borders but share cultural,
civilisational or economic linkages that have come up in policy parlance in recent times.
Former PM Atal Bihari Vajpayee had once famously said ‘You can change your friends but
not neighbours’. C. Raja Mohan argues that without enduring primacy in one’s own
neighbourhood, no nation can become a credible power on the global stage.

Indian neighbourhood is a very complex region as it is the largest regions in the world by
population, the least integrated with tremendous deficits in terms of infrastructure,
connectivity, and interdependence. And it is a region that is now being exposed to various
geopolitical competition dynamics, with China making a grand entry and the U.S. developing
relations on its own with some of India’s neighbours.

Although critics believe that India’s approach towards the neighbour had been defined as
adhoc and devoid of broader objectives, yet it must be kept in mind that India gave the
paradigm of Panchsheel, Gujaral Doctrine and very recently the Neighbourhood First policy .
India’s neighbourhood policy has been through several phases. The phase under colonial
times centred on ideas and slogans around anti-colonialism, anti-imperialism, anti-racism,
which cemented India’s relations with its neighbours and in a way, supported their
respective de-colonisation movements, for instance, in Myanmar and Sri Lanka. The post-
colonial phase, which broadly began in the late 1940s, again, has had a complementariness
which helped India and its neighbours to propel ideas such as non-alignment (reverberated
in the first Afro-Asian Conference at Bandung in 1955) in the international arena, which was
inspired by a macro-level “third worldism”, “South-South cooperation” and so on.

Though multilateralism prevailed in India’s foreign policy at the international level, there has
been a tremendous focus on bilateralism in India’s approach to its immediate
neighbourhood. During the cold war, India’s foreign policy approach towards its neighbours
were shaped by the “principle of balancing”. At the same time domestic level factors also
played a role in the shaping of neighbourhood policy. For example, the India-Sri Lanka
conflicts in the eighties and water sharing issues with Bangladesh. Parallel to this, the role of
super powers and their Cold War proclivities significantly contributed to India’s
neighbourhood policies.

In the post-Cold War period, which began in the 1990s, India set out to refashion its foreign
policy premises on non-alignment, its relations with Western bloc countries, regionalism
and so on, which in turn had a huge impact on India’s neighbourhood/regional policies.
India even undertook non-reciprocal initiatives to its South Asian neighbours to build ties
and instil a high degree of confidence. One of those initiatives was the “Gujral Doctrine” of
1996.

26
However, security related issues have dominated India’s relations with Pakistan, and of late
with China, though India-China trade increased exponentially in the period of post-Cold War
era. In the recent times, there have been many strains in ties with neighbours — for
instance, with Nepal over its Constitution in 2015 and now over the map, with Bangladesh
over the Citizenship (Amendment) Act (CAA),and Srilanka on the issue of Port development
as Sri Lanka unilaterally dropped a 2019 plan to jointly develop the East Container Terminal
at the Colombo Port with India and Japan. Also the new initiatives such as Vaccine Maitri
came under stress when India was unable to fulfil its promise on supplying vaccines and
other essential drugs.
However, the other side of coin is improved relations with Maldives, ceasefire with
Pakistan, sustained dialogue with China for de-escalation, settlement of maritime dispute
with Bangladesh etc. India has revived BIMSTEC (Bay of Bengal Initiative for Multi-Sectoral
Technical and Economic Cooperation) and worked in the BBIN (Bangladesh-Bhutan-India-
Nepal) quadrilateral for a framework on motor vehicle and water governance. Recent visits
by Foreign Secretary Harsh Vardhan Shringla and National Security Adviser Ajit Doval to
countries in the region appear to show new energy in India’s neighbourhood policy.

However, it is extremely important that our engagement with our neighbouring countries
should not be episodic. It should not be event-oriented; it should be process-oriented. And
we should have a plan for continuous engagement at various levels. It also important to
realise that it is a difficult region and there will never be great solutions or great setbacks.
The neighbourhood policy will always be a work in progress, ever evolving in accordance
with the emerging dynamics .

7(b).What are the repercussions of ethnic conflicts in South Asia on interstate relations?
Suggest a way forward to remove these impediments of regional cooperation.
South Asia is one of the emerging economies and most dynamic regions of the world. The
region is composed of Afghanistan, Bangladesh, Bhutan, India, Nepal, Pakistan, Sri Lanka,
and the Maldives. Though it is home to around a quarter of the world’s population, the
region accounts for just under seven per cent of the world’s GDP. As these countries shares
a common history, culture and preferences, the region has huge potential for cooperation.

It is generally being felt that while rest of the world has removed regional impediments and
are moving ahead through regional cooperation, South Asia remains a disjointed and
disorganised region. The countries in the region remain mostly in conflict since
independence, and suffer from high degree of insecurity and underdevelopment. There are
many regional irritants such as border conflagrations, water disputes, ethnic clashes etc.
which impede any holistic interstate cooperation in the South Asian region. Conflicts in
South Asia have impinged adversely on the autonomy of the region and its countries. They
have made the South Asian states porous and vulnerable to external interventions and
influences.

Conflicts in South Asia, or for that matter in most of the other parts of the world, can be
broadly put in four categories- those imposed and escalated by the global political,
strategic and developmental dynamics, including the role of great powers; those inherited
and strategically induced in inter-state engagements; those precipitated and nurtured by

27
the internal political turbulence, socio-cultural fault-lines and developmental distortions;
and those that are caused and covered by the non-state actors. For long, South Asia has
been an epicentre of conflicts including ethnic, political, religious and social ones. The region
has multi-cultural, multi-ethnic, multi-religious societies and this multiplicity has given rise
to a variety of clashes.Ethnic clashes just being one of many.

Ethnicity is a complex category, which has always been in a state of flux. It is closely
associated with the caste, class, politics and religion of majority–minority equations.
Ethnicity does not exist in a pure form, rather there have always been ingredients of class,
politics and religion in it. Therefore, ethnicity finds expression in economic exploitation,
political domination and psychological oppression. Ethnicity plays a major role in shaping
the conflict spectrum in South Asia. In fact, almost every conflict in South Asia is driven by
ethnicity. The post-colonial states particularly Bangladesh, India, Pakistan and Sri Lanka
bear testimony that all the major conflicts in the region have emerged on the basis of
ethnicity. South Asian societies suffer from ethnicity as the most pervasive of political fault
lines. Ethnic conflicts include the Chakma problem in Bangladesh, the Hindu, Muslim and
Sikh problem in India, the Hindu and Muhajir problem in Pakistan, the Tamil problem in
Sri Lanka,Rohingya issue in Myanmar,Madheshi problem in Nepal and the conflict over
Kashmir between India and Pakistan— the two most important states in South Asia.

Most ethnic conflicts in South Asia are political in nature. They could be resolved by framing
inclusive and comprehensive policies for all strata of society. The prevention of ethnic
conflicts before they emerge is the best strategy to tackle them. Governmental intent
should be conflict resolution and not merely conflict . Delegation of power to the grassroots
level is essential, as it provides opportunities to marginalised groups to participate actively
in democratic processes. Reviving regional cooperation in SAARC,and bringing out clarity in
the refugee policy in the region can be a way forward. There is a need to clearly define
terrorism and develop a compact against it as has happened in SCO because there is a close
linkage between ethnic conflicts and terrorism. Religious majorities in states should accept
the legitimate demands of minorities. Their problems and concerns should be addressed
through consensus.

In a nutshell, ethnicity in and of itself is not responsible for the rise of conflicts. It is the
politicisation of ethnicity, which generates conflicts in South Asia, ultimately affecting
regional cooperation and holding the region back economically and socially.

7(c). With major power fault line sharpening across the world, India’s growing
engagement with Africa can bring equative benefits to both. Discuss.
Political analysts across the world are of the view that , the golden era of rule based
governance is in decline , so much so that power faultlines have sharpened and the global
order is becoming Westphalian . In this backdrop , India’s engagement has been and
currently is embedded in democratic principles of mutual and equal benefits ,be it bilateral
or multilateral relations.

28
One such engagement is with Africa, based on the principle of partnership among equals.
India has always been at the forefront of championing Africa’s cause on the global stage it
supported the struggle against apartheid it has advocated representation from the African
continent in the UN Security Council and it has invested more than USD 13 Bn in capacity
building efforts which has contributed significantly towards economic and social
development in the region.

India has a long history of partnership with Africa, with solidarity and political affinity going
back to the early 1920s when both regions were fighting against colonial rule and
oppression. After India gained independence, it became a leading voice in support of African
decolonisation at the United Nations. Independent India, though extremely poor after two
centuries of colonial exploitation, strived to share its limited resources with African
countries under the banner of South-South cooperation. In 1964, India launched the Indian
Technical and Economic Cooperation (ITEC) programme to provide technical assistance
through human resource development to other developing countries, with African countries
the greatest beneficiaries of it and the Special Commonwealth African Assistance
Programme (SCAAP).

Much of India’s narrative, as it relates to engagement with Africa, has focused on “capacity-
building.” The ITEC programme, launched to share India’s lessons in development with
other developing countries, continues to remain an important pillar of Indian development
cooperation programme. Currently, about 98 Indian institutions run training courses in
fields such as agriculture, food and fertiliser, engineering and technology, and environment
and climate change. In addition to civilian training programmes, ITEC also conducts and
oversees defence training programmes, study tours, aid for disaster relief, the deputation of
Indian experts abroad and project-based cooperation. Africa is a key beneficiary of the
programme with nearly 50 percent of the ITEC slots reserved for countries from the region.
Indian government established 18 new diplomatic missions throughout Africa in March
2018. As stated by PM Modi , India is looking to “liberate” Africa’s potential, by creating
local opportunities to enable such an end. Africa is important for India in the arena of global
issues such as UNSC reforms,climate change,WTO reforms etc.
India has also aided African countries amid crises, including during the COVID-19 pandemic.
India has provided 270 metric tonnes of food aid to Sudan, South Sudan, Djibouti and
Eritrea, and supplied essential medicines to over 25 African countries. The Indian
government also organised an e-ITEC training course for healthcare professionals on COVID-
19 prevention and management protocols.
From a maritime perspective, working together for ensuring the freedom of the oceans with
an inclusive and cooperative approach underpinned by India’s SAGAR (Security and Growth
for All in the Region) is included in the guiding principles. The East coast of Africa is an
integral part of the Indian Ocean littoral with its waters washing the shores of 10 nations in
that continent. The ambitious India-Japan-Africa Growth Corridor (IJAGC), a developmental
project conceived through converging Indian and Japanese interests, aims to achieve closer
developmental cooperation with Africa. This proposed project represents a joint Indo-
Japanese effort aimed at building infrastructure in Africa.

29
Thus, the India-Africa engagement is mutually beneficial. According to Shashi Tharoor,
India’s engagement in Africa is an example of ‘DEVELOPMENTAL POWER’. India must
encourage greater African participation in global multilateral organisations. India’s non-
permanent membership in UN Security Council and India’s forthcoming Presidency of the
G20 will be an opportunity for India to strengthen its credentials amongst African countries.

8(a) In an era of new bipolarity, it appears that the path of India and Russia are set to
diverge along with their visions of future. Comment
The international system currently is one of emerging bipolarity, anchored by the US and
China. The global distribution of power determines the nature of any system; its
characteristics are set by empirical facts, not policy preferences. The US and China have
emerged over the past decade as the clear leaders on the most important indices of power
– military and economic. The US retains an advantage over China in the military realm;
Chinese nominal defense spending in 2019 was about 36% that of the US. The two are much
closer in the economic sphere, with the US ahead on nominal GDP and China leading on
purchasing power parity.
In this era of new bipolarity in international relations, both India and Russia are being forced
to gravitate toward one of the poles: the United States and China. It looks like the paths of
these two once-close partners are set to diverge, along with their visions of the future.
India is being pushed closer the United States—by its difficult relations with China, not least
the ongoing conflict between Indian and Chinese troops in the Himalayas. At the same time,
India is trying to keep some distance from the West due to its reluctance to damage
relations with Russia.
Today, Indian-U.S. relations are at the peak of their development. In the last year, India has
started actively cooperating with the Quad platform, which brings together the United
States, India, Australia, and Japan; held joint naval exercises with the Quad in the Bay of
Bengal in the Arabian Sea; and taken part in the expanded G7 summit. Both the countries
have also been cooperating more closely in the military sphere, with India allowing U.S. P-8
Poseidon patrol aircraft to refuel at its base in Port Blair in September 2020.
China isn’t the only reason for the Indian-U.S. rapprochement. The cooperation between the
two countries has been growing for about twenty years. Relations first began to noticeably
improve after Washington reviewed its policy in the region with a particular focus on India:
it recognized India’s nuclear status, restarted delivery of military technology which had been
stopped after India’s 1998 nuclear tests, and began holding joint military exercises with it.
The US now has an individual policy on India in its own right, separate from Pakistan.
India, for its part, has been set on a course of increased cooperation with the United States
since Prime Minister Narendra Modi came to power in 2014, having signed a series of
defense and security agreements since then. Economic ties between the two countries have
also improved, and for several years now, the United States has been India’s biggest trading

30
partner, with bilateral trade worth $80 billion (nearly $150 billion when services are taken
into account as well as goods).
There are two fundamental problems in Indo-Russian relations right now. The first is that
Moscow is incapable of expanding its relationship with New Delhi beyond the confines of
cooperation in the military and energy sectors. Economic ties are not developing as
expected: despite calls to increase the trade turnover to $30 billion by 2025, the indicator
has hovered around $10 billion for the last few years. The second problem is that Russia’s
confrontation with the United States is forcing its Asia policy to tilt toward China, which
cannot fail to impact its relations with India.
Another source of annoyance for India is Moscow’s relations with Islamabad. In 2015, Russia
announced it would be supplying Pakistan with four Mi-35 attack helicopters, thereby
ending its unspoken embargo on exporting military technology to that country. And since
2016, Russia and Pakistan have regularly held joint anti-terror drills dubbed “Friendship.”
Russia has also expressed interest in helping to build a gas pipeline from Karachi to Lahore,
and is cooperating with Pakistan on the issue of Afghanistan, bypassing India. However,
India remains the biggest importer of Russian arms, and Pakistan, with its one-off orders
and unstable economy, cannot possibly replace it.
The growing partnership between Russia and China has far more serious consequences.
Russia’s help building China’s missile attack early warning system, their cooperation on
technology and energy, the growth of trade and investment, and speculation about a
military alliance between the two countries is all of great concern for India.

During the Cold War, India was one of the initiators of the Non-Aligned Movement in
response to the polarized world of that era. Today, Russia and India are moving ever closer
to the two global centers of power: China and the United States. This divergence is gradually
burning the bridges of Russian-Indian friendship. There is still room for pragmatic dialogue
between Russia and India, but they no longer share a similar vision of the future.

8(b) India’s quest for deterrence stability with China has created the instability with
Pakistan. Do you think that the three way dynamics add complexity to India’s nuclear
policy?
Recently analyst Yogesh Joshi argued that India’s quest for deterrence stability with China —
the ability to have a secure second-strike option against that country — has created
crisis instability with Pakistan, where Islamabad/Rawalpindi is worried that the INS Arihant,
India’s sole nuclear ballistic missile submarine (SSBN), will be used for a first strike against
Pakistan’s nuclear weapons in a crisis. Such a belief is likely to generate a “use it or lose it”
pressure for Pakistan in a contingency involving India. Repeated Indian statements chipping
away at India’s official No First Use (NFU) policy have worsened the matters.

31
The India-Pakistan-China nuclear dynamics are referred to as Southern Asia strategic
space involving all three powers’ geopolitical preferences, nuclear postures and capabilities.
China’s support for Pakistan’s nuclear weapons program has a long history. China has
consistently provided technical help to Pakistan’s nuclear program. It has provided support
for not only testing purposes but also in creating a substantial number of arsenals.

For its own part, India’s nuclear-weapons capabilities increasingly emphasize contingencies
with regard to both. The country’s Agni V ICBM is reported to be capable of holding all of
China’s eastern coast at risk. As Hans Kristensen and Matt Korda noted in their most recent
update on India’s nuclear forces, “[w]hile India’s primary deterrence relationship is with
Pakistan, its nuclear modernization indicates that it is putting increased emphasis on its
future strategic relationship with China.” India’s sea-based nuclear deterrent shows a keen
awareness in ensuring a credible second-strike option when it comes to China.
This complex three-way dynamic creates a possibility that India may face a two-front
conventional military threat from China and Pakistan — either with both colluding, or with
one taking advantage of the other’s military action to open a new front against India. The
question here is the extent to which either nuclear threats from India or an Indian tactical
nuclear-weapons capability can forestall that possibility or, in the event of a conventional
deterrence failure, generate favorable outcomes. Analysts like Frank O’Donnell have
argued that the India-China conventional balance across the Line of Actual Control between
the two countries is not unfavorable to India and therefore, the country should take a lead
in promoting nuclear restraint globally.
India is unlikely to take part in any such initiative as long as China seeks to augment its
nuclear capabilities, if not in the number of warheads per se, then through pursuit of
technologies such as Multiple Independently Targetable Reentry Vehicles (MIRVs) or even
conventional Hypersonic Glide Vehicles (HGV). China is currently pursuing both as
countermeasures for Washington’s ballistic missile defense (BMD) plans. But China’s pursuit
of such technologies also stands to complicate India’s belief in the survivability of its own
second-strike ability — especially if it fails to increase the size of its own nuclear fleet in the
near future.

8(c).India is back as non-permanent member of UN Security council for 8th time. What
should be India’s agenda from the platform amidst the Geo-political turbulence?
On 1st January, India began its two-year tenure as a non-permanent member of the United
Nations Security Council (UNSC). As part of its tenure, in August 2021, it became the UNSC
president. India’s Permanent Representative to the UN, Ambassador T.S. Tirumurti said that
“India’s message will be to ensure how do we let diversity flourish in a united framework,
which is in many ways the United Nations itself. This is something which India as a country,
as what it stands for, will take to the council.”
For the month of August in 2021, the UNSC will also have on its agenda several important
meetings, including on Syria, Iraq, Somalia, Yemen, and the Middle East. The council will also

32
be adopting important resolutions on Somalia, Mali, and the UN Interim Force in Lebanon.
With the current turbulence in Afghanistan, India might have to face emergency meetings
on the status quo of Afghanistan. In this light, India’s agenda amidst the geo-political
turbulence should be:

 Counter terrorism- Indian should seek co-operation from other countries to reaffirm
that terrorism in all its forms and manifestations constitutes one of the most serious
threats to international peace and security, and that any act of terrorism, committed
by whomever, wherever and for whatever motives and purposes, is a crime and has
no justification. The countries must also recognize that acts, methods and practices
of terrorism in all its forms and manifestations are activities aimed at the destruction
of human rights, fundamental freedoms and democracy, threatening territorial
integrity, security of States, and that the international community should take the
necessary steps to enhance cooperation to prevent and combat terrorism, including
cross-border movement of terrorists.
 Peace-keeping: India is the largest contributor of troops to UN Peace Keeping
operations. More than 200,000 Indian troops have served in 49 of the 71 UNPKOs
deployed so far. However, the burden of sending troops for UNPKO falls on countries
like India, Ethiopia, Bangladesh etc while developed countries like US, Russia, China
etc rarely send troops for UNPKO. This lack of balance needs to be addressed.
 Maritime security: During a high-level debate at the UN Security Council, on the vital
issue of enhancing maritime security, on 9 August 2021, Indian Prime Minister
Narendra Modi said: “The ocean is our common heritage … (which) … is facing many
challenges.” He emphasised that “sea routes are being misused for piracy and
terrorism.”
He suggested the adoption of a five-point global template to include – removing
barriers to maritime trade; peaceful resolution of maritime disputes as per
international law; cooperation in addressing maritime threats from non-state actors
and natural disasters; conservation of maritime environment and marine resources
as well as responsible maritime connectivity.
 Reformed multilateralism- In November 2020, while addressing the 20th Summit of
Shanghai Cooperation Organization (SCO) Council of Heads of State, Indian Prime
Minister Narendra Modi had said that UN needs to accept “reformed multilateralism
that reflects global realities of the present times and discussed topics such as
expectations of all stakeholders, contemporary challenges and human welfare.”
Now, that India is at the helm of UNSC, it would definitely try to highlight the need of
reformed multilateralism.

In conclusion, India holds the presidency of the UNSC for a brief one-month period. It should
ensure that it is positive and productive. This is the time and opportunity for India to show
the world what it can expect from India if it were to sit permanently at the security high
table.

33
For any issue related to PSIR Test Series, write only at psirtestseries@shubhraranjan.com

34
PSIR Crash Course 2021
Model Answers - Test 5

For any issue related to PSIR Test Series, write only at psirtestseries@shubhraranjan.com

Section A

1(a). Philosophical approach to the study of politics. Comment

As Vernom Van Dyke points, in his book (Political Science : A Philosophical Analysis) argues
that a philosophical analysis is an efforts to clarify thought about the nature of the subject
and about ends and means in studying it. Put more generally, a person who take a
philosophical approach to a subject aims to enhance linguistic clarity and to reduce linguistic
confusion.
Secondly, the philosophical approach aims at evolving “Standard of right and wrong” for the
purpose of a critical evaluation of the existing institutions laws and policies.
Most of the classical political theory represents philosophical approach. Its themes are
generally concerned with moral reasoning which cannot be subjected to scientific test
although the empirical aspect of such reasoning can always be questioned. Moreover, the
moral aspect of such reasoning can also be questioned from the viewpoint of our modern
consciousness. Most of the classical thinkers, proceeding from a hypothesis about human
nature, dwelled on two main themes ‘art of government’ and ‘grounds of political
obligation’.
Leo Strauss is the chief exponent of philosophical approach. He sought to revive the
philosophical approach after it has been challenged by the behavioural approach. He
suggests that political theory is an attempt to understand the true nature of political things.
Philosophy being the quest for wisdom that is universal knowledge and hence, political
philosophy is to explore the true knowledge of a good political order. According to him
values are indispensable part of political philosophy and cannot be excluded from the study
of politics. Criticizing Behaviouralist he held that political action always aimed at either
preservation of change and is guided by evaluation of good and bad. Political Scientist must
possess the knowledge of good. The very assumption of political knowledge is to know the
character of opinions and to make judgments. He was highly critical of artificial distinction
between political science and political philosophy. According to him political philosophy is
identical with political science and all-embracing study of human affairs. The distinction
between the two cannot be sustained. There cannot be non- philosophical political science
and non-political philosophy. Thus, Leo Strauss re-emphasized on the revival of philosophical
approach.
1(b) Highlight the difference between Sex and Gender.

“One is not born a Woman, one becomes a Woman” -Simone de Beauvoir


According to Judith Butler, gender is a performative term. Having being born into one sex or
other, individuals are socialized according to specific gender expectations and roles.
Biological males take masculine roles and females take feminine roles. They are socialized to
think and act in specific ways. As pointed out by Simon de Beauvoir at the time of the birth
there is not much difference between male and female child but society makes differences
through gender constructions. According to Judith Butler, sex itself has become socially
constructed.
Sexual identities are fluid. Sex is not just an analytical category, it itself is a normative
category. Biology or medical science seen as a scientific discipline is more of representation
of social system. There is no logical basis to use the distinction. According to Shulamith
Firestone gender differences allow social fabric to function and hence, anybody trying to
change the role is challenged. She goes to the extent of suggesting the end of family to break
away from the oppression as family is the first site to confirm the gender norms. However,
cultural feminist, do not emphasize on whether the differences are natural or cultural, they
suggest celebration of womanhood, especially motherhood representing women disposition
towards nurturing. Judith Butler recommends that feminists rely too much on polarization of
sex and differences.

1(c) Enumerate the salient features of Buddhist political thought.


According to Bhikhu Parekh, Buddhism is not an entirely distinct tradition, can be considered
as a rebel child of Hinduism. According Thomas William Rhys Davids, Buddhist Sanghas are
the earliest examples of deliberative democracies.
“Do not build 50 palaces, Your Highness. After all you can only be in one room at a time.” –
Nagarjuna to an Indian King, 2nd Centure C.E.
The salient features of Buddhist political thought can be discussed as:
a) Origin of State/ King: Jatakas reject the divine origin theory and support the social
contract tradition. It talks about the election of king by the assembly of elders, hence
king is called as Mahasammat. Digha Nikaya challenges the Vedic dogma of divine
creation of social order.
b) Principle of Righteousness: In Buddhist literature, Danda does not have the central
role. Dhamma is to be upheld. However, the principle of dhamma/ righteousness is
different from Brahmanical conception of Dharma. It is closer to the western
conception of virtue. Righteousness is an ethical doctrine and mental discipline. The
principles of Righteousness for the king and the common man are the same.
Righteousness is applied in both domestic and foreign policy.
c) Attributes of the King: The virtues king is just not a successful administrator or a
warrior but a follower of the righteousness. He conquers heart by the force of
Righteousness rather than land by the force of violence.

The Principles of Righteousness are given in 8 fold path (Astangika marga) in Buddhism
which emphasises on right view, right thought, right speech, right action, right livelihood,
right effort, right attentiveness, right concentration.
To quote Dr. Schindler, “…the core of Buddha’s philosophy is the notion that Human life is
precious and endorsed with freedom and dignity.” According to Buddha, highest virtue is
awakening and compassion.

1(d) Overlapping Consensus .Comment.


Rawl’s in his best known book, A Theory of Justice, has articulated the theory of justice which
can be considered as universalist in Kantian terms. In his first book he adopted the method of
social contract by putting people in the original position, behind the veil of ignorance. His
approach was criticized by communitarian for its Universalist nature. In his book ‘Political
Liberalism,’ he engages into a critique of his earlier work and accept that he has not fully
accounted for the “Fact of reasonable Pluralism.” The principles of Justice given in the first
book has been from the moral stand point of autonomous agents in the form of
comprehensive liberalism and hence controversial. Communitarians deny that people are
autonomous and political values drives from such autonomy and rational.
Rawls accept that in liberal democratic societies, people holding various comprehensive
doctrines exists these people may disagree with each other on moral point of view. However
he believes that in a country with the democratic political culture, reasonable people may
arrive at a common conception of political justice. Citizens from different reasonable
comprehensive conceptions of good can develop “overlapping consensus” over liberal
political institutions. They can realise liberal values to relate to their comprehensive
conceptions. He also suggests that overlapping consensus differ from Modus Vivendi as the
people are reasonable and moral.
Differences arise in comprehensive doctrine because of multiple reasons, for example,
concepts are often vague, evidences may be conflicting and complex. As an example of
overlapping consensus we can give reference to the Muslims accepting liberal political
institutions for example Islam’s concepts of Jihad can be reinterpreted as individual spiritual
struggle, requirement to wear Veil may be interpreted symbolic with variation. Even
Christian, Jews and others may produce different reasons for endorsing liberal principles. The
task is not to agree on reasons, reasonable people will always disagree but they can
converge on set of institution diverse stand points.

1(e) Critically examine the concept of Asian Values.


Asian values are set of values promoted by some Asian political leaders and intellectual as
conscious alternatives to western political values like human rights, democracy and
individualism. They support Asian values by suggesting that the rapid development of many
East Asian economy in post Second World War period is due to shared culture of these
societies especially of Confucian heritage. They assert western values as unsuitable to East
Asia, with its excessive focus on individualism. It threatens to undermine social order and
economic dynamism. Among the frequently cited Asian values are discipline, hard work,
frugality, importance to society and respect to authority.
One of the prominent spokesperson of Asia values was former Prime Minister of Singapore
Lee Kuan Yew. Asian values claim also challenge the western assertions that liberal
democracy can be considered as “The End of History”. The prominent critic of Asian values is
Amartya Sen. He considers Asian values as a justification for authoritarianism as an attempt
to obscure weaknesses of Asian political and economic model. The discourse on Asian values
is based on the simplistic stereotypes of Asian cultures something similar to Orientalism
proposed by Europeans.
Feminists criticize Asian Values as an attempt to legitimize gender hierarchies embedded in
Asian cultures. Asian value debate extends to political theory where liberals suggest global
justice to be based on human rights whereas communitarians like Charles Taylor reflect the
need for more inclusive approach to the rights. Bhikhu Parekh also recommends that Asian
values can be deployed to enrich the concept of human rights.

2(a). How the concept of minimal state differs from the concept of welfare state? Discuss
the future of Neo-liberal state.

With the emergence of Behaviouralism the focus of political analysis shifted away from the
state towards socio-political processes, however, political scholars analysed the possibility of
the discipline losing its identity, called for centrality of the state. To quote: Theda Skocpol:
“bringing the state in.”
The concept of minimal state is associated with classical liberals like Adam Smith and Neo-
Liberals like Nozick, Hayek and others. The term minimal state reflects the minimal functions
to be performed by the state. The minimal functions are law and order and the maintenance
of external security. The minimal state is often described as Police State. The earliest
exposition is minimal state is found in the works of John Locke, the father of Liberalism.
According to John Locke, Government cannot deprive a person from his right to life, liberty
and property without his consent.
Acc. to Nozick “Minimal state is inspiring as well as right.” While giving the theory of
minimal state in his book “Anarchy, State and Utopia,” Nozick reasserts Man’s absolute right
over property, reject the legitimacy of welfare state and suggest that progressive taxation is
like bonded labour.
On the other hand, the concept of welfare state is associated with modern and social
liberals. The modern liberals like T H Green, Laski, and social liberals like Rawls, Dworkin,
Amartya Sen favours welfare state.
Welfare state differs from minimal state in terms of the range of functions performed.
Minimal state is based on the concept of negative liberty, goes for minimum interference in
the life of man and recommends only physical security as a public good. On the other hand,
welfare state based on the concept of positive liberty, recommends State’s role in providing
social security along with physical security.
Rawl’s Difference Principle, Dworkin’s Resource Egalitarianism and Amartya Sen’s Capacity
building approach reflect the notion of welfare state.
Neo-liberal state gained traction in 1990s, often seen as the beginning of globalization. Neo-
Liberal state was based on the recognition of the limitations of failure of the welfare state.
The welfare state introduced after great depression became almost a nanny state,
necessitating the need of rolling back of the state. According to some scholars Neo-Liberal
states emerged as the default logic which blurred the boundaries between the market and
state. Under welfare state, the onus of development was on the state but under neo-liberal
state the onus shifted to the individual.
However, the contradictions of the neo-liberal state were felt and documented in terms of
increasing inequalities (Thomas Picketty: Capital in 21st Century). Since Global financial crisis
in 2008, the neo-liberal state is facing crisis, Covid has further de-legitimised the neo-liberal
state. In context of Pandemic, people had to depend on the state. The states with better
health infrastructure could minimize the deaths on the other hand, the states run on neo-
liberal philosophy for example: USA has been one of the worst performers. Today, neo-liberal
state is at the cross-roads. Going back to the conventional welfare state is also not an
option, at the same time the pure neo-liberal state is also not viable. Hence, a combination
of the two is the need of the hour. Academic community has to think about a neo-liberal
welfare state as a bulwark against the disasters like Pandemic as well as against the
corruption which comes with the welfare state.

2(b) “Plato’s communism is a heroic remedy for a desperate evil, the union of political
power and economic temptation in the same hands.” Explain.
Plato, the father of Political idealism, conceptualized an ideal state ruled by “The Philosopher
King.” In the ideal state people will be in a position to lead the virtuous life. Plato’s concern
for the ideal state stems from his observation of the decline of Athens. Plato diagnosed the
fundamental problem in Athens as the rule of ignorance. It is because of the ignorance of the
ruling class that there was huge corruption. Plato suggests that Athens was not a single city
but two cities- the city of rich and the city of poor. In order to create the ideal state where
ruler is not corrupt, he gives two institutions- education system and Communism.
It is through the education system, Plato’s scheme of division of society based on Spiritual
qualities of Soul would come into existence. The men of reason will be made the Rulers.
Since, Ruler is a philosopher, he knows what is good and bad and what types of goods he
should pursue. According to Plato ideal ruler is a symbol of knowledge and sacrifice. Ideal
ruler knows that this world is a world of illusions and hence, would not be having the lust for
money and power. If, education system has been the prime mechanism, Plato proposed
Communism as a secondary mechanism to re-enforce the spirit of knowledge and sacrifice.
Plato proposed that the guardian class will neither purpose property nor family because of
their corrupting influence. Plato goes to the extreme of suggesting the institution of
Communism of Wives where state would be selecting the marriage partners. Thus, Plato’s
remedy is seen as heroic, often criticized for its extreme and radical suggestions which often
goes against the basic concepts of human psychology. It is often said that Plato’s diagnosis is
correct but his remedies are not desirable. One of the major critic of Plato’s “Heroic
remedies” has been his greatest disciple Aristotle who advocated the need to follow “middle
path.”

2(c) Critically examine the possible implications of Gandhi’s views on the relationship
between religion and politics.
As suggested by Prof. V R Mehta, unlike western scholars who had been seen the world in
dichotomies, Indian thinkers never looked for complete separation whether it is individual or
society, nature or culture, religion or politics. Gandhi as a representative of Indian culture,
also, does not make the distinction between religion and Politics. Gandhi believed that
Politics without religion is a “death trap.” Here it is essential to highlight the fact that
Gandhi’s religion should not be seen in narrow sectarian terms. Gandhi used to describe
Ahimsa as his creed. Gandhi believed that Ahimsa belonged to the world of Animals whereas
Ahimsa belonged to the world of Humans. Thus, Humanity becomes Gandhi’s creed.
The Gandhian approach to religion comes closer to the Buddhist approach where religion is
seen as Righteousness/ morality. Thus, like Buddha as well as Socrates, Gandhi believed in
the continuity of Politics and Ethics and was critical of the Machiavellian view of the
separation between the two.
Gandhi was inspired by Gokhale who was talking about the need for spiritualization of
Politics. It is unfortunate that some of Gandhi’s contemporaries like MN Roy have not
understood Gandhi’s view on the relationship between Religion and Politics, looked at
Religions in the conventional sense and declared Gandhi as a medieval Man and a
Reactionary. There is a tendency among the Historians to blame Gandhi for diluting the
secular nature of Indian national movement by bringing religion into politics.
Gandhi was criticised specifically for bringing the Khilafat issue. However, Neo-Gandhians
like Ashis Nandy believe in re-enforcing the Gandhian vision of co-existence of religion and
politics and the inapplicability of the Western model of secularism in Indian context. On the
other hand, scholars like Akeel Bilgrami, Achin Vanaik reject Nandy’s nostalgic vision and
believe that the combination of religion and politics would lead to strengthening of
communalism in Indian politics.
3(a) Define Value Pluralism. Critically examine Isaiah Berlin’s views on positive and
negative liberty?

Value Pluralism is a theory of meta-ethics rather than normative ethics. The credit for
popularising value pluralism goes to Isaiah Berlin. Value pluralism is critical of value monism.
Value monism believes that there is one foundational value which should be the ultimate
guide for all policies. For example – Utilitarianism believes utility to be the ultimate
determinant of all human action.
Value pluralism holds that there are certain values which may be equally correct and
fundamental, yet in conflict with each other. In many cases, values are incompatible. Since,
values are incommensurable. Hence, there is no basis to order them in any objective sense of
the term. The fundamental values can conflict with each other was initially captured by Max
Weber in his notion of polytheism. Value pluralism differs even from value relativism. For
value relativists, different people may have different values. While value pluralists believed
that for different people, different values do exists and different people prioritise value
differently. Berlin has given the example of value pluralism in the idea that the moral life of a
nun is incompatible with a house wife, yet both value their lives for their own reasons. Moral
decisions are made with varying rational calculus. In his essay , ‘The Hedge Hog and Fox’,
Berlin talks about the thinkers with the single central vision as Hedge Hogs and those who
pursue many ends often unrelated as Foxes.
Contrary to many 20th century thinkers, Berlin recognised that there could be number of
goods for which there can be disagreements which will require necessary trade-offs. Not all
problems are solvable and we cannot think that mankind can progress towards a realm in
which there can be complete consensus. The most important trade-off is going to be
between liberty and coercion. The wider is the area of non-coercion, wider is the freedom.
After Isaiah Berlin, scholars like Joseph Raz, Dworkin and William Galston and others have
worked further in the domain of value pluralism .
Isaiah Berlin in his ‘Two concepts of liberty’, makes distinctions between positive and
negative liberty.
Negative Liberty : When individual is not prevented from attaining his goals by other human
beings .
Positive Liberty : When individual is his own master.
In both cases, liberty is an absence of restraints. Political liberty belongs to the sphere of
negative liberty. Thus, state can only secure negative liberty by ensuring that state does not
prevent an individual from choosing his course of action. On the other hand, positive liberty
belongs to man’s own will and capacity , is beyond the scope of state.
Berlin suggests that availability/ non-availability of means is individual’s concerns and not
state’s concerns. He gave the example that if one cannot fly like eagle or swim like a whale,
it cannot be said that one lacks liberty. If man is too poor to afford bread when there is no
legal ban on purchasing. Man cannot complaint the absence of liberty. Berlin does not want
that social inequalities should be dealt from the perspective of liberty. The chief rationale
behind opposing positive liberty is Berlin’s fear that in the name of positive liberty, state
promotes totalitarianism. Norman P Barry, in his book ,’An Introduction to Modern Political
Theory ’, has elucidated Isaiah’s Berlin’s conception of liberty by making the differentiation
between material and moral sphere.
Berlin has defined positive liberty as self-mastery. Berlin divides self into two – higher self
and lower self. Person is free to the extent his higher self-commands his lower self. Thus,
person may be free from not being restricted by state, but remaining a slave to his irrational
appetites. Thus, positive liberty in moral sphere is self-controlled, where a man may be
deprived of his liberty due to lack of self-control and not due to coercion by others. State can
do little in moral sphere.
In material sphere, positive liberty can be hampered by different reasons. Berlin’s examples
are contradictory. He gives two examples.
First is the inability to fly like Eagle or swim like whale. This is a natural inability and not
much can be done. However, the second example of poor man unable to afford bread is a
product of social arrangement and alterable by political actions. He should have taken into
account the role of state in this context as it fits into his conception of liberty i.e. absence of
coercion, because poverty of a person may be due to coercion of other person.
Bhikhu Parekh also suggests that inequalities due to social arrangement are interference
that needs to be tackled. Even C.B. Macpherson calls Berlin’s view too narrow and suitable
only for market society.

3(b) Analyse how the theory of materialistic interpretation of history provides the
scientific account of the development of human societies.
Marx has developed the materialistic interpretation of history. The underline assumption of
the theory is the centrality of the role of the economic factors in shaping history. According
to historical materialism, economics form the basic structure, all other structures are the
part of superstructure. Superstructure is the reflection of the base, which means that
superstructure will change automatically with change in the basic structure. In the words of
Marx, “it is not our consciousness as men that determine their existence on the contrary
their social existence determines our consciousness.” Like Hegel, Marx also saw history
progressing towards the definite and inevitable goal. Marx writes that our conception of
history depends on our ability to expound the real processes of production. Marx turned
economic base as the mode of production. The mode of production consist of forces of
production, means of production and the relations of production. Marx has identified five
stages that is primitive communism, slavery, feudalism, capitalism and communism. In each
stage forces of contradiction made revolutions inevitable. For Marx it was not enough to
understand the general process of history but also the way it can be transformed.
Marx projected his theory of history as the scientific explanation of history against the
ideological explanation of history provided by previous scholar as ideological explanation.
Marx held that philosophers have sort to interpret the world, however what matters is how
to change it. However, Marx critics like Max Weber does not consider Marxian interpretation
as scientific. He calls Marxist approach as monocausal. Karl Popper criticised Marx for
committing the guilt of historicism. Thus, the theory provided the basis for totalitarianism.
Marxian theory was criticised as economic determinism which was later on overcome by
Gramsci in his writings.

3(c). Distinguish between formal equality and substantive equality. Compare equality of
resources with equality of capabilities.
Equality, like any political concept, is a contested one, but lies at the heart of normative
political theory. Equality is a feature of modern times because in ancient and in medieval
times there was more of a justification of inequality. Plato and Aristotle considered
inequality not only natural but also desirable. But with the modern revolutions like French
Revolution and American Revolution, the concepts of liberty, equality and fraternity became
the part and parcel of the political discourse in the western countries.
Formal equality denotes legal and political equality. In fact the earliest version of equality
was the legal equality when the emerging bourgeoise class called for ending the privileges of
the feudal class and for recognising equality before the law. However, Marxists criticize that
even when the legal and political equality has been extended in the western countries, there
are huge inequalities in society. Equality in the western countries is formal procedural. It is
not an equality in the real or substantive sense. Marxists held that the substantive equality
can be achieved only in communism with the abolition of private property and when there is
a social ownership of means of production. Marxists explained their idea of equality which
they have defined in social and economic sense as substantive equality.
In Indian context we can understand the difference between the formal and substantive
equality in the views of Ambedkar. Ambedkar has held that that political democracy in the
absence of social and economic democracy is a contradiction. That’s why Ambedkar prefers
‘one man ,one vote and one value’ over ‘one man ,one vote’ . The Indian constitution grants
Fundamental rights as well as Directive Principles of State Policy. It is based on the
realisation of existing inequalities in the society. It is based on the realisation that without
social and economic inequality the fundamental rights will remain hollow and hence the
substantive equality has to be incorporated.
Within liberalism there is a debate of equality in what sense? The utilitarian perspective
strives for equality of welfare or equality of happiness based on the formulae ‘maximum
happiness of maximum numbers”.
Social liberals advanced the ideas of Equality of resources, seen in the work of Dworkin and
Equality of capabilities as given by Amartya Sen and Martha Nussbaum. Dworkin’s work is in
the context of his critique of Rawls’ view on equality and justice and Nozick’s libertarian
view. The Libertarian perspective represented by scholars like Nozick is against the welfare
state and consider progressive taxation as a bonded labour and support night watchman
state.
Equality of capabilities is considered as an improvement over equality of resources because
Amartya Sen believes that even if we provide equal resources yet we will not be able to
provide the equal sense of wellbeing as people may not have equal capacity to utilise the
resources. Along with giving the equal resources, state should make an attempt to provide
equal capabilities. For Amartya Sen, capability approach is more result oriented, action
oriented because it takes into account the diverse circumstances of the people. To make the
equality substantive in a real sense and to make initial distribution really fair we have to go
for equality of capacity.

4 (a) “A wise king trained in politics, will, even if possesses small territory, conquer the
whole earth with the help of the best fitted elements of his sovereignty and will never be
defeated”. Explain with reference to the views of Kautilya.
Kautilya is considered as the greatest political thinker of Ancient India. He has given an
elaborate theory of state craft with respect to internal and external aspects. His theory of
state is known as Saptanga theory. Saptanga theory is an organic theory of state where a
state comprises of seven essential elements. Though, King holds the centrality yet like the
wheels of chariot all parts are inter-dependent. The seven elements of the sovereignity are
the king, amatyas, janapadas, durga, kosha, bal/ Danda, mitra.
According to Kautilya, King is the spirit that regulates and guides the entire body politic.
According to him, if other elements of the state are weak but King is strong, he will convert
the weak elements into the elements of the strength. On the other hand, if the King is weak,
the other elements even if are strong will lose their relevance. Thus, Kautilya has emphasized
on the importance of the leadership. At the same time, he was careful to tell that the king
must not under value the significance of the other elements. Thus, Kautilya depicts the
various facets of the State. He presented the state as not as a thing in itself but as an entity
in relation to many. His theory is described as pluralistically dominated monism.
Subscribing to the monarchy as the ideal form of the state, Kautilya accords to the King the
highest place in body politic. King is the chief executive head and hence consummation of all
elements. Kautilya has given the qualities of the ideal king which include
a) King is supposed to of high worth
b) Approachable to the people (abhigamika guna)
c) King must possess qualities of intellect (pragyana guna)
d) Qualities of enthusiasm (utsah guna)
e) Qualities of self-restrain (atma-sampad guna)

King is a swamy, who is not under any one yet not a despot and helps his people to achieve
yoga ksheme.
4(b) Explain the concept of distributive Justice with reference to the views of John Rawls.
Differentiate the concept of procedural justice with that of substantive justice.

Justice is a central concept in Political philosophy. In political philosophy theory of Justice


deals with the distributive aspect of Justice. The core idea in Justice is fairness. Thus, Justice
is a distributive concept which is about the impartial distribution of goods or resources in the
society. Thus, since the time of Plato, political philosophers are discussing Justice as a
distributive concept. Since, justice is also a normative concept different scholars have
different views with respect to the principles of the justice. For example: Plato goes for the
distribution on the basis of the spiritual quality of soul. In general the basis of distribution
proposed by the philosophers has been: need, choice, utility or equality. In modern political
philosophy Rawls theory of Justice is a prominent theory of distributive Justice. Rawls talk
about creating a more equal and a just society by bringing the difference principles.
Rawls gives maximum equal liberty to all but provide two conditions: first, it should be done
under equality of opportunity; second, it should be done in a manner which works to the
maximum advantage of the least advantaged group in the society.
Thus, according to Rawls, all social values like liberty, opportunity, income and wealth, and
self -respect are to be distributed equally. Unless an unequal distribution of any of these
values is to everyone advantage. Thus, for Rawls, injustice is prevalence of those inequalities
which do not benefit all.
The procedural theory of Justice emphasize on just procedures for ensuring Justice.
According to procedural theory of justice, results are not important. Procedure has to be fair.
Procedural theory primarily focuses on individual liberty. It emphasize individual as rational
being, aware of the choices available to them, state should not interfere in the entitlement
and individual should be held responsible for their actions and consequences. Nozick
entitlement theory is a good example of procedural theory. According to Nozick, outcome
should not be predetermined or determined by the state. The Substantive Theory of Justice
not just emphasizes on just procedure but also how to achieve fairness in the society. It gives
equal emphasis on fare principles for distribution. In contrast to procedural theory,
substantive theory emphasize on just outcomes. Since, substantive theory focus on just
results and that is why Nozick call it as end state theory. Thus, Rawls theory is a good
example of the substantive theory along with procedural theory. Rawls focus on just
allocation.

4(c) “The possession of rights does not mean the possession of claims that are empty of all
duties” Laski. Explain the relationship between rights and duties with reference to the
views of Gandhi.
Laski in his book: ‘A Grammar of Politics’ has investigated the moral foundation of rights and
postulated a synthesis of liberal and socialist values within the framework of democracy.
Laski believes that rights are not the concessions granted by the State. On the contrary, they
are superior to the state as they provide a standard to judge the state. In the words of Laski,
rights are those conditions of social life without which no man can seek in general to be at
his best. He further writes that every state is known by the rights it maintains. Laski believes
that man cannot have rights against the community. He tries to accommodate the liberal
individualistic position with the requirements of the public welfare. As rights are erected on
moral foundations they are essentially accompanied with duties. He carefully points out that
possession of rights does not mean the possession of claims empty of all duties. Rights are
not independent of society rather inherent into it. Rights are therefore co-relative with the
duties or functions.
Man in entitled to Rights to enable him to contribute to social good. Thus, enjoyment of
rights depends upon the fulfilment of duties by both individual as well as the State. State’s
duty is to help in the realization of Rights, individuals duty is to contribute to the common
good. He also adds that if state fails in duty, it becomes the duty to resist the state. Men
must learn to subordinate the self-interest to the common interest, the privileges of some
must give way to the Rights of all.
Thus,Laski’s theory of Rights seeks to evolve the synthesis of individual Rights with common
welfare. He gives valuable insights on the re-organisation of the society according to the
principles of social justice. He takes care of the interest of the oppressed and the vulnerable
by integrating the rights and duties.
Gandhi’s discourse on the relationship between rights and duties show a continuity of
relation with Laski . Gandhi also does not make a separation between rights and duties .
However, Gandhi reflects more of an Indian tradition which give greater emphasis on duty
as found in Gita . Gandhi speaks of rights in context of duties . He suggests that real rights
are the results of performance of duty. Rights can be deserved and preserved only as a
derivative of duties performed well. As suggested by V.R. Mehta Indians do not think in
dichotomous categories and so we do not find the separation between the two. Gandhi’s
ideas are inspired by Gita which suggest that when actions are performed with detachment ,
we are free from anxiety of its future consequences .Detachment does not mean lack of
clarity . For Gandhi, the important thing is to let people do without attraction or lure of
inducements or threats of theological sanctions . Its not that Gandhi ignores rights based
individualism , but here rights are connected with the idea of duty so that Individual can lead
moral life . Gandhi was concerned about the basic rights of the vulnerable sections.
However, he does not believe that the rights of vulnerable sections have to be solely
dependent on state , rather it should be dependent on morality of self .
To conclude, Gandhi wants people to perform their duty morally regardless of the
consequences .
Section B
5(a) Discuss the features of Asymmetrical federation in Indian constitution.
Federation is a union of partially self-governing provinces, states, or other regions under a
central federal government . The division of power between them and the central government,
is typically constitutionally entrenched and may not be altered by a unilateral decision.
Federations are often multi-ethnic and cover a large area of territory. Dicey calls federalism
as a political contrivance (arrangement) among those who desire. In Bommai case (1994), the
Supreme Court laid down that the Indian Constitution is federal and characterised federalism
as its ‘basic feature’.

Asymmetric federation is one in which some states have more autonomy than others. This is
in contrast to symmetric federalism, where no distinction is made between constituent states.
It is the result of an ethnic, linguistic or cultural difference.

The governance of India is based on a tiered federal system and is designed to be asymmetric
where necessary. One comes across various kinds of asymmetries in Indian federation. There
is universal asymmetry with regard to the constituent provinces because they are represented
in Rajya Sabha on the basis of their demographic strength unlike the American system where
each state has two members in the Senate regardless of the strength of population. There are
specific asymmetries as regards administration of tribal areas, intra-state regional disparities,
law and order situation and fixing the number of seats, as per Article 371 of the Constitution,
in states like Maharashtra, Gujarat, Manipur, Assam, Andhra Pradesh, Arunachal Pradesh,
Sikkim, and Goa. The areas identified as union territories enjoy special constitutional status.
The Sixth Schedule to the Constitution contains provisions for the administration of tribal areas
in Assam, Meghalaya, Tripura and Mizoram. These create autonomous districts and
autonomous regions administered by District Councils and Regional Councils.
The inherent structure of the Indian polity strengthens the Union government more than the
states, and the attempts of the Centre to politically control the regional political landscape
serve to centralise the Indian federal design

To be sure, however, a diverse and large country like India requires a proper balance between
the six pillars of federalism: autonomy of states, national integration, centralisation,
decentralisation, nationalisation, and regionalisation.

5(b) Elaborate the role of supreme court as a conscience keeper of Indian constitution.
The Supreme Court is the living outlet of the Constitution. It is basically the will of the people
which is expressed in the Fundamental Law that they have resolve to retain themselves from
hasty and unjust action by placing their representative under the restriction of permanent
law

The Supreme Court is seen as the protector of the philosophy and wisdom of the
constitution. The philosophy of the constitution has clearly evident from Preamble ,
Fundamental Rights, Directive Principles etc. Preamble declares India to be a sovereign,
socialist, secular and democratic republic. The objectives stated by the Preamble are to
secure justice, liberty, equality to all citizens and promote fraternity to maintain unity and
integrity of the nation. In the 1995 case of Union Government Vs LIC of India, the Supreme
Court held that Preamble is the integral part of the Constitution

It is well said that ‘A right without a remedy is a legal conundrum of a most grotesque kind’.
Fundamental Rights is meaningless unless there is an operative machinery for the
enforcement of the rights. It is well said that if there is no remedy, there is no right at all.
Under Article 32, the Supreme Court can be directly approached in case of violations of the
FRs mentioned in Part III of the constitution. For Ambedkar, Article 32, is one such Article
without which this Constitution would be a nullity . In Romesh Thappar vs State of Madras
(1950), the Supreme Court observed that Article 32 provides a “guaranteed” remedy for the
enforcement of fundamental rights.

Under Article 13, the Supreme Court is can declare any act as null and void if it violates the
rights provided in Part III of the constitution.

Similarly, in Kesavanand Bharati Case , 1973, the Supreme Court ruled that the constituent
power of Parliament under Article 368 does not enable it to alter the ‘basic structure’ of the
Constitution. Parliament cannot abridge or take away a Fundamental Right that forms a
part of the ‘basic structure’ of the Constitution.

In conclusion , the Supreme Court has been truly seen as the conscience keeper of the
constitution .

5(c) Write short note on Dalit perspective of Indian National Movement.


Indian historian Bipan Chandra calls the Indian National Movement as the most spectacular
mass movement . However, the challenge before the leadership of national movement was
to integrate the divergent interests of different social groups in India in a united movement
against the colonial rule.
Dalit intelligentsia at the regional as well as national levels tried to mobilise people
belonging to their social groups in order to assert their social and political rights. Liberation
from internal oppression rather than liberation from the British rule was the desired goal.
Granville Austin observes that two revolutions, National and social, had been running
parallel during the INM, focusing on freedom from the colonial rule and ‘medievalism’
respectively.

The Dalit perspective represents an alternative imagination of India as proposed by


Ambedkar, Phule and Periyar and some other scholars. Ambedkar considers India both as
“prabuddha Bharat” (enlightened India) and “bahishkrit Bharat” (ostracized India).

Mahatma Jotiba Phule , who is considered the first ideologue of anti-caste movement in
modern India, in his book Gulamgiri (1873), described Brahmans as Aryans who came from
outside and subdued the indigenous people who were Shudras and he gave a call to discard
caste.
Periyar was initially a member of the Indian National Congress (INC) but eventually got
frustrated with the Brahminical attitude of the INC leaders. He started the ‘self-respect’
movement rejecting Brahmanism and Hinduism.
The Dalit attitude has been criticized as anti-national by nationalists and right-wing
historians like Arun Shourie. However, Prof. Gopal Guru says Arun Shourie’s criticism is
politically motivated and there is a need to understand Ambedkar’s perspective more
scientifically.

Suhash Palshikar in his article ‘Gandhi-Ambedkar interface: when shall the Twain meet?,’
says that both the Gandhian and Ambedkar’s discourses were fundamentally concerned with
the emancipation of the oppressed sections of the society. However, they differed in their
approaches and areas of concentration.

5(d) Highlight the role women in Freedom Struggle.


According to Mahatma Gandhi , “To call women the weaker sex is a libel, it is men’s injustice
to women.” This statement reflects the importance that national leadership gave to the role
of women in the national movement.
The then Home Secretary of the British govt had to confess that nothing had disturbed him
more than the great awakening among the Indian Women and the part played by them in
Indian politics.
When most of the men freedom fighters were in prison the women came forward and took
charge of the struggle.
Woman's participation in India's freedom struggle began as early as in1817 .Bhima Bai
Holkar fought bravely against the British colonel Malcolm and defeated him in guerilla
warfare. Many women including Rani Channama of Kittur, Rani Begam Hazrat Mahal of
Avadh, Rani Lakshmi Bai of Jhansi fought against British East India company in the 19th
century.
Starting with Sarojini Naidu, Vijayalakshmi Pandit, Kamaladevi Chattopadhyay and Mridula
Sarabhai at the national level, we may go on to provincial level leaders like Annie Mascarene
and A.V. Kuttimaluamma in Kerala, Durgabai Deshmukh in Madras Presidency, Rameshwari
Nehru and Bi Amman in U.P., Satyawati Devi and Subhadra Joshi in Delhi, Hansa Mehta and
Usha Mehta in Bombay and several others.
Besides all these Indian women, there were also Irish women like Annie Besant and
Margaret Cousins, who brought their own knowledge of the Irish experience of British
exploitation to bear on India.
Mahila Samitis like the Mahila Shilpa Samiti and the Lakshmir Bhandar were set up.
Women’s journals like the Bharat Mahila played their role mainly in rejecting British
goods.From the inception of Indian National Congress (1885) its membership was open to
women also. Women organized Swadeshi fairs and Saraladevi Chaudhurani opened
‘Lakshmir Bhandar’ selling only indigenous articles. Women’s wing of Abhinav Bharat named
it Atma Night Yuvati Sabha secretly assisted the revolutionaries.
With his experience of South Africa behind him, Gandhi was conscious of the female
potentialities as passive resisters. During the Champaran Satyagraha,1917, Ahmadabad
labour movement of Feb 1918, women fought a brave battle. Thus, Madhu Kishwar argues
that it was in the course of the Gandhian movement that the single woman acquired a sense
of dignity and came to be respected for her political work.
IN conclusion, Women’s participation has been there in the Indian Nationalist Movement
right from 1857 till the Independence in 1947. Women’s participation enriched India’s
freedom struggle across geographies, time and mode of resistance .

5(e) Indian parliament is in a state of decline. Comment.


India's founders saw parliament as an institution that would provide social and political
unity. The fragmentation that followed the decline of the Congress has not rejuvenated the
country's legislature. While parliament has become more representative, it has declined in
status and effectiveness. While it continues to be a ‘reactive legislature’, parliament's role in
India's political system is more marginal than it was in the country's early years.
Some of the major functions of the parliament include legislative functions i.e. making laws
on all subjects listed in the Union List and the Concurrent List. Parliament maintains financial
Control over executives . The existence of opposition ensures that the nation gets to know
about the alternative points of view. Parliament is the most powerful organ so far
information about the functioning of the government is concerned. The information provided
in the Houses is authoritative and Ministers are bound to provide information on matters of
government when so desired by the members. The power to amend the Constitution vests
with the Parliament.
The analysis of the role of parliament can be carried out on the basis of three governance
indicators - law-making, social inclusion and institutional effectiveness.
In the 1950-60s, Parliament, the law-making organ of the State, used to meet, on an
average, 125- 130 days in a year. This has come down to just 65-70 days in the last decade.
The second indicator is the social inclusion of Parliament - to what extent Parliament has
been able to accommodate the different social communities and groups, making it a
representative of Indian society. The current Lok Sabha, in terms of Religions, 90.4% are
Hindus, Muslims constitute 5.2% and Other religious minorities 4% . Women makes just
14.6% of the number. The third indicator is the institutional decay of parliamentary efficacy.
As per data from the Association of Democratic Reforms 43% of the 17 th Lok Sabha members
have criminal charges against them.
Successive governments have exploited the spirit of this constitutional provision.Bringing in
law through the ordinance route also bypasses parliamentary scrutiny. PRS Legislative
Research shows that much of the desired expert consultation and deliberation does not
occur anymore. The committees run on poor infrastructure and logistics, with members
attending infrequently.
To sum up, Parliament appears to be quite ineffective in all its functions. The reason for
having a legislature separate from the executive is to have a check on executive power.
Our country needs a robust lawmaking process, one that does not bend to executive urgency
or succumb to political expediency. The government must ensure that it identifies the gaps in
our legal system proactively. Enacting statutes without proper scrutiny wastes the
legislature’s time when the government approaches Parliament to amend such laws.
There is an urgent need for reforms in the composition and conduct of Parliament. We, the
people, have to exercise the power of active and conscious citizenship, not only to send
honest individuals to the Lok Sabha, but also to sustain our democracy and the supremacy of
Parliament.

6(a).The creation of independent Union Ministry of Cooperation and the recent verdict of
Supreme court on cooperatives has brought a renewed focus on the relevance of
cooperatives in India’s Political economy of reforms. Comment. Suggest the pathways to
revitalize cooperatives in India.
The International Cooperative Alliance (ICA) defines a Cooperative as “an autonomous
association of persons united voluntarily to meet their common economic, social, and
cultural needs and aspirations through a jointly-owned and democratically-controlled
enterprise.”

The cooperative movement began in Europe in the nineteenth century, primarily in England
and France. The industrial revolution and the increasing mechanization of the economy
transformed society and threatened the livelihoods of many workers. Robert Owen is
considered the father of the cooperative movement. Owen had the idea of forming "villages
of cooperation," where workers would drag themselves out of poverty by growing their own
food, making their own clothes, and ultimately becoming self-governing.

The Ministry of Co-operation was formed in July 2021. The ministry provides a separate
administrative, legal and policy framework for strengthening the cooperative movement in
the country with a vision statement of "Sahkar se samriddhi". The ministry works in
strengthening co-operatives at the grassroot level, working to streamline processes for 'Ease
of doing business' for co-operatives and enabling the development of Multi-State Co-
operatives (MSCS).
Co-operative societies, being a subject of State List under the Seventh schedule of the
Constitution, many experts raised concerns that creation of a new Cooperation ministry is an
infringement upon the federal rights of the state governments.
Recently, in UoI Vs Rajendra N Shah Case, 2021, the Supreme Court struck down parts of the
97th Constitutional Amendment which shrank the exclusive authority of States over its co-
operative societies. The court held that co-operative societies come under the “exclusive
legislative power” of State legislatures. However, the court did not strike down the portions
of Part IXB of the Amendment concerning ‘Multi State Cooperative Societies’ due to the lack
of ratification. The “Ministry of Cooperation” will continue to have the power to look into the
work of “multi state” co-operative societies.

The importance of Cooperative sector for India lies in the fact that substantial chunk of
Indian population is poor, illiterate and unskilled. The cooperatives penetrate into crucial
areas like agricultural credits. It provides crucial Inputs for the agricultural-sector, consumer
societies meet,remunerative price. Cooperatives are also involved in building rural agri-
infrastructure. Cooperatives function beyond bureaucratic framework, keeps unity among
members and encourages democratic values. Some successful cooperatives in India include
Pravaranagar Udhyog Samuha, Lijjat Papad cooperative, Amul etc. Their success is owed to
following features which need to be replicated elsewhere:
1. An integrated approach with participation of community and the management being
attuned to the need of most marginalised.
2. Strong democratic ethos and fair elections with single voting right for each member
irrespective of number of shares.
3. Use of modern technology.
4. Mobilization of women and rural workers.

There is a need to replace the existing government dominated cooperative laws by a new
people centric legislation. Reducing the politicisation of cooperatives by ensuring periodic
free and fair elections will be helpful. Accountability ,periodic audit and bringing them in the
purview of RTI will make their functioning more transparent.

These cooperatives have immense potential to deliver goods and services in areas where
both the state and the private sector have failed. Co-operatives need to run on well
established democratic principles and elections held on time and in a free and fair manner.

6(b).Critically examine the role played by National Commission for Women with respect to
the empowerment of Women in India.

The principle of gender equality is enshrined in the Indian Constitution in its Preamble,
Fundamental Rights, Fundamental Duties and Directive Principles.
The Constitution not only grants equality to women, but also empowers the State to adopt
measures of positive discrimination in favour of women. India has also ratified various
international conventions and human rights instruments committing to secure equal rights
of women like the Convention on Elimination of All Forms of Discrimination against Women
(CEDAW) in 1993.
The National Commission for Women was set up as statutory body in January 1992 under
the National Commission for Women Act, 1990 to review the Constitutional and Legal
safeguards for women ; recommend remedial legislative measures ; facilitate redressal of
grievances and advise the Government on all policy matters affecting women. The report
prepared by the commission and handed over to Central and state Governments are laid
before respective legislatures.
NCW has been given powers of a Civil Court like summoning and enforcing the attendance of
any person from any part of India and examining him on oath;requiring the discovery and
production of any document;receiving evidence on affidavits etc.
NCW’s performance has been mixed . The commission was successful in securing the release
of female who were allegedly gang raped by BSF personnel in early 2002.The ‘Legal cell’ of
the commission has recommended modifications in a number of acts and proposed a
number of new bills. It has recommended amendments to Hindu Manage Act, medical
termination of pregnancy act and Indian Penal Code on Adultery to make it a civil offence.
The Commission has initiated various steps to improve the status of women and worked for
their economic empowerment.NCW acts suo-moto in several cases to provide speedy justice.
It has taken up the issue of child marriage, sponsored legal awareness programmes,
Parivarik Mahila Lok Adalats and reviewed laws such as Dowry Prohibition Act, 1961, PNDT
Act 1994, Indian Penal Code 1860 and the National Commission for Women Act, 1990 to
make them more stringent and effective.
On the other hand NCW suffers with constitutional Weaknesses. It has been constituted as a
less powerful body . The NCW suffers from Organisational infirmity as it has two centres of
power - The Chairman and The Member Secretary
Despite being statuted as an autonomous body, in terms of its financial and administrative
powers, the Commission remains subordinated to bureaucracy. NCW gets their funds and
functionaries from the govt.,thus hesitate from being critical against govt. Politicized nature
of NCW is evident from the fact that it took a pro-govt stand in Gujarat riots. The approach
of NCW in Manorama Devi case, Soni Suri case, in Muzaffarpur shelter home case and very
recently in Hathras has not been satisfactory.
Sadhana Arya in her article titled “National Commission For Women: Its Overview And
Performance” published in EPW, has highlighted that the present composition is not in the
manner suggested by the women organisations. The selection process is not transparent and
number of times such persons are appointed which themselves reflect patriarchal attitude.
Therefore, the Commission must be granted the power of choosing its members. The
members should be chosen with no prejudice and will have a good knowledge of the law and
understand society and human behaviour. The selection of the chairperson and members
should be transparent and democratic.
It is high time government should take to empower the institution to address the interest of
women in India in line with the constitutional commitment of gender justice

6(c) While the Apex court has delivered several progressive pronouncements over the
decades to root out the evil of criminalisation of politics yet the sad reality is there is
hardly any change on the ground level. Comment
Gandhi followed the maxim that in politics ‘means are after all everything’. Unlike
Machiavelli, he never subscribed to the principle that the end justified the means. He
compared the means to a seed and the end to a tree, and emphatically said: “There is just
the same inviolable connection between the means and the end as there is between the seed
and the tree”.
The Supreme Court has time and again ruled in favour of arresting the increasing
criminalisation of politics. In Union of India v. Association for Democratic Reforms, 2002, the
SC ruled that candidates shall disclose their assets and liabilities, criminal conviction, if any,
and pending cases-nomination papers. In Lily Thomas v. Union of India, 2013, the SC held
that that Section 8(4) of The Representation of the People Act, 1951 is unconstitutional
which allows MPs and MLAs who are convicted to continue in office till an appeal against
such conviction is disposed of. The court held that MP/MLA convicted for two years or above
would be disqualified immediately. In Public Interest Foundation Vs UoI 2014 , the SC
directed the trial courts to complete the trial of cases involving the legislators within one
year to introduce internal democracy, financial transparency and accountability in the
working of the political parties.
Similarly , in Rambabu Singh Thakur Vs Sunil Arora & Ors, 2019 ,the Court directed that
parties should publicise with regard to the criminal cases pending against their
candidates.The political party concerned shall be obligated to put up on its website the
aforesaid information pertaining to candidates having criminal antecedents. The SC issued
the following directions in exercise of it's constitutional powers under Articles 129 and 142 of
the Constitution of India.
Inspite of all these rulings , the Association of Democratic Reforms reports says that 43% of
the 17th Lok Sabha members have criminal charges against them, a 26% increase as
compared to 2014. The party wise distribution is such that BJP has 116 MPs or 39% of its
winning candidates with criminal cases, followed by 29 MPs (57%) from the Congress, 13
(81%) from the JDU, 10 (43%) from the DMK and nine (41%) from the TMC.
The years immediately after independence in India, were full of hope and enthusiasm. Rajni
Kothari has described these years as period of ’politics of trust’.However, transformation of
Indian politics from an ‘organic’ entity into a mere mechanical electoral system, narrowed
the domain of politics.
In conclusion, we can agree with the Supreme Court’s observation that the increasing trend
of criminalisation of politics strikes at the very root of democracy. There is a need to
empower Election Commission of India to firmly deal with the situation by securing its
independence through a bipartisan selection process, and through more resources and
manpower. In addition, more transparency in political funding; a cap on party expenditure
or State funding of election and ban on donations by government companies as suggested
by Indrajit Gupta committee can be introduced.
7(a) Trace the trajectory of UCC debate in India. Do you think Uniform Civil Code is suitable
for the country which pride itself on the Idea of “Unity in Diversity”.
A uniform civil code here refers to a single law, applicable to all citizens of India in their
personal matters such as marriage, divorce, custody, adoption and inheritance.
It is intended to replace the system of fragmented personal laws, which currently govern
interpersonal relationships and related matters within different religious communities.

The code comes under Article 44 of the Constitution, which lays down that the state shall
endeavour to secure a Uniform Civil Code for the citizens throughout the territory of India.

The origin of the UCC dates back to colonial India. There was division on the issue of putting
Uniform Civil Code in the fundamental rights chapter. Dr. B R Ambedkar, while formulating
the Constitution had said that a UCC is desirable but for the moment it should remain
voluntary. Mohammed Ismail, who thrice tried unsuccessfully to get Muslim Personal Law
exempted from Article 44, said a secular state should not interfere with the personal law of
people.

Alladi Krishnaswami, who was in favour of a Uniform Civil Code, conceded that it would be
unwise to enact Uniform Civil Code ignoring strong opposition from any community.

In June 1948, Rajendra Prasad, President of the Constituent Assembly, warned Jawaharlal
Nehru that to introduce “basic changes” in personal law was to impose “progressive ideas”
of a “microscopic minority” on the Hindu community as a whole.

Article 44 of the Constitution lays down that the state shall endeavour to secure a Uniform
Civil Code for the citizens throughout the territory of India. Being a directive principle and as
defined in Article 37, is not justiciable but is fundamental in governance.

The courts of India have been prodding successive governments to bring in a UCC as
enshrined in Article 44 of Indian Constitution. They raised this issue in 1985, 1995, 2015,
2017, while overturning triple talaq.
In recent times ,the debate was renewed with the judgment pronounced in the Mohd
Ahmed Khan v Shah Bano Begum and Others, 1985. The Court reiterated the same issue
later in a slightly different context, that of a marriage between a Christian woman and a Sikh
man under the Indian Christian Marriage Act, 1872 in Jordan Diengdeh v S S Chopra (1985).

In Lily Thomas v Union of India and Others (2000), the SC said that the Court could not direct
the centre to introduce a UCC, but that did not stop the Supreme Court from once again
affirming the necessity and desirability.
Recently, the SC has put the spotlight back on the debate in Jose Paulo Coutinho vs Maria
Luiza Valentina Pereira, 2019. The SC described Goa as a “shining example” with a Uniform
Civil Code and said that the founders of the Constitution had “hoped and expected” a UCC
for India but the government had made no attempt yet.

UCC has received the support of the courts, many organisations working for women’s rights,
and, of course, one major political party. UCC would, in theory, provide equal status to all
citizens irrespective of their community. Personal laws, because they derive from tradition
and custom, also tend to give undue advantage to men. The Law Commission observes in its
2018 consultation paper, “Various aspect of prevailing personal laws disprivilege women.”
Examples - Muslim men being allowed to marry multiple wives, but women being forbidden
from having multiple husbands. UCC de-links law from religion which is a very desirable
objective to achieve in a secular and socialist pattern of society.
However, the idea of a UCC clashes with the right to freedom of religion (Article 25) , which
has been particularly important for minorities. Article 26(b) upholds the right of each
religious denomination or any section thereof to “manage its own affairs in matters of
religion”; Article 29 defines the right to conserve distinctive culture. Critics hold that a state
action to introduce the UCC is against the quintessence of democracy.
According to the Law Commission(2018), the best way forward is to reform the personal
laws i.e. preserve the diversity of personal laws but at the same time ensure that they are
not discriminatory across genders and do not violate the fundamental rights as enshrined in
the constitution of India.

7(b) Highlight the contours of the right to privacy given by supreme court in Puttaswamy
Case. What concerns emerge with respect to the future of India’s democracy due to
software’s like Pegasus.
“If you want to keep a secret , first hide it from yourself.”George Orwell , 1984
The status of right to privacy has once again come into question with the pegasusu
controversy . It has been alleged that the government utilising modern technology has
invaded into the right to privacy against the democratic norms .
The Supreme Court, in Puttaswamy Case has declared Right to Privacy as a fundamental
right under Art.21 , interpreting it in conformity with Art.12 of the Universal Declaration on
Human Rights and Art.17 of the International Covenant on Civil and Political Rights, 1966.
Both of these international documents provide for the right of privacy.
The fundamental right to privacy has been developed by the courts in this manner for over
60 years. It all began when the attorney general of India, while defending the Aadhaar
project, argued that the Constitution does not include within it a fundamental right to
privacy. He based his conclusion on two cases decided by the Supreme Court—one,
MP Sharma v. Satish Chandra, decided by an eight-judge bench in 1954 and the other,
Kharak Singh v. State of Uttar Pradesh, by six judges in 1962. Both cases had held, under
different circumstances, that the Constitution of India does not specifically protect the right
to privacy.
In 2012, Justice K.S. Puttaswamy (Retired) filed a petition in the Supreme Court challenging
the constitutionality of Aadhaar on the grounds that it violates the right to privacy. During
the hearings, the Central government opposed the classification of privacy as a fundamental
right. The government's opposition to the right relied on two early decisions—MP Sharma vs
Satish Chandra in 1954, and Kharak Singh vs State of Uttar Pradesh in 1962—which had held
that privacy was not a fundamental right.
However, in Justice K.S. Puttaswamy (Retd) vs Union of India case, finally the contours of
Right to Privacy was detailed out. In the judgment, the nine-judge bench said, “The right to
privacy is protected as an intrinsic part of the right to life and personal liberty under Article
21 and as a part of the freedoms guaranteed by Part III of the Constitution.”
Justice Chandrachud held that, though right to privacy is not a fundamental right, yet any
invasion of life or personal liberty must meet the three requirements of
A. legality, i.e. there must be a law in existence;
B. legitimate aim, which he illustrates as including goals like national security, proper
deployment of national resources, and protection of revenue; and
C. proportionality of the legitimate aims with the object sought to be achieved.
The lead judgment calls for the government to create a data protection regime to protect
the privacy of the individual. It recommends a robust regime which balances individual
interests and legitimate concerns of the state. The judgment has ramifications for a number
of contemporary issues pending before the supreme court. In particular, two proceedings—
WhatsApp-Facebook data sharing and on Pegasus—has become the test grounds for the
application and contours of the right to privacy in India.
The Pegausus issue raises serious question about right to privacy , especially in context of
digitalisation. It is alleged that Pegasus can compromise the digital security through zero
click vulnerabilities. It has the capacity to access the entire stored data. Since, the NSO Group
purportedly sells the softwares only to government agencies , the matter give rise to the
possibilities of Orwellian state in India. We live in an era of information revolution where the
entire lives of the people is stored in the cloud . If the technology can be used to improve
lives , it can also be used to breach the sacred sphere of Man’s privacy. Every member of a
civilised society has a reasonable expectation of privacy to enable them to exercise choices,
liberties and freedom. In Puttaswamy case , it is seen as sacrosanct as human existence. As
per judgement Right to Privacy can be made limited only by law subjected to Judicial Review
. However, in this case there is a direct infringement of rights. Though state can interfere ,
yet it needs to follow the procedure even if it is for national security .Since, the right to
privacy can also be linked to Freedom of Press , it has impact on democracy.
The Supreme Court of India ordered an independent probe into the issue by a three-member
technical committee. The three-judge bench, headed by Chief Justice of India N V Ramana
said right to privacy was sacrosanct even as it clarified that such a right can’t be said to be
an absolute as the Constitution envisaged certain limitations on it.

7(c) How caste census is linked to Politics of distributive Justice in India. What are the
various dimensions of debates within India’s reservation policy?
Distributive justice concerns the socially just allocation of goods. Principles of distributive
justice provides moral guidance for the political processes and structures that affect the
distribution of benefits and burdens in society. The basic principle of distributive justice is
that equal work should produce equal outcomes and some people should not accumulate a
disproportionate amount of goods.
There are 5 types of distributive norms:
A. Equality: Regardless of their inputs, all group members should be given an equal
share of the rewards/costs. It is also known as 'Strict Egalitarianism'
B. Equity: Members' outcomes should be based on their inputs. Equals should be treated
equally and unequal, unequally
C. Power: Those with more power should receive more than those in lower-level
positions.
D. Need: Those in greatest needs should be provided with the resources needed to meet
those needs.
E. Responsibility: Group members who have the most should share their resources with
those who have less.
India’s caste census is an attempt to remedy the past injustices suffered by those who are at
the lower levels of India's four-tier caste hierarchy. Caste Census is the caste-wise tabulation
of population in the census exercise. Caste, was last included in the Census of India back in
1931. The practice was stopped by the British in 1941 and the post 1947, the government did
not revive it.
In recent months, there has been a strong demand by various political parties to have a
caste-based enumeration in the 2021 Census which has triggered a serious debate.
The critics argue that reasonable estimates of the broad social break-up of India’s population
is already available as various government surveys such as the ones conducted by the
National Sample Survey Office (NSSO) and National Family and Health Survey (NFHS). A full
caste census, including a jati-wise break-up of all ‘upper castes’, would pose some
difficulties, since we don’t have an official list of all castes in the country. In India voters
don’t cast their votes, they vote their caste. Break up of population in various caste would
further strengthen caste-based politics in India. There can be rise in demand for reservation
as being demanded by Jats, Patels and Marathas.
On the other hand from academic and policy point of view, the collection of such data will be
useful to understand people’s socioeconomic status by caste and sub-caste. It will help know
whether educational and job opportunities are spread evenly across castes, along with data
on household assets and life expectancy. Caste Census may provide solid evidence, missing
so far, that some castes are rather privileged and do not deserve to be on the OBC list
anymore. Yogendra Yadav argues that for a country like India that runs such a large
affirmative action programme based on caste , data on educational and economic profile of
castes is a must. Regardless of the ongoing debate, it is important to continue the discussion
on the existing caste data, how it has been used and understood by the government and its
various departments to grant or withdraw benefits. Also its importance and utility for the
important academic exercise of mapping social inequalities and social change.

The Reservation, whether on the basis of economic status or social and educational
backwardness has been the centre for debate. Scholars like Prof. Satish Deshpandey and
Yogendra Yadav have supported the reservation policy in India. On the other hand
academicians like Ramchandra Guha, Pratap Bhanu Mehta believe that the policy has led to
the race to bottom as the demand for being classified as “backward” has been on the rise .
over the period, various dimensions of the debate has emerged within India’s reservation policy.
The debate is over the criteria on the basis of which should the reservations be given like gender,
economic status, social backwardness etc. What should be the quantum of the reservation and for
what duration . There are certain super speciality jobs in which reservation policy is not applied.

Thus, still the reservation question is attracting lot of public debate and it has been rightly said by
Ram Chandra Guha , that the debate on reservation policy in India generates more heat than light.

8 (a) Discuss the changing Trends in India’s electoral behaviour on the basis of the recent
elections. How does the understanding of the changing Trend in the Electoral behaviour is
key to understand countries democratic trajectory in the long run ?
The study of electoral behaviour is a result of the growth of Behavioural Movement in
Political Science. According to Milan Vaishnav ‘The study of electoral behaviour in India is a
challenging task because of size and diversity.” Kenneth Arrow has given “Impossibility
Theorem” in which he has said “In case of election, it is difficult to determine the preference
when voters have more than three choices. Still, India is a very interesting case study of the
electoral behavior.”
In democratic systems, each adult citizen uses ‘voting’ as a means for expressing his
approval or disapproval of governmental decisions, policies and programmers of various
political parties and the qualities of the candidates who are engaged in the struggle to get
the status of being the representatives of the people. In India, the behaviour of voter is
influenced by several factors such as religion, caste, community,language, money, policy or
ideology, purpose of the polls, extent of franchise, political wave etc.
In India following main political and socio-economic factors which act as determinates of
voting behaviours in our Democratic system:
(1) Charisma, like Nehru, Indira, Vajpayee ,Modi.
(2) Caste continues to be a determinant of voting behaviours in India.
(3) Religion, due to existence of such political parties and neo-political groups as stand
linked with a particular religion, for example, Akali Dal, Hindu Maha Sabha, Shiv sena
etc.
(4) Language also serves as factor in voting behavior.
(5) Money
(6) Communalism, regionalism and sub-nationalism are the hard realities of Indian
society.
According to LokNiti, a part of Centre for the Study of Developing Societies, we can see the
following trends in Indian electoral behavior
 Caste and Religion remain the major long term determinant
 Corruption and Anti-incumbency does not matter. No difference in the preference of
Men and Women, Rural and Urban scenarios
For Pranav Dhal , the recent elections show the following shifts:
A. Nationalism as a political arguments is making a strong comeback which was
overshadowed by regionalism and secularism .
B. There was a time when national parties were depended on regional parties but the
trend is getting reversed .
C. There is an attempt to redefine caste as NDA had introduced a more flexible
economically weaker section category on the basis of deprivation index developed by
SECC (Social and economic caste census). Thus , old caste alliances are shifting for the
new.
D. There is a big shift against the sense of entitlements and elites coming from dynasty
politics .
However, economists Abhijit Banerjee, Thomas Piketty and Amory Gethin published a paper
on Indian electoral behaviour , in which they argued “voters seem to be less driven by
straightforward economic interests than by sectarian interests and cultural priorities. In
India, as in many Western democracies, political conflicts have become increasingly focused
on identity and religious-ethnic conflicts rather than on tangible material benefits and class-
based redistribution.”
Prannoy Roy, Ashok Lahiri, David Butler in their book “India Decides” show that that the
voting behavior of Indians is many times more mature that then voting behavior of the
voters in Western Countries.
According to Prof. M P Singh: The credit for success for India’s democracy go to the great
common sense shown by ordinary voters in India.

8(b) To interfere with Freedom of Speech and Expression is the worst form of “Tyranny”. In
the context of the statement do you think that there is strong case to question the
continuation of sedition law in India?
“I disapprove of what you say, but I will defend to the death your right to say it.”― S.G.
Tallentyre, The Friends of Voltaire.

Freedom of speech is a fundamental human right that supports the freedom of an individual
or a community to articulate their opinions and ideas without fear. The right to freedom of
expression is recognized as a human right under Article 19 of the Universal Declaration of
Human Rights (UDHR) and under Article 19 of the International Covenant on Civil and
Political Rights (ICCPR). In the United States, freedom of speech and expression is strongly
protected from government restrictions by the First Amendment to the United States
Constitution, many state constitutions, and state and federal laws.
Being a truly liberal and democratic document, the Indian Constitution too grants Right to
freedom of speech and expression under Article 19(1)(a). The Preamble explicitly mentions
‘liberty of thought and expression’ principle thereby ensuring freedom of speech and
expression to Indian citizens.

However, the constitution also ensures that right to free expression is not equated or
confused with a license to make accusations unfounded and are irresponsible. Therefore, the
right to speech and expression is not an absolute right, and the State can enforce reasonable
restrictions under Article 19(2) of the Constitution. Every constraint on the practice of the
right referred to in Article 19(1)(a) which does not come within the four corners of Article
19(2) might not be applicable. The State can enforce reasonable restrictions on the exercise
of the right to freedom of speech and expression based on grounds of defamation, Contempt
of court, decency or morality, security of the state, friendly relations with other states,
sedition, public Order and incitement to an offence.

In June , 2021, while hearing a case involving sedition charges on two Telugu news channels,
the apex court called for renewed scrutiny of the “ambit and parameters” of the laws
governing sedition. In Vinod Dua Case, the court observed that “Every journalist is entitled
to protection under the Kedar Nath Singh judgment ”. In 1962 Kedar Nath Judgement, a
constitution bench of the Supreme Court considered the constitutionality of sedition in
Section 124A of IPC and held that a citizen has a right to say or write whatever he likes about
the Government, or its measures, by way of criticism or comment, so long as he does not
incite people to violence against the Government established by law or with the intention of
creating public disorder.”

Recently, Chief Justice of India N V Ramana has ignited a passionate debate concerning
whether “sedition” should be an offence at all, and how to prevent its misuse or abuse, were
it to remain.
Sedition is defined as per Section 124-A of the Indian Penal Code (IPC) as, “whoever, by
words, either spoken or written, or by signs, or by visible representation, or otherwise, brings
or attempts to bring into hatred or contempt, or excites or attempts to excite disaffection
towards, the Government established by law in India, shall be punished with imprisonment
for life, to which fine may be added, or with imprisonment which may extend to three years,
to which fine may be added, or with fine.” This law was enacted in 1860, under the British
Raj, to prevent any offences against the state.

The critics argue that incitement to violence or disorder are already offences under different
sections of the penal code. Thus the there is no need to have a separate Section 124A.
Moreover, mere criticism cannot be called as sedition unless it has potential to encourage
people to be violent. In a democratic setup right to speech and expression under Article
19(1)(a) is as valuable as right to life guaranteed under Article 21 of the Indian constitution.
Expression of dissatisfaction against government ought to be allowed as it functions in the
same way as safety valves work in pressure cooker to ease out the pressure. Section 124A
has been repeatedly misused by government to suppress political dissent, constructive
criticism of government and its policies thereby stifling democratic governance.

Upendra Baxi in his article, “Why India at 75 is ready for a sedition-less future” argues that
Section 124-A needs to be wholly judicially repealed at the bar of the fundamental human
right to free speech.

While it is important for citizens to exercise our fundamental rights within reasonable limits
laid down by law, there is a greater obligation on the establishment to ensure that the laws
are not twisted, misused or abused in such a manner that citizens are deprived of
fundamental rights that impact the liberty of an individual. It is to be realised that in any
democracy, there are bound to be different points of view. These must be respected –
otherwise the fabric of our society might disintegrate, and fraternity, one of the key words in
the preamble to our Constitution might just become another dead idea.

8(c) Though the constituent assembly has given special focus to the Inter-state water
disputes yet it has overlooked the political ramifications of Inter-state boundary dispute
in context of the smooth functioning of India’s cooperative federalism. Discuss.
The Inter-state relations in India run along the lines of both conflicts and cooperation. Sharing
of water and territories are among the contentious issues’ between two or more states. For a
country like India with considerably low per capita availability of water, agrarian economy
and where most of the rivers are inter-state, it is natural that constituent assembly has given
special attention to this category of dispute

Constituent Assembly provided several constitutional provisions for dealing with federal
disputes. Article 131 deals with the original jurisdiction of SC to deal with federal disputes of
legal nature. Up till 1956, the mechanism used for the resolution of water dispute has been as
per article 131. In 1956, government has brought two acts invoking article 262. As per Article
262, if parliament thinks fit it can provide for the resolution of disputes by bringing a law. It
even suggests that parliament may exclude the jurisdiction of courts including Supreme Court.
Inter-state councils (Article 263) were envisaged to be a platform to tackle such disputes.
Besides, Zonal Councils are extra-constitutional bodies, which were created by the state
Reorganisation Act 1956 to offset the negative consequences of state reorganization,
especially in case of water disputes.

Inspite of these provisions , the Inter-State River Water Disputes are one of the most
contentious issues in the Indian federalism today. The recent cases of the Cauvery Water
Dispute and the Satluj Yamuna Link Canal are some examples.

Added to these, many inter state territorial disputes have also emerged like territorial Disputes
related to Belgaum between Maharashtra and Karnataka; Chandigarh, Abohar Fazilka town
between Punjab and Haryana. There are many territorial disputes in North-East India.
Recently, the escalating tensions between two north eastern states – Assam and Mizoram –
consumed the lives of six Assam police personnel.

Abdul Gafoor Noorani, a senior Indian constitutional expert, believes that the draftsmen of
India’s constitution had erred grievously in ignoring the problem of interstate disputes,
especially interstate boundaries. The constitution makes no provision for a swift and binding
decision of such disputes. Article 262 is on the adjudication of disputes relating to waters of
inter-state rivers or river valleys. There is no comparable provision on disputes on land.

According to Pushpita Das (Institute for Defence Studies and Analyses), the interstate border
disputes in the northeast of India have persisted for a long time. Several attempts at resolving
these disputes have been unsuccessful indicating the collective failure of union and state
governments. After every major incident, the government or the Supreme Court have
appointed commissions, which recommended ways to resolve the issue. However, since these
recommendations were not binding upon the states, one or the other state government
rejected the reports if its claim was not favored.
India needs multi-dimensional approach—legal, administrative, constitutional and
political— to address these federal disputes . To strengthen the cooperative federalism,
parochial mindset making regional issues superior to national issues should not be allowed.
Disputes must be resolved by dialogue and talks and the political opportunism must be
avoided.A robust and transparent institutional framework with cooperative approach is need
of the hour.
PSIR Crash Course 2021
Model Answers - Test 6

For any issue related to PSIR Test Series, write only at psirtestseries@shubhraranjan.com.

Section A

1(a) Discuss the limitations of structural functional approach to the study of comparative
politics.

Structural Functional approach is an attempt to address the limitations of Systems approach


which suffers from the following defects:

 Too general/ Macro View


 Status Quoist/ Modelled on Western Countries

In order to develop more universalist model which is relevant for the study of developing
societies, he incorporated the functionalist approach of Anthropologists like Malinowski and
Radcliffe Brown. According to these scholars every society performs certain essential functions,
though the structure performing the functions may vary. It also incorporates some aspects of
institutional approach.

Almond highlighted seven input-output functions common to all political systems. On the input
side he listed these functions as: political socialization, political interest articulation, political
interest aggregation, and political communication. On the output side, he listed rule-making,
rule implementation, and rule adjudication.

Thus, Almond has given the four features of the Political System:

1. Every Political system has its own structures


2. Political Structures are multi-functional
3. Political Systems are Dynamic i.e. they keep on evolving
4. Despite differences in the structure, all perform similar functions

In Politics of Developing areas, Almond and Coleman defined Political System as ‘a system of
interaction found in all independent societies, performing the functions of integration and
adaptation by the means of legitimate physical force, aiming at maintaining the order in the
system.’

Limitations of Structural Functional Approach to the study of Comparative Politics can be


discussed as:

(1) Status Quoist:

Despite aim of addressing the status quoist nature of Systems approach, even structural
functional approach continue to be concerned, with the problems of systemic survival, the
requirement of stable adaptation and the operation of various functions and structures
oriented towards system maintenance.

(2) Problems of Operationalisation

It ignores the fact that there is no scientific way to determine either the essential nature,
structure and the functions of a political system.

(3) Fails to lay down Priorities of Functions:

While suggesting the study of functions, it fails to specify which functions are more important
than the others. Without such a standard, one has to accept an unacceptable proposition that
all functions are basic to all the political systems.

(4) Ignores the study of Crises and Revolution:

This approach ignores the study of political crises and revolutions or military and civil coups,
which characterise or can characterise all political systems, particularly the Asian and African
political systems.

(5) Excludes the study of Values:

The Structural-Functional Approach is again inadequate because it gives no place to the study of
goals and values which condition the political behavior in all the political systems.

Despite these limitations, one cannot deny the utility of Structural-Functional Approach in the
study of Comparative Politics. In fact, several political scientists have successfully and
productively used this approach in the study of both Politics and Comparative Politics. Almond
and Powell have been its strongest protagonists. It is a good conceptual framework for
organizing the data at the initial level of research.

1(b) Comment on Morgenthau’s views on national interest.


“The meaning of national interest is survival—the protection of physical, political and cultural
identity against encroachments by other nation-states”—Morgenthau.

Hans Morgenthau’s concept of “the national interest” first appeared in the essay “The Primacy
of the National Interest” as part of the 1949 issue of The American Scholar titled “The National
Interest and Moral Principles in Foreign Policy.” As William Scheuerman observes, “The
concept of the ‘national interest’ first takes on a special analytic status in this essay.” In the
essay, the national interest is first presented as a necessary corrective to what Morgenthau
had already characterized in Scientific Man vs. Power Politics as legalism, moralism, and
sentimentalism in American politics, and as a more effective guide to foreign policy than the
American tradition seemed able to provide.

The political context is critical to understanding Morgenthau’s thinking on the subject. At the
time of the publication, the cold war was in its early stages, and there was growing public
unease not only about Soviet intentions but also American responses. Morgenthau used the
framework of national interest for understanding the new requirements for an American
foreign policy due to the steady institutionalization of the cold war that had occurred
throughout 1948.

According to Morgenthau, “the concept of national interest is similar in two respects to the
‘great generalities’ of the constitution, such as the general welfare and due process. In
Morgenthau’s opinion, the minimum requirement of nation states is to protect their physical,
political and cultural identity against encroachments by other nation-states.

National Interest defined in terms of National Power: This principle holds that nations always
define and act for securing their national interests by means of power. Each nation
conceptualizes its national interests in terms of power and then acts to secure these by means of
power. In the words of Morgenthau, “The main sign-post that helps political realism to find its
way through the landscape of international politics is the concept of interest defined in terms
of power.”

Instruments and methods for the promotion of national interest according to Morgenthau:

1. Diplomacy: Diplomacy is a universally accepted means for securing national interests. It is


through diplomacy that the foreign policy of a nation travels to other nations. It seeks to secure
the goals of national interests.

2. Alliance: Alliances and Treaties are concluded by two or more states for securing their
common interests. This device is mostly used for securing identical and complementary
interests. However, even conflictual interests may lead to alliances and treaties with like-minded
states against the common rivals or opponents.
3. Propaganda: Propaganda is the art of salesmanship. It is the art of convincing others about
the justness of the goals and objectives or ends which are desired to be secured. It consists of
the attempt to impress upon nations the necessity of securing the goals which a nation wishes
to achieve.

4. Psychological and political warfare: It is an unwritten law of international intercourse that


nations can use force for securing their national interests. International Law also recognizes
coercive means short of war as the methods that can be used by states for fulfilling their desired
goals and objectives. Intervention, embargoes, boycotts, reprisals, retaliation, severance of
relations are the popular coercive means which can be used by a nation to force others to
accept a particular course of behaviour or to refrain from a course which is considered harmful
by the nation using coercive means.

5. Economic methods: The rich and developed nations use economic aid and loans as the means
for securing their interests in international relations. The existence of a very wide gap between
the rich and poor countries provides a big opportunity to the rich nations for promoting their
interests vis-a-vis the poor nations.

All these means are used by all the nations for securing their national interests. Nations have
the right and duty to secure their national interests and they have the freedom to choose the
requisite means for this purpose. They can use peaceful or coercive means as and when they
may desire or deem essential. Thus, Morgenthau made a significant contribution to the theory
of National interest.

1(c) Difference between defensive and offensive realism.

Structural realism as developed by Kenneth Waltz argues that the anarchic system and the
distribution of capabilities are powerful constraints and inducements which produce “sameness”
in the behavior of states.

There are differences within structural realists with respect to the amount of power state must
acquire. There are two views. The first one is defensive realism, proponents include Kenneth
Waltz and Jack Snyder. Defensive realists argue that since states want security, it is possible to
have an international equilibrium that is stable through balancing. They reject the argument of
offensive realists that states seek hegemony and say that it is strategically foolish to pursue
hegemony. States want an appropriate amount of power, not hegemony due to a number of
factors. First, if any state becomes too powerful, other states will balance against it. Second,
conquest is feasible but it would not pay as its costs outweigh the benefits.
John Mearsheimer in his The Tragedy of Great Power Politics (2001), has portrayed offensive
realism, the second view. He argues that states seek to maximize power instead of security.
States constantly seek opportunities to maximise their power and hegemony is their ultimate
goal. This makes it harder to achieve equilibrium in international politics through balancing.
Offensive realists argue that often, balancing is inefficient which allows an aggressor to take
advantage of its adversaries.

The main distinction between these two versions of realism is the role of the anarchic
international system and whether it encourages states to maximize their security or to
maximize their power and influence. For offensive realists security is scarce. The anarchic
nature of the international system compels states to maximize their share of world power and
to seek superiority, rather than equality, in order to make themselves more secure and thereby
increase their odds of survival. John Mearsheimer is clear that “states quickly understand that
the best way to ensure their survival is to be the most powerful state in the system”

For defensive or positional realists, security is plentiful. Major powers seek to maximize their
security by preserving the existing balance of power through mostly defensive strategies.
Defensive realists maintain that the international system encourages states to pursue moderate
and restrained behavior to ensure their survival and safety, and provides incentives for
expansion in only a few select instances. The rationale is that aggression, competition, and
expansion to maximize power through primacy and preponderance are unproductive because
they will provoke the security dilemma and counterbalancing behavior, and thereby thwart the
state’s effort to increase its security. As Christopher Layne concisely notes, “states balance
against hegemons”

In an international system filled with such uncertainty regarding states’ intentions, the nature of
states’ military capabilities and other states’ assistance in a struggle against hostile states, the
two theories of offensive and defensive realism provide a framework to study the emerging
new world order i.e. the decline of USA and the rise of China.

Mearsheimer in the Tragedy of Great Power Politics (2001) has argued that it is difficult to distinguish
between China’s defensive and offensive military capabilities. Thus, both USA and China states will react to
this mutual uncertainty by assuming that the other has the worst intentions. In assuming the worst, both
states, being rational, will attempt to amass power, which gives rise to a security competition with a potential
for war. The rise of China can therefore not be peaceful. China will inevitably make a bid for regional
hegemony while the US and most of China’s neighbors attempt to contain China. Thus, despite starting from a
Defensive Realism, China would slowly become an Offensive power.
1(d) Write a short note on Religious Terrorism with reference to Al Qaeda

“Religion is the Opium of Masses”- Karl Marx

Since 1990s Terrorism has emerged a major security concern in International Politics. The 9/11
2001 incident has dramatically changed the way the issue of Terrorism has been conceived. It
has given rise to the new discourses like “New Terrorism, Global Terrorism, Catastrophic
terrorism, and Religious Terrorism.”

Religious Terrorism is primarily seen as Jihadist/ Islamist terrorism. According to Azzam, Islamic
terrorism has been portrayed as nihilistic movement or manifestation of religious revivalism. It is
better understood as a violent response to the political conditions and the crisis which have
found expression in politico-religious ideology. The reasons for the rise of Religious terrorism are

The large number of government of Muslim states in the Middle East experiencing crisis of
Legitimacy

USA’s geo-political aims in the region specifically the anger against the creation of Israel

The emergence of politically engaged form of religious fundamentalism starting from Iran

The war in Afghanistan has also facilitated the rise of religious terrorism

As far as the relationship between Religion and terrorism is concerned, it can be said that no
particular religion can be seen as promoting terrorism however since Ancient times, religion has
been used and misused in politics as a tool. Terrorism clothed in the Religious ideology gains
larger than life picture, makes struggle for political- economic power a civilizational one with
the aim to gain legitimacy.

According to Mark Juergensmeyer, “Religious terrorism is intimately connected to current forces


of geopolitics.”

Bruce Hoffman has characterised modern religious terrorism as having three traits:

1. The perpetrators must use religious scriptures to justify or explain their violent acts or to
gain recruits.
2. Clerical figures must be involved in leadership roles
3. Apocalyptic images of destruction are seen by the perpetrators as a necessity.
Al Qaeda was founded in 1988. It emerged in the context of the struggle in the Afghanistan
after the Soviet intervention of 1979. Like other groups of anti-Soviet fighters, it was supported
during this period by US funds and arms supplied. After the overthrow of the Taliban regime in
2001, it operated from from the Tribal lands on the Af-Pak border. Its founder Osama Bin Laden
was killed in 2011 by US forces in Pakistan. Apart from September 11, al Qaeda has been
associated with the 1993 attack on the World Trade Centre, the 1996 Khobar Towers bombing,
the 1998 bombings of the US embassies in Tanzania and Kenya, the 2000 attack on the USS
Cole, the 2004 Madrid train bombings and the 2005 London bombings.

Al Qaeda is often credited with having redefined in the nature of terrorism. It is the chief
inspiration behind the sprawling network of Islamic terrorist groups like ISIS and Boko Haram.

1(e) Enumerate Reforms needed in UN peacekeeping.

United Nations Peacekeeping was created in 1948. Its first mission involved the establishment
of the UN Truce Supervision Organization (UNTSO), which served to observe and maintain
ceasefire during the 1948 Arab-Israeli War. UN peacekeeping is a unique global partnership. It
brings together the General Assembly, the Security Council, the Secretariat, troop and police
contributors and the host governments in a combined effort to maintain international peace
and security.

UN Peacekeeping is guided by three basic principles:

1. Consent of the parties.


2. Impartiality
3. Non-use of force except in self-defence and defence of the mandate.

Significance:

United Nations Peacekeeping helps countries torn by conflict create conditions for lasting peace.

Peacekeeping has proven to be one of the most effective tools available to the UN to assist host
countries navigate the difficult path from conflict to peace.

Peacekeeping has unique strengths, including legitimacy, burden sharing, and an ability to
deploy and sustain troops and police from around the globe, integrating them with civilian

peacekeepers to advance multidimensional mandates.


Action for Peacekeeping (A4P)

In 2018, the UN Secretary General had launched Action for Peacekeeping which had highlighted
reforms in the same:

 Advance political solutions to conflict and enhance the political impact of peacekeeping
 Implement the women, peace and security agenda
 Strengthen the protection provided by peacekeeping operations
 Improve the safety and security of peacekeepers
 Strengthen conduct of peacekeeping operations and personnel
 Improve peacekeeping partnerships to enhance collaboration and planning
 Strengthen the impact of peacekeeping on sustaining peace
 Support effective performance and accountability by all peacekeeping components

Other Reforms:

Increase incentivisation of women peacekeepers. As of July 31, women peacekeepers


constituted 6 per cent. There are 5,243 female peacekeepers, out of a total of 86,687
peacekeepers.

Innovative options such as co-deployment of peacekeepers from different countries encourages


a genuine spirit of partnership for peace

There is need for expansion of online initiatives to develop capacities of future commanders and
managers so that they lead by example and raise awareness of UN standards of conduct among
their personnel

India, which has a long tradition of peacekeeping, has contributed more than 250,000 troops in
49 UN peacekeeping Missions. India has given few suggestions to strengthen peacekeeping
which involved pointing out that "Peacekeeping missions cannot be a long-term response to
what are fundamentally political problems" and "it is for the Council to address these issues
rather than burden peacekeeping missions beyond their mandate."

External Affairs Minister S Jaishankar chaired an open-debate under India's presidency of the
Security Council on UN peacekeeping operations with the theme of "Protecting the protectors:
technology and peacekeeping", the second signature event held as India presides over the
powerful 15-member UN body for the month of August.

UNITE AWARE platform -increase situational awareness and provides terrain-related


information to peacekeepers through technology
carry out realistic assessment of contingents being selected and deployed in the mission area.
Troop competencies need to be a critical requirement of selection criteria.

Capacity building and training benchmarks relating to prevention of Sexual Exploitation and
Abuse, gender sensitivity, requirements of conduct and discipline, environmental standards

Increase accountability of troop-contributing countries to adhere to training plans, including


pre-deployment and in mission training, so that key mission tasks are not left to 'learning on the
job"

2(a) The rivalry between the USA and China has become a guiding paradigm of international
relations shaping both scholarly debates and real world of politics. Discuss

Rivalry between the United States and China shapes strategic debates and real political, military
and economic dynamics, and is likely to continue to do so for some time. For the United States,
it can be said that strategic rivalry with China has edged out the “War on Terror” paradigm
that had prevailed since 2001.

Since 2017 China has been treated as a “long-term strategic competitor” in official US
government strategy documents. And in its London Declaration of December 2019 NATO spoke
for the first time of the challenges (and opportunities) presented by China’s influence and
international policies. China’s political elite is – rightly – convinced that the United States is
seeking at the very least to prevent any further expansion of Chinese influence. And while
disputes over trade policy and trade balances feature most prominently in the US President’s
statements and directly affect the global economy, they in fact represent but one aspect of the
rivalry and by no means the most important. The conflict is, as political analyst Peter Rudolf
shows, multidimensional.

The future of USA-China relations has dominated scholarly debates in recent years. Graham
Allison gave the concept of Thucydides trap to project the future of the relation which he argued
would culminate into war. John Mearsheimer in his work on The Tragedy of Great Power also
pointed towards a similar future. Further, in his article ‘the Gathering Storm: China’s challenge
to US power in Asia’ Mearsheimer suggested how the global order is being transformed with the
rise of China in Asia. Joseph Nye suggested that both the countries are likely to engage in
intense rivalry and managing China will be the biggest challenge for USA. In Washington Post,
Fareed Zakaria argued that USA and China are locked in ‘Cold Peace.’

The US regards superiority – and above all military dominance – as an end in itself rather than
simply a means to promote particular interests and values. On the other hand, President Xi
Jinping appears to be driven more by a Chinese vision of world order in which superiority is both
means and end.
From the Chinese perspective, the United States will never voluntarily cede significant
international influence to China. America regards China as a revisionist power whose long-term
aim is global supremacy.

For that reason hard security challenges escalate, leading to the emergence of a classical
security dilemma. As Michael Paul and Marco Overhaus argue that this applies especially
strongly to China as a great power that is expanding its radius of action and in the process
transitioning incrementally from the doctrine of coastal defence to maritime “active defence”.
But it also applies to the United States, which sees China’s growing military capabilities as a
threat not only to its own military bases in the Pacific, but also to its system of partnerships and
alliances in the Asia-Pacific region – and in the longer term to its nuclear deterrent.

Economic competition and conflicts over trade, economic and financial policy form a real
dimension of rivalry in their own right, which predates the protectionist course adopted by the
United States under President Trump. Washington’s criticisms of Chinese trading practices as
unfair competition and rule violations are widely shared in Europe.

Bilateral trade between the United States and China is no longer a stabilising factor capable of
ameliorating political conflicts. Instead trade conflicts are politically instrumentalised, although
they may also represent the most easily untangleable knots in the complex web of US-China
rivalry.

The technological dimension of the rivalry runs deeper. Both absolute and relative prizes are at
stake: the question of who will secure the largest piece of the cake in the long term, for example
by defining the technical standards. And technological competition is always also a question of
security.

Even if the constellation of conflict and competition described here is understood as a bilateral
rivalry and to some extent plays out as such, its significance and consequences are global: It
affects relationships with other powers, influences regional dynamics even in Europe, shapes the
work of international organisations and forums (such as the G20 or the United Nations and its
agencies), and, often enough undermines multilateral institutions.

This is especially clear in the case of the World Trade Organisation, whose rules have been
violated by both sides and whose very function the Trump Administration has sought to impair.

China is establishing new international forums and organisations in line with its own Sinocentric
concepts of order, especially in its own regional environment. But unlike the United States,
China is showing no signs of withdrawing from international and multilateral institutions.
Instead it is working actively to expand its influence at the United Nations and within its agen-
cies and programmes.
The strategic rivalry between the United States and China risks spiralling into a multi-layered
world conflict that presents economic and military dangers.* The rivalry between the two great
powers is beginning to structure international relations and bears the potential to bring forth a
new “geo-economic world order”. In comparison to past decades, the question of who gains
more from economic exchange and concern over the problematic security implications of
economic interdependence now plays a much more important role.

2(b) “The center of gravity of International politics is shifting towards Indo -pacific”.
Comment. Analyse the role of Quad in advancing the areas of cooperation and Mutual
interest in the region.

The Indo-Pacific region, uniting the Indian and Pacific oceans, is the world’s economic and
geopolitical hub. It is home to the world’s most populous nations, largest economies, and
largest militaries. More significantly, the Indo-Pacific is emerging as the center of global power
and wealth. Building a stable balance of power that keeps the peace is at the heart of the
security challenges in this sprawling region.

Geopolitical rivalry in the Indo-Pacific, however, is already sharpening. For example, several
boundary, sovereignty and jurisdiction disputes threaten freedom of navigation. China is
challenging the existing balance of power, with its territorial and maritime revisionism injecting
greater instability and tensions. Indeed, China is seeking to checkmate, if not supplant, the
United States, the dominant power in the Indo-Pacific. This has brought the attention of the
world to the Indo-Pacific Locus.

The “Indo-Pacific” emerged circa 2010 as a regional framework for US strategic discourse under
the Obama administration, and became a key regional term for official US discourse by 2017
under the Trump administration. There are two reasons for this recent shift in strategic
language, one is geo-economic and the other is geopolitical.

The geo-economic shift is to do with the general volume of trade, including particularly
significant energy flows between the Indian and Pacific oceans. The geopolitical shift is to do
with the rise of China in the region. The US has been the leading power in the Pacific since 1945
and a prominent power in the Indian Ocean since the 1980s. Now the US, Japan and Australia
are faced with Chinese assertiveness in the Western Pacific; while in the Indian Ocean the US
and another key rising power, India are faced with an increasing Chinese presence. Not
surprisingly, in the face of this Chinese challenge, the US has crafted an Indo-Pacific response.

Just over a decade ago, John Mearsheimer (2006) argued that “to predict the future in Asia,
one needs a theory of international politics that explains how rising great powers are likely to
act and how the other states in the system will react to them”. The US Indo-Pacific strategy
reflects three processes which follow from each other.

First is that China’s rise presents “power transition” challenge to the US.

Second is that the US is responding to that challenge by pursuing “balancing”, both in terms of
building up its own strength (“internal balancing”) and in strengthening its alliances and
strategic partnerships (“external balancing”).

Third is that in balancing terms “balance of threat” considerations are in operation not only for
the US but also for Japan, Australia and India vis-à-vis China.

The significant shift from Pacific to Indo-Pacific security arrangements being made by the US are
primarily focussed around India. The New Framework for Defense Cooperation (NFDC)
established in 2005, and strengthened further in 2015, has been followed by increasingly
significant naval and air force bilateral exercises in both the Indian and Pacific Oceans between
the two countries, which are strategic partners if not formal allies. Their Logistics Exchange
Memorandum of Agreement (LEMOA), signed in August 2016 further opens the way for mutual
use of each other’s bases in both oceans. Specific regional underpinnings were apparent in the
drawn up in January 2015 between Obama and Indian Prime Minister Narendra Modi (US-India
2015). Joint common concerns have been expressed by US and Indian officials over maintaining
freedom of navigation and airspaces in the South China Sea, and for UNCLOS tribunal findings
to be upheld – both of which are comments aimed at China.

In the light of these bilateral and trilateral links, the strategic logic was straightforward: renewal
of the Quadrilateral format between the US, Australia, India and Japan. This had first surfaced
in 2007 but had been halted following Chinese criticisms and subsequent Australian and Indian
hesitations. A decade later, US officials met their quadrilateral counterparts in November 2017
for Consultations on the Indo-Pacific. On 12 March 2021, the inaugural QUAD (Quadrilateral
Security Dialogue) Leader’s Summit took place wherein a joint statement known as ‘The Spirit
of the Quad’ was released in which the members has described, "a shared vision for a Free and
Open Indo-Pacific," and a "rules-based maritime order in the East and South China Seas," for
which Quad member states are needed to counter Chinese maritime claims.

In August 2021, senior officials from the Quad country met to discuss ways to advance co-
operation and mutual interest in the region. This included countering disinformation,
promoting democracy and human rights, strengthening international institutions including
the United Nations and related organisations, and supporting countries vulnerable to coercive
actions in the Indo-Pacific region.
They also discussed the importance of peace and security in the Taiwan Strait, the ongoing
crisis in Myanmar, and reaffirmed the Quad’s strong support for ASEAN centrality and the
ASEAN Outlook on the Indo-Pacific.

The reality is that the Indo-Pacific has emerged as the new global center of trade and energy
flows, in keeping with the shift in world power from the trans-Atlantic region to the East. In fact,
the Indo-Pacific is likely to determine international geopolitics, maritime order, and balance of
power. Yet, it is also true that in no part of the world is the security situation so dynamic and in
such flux as in the Indo-Pacific — the hub of global geopolitical competition. In this scenario,
QUAD can play a major security platform.

2 (c) Since the end of the cold war nothing had a more decisive impact on the structure of the
international system than the rise of China. Elucidate.

G. John Ikenberry said, “The rise of China will undoubtedly be one of the great dramas of the
twenty-first century.” China's extraordinary economic growth and active diplomacy has made a
huge impact in the transforming East Asia, and future decades will see even greater increases in
Chinese power and influence. This has made many observers feel that the American era is
coming to an end, as the Western-oriented world order is replaced by one increasingly
dominated by the East. The historian Niall Ferguson has written that the bloody twentieth
century witnessed "the descent of the West" and "a reorientation of the world" toward the
East.

Power transitions are a recurring problem in international relations. As scholars such as Paul
Kennedy and Robert Gilpin have described it, world politics has been marked by a succession of
powerful states rising up to organize the international system. A powerful state can create and
enforce the rules and institutions of a stable global order in which to pursue its interests and
security.

But nothing lasts forever: long-term changes in the distribution of power give rise to new
challenger states, who set off a struggle over the terms of that international order. Rising states
want to translate their newly acquired power into greater authority in the global system-to
reshape the rules and institutions in accordance with their own interests. Declining states, in
turn, fear their loss of control and worry about the security implications of their weakened
position.

The size of China’s economy has quadrupled since the launch of market reforms in the late
1970S and, by some estimates, will double again over the next decade. It has become one of the
world's major manu facturing centers and consumes roughly a third of the global supply of iron,
steel, and coal. It has accumulated massive foreign reserves. China's military spending has
increased at an inflation-adjusted rate of over 18 percent a year, and its diplomacy has
extended its reach not just in Asia but also in Africa, Latin America, and the Middle East.

These recent years are fraught with danger. When a state occupies a commanding position in
the international system, neither it nor weaker states have an incentive to change the existing
order. But when the power of a challenger state grows and the power of the leading state
weakens, a strategic rivalry ensues, and conflict perhaps leading to war-becomes likely.

Any international order dominated by a powerful state is based on a mix of coercion and
consent, but the U.S.-led order is distinctive in that it has been more liberal than imperial. Its
rules and institutions are rooted in, and thus reinforced by, the evolving global forces of
democracy and capitalism. It is expansive, with a wide and widening array of participants and
stakeholders.

But as the scholar Marc Lanteigne argues, "What separates China from other states, and
indeed previous global powers, is that not only is it 'growing up' within a milieu of
international institutions far more developed than ever before, but more importantly, it is
doing so while making active use of these institutions to promote the country's development
of global power status."

China, in short, is increasingly working within, rather than outside of, the Western order. It is
challenging the order by being part of the same. Thus, the rise of China is making a decisive
impact on the World order which is currently based on USA hegemony.

Since assuming power in October 2012, Xi Jinping has given clearer indications that China is
seeking not only to enjoy a central position on the global stage, commensurate with its
economic and military power, but also to reshape, alter, and redefine elements of the existing
system to better fit its views and interests.

However, one thing is clear that the rise of powerful China and a declining United States has
altered the International system. And as the world's largest country emerges from within the
established post-World War II international order, it is a drama that will end with the grand
ascendance of China and the onset of an Asian centered world order (Fareed Zakaria).

3 (a) Do you think that the International institutions played significant role in making
cooperation possible among the nation states? Give examples to substantiate your point of
view.
International organizations are increasingly a prominent feature of international politics today.
Both intergovernmental organizations (IGOs) and nongovernmental organizations (NGOs) have
grown at a rapid rate in recent years.

The Former Secretary-General Kofi Annan spoke about how the challenges of the twenty first
century would not be conquered if it weren’t for international organizations. “More than ever
before in human history, we share a common destiny. We can master it only if we face it
together. And that, is why we have the United Nations.” International Organisations are seen
to be responsible for the maintenance of international peace and stability, be this economic,
social or political, and that they act in the interest of the international community.

The creation of an international forum for multi-lateral negotiations came about with the Inter-
Parliamentary Union (IPU) in 1889, which is still active today and has membership of 157
national parliaments. The IPU was the predecessor to the League of Nations, created in 1919
after the end of the First World War; this later became the United Nations after the failure of
the League to prevent international conflicts. The legacy of the IPU, the League of Nations, and
other early international alliances was not the institutions’ effectiveness as an actor, but rather
as a forum, for nations to voice their opinions and promote dialogue. This was arguably their
greatest achievement, as even after the failure of the League, nation States still felt the need for
an institution that would allow them to share their ideas and provide an opportunity to settle
disputes peacefully. Thus, emerged the United Nations, which to this day remain the only
institution with universal membership and is the largest of all international organisations.

The pioneer towards Institutionalism was Woodrow Wilson who held that International
organization can convert the jungle of International Politics into Zoo. Neo-Liberal scholars like
Joseph Nye and Robert Keohane believe that once cooperation amongst States is
institutionalised, States would be reluctant to leave it, in fear of International repercussion. By
bridging the gap between States and giving them this forum for debate, institutions help trigger
important coalitions, and with its congenial approach to weaker States, aids in their pursuit of
linkage strategies. Hence, States feel welcome in what was previously a hostile international
environment. However Realists/ Neo-Realists have little faith in the potential of international
organisations.

Neo-liberal institutionalism stresses the importance of the UN’s work with regional
organizations, as they become indispensable in the international diplomatic process predicting,
“the international community will increasingly direct itself towards combined action of the
universal Organization with regional bodies.” This can be observed in the recent links between
the UN and regional organizations such as the Organisation of American States (OAS), the
African Union (AU), the European Union (EU), the Arab League, and the Association of South
East Asian Nations (ASEAN).
On the other hand, Constructivist theory challenges both neo-realism and neo-liberal
institutionalism by claiming that anarchy is not inherent in the state system, as affirmed by neo-
realists, neither is it inexistent, as affirmed by neoliberal institutionalists, it is, in truth, what
States make of it. It asserts that institutions and structures, within the international system,
are mutually constructed concepts by actors that employ social practice to define the
‘international realm.’ Neo-realism and neo-liberal institutionalism, take for granted the idea
that economic and military power is the primary source of influence in world politics.
Constructivist theorists counter this, as they believe discursive power also plays a fundamental
role in the understanding of the global political system. As Alexander Wendt said, “if today we
find ourselves in a self-help world, this is due to process, not structure.”

International institutions and their role in ensuring in peace and order can be seen as a mixed
one. It can be said that the glass is either half-filled or half empty (Manmohan Singh on SAARC).
If we take organization like UN, though there is a huge disappointment with respect to its role in
Israel Palestinian issue or in Afghanistan but UN has played considerable role towards
development, health and climate change. Similarly WTO is ina deadlock but it did play a role in
establishing the rule based order. Similarly, World Bank, IMF, WHO, ILO, ICJ and other
institutions give a mixed scenario. In this context, we can give reference to the views of Hedley
Bull who believe that these institutions have transformed the anarchy into anarchical society.

3(b) How do feminist approach to International Politics contribute in addressing the


subordination of Women in Global politics?

The academic interest of feminism in International Relations started only from the late 1980s.
The traditional IR theory was re-evaluated during the post-Cold War during the 1990s which
created the need to study how International Relations have been gendered. Historically, the
study of IR has been largely concerned with the study of relations between sovereign states and
it had been assumed that the kind of issues raised by feminist are not in consonance with the
process and structures raised by International Relations and the discipline could be studied
without any reference to gender. As a consequence of which the discipline of International
Relation has become gender blind. However the scholarships of Feminist in International
Relations have made the discipline more open as they have pointed out that the issues of gender
are embedded with the discipline.

The main crux of Feminist IR is to examine how the core concepts employed within the discipline
of IR are highly gendered such as in the concepts of war and security, etc.

The main focus of feminism has been on the argument that the boundaries of state have
historically excluded women from domestic and international political life and has treated
international relations as the exclusive domain of men and masculinity where it thrives on the
domination over women.

From the outset, feminist theory has challenged women’s near complete absence from
traditional IR theory and practice. This absence is visible both in women’s marginalisation from
decision-making and in the assumption that the reality of women’s day-to-day lives is not
impacted by or important to international relations. Beyond this, feminist contributions to IR
can also be understood through their deconstruction of gender – both as socially constructed
identities and as a powerful organising logic. This means recognising and then challenging
assumptions about masculine and feminine gender roles that dictate what both women and
men should or can do in global politics and what counts as important in considerations of
international relations. These assumptions in turn shape the process of global politics and the
impacts these have on men and women’s lives.

If we start with feminism’s first contribution – making women visible – an early contribution of
feminist theorists is revealing that women were and are routinely exposed to gendered
violence. In making violence against women visible, an international system that tacitly
accepted a large amount of violence against women as a normal state of affairs was also
exposed. For example, former UN Secretary General Ban Ki-moon’s ‘UNiTE’ campaign to end
violence against women estimated that up to seven out of ten women will experience violence
at some point in their lives – and that approximately 600 million women live in countries where
domestic violence is not yet considered a crime.

In making women visible, feminism has also highlighted women’s absence from decision-
making and institutional structures. For example, in 2015 the World Bank estimated that
globally women made up just 22.9% of national parliaments. One of the core assumptions of
traditional perspectives that feminism has challenged is the exclusionary focus on areas that are
considered ‘high’ politics – for example, sovereignty, the state and military security. The
traditional focus on states and relations between them overlooks the fact that men are
predominantly in charge of state institutions, dominating power and decision-making
structures. It also ignores other areas that both impact global politics and are impacted by it.

This brings us to the third key contribution of feminism – exposing and deconstructing socially
constructed gender norms. In making sense of IR in a way that takes both women and gender
seriously, feminism has demonstrated the construction of gendered identities that perpetuate
normative ideas of what men and women should do.

Femininity is often associated with irrationality, in need of protection, domesticity and the
private sphere. These socially and politically produced gender identities shape and influence
global interactions, and IR as theory – and global politics as practice – also produces such
gendered identities in perpetuating assumptions about who should do what and why. These
gender identities are also imbued with power, in particular patriarchal power, which
subordinates women and feminine gender identities to men and masculine gender identities.

Cynthia Enloe (1989) asked the question ‘where are the women?’, encouraging IR scholars to
see the spaces that women inhabit in global politics and demonstrating that women are
essential actors in the international system. She focused on deconstructing the distinctions
between what is considered international and what is considered personal, showing how global
politics impacts on and is shaped by the daily activities of men and women – and in turn how
these activities rest on gendered identities.

Feminist research has demonstrated the value in taking women’s experiences and
contributions seriously and used that as a base to demonstrate how IR rests on, and
perpetuates, gendered ideas about who does what, who experiences what – and why – in global
politics. Beyond this there is also recognition that women are important agents in political,
economic and social processes.

In highlighting both inequality and relations of power, feminism reveals gendered power and
what it does in global politics. Being concerned with women’s subordination to men, gendered
inequality and the construction of gendered identities, feminism has highlighted the need to re-
evaluate IR holistically.

3(c) Discuss the constructivist critique of realism. How does the Constructive critique differs
from the postmodernist critique of realism?

Constructivism (sometimes called ‘social constructivism’) has been the most influential post-
positivist approach to International Theory, gaining significant attention since the end of Cold
War. The Constructivist approach to analysis is based on the belief that there is no objective
social or political reality independent of our understanding of it. People whether acting as
individuals or as social groups construct the world in which they live and act according to those
constructions. Constructivist analysis highlights the missing dimension in the structure-agent
debate in global politics.

One of the most influential formulations of constructivism was Alexander Wendt’s assertion
that ‘anarchy is what states make of it.’ This implies that state behavior is not determined as
neo-realists assert by the structure of the International system but by how particular states
view anarchy. While some states may view anarchy as dangerous and threatening, other may
see it as the basis for freedom and opportunity.
This is also seen in relation to nation and nationalism. Nations are not objective entities, groups
of people who happen to share a common cultural heritage; rather, they are subjective entities,
defined by their members, through a particular set of traditions, values and sentiments.
Constructivist analysis highlights the fluidity of world politics, contrary to what realists argue:
as nation states and other key global actors change their perception of who or what they are,
their behavior will also change.

The fluidity of world politics also leaves open the possibility that states may transcend
perception of national interests and embrace global concepts like climate change,
globalization, and cosmopolitanism. Wendt argues that both neo-liberalism and neo-realism are
defective because both fail to take account of the self-understandings of state actors.

On the other hand, post-modernism emphasizes that all ideas and concepts are expressed in
language which itself is enmeshed in complex relations of power. Inspired by Michel Foucault,
post-modernists draw the attention to the link between power and systems of thought using the
idea of discourse. In the words of Jacques Derrida, “There is nothing outside the text.”

Post modernists draw attention to the fact that any political event will always be susceptible to
interpretations: For example, the events of 9/11 can be interpreted as a terrorist act or an act
of revenge. Der Derian has argued that ‘the complexity, ambivalence, and indeterminacy of
human relations, magnified, mediated, and estranged in the international arena, make it all the
more evident why a single … theory cannot explain the workings of international relations.’

Post Modernists believes that deconstruction is one of the only ways one can successfully
analyse the international in the face of what he has heralded as a ‘crisis of modernity’; a
situation in which objective reality is displaced by textuality.

Post Modernists undermine the fundamental tenets of the traditional, state-centric


international theories, leaving a theoretical vacuum in their wake.

Scholars like Der Derian argue that Realism has failed to look at the issues emerging from the
margin. They argue, “in view of multiculturalism, ethnic diversity and interpenetration,
minority and indigenous rights, diasporic peoples, environmental degradation, migration and
the general movement of peoples, globalization, interdependence and so on.’

In conclusion, both constructivists and post modernists critic Realism especially on the aspect of
Anarchy which the latter considers the definitional aspect of IR. Realists subscribe to the belief
that states are the only relevant actors in international politics and as there is no central
authority to regulate or govern nation-states, a state of anarchy exists, where conflict and war
is a constant threat as each state seeks to ensure its own survival at the expense of others. Both
Constructivists and post modernists point out alternative understanding of state behavior ---
subject-agent relationship and power.

4.(a) Write a note on sustainable development goals and elucidate their critical importance
in the achievement of new international order.

The Sustainable Development Goals (SDGs) or Global Goals are a collection of 17 interlinked
global goals designed to be a "blueprint to achieve a better and more sustainable future for
all". The SDGs were set up in 2015 by the United Nations General Assembly and are intended to
be achieved by the year 2030. They are included in a UN Resolution called the 2030 Agenda or
what is colloquially known as Agenda 2030. The SDGs were developed in the Post-2015
Development Agenda as the future global development framework to succeed the Millennium
Development Goals which ended in 2015.

Objectives:

 End poverty in all its forms everywhere


 End hunger, achieve food security and improved nutrition and promote sustainable
agriculture
 Ensure healthy lives and promote well-being for all at all ages
 Ensure inclusive and equitable quality education and promote lifelong learning
opportunities for all
 Achieve gender equality and empower all women and girls
 Ensure availability and sustainable management of water and sanitation for all
 Ensure access to affordable, reliable, sustainable and modern energy for all
 Promote sustained, inclusive and sustainable economic growth, full and productive
employment and decent work for all
 Build resilient infrastructure, promote inclusive and sustainable industrialization and
foster innovation
 Reduce inequality within and among countries
 Make cities and human settlements inclusive, safe, resilient and sustainable
 Ensure sustainable consumption and production patterns
 Take urgent action to combat climate change and its impacts
 Conserve and sustainably use the oceans, seas and marine resources for sustainable
development
 Protect, restore and promote sustainable use of terrestrial ecosystems, sustainably
manage forests, combat desertification, and halt and reverse land degradation and halt
biodiversity loss
 Promote peaceful and inclusive societies for sustainable development, provide access to
justice for all and build effective, accountable and inclusive institutions at all levels
 Strengthen the means of implementation and revitalize the global partnership for
sustainable development

Though the goals are broad and interdependent, two years later (6 July 2017) the SDGs were
made more "actionable" by a UN Resolution adopted by the General Assembly. The resolution
identified specific targets for each goal, along with indicators that are being used to measure
progress toward each target.

To facilitate monitoring, a variety of tools exist to track and visualize progress towards the
goals. All intention is to make data more available and easily understood. The SDGs pay
attention to multiple cross-cutting issues, like gender equity, education, and culture cut across
all of the SDGs. On July 7, 2020, the Sustainable Development Goals Report 2020 was
released. As per the report, the COVID-19 pandemic unleashed an unprecedented crisis which
has lead to further disruption to SDG progress.

In 2019 five progress reports on the 17 SDGs appeared. Three came from the United Nations
Department of Economic and Social Affairs (UNDESA), one from the Bertelsmann Foundation
and one from the European Union. In all the reports, the progress on all 17 counts was shown
only limited success in various countries. The UN released a statement in 2019 urging all
member countries to allocate more resources in order to meet the SDG by the target year. It
further mentioned that the SDGs have resulted in the decline of the maternal mortality rate,
reduced poverty, worked for the improved health of the people, spread awareness about the
communicable and non-communicable diseases and also the vaccines required for children.

Present International order based on Westphalian notion of sovereignty and the neo-liberal
approach to Capitalism is creating challenges to address the global concerns like Climate
Change, Terrorism, Poverty etc. which have gone beyond the capacity of a single country even if
it is a super-power to deal with. Today there is an existential crisis for humanity and hence there
is a need for more equitable inclusive representative world order. It is a time when countries of
the world need to come together to help each other to save our collective future. In this context,
sustainable development goals can usher a new World Order based on mutual benefits and a
non zero-sum approach. We cannot address Climate Change, terrorism, organised crimes
without addressing the poverty in global south.

Overall the Sustainable Development goals tend to make the world a better place to live by
fulfilling its targets within the stipulated time of 15 years and remove poverty, improve health,
provide employment, empower women, reduce inequalities and follow all the seventeen targets
that have been set by the UN. Although there has been limited progress till now, these goals are
meant for improving the lives of every human being across the globe.

4(b) Discus the three generations of Human Rights. How are they related to each other? Why
western nations do not accept the Asian value perspective on human rights?

When human rights are being discussed, they are often divided up into three categories called
generations. A reflection of the three generations of human rights can be seen in the popular
phrase of the French Revolution: liberty, equality, fraternity. These generations of human rights
were first formally established by Karel Vasak, a Czech jurist, in 1979. This division of the types
of human rights helps improve conversations about rights, especially those involving legislation
and the role that governments play in human rights.

The First Generation: Liberté

The first generation of human rights encompasses an individual’s civil and political rights. First
generation rights can be divided into two sub-categories. The first sub-category relates to
norms of “physical and civil security.” This includes not committing acts of torture, slavery, or
treating people inhumanely. The second sub-category relates to norms of “civil-political liberties
or empowerments.” This includes rights such as freedom of religion and the right to political
participation.

First generation rights are based around the rights of the individual person and are often the
focus of conversations about human rights in western countries. They became a priority for
western nations during the Cold War. Some documents that focus on first generation rights are
the United States Bill of Rights and Articles 3 through 21 of the Universal Declaration of Human
Rights (UDHR).

The Second Generation: Égalité

The second generation of human rights encompasses socio-economic rights. Second generation
rights can also be divided into two sub-categories. The first sub-category relates to norms of the
fulfillment of basic needs, such as nutrition and healthcare. The second sub-category relates to
norms of the fulfillment of “economic needs.” This includes fair wages and sufficient standards
of living.

Second generation rights are based on establishing equal conditions. They were often resisted
by western nations during the Cold War, as they were perceived as “socialist
notions.” The International Covenant on Economic, Social, and Cultural Rights and Articles 22
through 27 of the UDHR focus on these rights.
The Third Generation: Fraternité

The third generation of human rights encompasses broad class rights. Third generation rights
can be divided into sub-categories as well. The first sub-category relates to “the self-
determination of peoples” and includes different aspects of community development and
political status. The second sub-category is related to the rights of ethnic and religious
minorities.

Third generation rights are often found in agreements that are classified as “soft law,” which
means they are not legally binding. Some examples of these agreements include the UDHR and
the 1992 Rio Declaration on Environment and Development. This generation of rights is
challenged more often than the first and second generations, but it is being increasingly
acknowledged on an international level. These rights started gaining acknowledgement as a
result of “growing globalization and a heightened awareness of overlapping global concerns”
such as extreme poverty.

Though they appear different from each other as they are also based on different ideological
concerns for Example: the first generation rights are based on Liberalism, the second
generation is based on socialism and the third generation is based on multi-culturalism. Yet
they cannot be seen in a compartmentalized format. It is believed that they are organic
linkages and one creates the condition for the other. We can understand the linkages through
the statement of Ambedkar who held that “Political democracy without Social and Economic
is a contradiction.”

Asian Value Perspective

The 'Asian values' perspective emerged during the 1990s after the collapse of the Soviet Union
bloc and as a result of the rise of East Asian economies. It refers to values which, distinct from
those emerging from European discourse, advocate the particularity of human rights and deny
their universality. It can be summarised as having the following major components: a relativistic
approach to human rights, a communitarian outlook, a strong emphasis on the family at the
core of society, the need to acknowledge the significance of economic, social and cultural
influences, human dignity as the foundation of human rights, an identified correlation between
rights and duties, and emphasis on national sovereignty. This concept has been widely debated
by many prominent East Asian politicians like Lee Kwan Yew and scholars like Li Xiaorang, and
used to challenge the universality of human rights.

The cultural relativists maintain that human rights, far from being universal, essentially
represent Western (more specifically, Christian and European) values . Amartya Sen and other
Asian scholars have been critical of the claim that Asia had ‘long expelled’ political freedom
from their system and that democracy as a fundamental and ancient feature of Western culture
was not to be easily found in Asia.

Amartya Sen distinguishes between East Asian and south Asian values. He points out that in
south Asia there exists a tradition of respect for tolerance (a pre-conditional value of human
rights), whereas East Asian society is mainly based on Confucianism that advocates the
significance of society over the individual. He emphasises the impact of this tolerant attitude on
public policy-making, with governments being influenced towards values of non-injury, restraint,
impartiality, and mild behaviour. Sen argues that unlike ancient India, in China (and in East
Asian societies that have been influenced by Chinese civilization, such as Japan and Korea) the
political order presides in principle over every area of socio-political life.

In 1993, thirty-four Asian and Middle Eastern states adopted the Bangkok Declaration claiming
that human rights ‘must be considered in the context of a dynamic and evolving process on
international norm-setting; bearing in mind the significance of national and regional
peculiarities and various historical, cultural, and religious background’. This sentence was added
to the 1993 Vienna Declaration of Human Rights.

However politicians and scholars of the Western world argue that Human Rights is an universal
issue affecting Humanity, rather than particular sections of the world. All cultures—Western or
otherwise—are acquainted with the experience of injustices, which beget demands for the
recognition of human rights from oppressed populations. It is this “collective experience with
injustice” which constitutes an authoritative foundation on which to build a theory of Universal
Human rights.

Moreover, modern conceptualizations and articulations of human rights norms and standards
were formed as political responses to the atrocities of the last World War. The international
expressions of rights themselves claim no philosophical foundation, do not reflect any clear
philosophical assumptions, and “articulate no particular moral principles or any single,
comprehensive theory of the relation of the individual to society.” They are the outcome of
diplomatic initiatives, involving political strategy and negotiated agreements.

4(c) Elaborate on Marxist critique of realism in International politics. How the Neo-gramscians
like Robert Cox apply the concept of Hegemony in international relations?

The Marxist perspective is one of the most lively approach to the study of International Politics
which is different from traditional theories of international relations such as realism and neo-
realism in manner; that it does not support the status-quo in the international system. Rather,
it attempts to bring the radical change in the prevailing social and political order. Marx wrote
“philosophers had only interpreted the world whereas the real point was to change it.”
The basic assumption of Marxist Approach to International Relations is that Economy provides a
tool/lens to understand international relations.

In the Marxist orientation the World System theory was developed to analyse the postcolonial
international order. The roots of the world system theory can be traced to the writings of Lenin.
In his monumental work, “Imperialism-The Highest Stage of Capitalism”, Lenin contended that
imperialism created a two-tier structure within the capitalist world economy. He identified the
dominant structure as the “core” and the less-developed structure as the “periphery”.

The world system theory was further developed by Wallerstein who provided powerful insights
into the working of the world capitalist economy. Wallerstein in his seminal work “Modern
world System: Capitalist Agriculture and the Origins of the European World Economy in the
Sixteenth Century (1974)”, represents the powerful exposition of the modern world system
theory. Tracing the emergence of capitalism in the sixteenth century Europe, he examines its
evolution into a world capitalist system that contains Core, Periphery and Semi-Periphery in
terms of wealth accumulation and economic development.

Robert W. Cox has applied Gramscian approach to address the economic reductionism of the
conventional Marxism and highlighted the role of ideas and institutions along with material
power in building US Hegemony. Andrew Linklater has given an emancipatory approach,
inspired by Cox. Some of the points of criticism of Realism by Marxist are:

Realists give primacy to the national interest whereas Marxist want to highlight that National
interest is a misnomer to legitimise the interests of the corporate class.

Realists suggest that structure of Int. Politics is anarchical, which give rise to wars. However
Marxists believe that International Politics has a hierarchical structure, where states can be
categorized into Core, semi-periphery, and Periphery. The cause of war is not the anarchical
structure rather the capitalist search for profit. Acc. Rosa Luxembourg, Capitalism, Nationalism,
Imperialism and Militarism co-exist.

Realists are status-quoists whereas Marxists look for emancipation.

Hegemony is a concept used in International Politics since the time of Peloponnesian war.
According to Thucydides and later day Realists, Material power is the basis of Hegemony.
Robert Cox incorporating Gramscian tradition suggested that ideas and institutions also play the
role in building Hegemony. Scholars should also study how US has used cultural power to
establish its dominance.

Internationally, a hegemon exercises pressure on subaltern states to align their structures with
those of the hegemonic state often through the processes of creating economic
interdependence. In contrast to realism wherein international hegemony is based on the
concentration of material power in one dominant state, neo Gramscians like Robert Cox claim
that hegemonies are created primarily through consensus creation (sometimes coercion remains
latent) between the hegemonic and hegemonised states rather than on crude power politics
only.

“Hegemony is a structure of values and understandings about the nature of order that permeates a whole
system of states and non-state entities. In a hegemonic order these values and understandings are
relatively stable and unquestioned. They appear to most actors as the natural order. Such a structure of
meanings is underpinned by a structure of power, in which most probably one state is dominant but that
state’s dominance is not sufficient to create hegemony. Hegemony derives from the dominant social
strata of the dominant states in so far as these ways of doing and thinking have acquired the acquiescence
of the dominant social strata of other states.”

– Robert Cox
Section B

5 (a) Soft power as a major foreign policy tool for India. Discuss.

The term ‘soft power’ was coined by the American Scholar Joseph Nye, in his book, “Bound to
Lead: The Changing Nature of American Power”. Nye developed the concept further in his book,
“Soft Power: The Means to Success in World Politics.” Soft power in foreign policy arises from
factors such as the dominant values, internal practices and policies, and the manner of
conducting international relations of a State. Soft power is getting others to want the
outcomes that you want, co-opt people rather than coercing them. Soft power can be helpful in
preparing the ground for certain foreign policy decisions and gathering required international
support to ultimately legitimise one’s foreign policy.

It can be contrasted with 'hard power', which is the use of coercion and payment. Soft power
can be wielded not just by states but also by all actors in international politics, such as NGOs or
international institutions. It is also considered the "second face of power" that indirectly allows
you to obtain the outcomes you want. A country's soft power, according to Nye, rests on three
resources: "its culture (in places where it is attractive to others), its political values (when it lives
up to them at home and abroad), and its foreign policies (when others see them as legitimate
and having moral authority)."

India’s arsenal of soft power

Capacity Building: India focuses on three main components: providing training in India, sending
teams of experts to partner countries and providing equipment for project sites. India has also
taken up larger issues at various multilateral forums including the World Trade Organization
(WTO) and the World Intellectual Property Organization (WIPO).

Concessional Finance: Concessional financing constitutes close to 70% of India’s development


cooperation portfolio. Development assistance in the form of concessional Lines of Credit (LOCs)
is extended by the Government of India under the Indian Development and Economic Assistance
Scheme (IDEAS) through the Exim Bank of India. In total 306 LOCs worth US$ 30.59 billion have
been extended to 65 countries.

Technology Sharing: Innovation and entrepreneurship should be the soft power of choice both
within the country and outside. For instance, in Ethiopia, Indian engineers provided support and
training in areas such as irrigation, electrical power and railway management.

Grants: India provides development assistance of $6.48 billion and receives assistance of $6.09
billion annually from key partners as Official Development Assistance (ODA).
Trade: By providing duty-free and quota-free access to the Indian market. India was one of the
first few countries that announced duty free, quota free access to low income countries. Indian
private investment in the world has also surged over a period of time with major investments in
telecommunications, IT, energy, and automobile sectors.

Ideas: India's spiritualism, yoga, movies and television, classical and popular dance and music,
its principles of non-violence, democratic institutions, plural society, and cuisine have all
attracted people across the world.

Religion: India is a civilisation which has offered refuge to cultural and religious freedoms to
Jews, Parses, Christens and Muslims. The imprint of Indian civilization is the strongest
throughout the world.  India is a country in which all major religions like Hinduism, Islam,
Christianity and Sikhism - coexists which has been the strength of India's soft power. Buddhism
is an important bridge between not just India and South East Asia and East Asia, but also with
South Asia.

Yoga: International Day of Yoga reflects yoga’s immense popularity worldwide, underscoring its
richness as a soft power resource

Diaspora: Indian Diaspora is seen as ambassadors of carriers of our soft power. To project New
Delhi’s soft power across world capitals, the MEA has embarked on a program to build
diplomatic missions and cultural centres using the country’s diverse styles of architecture.

Bollywood: Bollywood has been projected as a great Soft Power tool for India.

Covid Vaccines: Under the Covax initiative and India’s own Vaccine Maitri diplomacy,
Bangladesh, Myanmar, Nepal, Bhutan, Maldives, Mauritius, Sri Lanka, Brazil, Morocco, South
Africa, Afghanistan, Mexico, DR Congo, Nigeria and the UK were among some of the
beneficiaries of the Vaccine Maitri initiative. As of 29 May, data from the Ministry of External
Affairs showed that India had exported 6.63 crore doses to other countries of which 10.7 crore
went as a grant, whereas 35.7 crore doses were given in a commercial capacity.

Limitations of India’s Soft Power Diplomacy

Lack of Institutional Framework: India needs an independent development partnership agency


that develops long-term and short-term strategies, identifies priorities, builds knowledge and
facilitates learning. It needs to remove Internal institutional hurdles, such as policy and
bureaucratic delays, to meet its infrastructure targets.

Lack of Funds: With limited capacity to fund infrastructure projects, India needs to rationally
allot its funds, keeping in mind its strategic objectives. Moreover, improving the Indian economy
and opening up the market can help India to accumulate funds for international projects.
Delivery-Deficit Nation: India’s neighbors have often complained of India making big promises
but suffering from a delivery deficit. This is the case with most countries in which India has taken
up projects, for instance, road and railway lines, establishing integrated border checkpoints, and
hydropower projects.

Protectionism: Protectionism is leading to a significant impact on economic diplomacy. India


stood at rank 24 out of 25 countries in the Asia-Pacific region in terms of trade openness as per
an Asian Development Bank (ADB) report.

Recent Examples of India’s Soft Power Diplomacy

PM Modi has also gone to great lengths to court foreign governments through soft power. For
example, he has built up India’s Look East Policy by emphasising India’s historical links with
Buddhism. Another platform for projecting India’s soft power has been the adoption of an
International Yoga Day by the United Nations. Modi described yoga as an “invaluable gift of
India’s ancient tradition”. Every year, on International Yoga Day, Modi leads large
congregations in yoga practice. In addition, India has attempted to leverage Mohandas
Gandhi’s worldwide image as an apostle of peace through carefully curated public relations
gestures, such as op-eds, speeches, and foreign dignitary photo-ops.

This holds true in the case of India as, Shashi Tharoor reminds us that soft power, “is created
partly by governments and partly despite governments; partly by deliberate action, partly by
accident.”

5(b) Lack of trust between India and Sri Lanka has made the relationship transactional.
Comment.

The relationship between India and Sri Lanka is more than 2,500 years old. Both countries have
a legacy of intellectual, cultural, religious and linguistic interaction. But in recent times, it is
felt that due to change in geopolitics of the region, a trust deficit has emerged. It has rendered
an all encompassing strategic relationship into a mere transactional one (meaning a
relationship based on bargaining, and therefore, lacking in warmth and a natural flow).

There is truth in the assessment that the historic relations between the two nations have long
since given place to self-centric claims, expectations and denials by both. Neither does India
possess the elder brother/sister attitude any more, nor is Sri Lanka willing to stop playing the
recently-innovated ‘China card’ viz its northern neighbour. Although old festering issues such
as 13th amendment, elections to provincial councils, human rights violations, ethnic
tensions,fishermen issue etc. persist but the Chinese influence in this geostrategically located
island has complicated the old bonhomie between India and Srilanka. According to Asanga
Abeyagoonasekera the island faces triple spheres of influence, from neighbouring India, the
United States, and China. Among the three spheres, Chinese role and influence have proceeded
the others in terms of financial assistance through commercial loans, infrastructure
development, tourism sector, foreign direct investments, and defence support to disaster relief
assistance.

Geopolitical Significance of Sri Lanka

Sri Lanka’s location in the Indian Ocean region as an island State has been of strategic
geopolitical relevance to several major powers. It was the massive Chinese involvement during
the Rajapaksa tenure that garnered the deepest controversy in recent years. China is building
state of the art gigantic modern ports all along the Indian Ocean to the south of it, in Gwadar
(Pakistan), Chittagong (Bangladesh, Kyauk Phru (Myanmar) and Hambantota (Sri Lanka).
China’s string of pearl’s strategy is aimed at encircling India to establish dominance in the Indian
Ocean.

Post 2015, Sri Lanka still relies heavily on China for Port city project and for continuation of
Chinese funded infrastructure projects in Sri Lanka. Although the Hambantota harbour is
reportedly making losses, it too has potential for development due to its strategic location.

Sri Lanka has a list of highly strategic ports located among busiest sea lanes of communication.
Sri Lanka’s Colombo Port is the 25th busiest container port in the world and the natural deep
water harbor at Trincomalee is the fifth largest natural harbour in the world.Port city of
Trincomalee was the main base for Eastern Fleet and British Royal Navy during the Second
World War. Sri Lanka’s location can thus serve both commercial and industrial purposes and be
used as a military base.

Political Relations

As a country that emerged from a civil war facing human rights allegations; the domestic
politics and international relations of Sri Lanka are heavily geopolitical with foreign powers
having vested interests.

Political relations between India and Sri Lanka have been marked by high-level exchanges of
visits at regular intervals. In February 2015, Sri Lanka’s newly elected President Maithripala
Sirisena undertook his first official visit to India, and Modi paid a return visit to Colombo in
March 2015. He was the first Indian prime minister to do a stand-alone visit to Sri Lanka in 28
years. In June 2019, the first overseas visit of Indian Prime Minister to Sri Lanka, in his second
term, is an important symbolic gesture reflective of the special relationship between the
countries.
Sri Lanka is a member of regional groupings like BIMSTEC (Bay of Bengal Initiative for Multi-
Sectoral Technical and Economic Cooperation) and SAARC in which India plays a leading role.
Recently, India has invited leaders of BIMSTEC member countries to attend the swearing-in of
Prime Minister Narendra Modi and his council of ministers. This is in line with the government’s
focus on its ‘Neighbourhood First’ policy.

Sri Lanka has long been in India’s geopolitical orbit, but its relationship with China has
strengthened in recent years. Former President Rajapaksa took Sri Lanka closer to China and
sidelining Indian concerns including over the rehabilitation of Tamils displaced by the long-
running Sri Lankan civil war.

Recently Sri Lanka scrapped its $500 million agreement with India and Japan to develop the East
Container Terminal (ECT) project, instead seeking investments from the two countries for the
West Container Terminal of the Port of Colombo under a public-private-partnership model. Also
Colombo handed over a $12 million energy project to a Chinese firm for the joint development
of three renewable power plants in as many islands off the Jaffna peninsula, about 50 km away
from the Tamil Nadu coast.

Amidst such irritants there is also a ray of hope. India’s Neighbourhood first policy has tried to
kick-start the relationship with neighbours. The new Gotabaya Rajapaksa government has
recalibrated the Indo-Lanka relationship which was strained at the tail end of Mahinda
Rajapaksa presidency. But there is a need for increased, all round, frequent and structured
interactions from the top-most level to the bottom. Given the millennia of multifarious and
unbroken interactions between Sri Lanka and India, the relationship ought to be a “special one”
marked by “inter-dependence, mutual respect and affection” as noted in a recent concept paper
called ‘Integrated Country Strategy’ which focused on improving the Indo-Sri Lanka ties.

5(c) Explain the New horizons in India Africa relations.

Africa is a continent on the move, characterised by rapid economic growth, rising educational
and health standards, increasing gender parity, and expanding infrastructure and connectivity.
India and Africa share a rich history of cultural, economic, and political interactions, rooted in
the spirit of developing together as equals. Accordingly Rajiv Bhatia in his new book ‘India-Africa
relations, New Horizon’, writes about growing continental, regional and bilateral facets of
India–Africa relations.

In July 2018, Prime Minister Narendra Modi addressed the Ugandan Parliament during his state
visit and outlined a vision for not just a bilateral partnership with Africa, but also a partnership
in multilateral forums by espousing the ‘10 guiding principles for India-Africa engagement’. They
represent continuity as well as change in India-Africa engagement.

The spirit of “developing together as equals” defines this bilateral partnership. Four main pillars
embody India’s development partnership approach with Africa: capacity building and training
under Indian Technical and Economic Cooperation (ITEC); Lines Of Credit (LOC) or concessional
loans and grant assistance; trade; and investments. It is expected that there would be a synergy
between the outcome of India-Africa Forum Summit-IV and Africa’s Agenda 2063.

A resurging Africa and a rising India can give a strong impetus to South-South Cooperation,
especially when it comes to addressing challenges in areas like clean technology, climate-
resilient agriculture, maritime security, connectivity, and Blue economy. Africa’s importance to
India as a development partner first became visible with the decision to enter into a structured
engagement with African countries under the framework of India Africa Forum Summit (IAFS).

The first-ever India-Africa Defence Ministers’ Conclave was held in Lucknow in February 2020 in
which 146 delegates from 33 African countries had participated and which had adopted the
Lucknow Declaration which appreciated that India and Africa were a significant part of the Indo-
Pacific continuum and that the AU Vision for peace and security in Africa coincided with India’s
vision of SAGAR.

From being the fourth-largest investor in the continent, to being the largest employer, to
capacity-building initiatives for local youth, India is continuously working on developing its
multi-dimensional relationship with Africa. The goal is to scale up bilateral trade volume to
US$150 billion and double Indian investments in Africa to US$100 billion in the next few years.

India has also aided African countries amid crises, including during the COVID-19 pandemic. And
even as developed countries have focussed on securing large vaccine supplies for their own
populations, India is being hailed for its vaccine diplomacy — it has exported over 1.6 crore
doses of vaccines globally, of which about 62.7 lakh doses (or about 37 percent) are as grant
assistance.

In conclusion, the ten guiding principles show a new dynamism in relations but India must chart
out a roadmap for its development cooperation program in Africa that outlines a long-term
strategy and delineates how it will deploy state capacity to pursue common development goals.
Doing so will become even more important for India in the aftermath of the COVID-19 pandemic
given the harsh economic impacts and the resultant inability to keep increasing its development
cooperation budget without any tangible outcomes. India and Africa must also look to sign
more legal frameworks to improve economic engagements like BIPA and FTAs. To generate and
incubate more ideas, both India and Africa should look to engage with one another in more
program and platforms at the Track I and Track II levels.
5(d) Elaborate the reasons why India needs more FTAs.

The signing of the Regional Comprehensive Economic Partnership (RCEP) agreement by 15


countries of the Asia-Pacific, accounting for 30% of the global trade, late last year, and the
launch of the African Continental Free Trade Area (AfCFTA) comprising 54 countries in January
2021, have revived the narrative on free trade agreements (FTA). Recently the government has
signaled desirability and willingness of entering into new FTAs.

FTAs between two or more countries increase trade by reducing customs duties and non-tariff
barriers on substantially all trade. They also cover services and non-trade issues like investment.
‘High standard’ FTAs, being aggressively promoted by the US, also include rules on e-commerce,
intellectual property, labour standards and environment protection measures. Today, the
spaghetti bowl of FTAs includes about 500 arrangements with linkages and overlaps.

India’s first comprehensive FTA was in 2005 with Singapore. FTAs can help India gain substantial
access to large markets at concessional duty for products where we are competitive. Sectors like
automotive, textiles, handicrafts, leather, pharmaceuticals, light electricals, some chemicals,
many agricultural items, jewellery and professional services, which are all employment-
intensive, could benefit. It can trigger huge job creation riding on exports. In textiles and
clothing, our competitors Vietnam and Bangladesh enjoy tariff-free access to the large and
lucrative EU and US markets on account of their FTAs or LDC status. Tariff elimination under
FTAs can provide our exporters a level-playing field and stop erosion of our market share and
profits. The government has identified 27 champion sectors that can be globally competitive
with the help of initial hand-holding. However, in sectors/industries where domestic capability
has been lacking, the dependence on good quality and competitive imports is inevitable. It is
here where the role of FTAs come in.

Government is fast-tracking six of the 20 FTAs that are currently under negotiation. Some low
hanging fruits are Australia, UK, UAE etc. There are also plans to revive negotiations with the
EU. It is noteworthy that India has forged major free trade alliances with Asian countries
(ASEAN, Japan and Korea).

But there are a number of issues that need to be considered in India’s approach to FTAs and
trade agreements, in general.

-Our future FTA negotiation strategy and tariff schedules need to be completely aligned with the
Aatmanirbhar Bharat strategy for scaling up the champion sectors.
-it is also imperative that we strengthen the safeguard clauses within existing FTAs under review
as well as in the new ones. Safeguard clauses need to be designed in a way that the domestic
industry gets protection in case of material injury well within time.

-the post-Covid world trade order will most likely be influenced by geo-political decisions. India
has a fine opportunity here and should fill the void by scaling up its champion sectors with
immediate urgency. India can pick up cues from Vietnam and Bangladesh who are doing
exceedingly well as alternative destinations for companies moving their production away from
China.

-it is imperative for India to now carefully choose its new FTA partners. While the focus should
be on countries with maximum trade complementarities, which makes the EU, UK and US
natural allies, one should note that these countries are tough negotiators. The India-EU FTA has
been lying in limbo since 2007.

Nobel laureate Milton Friedman has praised the free trade. However, good economics is not
always good politics. From India, which in the last five years has hiked tariffs on a fourth of all
products traded, to the US, which promotes buy American, protectionism is the flavour of the
season. Trade, one of the engines of economic growth, has not fired for India in the last decade.
Negotiating FTAs can be a challenge as it involves an element of give and take. Since companies
trade and not countries, some benefit while others lose out. That is the nature of the beast!

5(e) Write short notes on India’s humanitarian diplomacy.

Humanitarian assistance is usually categorised as short-term relief in the aftermath of disasters,


compared to long-term assistance classified as development aid. Historically, the West has been
the leading provider of humanitarian assistance but rise of countries in the Global South,
particularly India has reduced this dependence on Western humanitarian assistance, especially
in South Asia.

Although India does not have a designated Humanitarian policy, it has provided humanitarian
assistance and been involved in relief measures in and outside of its neighbourhood. India has
also utilised its humanitarian assistance as a diplomatic tool to deepen bilateral relations and
assert its claim to regional and global leadership. India’s outlook is broadly in line with global
standards such as the ones laid down by the UNOCHA (The United Nations Office for the
Coordination of Humanitarian Affairs), which advocate providing humanitarian assistance in
accordance with the principles of “humanity, neutrality, and impartiality.
Certain principles and priorities inform the way India goes about dispersing humanitarian aid.
They can be summed as:

-The imperative to respect the sovereignty of the affected state is the most important guiding
principle. It is an important legacy of the country’s struggle against colonialism and was the
defining commitment of the non-aligned movement.

-In line with this view on sovereignty, India aims to provide assistance according to the
requirements and needs as defined by the affected government, an approach that Indian
decision makers have labeled “demand-driven”.

-India strives to adopt a strictly non-political approach to humanitarian assistance, stressing


that humanitarian aid should not be linked to political objectives. This stance is a natural
extension of India’s traditionally non-aligned worldview.

-As a consequence, India eschews the “donor” category and instead views itself as a “partner”
who wants to stand in solidarity with its sister developing countries in distress.

-Furthermore, India emphasizes the importance of a smooth transition from immediate relief to
the long term development phase.

India prefers bilateral government-to-government aid to other aid modalities. Because of the
institutional reasons, principles and priorities discussed above, humanitarian assistance is
heavily anchored in the respective regional foreign policy frame. The bulk of India’s
humanitarian assistance goes to its extended neighborhood in South and Central Asia.
Contributions to countries in Africa and South America remain at a low level but have risen in
the past 10 years. India addresses two target groups with its humanitarian engagement: the
individual recipient countries and the international community.

Recently in Covid crisis India donated vaccines to sixty nine countries. India has a history of
providing drugs at affordable prices for example antiretroviral drugs. Shashi Tharoor says that
humanitarian assistance is explicitly part of India’s aspiration to lead by the “power of
example”. India is the first responder and a net security provider for the non-traditional threats
in its neighbourhood and believes in winning the hearts and minds of people. Former
Ambassador Vishnu Prasad opines that we can not call it as a humanitarian diplomacy because
diplomacy implies a give and take but India has no agenda when it comes to aid and India
believes in Vasudevay kutumbkam.

6(a) Fight for South China Sea is the fight for the soul of international order. Comment. What
is India’s approach to South China Sea?
Present international order which was envisaged by the west is in flux as it is not able to cater to
new geopolitical facts such as the enormous rise of China in the east. One of the many outcomes
of it is the rising contention between the USA and China. It is most visible in Asia-pacific and
South China Sea is one of a glaring example where China makes expansive claims in the form of
nine dash lines. Beijing claims roughly 90% of the sea, which encompasses an area of around 3.5
million square kilometers.

One-third of the world’s shipping carrying about US$3 trillion worth of trade passes through the
South China Sea each year, and the area is also the site of major oil and gas deposits. It is also a
fishing ground crucial for food security. Over time China has built up and enlarged several
islands, putting military facilities, runways, and fighter aircraft on them. It has used a variety of
methods including fishing fleets, fisheries administration, coast guard, and a militia to assert
sovereignty and plant its flag. Today, the South China Sea is, in practice, a Chinese lake.

According to Shashi Tharoor and Samir Saran ,the outcomes of posturing between USA,china
and other nations in the region surrounding south china sea will reflect the shifts in
international order because China’s aggressive naval build-up in the South China Sea (SCS) is
displacing American military power in the Pacific, and sowing discord among the member states
of Association of Southeast Asian Nations (ASEAN).China has used the South China Sea issue to
make it clear that a distant United States is not a reliable partner for others with claims in the
area, nor are international institutions and international law of any benefit. Instead, the
message is that parties must deal directly with China. The political effect of this behavior in the
South China Sea has been to worsen China’s relationship with every country in the region. By
expanding its security perimeter and militarizing the South China Sea, China has complicated the
unrestricted freedom of maneuver that the United States has enjoyed in the western Pacific and
seas near China since 1945.

China has successfully changed the military balance in its immediate periphery in its favor. Its
new strategy has been profitable, increasing its influence among south-east Asian countries and
in its maritime periphery.A breakdown in international order is signaled when the great powers
start to disagree about the rules, as we see in the South China Sea. In the South China Sea, China
is showing that she can break rules set by others, namely, the Law of the Sea (The Permanent
Court of Arbitration ruled that Chinese claims over 90 percent of the South China Sea area are
illegitimate and under UNCLOS, China is intruding into the Philippines’ sovereign waters as the
9-dash line which includes the Scarborough shoal and crosses into the Philippines EEZ. China out
rightly rejected the ruling), and that the United States and the international community choose
to do little about it. Since 2008 China has used issues like the South China Sea to effectively
weaken U.S. credibility in the region. ASEAN countries, once U.S. allies, then reluctant to choose
between the two, are now unable to agree on a common public position even on their own
issues with China such as the South China Sea, let alone stand with the United States against
China. The three pillars of the Westphalian order—states, sovereignty, and non-intervention—
are all breaking down, and are under attack.

Both India and China have developed and changed since the strategic framework was put in
place in the 1980s. New issues such as maritime security and cyber security have come to the
forefront. As a result of development, their interests have grown and expanded, and they now
rub up against one another in the periphery they share, extending to the South China Sea. About
38 percent of Indian trades and US$66 billion worth of its exports transits the South China Sea,
making freedom of navigation there vital. As a consequence, India’s stakes in the peace and
stability of the area have grown. Thus India has been focussing on both internal as well as
external balancing. India works with partners in the region such as Singapore, Japan, and
Vietnam in new ways extending to defense and security issues. Despite China’s protests, India
continues its oil exploration in Vietnam’s exclusive economic zone (EEZ) in the South China Sea
from where ONGC Videsh Limited supplies oil to Vietnam. In contrast to China, India amicably
settled its maritime dispute with Bangladesh and accepted Hague Tribunal’s ruling.

Recently, Prime Minister Modi gave maritime Panchsheel at the UNSC virtual debate. The
principles are removing barriers from legitimate maritime trade, encouraging responsible
maritime connectivity, settling maritime disputes through peaceful means and on the basis of
international law, jointly facing natural disasters and maritime threats created by non-state
actors, and preserving the maritime environment and resources. He emphasized that we want
to make an inclusive framework on maritime security in our region based on the vision of SAGAR
(Security and Growth for All in the Region). This vision is for a safe, secure, and stable maritime
domain. Also, India has become comfortable with the idea of Indo-pacific and the QUAD
grouping is a reality now. There are plans afoot to make it an economic and security grouping.

According to Shiv Shankar Menon, if the South China Sea is a proxy for the larger strategic
adjustment playing out between China and the United States across the region, China’s
intentions and U.S. responses are on display for the region and the world to draw their own
conclusions. When two big powers start to differ on common principles then indeed the world
order is in danger. Countries like India should not go for false choices of taking sides. Rather
Each nation can balance, hedge, or even bandwagon, as needed, and also work with other rising
and established powers in the region—most of whom reject assumptions of hierarchy,
centrality, and superiority. In other words, each can pursue its interests independently.

6(b).USA’s exit necessitates paradigmatic shift in India’s Afghanistan policy. Explain?


India, in recent times, has been confronted with some hard choices in Afghanistan. A decade-
long policy of providing huge humanitarian and developmental assistance, which has accrued
tremendous goodwill among the Afghans, is perceived to be in imminent danger of being
disrupted and overwhelmed by the withdrawal of the United States. This is in addition to the
recently shifting discourse of negotiating with the Taliban, which is interpreted as an upsurge of
Pakistani influence in Kabul. The choice for India was never whether it should stay engaged in
Afghanistan or not. Even in the face of repeated onslaughts on its personnel and mission, India
was committed to staying the course.

India’s role in post-9/11 Afghanistan has been a subject of intense policy deliberations, at times
bordering on speculations. On one hand, India is seen to play a critical role in the reconstruction
of the war-ravaged country, and on the other, India’s growing influence in Afghanistan as a
result of its aid diplomacy is said to have increased Pakistan’s real or imagined concerns of
encirclement in what it perceives to be its ‘strategic backyard’. This ‘zero-sum’ geopolitical
rivalry between India and Pakistan, dubbed by Western analysts as the ‘new great game’, is
seen as a source of further instability. India’s interests in post-9/11 Afghanistan have centred on
three broad objectives – security concerns, economic interests and regional aspirations.

In understanding India’s strategic thinking on Afghanistan, it would be useful to locate India‟s


strategic mind within the broad parameters of three schools of thought/paradigms:

1. Realist,
2. Neo-liberal, and
3. Nehruvian-Marxist.

The Realist perspective envisions India’s aid and development assistance as a useful tool
directed at supporting the nascent democratic regime, the strengthening of which denies the
space for return of the Taliban to the seat of power.

The Neo-liberals prod India to take a more proactive role as a major regional and economic
power with ambitions of extending its influence beyond its immediate neighbourhood for
tapping enormous energy and trade potential in the region. India has worked towards this
objective by reviving the role of Afghanistan as a land bridge, which connects South Asia with
energy rich Central Asia for economic and energy interests. By linking stability with greater
economic integration, India has been actively promoting greater trade and economic
integration of Afghanistan with South Asia through the regional mechanism of South Asian
Association of Regional Cooperation (SAARC). The thinkers in the Nehruvian tradition, while
arguing against interference in internal affairs, emphasise the need for regional cooperation
and restoring ‘peace and neutrality’ in Afghanistan, preferably under the aegis of the United
Nations (UN).
To achieve these objectives, as indicated by various strands of Indian thinking, India adopted the
‘soft power’, reviving its historical, traditional, socio-cultural and civilisational linkages with the
war-ravaged country. This ideational and enabling role which has been well received by the
Afghans, has important lessons for the international community for the long-term stabilisation
of Afghanistan.

India’s interests in Afghanistan need to be viewed in the security paradigm, in the context of
India’s concerns of terror emanating from the extremely volatile Pakistan-Afghanistan border
spilling into India. A strong, stable and democratic Afghanistan would reduce the dangers of
extremist violence and terrorism destabilising the region. The Realist line of thinking is driven by
‘fear’ of security considerations. This could be real or perceived fear of Pakistani military and
intelligence agencies regaining their ‘strategic depth’ by reinstalling a pliant Taliban regime in
Afghanistan much to the detriment of India’s security imperatives. Thus, this stream of thinking
emphasises on the need for India to use military and diplomatic tools to secure its ‘outer
periphery’ or ‘extended neighbourhood’.

Since 9/11, New Delhi’s policy has broadly been in congruence with the US objectives of
decimating the Taliban-Al Qaeda combine and instituting a democratic regime in Kabul.
Alongside the United States, India has spent the past 20 years trying to foster a democratic
system in Afghanistan. It invested $3 billion into building Afghan roads, bridges, schools and
clinics. In 2015, Indian Prime Minister Narendra Modi traveled to Kabul to inaugurate the $90
million parliament building, made with marble quarried from Rajasthan. He and then-Afghan
President Ashraf Ghani spoke about their "special friendship" that was "bound by a thousand
ties."

However, today, with the Taliban in power, India has had to rethink its strategy towards the
group by engaging with them. By holding the Third Regional Security Dialogue on Afghanistan
recently, chaired by NSA Ajit Doval, India has sent out three strong messages: one, that it wishes
to remain an important and engaged player in the future of Afghanistan; second, that with the
exit of U.S.-NATO troops, the ideal solution to the situation is through consensus in
Afghanistan’s extended neighbourhood including Russia; and third, that the Afghan
humanitarian crisis should be the region’s immediate priority and political differences can be set
aside to help.

The Delhi Declaration issued by the eight participating nations, including Iran and Russia, is a
milestone in keeping India inside the discussion on Afghanistan. The declaration goes farther
than the previous such regional discussion of SCO countries in Dushanbe in September 2021, in
its strong language on terrorism, terror financing and radicalisation. It also expands on the need
for an inclusive government in Kabul that will replace the Interim Taliban regime, and promotes
a national reconciliation process. The Taliban have welcomed the Declaration.
More than the list of participants or the outcome, the fact that this dialogue was the first-of-its-
kind hosted by India is a significant diplomatic statement. By throwing its hat in the ring, India is
signalling that it isn’t going to just roll over and play dead after the Taliban takeover of
Afghanistan. The message is that going forward, India will not just be a bystander but a player.
India will use her diplomatic heft, and bring to bear all her equities and influence with regional
players to regain agency in Afghanistan, if not directly, then through its partners. At the same
time, India will explore avenues and opportunities to forge a common front with all the
participating countries on issues of common concern—terrorism, radicalisation, migration and
cross-border movement of people, narcotics trafficking, proliferation of small arms.

6(c) New Delhi faces challenges in managing its relations with Iran within its competing
interests and conflicting relations with other countries. Discuss?

Iran, one of the richest countries in hydrocarbon and India, a rapidly growing economy with a
deep thirst for energy are the hallmarks of the India-Iran relationship. Nevertheless, Iran’s
strategic significance to India is crucial, compelling India to remain tied to Iran in face of
insurmountable difficulties. Under the current scenario, had relations been solely based on
energy, Iran would have drifted away from India. However, Iran’s geographical position is
paramount to India’s geopolitical outreach, especially to Central Asia, a rich reservoir of natural
resources. Similarly, Iran is vital for India’s access to Afghanistan in which India has immense
strategic and security interests.

The benefits of India-Iran ties are mutual. An important element in the India-Iran relationship is
India’s status as a huge market for Iranian products. India’s choice of Iran as a gateway to
Central Asia and Afghanistan also has positive repercussions on Iran’s aspirations of becoming a
regional hub for trade and transit. Finally, under the backdrop of Iran’s nuclear controversy and
US efforts to isolate Iran, the continued relationship of the Islamic Republic of Iran with the
world’s largest democracy has immense ideational value.

Such an understanding of reciprocally advantageous consequences motivated India and Iran to


formally sign two consecutive comprehensive joint documents reflecting their vision of strategic
partnership. The first is the ‘Tehran Declaration’ signed in April 2001, and the second is the ‘New
Delhi Declaration’ signed in January 2003. While these documents serve as a template for
greater regional and global cooperation, they also highlight the realisation of the scope of such
a partnership. Notwithstanding these efforts, the cloud of the Iranian nuclear controversy and
India’s need for closeness to the United States had implications on India-Iran relations.

Before the two countries could fully implement the tenets of the New Delhi Declaration, the
Iranian nuclear controversy intensified, leading to a vote on a resolution at the International
Atomic Energy Agency (IAEA) in September 2005, to refer the Iranian nuclear file to the United
Nations Security Council. By virtue of India’s membership on the IAEA board at that time, India
was compelled to take a stand. The fact that India voted against Iran at a time when it was also
negotiating a civil nuclear deal with the United States, created mistrust in the minds of the
Iranians.

The maturity and compulsions of India-Iran relations along with the countries’ leaders’
pragmatism effectively manoeuvred this relationship, leaving this teasing episode behind sooner
than expected. Still, the issues surrounding the Iranian nuclear programme, its regional role, and
resultant antagonism with other countries have its share of pressure on the potentially deep and
broad bilateral ties between India and Iran. Since the onset of this controversy, India has
indulged in a balancing act between its relationship with Iran, on one hand, and with the US,
Saudi Arabia, and Israel, on the other. The current US effort to impose “maximum pressure” on
Iran to ensure its seclusion has its toll on the India-Iran relationship. Amongst the visible effects
of the Iran-US enmity is Iran’s increased ties with China, an adversary of India.

In order to understand the current depth and anticipate the future course of India-Iran relations,
three core issues could be underlined, which serve as a pointer to this trajectory. The first is
Iran’s Farzad-B Gas Field in which an Indian state-owned company, ONGC, was involved at the
discovery stage. The second prominent issue that has become a marker of the evolving India-
Iran relationship is the proposed railway line connecting Chabahar to Zahedan, the capital of
Iran’s Sistan and Baluchestan Province. The third important issue is the Chabahar Port project.
In this realm, India has achieved considerable success, specifically in terms of starting the
operation of this project after the initial investment for its development.

Since May 2019, under pressure of stringent economic sanctions from the Trump
administration, India had brought oil imports from Tehran down to zero. However, in recent
weeks, Iran has emerged as a strong Indian ally in the region vis a vis India’s relation with
Afghanistan. Due to which, there is a strong possibility that India woud resume its oil imports
from Iran shortly. Iran’s ambassador to India Ali Chegeni said “We have no limit, no ban, no
barrier in developing our relationship with India … India used to be our big customer of oil, and
we were the first country to export oil to India based on the rupee. It was recognition of the
rupee as international money, instead of using the dollar or some other currency. We did that
also to buy commodities from India. For oil, we expect India to begin (purchases).”

Connectivity is another imperative for India’s engagement with Iran. With the complicated
situation in Afghanistan, along with Pakistan’s obdurate refusal of transit passage across its
territory to Indian goods, Iran remains the portal for India to the coveted markets and resources
of Central Asia. All these factors are causing India and Iran to reboot their age-old relations. The
election of President Raisi and the possibility of a US-Iran rapprochement may open a new
chapter in Indo-Iranian relations. Nevertheless, given the many other fault lines in the region,
primarily that between Iran on the one hand and some Gulf states and Israel on the other, with
whom New Delhi enjoys excellent bilateral relations, India will have some deft balancing act to
do.

7 (a) Enumerate the success and failures of India’s diplomacy during the Covid?

India’s Diplomacy led by PM Modi during the Covid was on three issues

1. Appealing for aid from the International Community


2. Pushing other countries for support at the WTO for waiving patents on Covid 19 vaccines
3. India’s image management during the crisis

The appeal that went out from the govt. amplified through its embassies have met with success.
Offers of aid have poured in. Acc. To govt. figures from 27 April to 6 May, India received 1841
Oxygen Concentrators, 1800 Oxygen cylinders, 9 big Oxygen generation plants, 2403
Ventilators/ BiPAPs/ CPAP and nearly 3 lac vials of Remdesivir. Germany provided the largest
oxygen plant which arrived in 2 plane loads which had the potential for 4,00,000 litres of oxygen
per day. The same was established by DRDO in Delhi during the peak of the 2nd wave.

India’s vaccine diplomacy- vaccine maitri was heralded as a success but the disastrous 2nd wave
of Covid in 2021 led many to challenge India’s intent and status as a vaccine hub in the fight
against Covid 19. An embattled India sought help and gratefully accepted what it got. This
assistance came with a sense of camaraderie—with confidence, eagerness, and without
prejudice. The tone and tenor of aid being offered is markedly different from ‘north to south’,
‘rich to poor’, ‘developed to developing’, or ‘conditions-based’ grants of the past. It is also
qualitatively different from the all-round aid India received in the 1960s or 1970s. This time,
India was short of only essentially two commodities—oxygen and specific medicines like
Remdesivir. As such, its requests were focused and purposeful.

The success of India in the international sphere despite the failure of management of the 2 nd
wave of Covid was due to couple of reasons. One reason is India’s emerging salience in global
affairs. India today is a strategic partner to many nations. It is a dependable stakeholder in the
Indo-Pacific, a dynamic market-based emerging economy with a direct bearing on
geoeconomics, and a core member of the club of democracies. How India responds and recovers
will be a test for emerging arrangements on economic interdependence, regional security,
artificial intelligence, reforming multilateralism, and trade negotiations. India, despite its low
per capita income is too strategic to fail—or even to be put out of action for too long.
As Australia sent support, it spoke of India as a key partner in the Quad in vaccine production.
The Australian government emphasised the criticality of helping India recover quickly, for its
production capacity is important for the world to fight COVID. The United States’ move to waive
IP protection for COVID-19 vaccines, its willingness to negotiate at the WTO, and its delivery of
raw materials required for India to make millions of vaccine doses showed that shoring up
India’s domestic manufacturing capacity is a key motivation for external assistance. A similar
perspective came from the India-EU Summit. It highlighted cooperation on “resilient medical
supply chains, vaccines, and the Active Pharmaceutical Ingredients (APIs)” as central to the joint
fight against COVID.

The speed and scale of global support could very well be attributed to India’s active and
effective foreign engagements. It demonstrates how Indian diplomacy punches above its weight
on the international stage and can mobilise help when needed. India today is understood more
comprehensively around the world. Its actions and policy positions on global affairs are more
pronounced and appreciated.

Despite the various failures and successes, the pharmacy of the world has to live up to its
promise as a vaccine source for not just itself, but also its neighbours and the Global South.
Without India being vaccinated and without Indian vaccine capacity being widely available, the
world will not be able to defeat COVID-19. As many have repeated over the past year, “No one is
safe until everyone is.”

7 (b) Discuss the context of emergence of BRICS. What should be the Indian imperative in the
BRICS?

BRICS is the acronym coined to associate five major emerging economies: Brazil, Russia, India,
China, and South Africa. The BRICS members are known for their significant influence on
regional affairs. It does not exist in form of organization, but it is an annual summit between the
supreme leaders of five nations. The Chairpersonship of the forum is rotated annually among
the members, in accordance with the acronym B-R-I-C-S. Together, BRICS accounts for about
40% of the world’s population and about 30% of the GDP (Gross Domestic Product), making it a
critical economic engine. It’s an emerging investment market and global power bloc.

The acronym "BRICS" was initially formulated in 2001 by economist Jim O'Neill, of Goldman
Sachs, in a report on growth prospects for the economies of Brazil, Russia, India and China –
which together represented a significant share of the world's production and population. In
2006, the four countries initiated a regular informal diplomatic coordination, with annual
meetings of Foreign Ministers at the margins of the General Debate of the UN General Assembly
(UNGA). This successful interaction led to the decision that the dialogue was to be carried out at
the level of Heads of State and Government in annual Summits.

The BRIC grouping's 1st formal summit, held in Yekaterinburg, Russia commenced on 16 June
2009, with Luiz Inácio Lula da Silva, Dmitry Medvedev, Manmohan Singh, and Hu Jintao, the
respective leaders of Brazil, Russia, India, and China in attendance. The summit's focus was on
improving the global economic situation and reforming financial institutions, and discussed how
the four countries could better co-operate in the future. There was further discussion of ways
that developing countries, such as 3/4 of the BRIC members, could become more involved in
global affairs. South Africa officially became a member nation on 24 December 2010, after
being formally invited by China to join. In April 2011, the President of South Africa, Jacob Zuma,
attended the 2011 BRICS summit in Sanya, China, as a full member.

The 13th BRICS summit is set to be held on 9 September in digital format under India’s
chairmanship. As the current chair, India has outlined four priorities.

The first is to pursue reform of multilateral institutions ranging from the United Nations, World
Bank and the International Monetary Fund to the World Trade Organization and now even the
World Health Organization.

The second is the resolve to combat terrorism. Terrorism is an international phenomenon


affecting Europe, Africa, Asia and other parts of the world. In this context, BRICS is attempting
to pragmatically shape its counter-terrorism strategy by crafting the BRICS Counter Terrorism
Action Plan containing specific measures to fight radicalisation, terrorist financing and misuse of
the Internet by terrorist groups. This plan is expected to be a key deliverable at the forthcoming
summit and may hopefully bring some change.Promoting technological and digital solutions for
the Sustainable Development Goals expanding people-to-people cooperation.

For BRICS to remain relevant over the next decade, each of its members must make a realistic
assessment of the initiative's opportunities and inherent limitations. BRICS nations need to
recalibrate their approach and to recommit to their founding ethos. BRICS must reaffirm their
commitment to a multi-polar world that allows for sovereign equality and democratic decision
making by doing so can they address the asymmetry of power within the group and in global
governance generally.

The five-power combination has succeeded, albeit up to a point. But it now confronts multiple
challenges: China’s economic rise has created a serious imbalance within BRICS; Beijing’s
aggressive policy, especially against India, puts BRICS solidarity under exceptional strain; and
BRICS countries have not done enough to assist the Global South to win their optimal support
for their agenda. It is necessary for leaders and officials of this grouping to undertake serious
soul-searching and find a way to make the organization relevant in the post-pandemic world
order.

7(c) Do you think BIMSTEC has emerged as the preferred grouping of countries in South Asia?
Enumerate some recent initiatives in BIMSTEC?

After twenty-three years and a sluggish pace of growth, the Bay of Bengal Initiative for Multi-
Sectoral Technical and Economic Cooperation (BIMSTEC) has gained salience in recent times
against the backdrop of changing geopolitical dynamics in the Bay of Bengal region.

The Bay of Bengal Initiative for Multi-Sectoral Technical and Economic Cooperation (BIMSTEC)
is a regional multilateral organisation. Its members lie in the littoral and adjacent areas of the
Bay of Bengal constituting a contiguous regional unity. Out of the 7 members, five are from
South Asia – Bangladesh, Bhutan, India, Nepal and Sri Lanka. Two are from Southeast Asia –
Myanmar and Thailand.

BIMSTEC is an economic bloc that came into being on 6 June 1997 through the Bangkok
Declaration. It aims to accelerate economic growth and social progress among members
across multiple sectors — trade, technology, energy, transport, tourism and fisheries,
agriculture, public health, poverty alleviation, counter-terrorism, environment, culture, people
to people contact and climate change. The BIMSTEC region is home to roughly 22 per cent of the
global population with a combined GDP of over $2.7 trillion.

Initially, the economic bloc was formed with four countries with the acronym ‘BIST-EC’
(Bangladesh, India, Sri Lanka and Thailand Economic Cooperation). With the entrance of
Myanmar in 1997, the grouping was renamed ‘BIMST-EC’ (Bangladesh, India, Myanmar, Sri
Lanka and Thailand Economic Cooperation). Finally, with the entrance of Nepal and Bhutan at
the 6th Ministerial Meeting in 2004, the grouping was named Bay of Bengal Initiative for Multi-
Sectoral Technical and Economic Cooperation (BIMSTEC).

Reasons why BIMSTEC has emerged as the preferred grouping of countries in South Asia:

a) The Bay of Bengal has grown in strategic significance within the Indo-Pacific, especially
due to the contest between India and China. What was once a region bereft of any major
conflicts is now poised to become “a zone of geopolitical rivalry among major powers
and of regional conflict”, argued C. Raja Mohan.
b) Regional Blocs like BIMSTEC help small nations to gain higher economic dividends than
fragmented multilateralism promoted by the World Trade Organization (WTO).
c) Due to setbacks to the South Asian Association of Regional Cooperation (SAARC),
particularly when its 2016 summit scheduled to be held in Pakistan was suspended after
member countries declined to participate, BIMSTEC has emerged as the “preferred
platform” for regional cooperation in South Asia.

Some key agreements signed by BIMSTEC members include a convention for combating
terrorism, transnational organised crime and illicit drug trafficking. However, this awaits
ratification. Another is the BIMSTEC Grid Interconnection, signed during the BIMSTEC Summit in
Kathmandu, Nepal, in 2018, which aims to promote an optimal power transmission in the
BIMSTEC region. In April 2021, the foreign ministers of BIMSTEC met in a virtual conference. This
is the first ministerial since the globe has been hit by the Covid-19 pandemic. They cleared the
draft for the BIMSTEC charter, recommending its early adoption. They endorsed the
rationalization of sectors and sub-sectors of activity, with each member-state serving as a lead
for the assigned areas of special interest. The ministers also conveyed their support for the
Master Plan for Transport Connectivity, which will be adopted at the next summit in Sri Lanka.
Preparations were completed for the signing of three agreements relating to mutual legal
assistance in criminal matters, cooperation between diplomatic academies, and the
establishment of a technology transfer facility.

As BIMSTEC readies itself to celebrate the silver jubilee of its formation next year, it faces a
serious challenge i.e. to become relevant to its member states. In recent years, Bangladesh has
started viewing BIMSTEC as a platform to strengthen its economic development while Sri Lanka
intends to fulfill its aspirations to emerge as a major transhipment hub for the wider Indo-Pacific
region. For the two landlocked Himalayan member states, Nepal and Bhutan, BIMSTEC is an
opportunity to get access to the Bay of Bengal. For Myanmar and Thailand, connecting more
closely with India across the Bay is an opportunity to get access to India’s consumer market and
to address overdependence on China. For India, BIMSTEC is a platform to fulfill India’s key
foreign policy priorities of ‘Neighbourhood First’ and ‘Act East.’

8(a) With no political solution to the border issue on the horizon, the military disengagement
merely provides temporary relief for a chronic problem. Comment. Do you think India and
China need more than a border pullback to mend fences?

After many months of tense military confrontation, which included the first deadly clash in
decades between Indian and Chinese troops along their disputed border, the two sides began
withdrawing from their positions on the southern and northern banks of the lake as part of a
phased, synchronized military disengagement from 11 February.

By mitigating the risk of another skirmish or accident, the move has brought Beijing and New
Delhi back from the brink in their border standoff. The successful disengagement was followed
up on 20 February with a tenth round of meetings between the Indian and Chinese commanders
in the region. Five days later, Indian Foreign Minister S. Jaishankar spoke by phone with his
Chinese counterpart, Wang Yi, affirming the disengagement as “a significant first step,”
according to an Indian Foreign Ministry spokesperson. The two sides also unveiled plans to
establish a diplomatic hotline to aid in future crisis management. Affirming the disengagement
as “a significant first step,” according to an Indian Foreign Ministry spokesperson, the two sides
also unveiled plans to establish a diplomatic hotline to aid in future crisis management.

While positive, these de-escalatory steps do not guarantee peace, much less a return to
conciliatory Sino-Indian ties. “This is a tactical withdrawal, and it is temporary on both
sides,” cautioned Shivshankar Menon, a former Indian national security adviser. A political
agreement on how to resolve the larger border dispute is still lacking, and there is little trust left
to speak of between New Delhi and Beijing, making the path ahead perilous.

The two rivals continue to blame one another for the deadly brawl last June in the Galwan
Valley, offering conflicting accounts of what precipitated it. Both sides maintained that it was
the other advancing beyond their agreed-upon patrol areas that led to the clash. At the time,
India said that 20 of its soldiers were killed in the hand-to-hand combat, while China belatedly
announced last month that four of its soldiers had died.

Now, Indian officials are emphasizing the need to verify that all agreements are being “observed
in letter and spirit,” as Gen. Manoj Naravane, India’s army chief, put it in recent public remarks.
Both militaries will be watching closely to ensure the other does not renege on their
commitments.

Synchronized pullbacks are fragile; it is worth remembering that the Galwan Valley clash
occurred in the midst of a failed effort at disengagement. Even if it sticks, the Pangong Lake
withdrawal is merely the start of a complex process. Naravane has admitted there is still “a long
way to go,” and that both sides must now focus on de-escalation and troop withdrawal from
other areas along the Line of Actual Control, the de facto demarcation that separates Indian-
and Chinese-controlled territory.

Kanti Bajpai in his book, “India Versus China: Why They Are Not Friends,” writes that “India and
China are not friends for four key reasons: deep seated differences over their perceptions of
each other, their territorial perimeters, their strategic partnerships and, most importantly, the
asymmetry of power.”

In recent years, the bilateral perceptions have deteriorated further. On one hand, China from its
position of strength does not view India as a hegemonic rival. It does not see the need to
accommodate India on various issues. On the other hand, India, as a rising power feels the need
to push-back Chinese ambition in order to grow as a regional power. India feels that succumbing
to Chinese pressure would lead to erosion of strategic autonomy and thus, continues to project
China as a looming enemy in the periphery.

In this context, one can analyse the power asymmetry between the two nations. When it comes
to soft power, China using its economic might has been able to influence several countries in
various regions. India on the other hand is lagging far behind in terms of investments or
economic reach. However, even in this battle, all is not lost for India, as it has been able to
create a sense of good will amongst many countries in the world through diplomatic aid and
other means. Nonetheless, in a battle between economic might and good will, one can safely
bet which one would trump the cards.

When it comes to hard power, there is a clear disparity which is obvious. Although India has the
larger army, China has three-and-a-half times more aircraft, three times more submarines and
twice as many nuclear weapons. But it’s the difference in indigenous defence production and
China’s ability to produce new technologies that gives Beijing the decisive edge.

The disparity in hard power is obvious when it comes to Chinese aggression of India’s borders
especially in Arunachal Pradesh and Ladakh. The lack of agreement on the border lines have
continued to make Perimeter as a matter of concern for both the countries.

This brings us to the last aspect: Partnerships. When it comes to India-Chinese relations,
bilateral trade is one of the biggest features. Despite tensions in Galwan and Ladakh regions, in
2021, Bilateral trade between India and China grew over 62 per cent. The total bilateral trade
between the two countries increased to $57.48 billion in the first half of 2021.

In conclusion, the four aspects of Perception, Power, Partnership and Perimeter govern the
relation between India and China. The growing disparity between the two countries in these
parameters has led to a very limited prospect of a lasting rapprochement.

8(b) The three major tenets of India’s Nuclear doctrine, No First Use, Massive
retaliation,Credible Minimum Deterrence are no longer relevant to India’s Nuclear
conversations and actions. Comment?

A nuclear doctrine states how a nuclear weapon state would employ its nuclear weapons during
both peace and war. By communicating its stated intentions and resolve, nuclear doctrine helps
states to establish deterrence vis-à-vis its adversaries during peace and once deterrence fails,
guides the states response during war. India’s nuclear doctrine is an important variable
determining nuclear stability in south Asia, especially because the doctrine is considered to be
'restrained'. So any indication of change in the doctrine is a cause for concern. Such an
indication of change happened most recently in 2014, when the Bharatiya Janata Party (BJP)
released its election manifesto, in which it promised to “revise and update” India’s nuclear
doctrine to “make it relevant to challenges of current time.” This led to some speculation that a
key elements of India’s nuclear doctrine.

India’s Nuclear Doctrine:

No-First Use (NFU) Posture.

India’s nuclear doctrine envisages NFU posture and also that India will use nuclear weapons in
Retaliation. According to some analysts, NFU represents India’s cultural and traditional
abhorrence to nuclear weapons. However, according to Bharat Karnad NFU may be useful as a
political rhetoric but is a liability in a serious war planning. It requires India to absorb a nuclear
before responding in kind. According to P R Chari NFU frees Pakistan from fearing an Indian
nuclear response to either terrorism or limited war.

Credible Minimum Deterrence.

This has been criticised on the ground that minimum deterrence can't be credible. According to
Jayant Prasad it should be minimum credible deterrence. Basrur argues that India has moved
from deterrence to compellence after terror attack on Indian Parliament(2001).

No Use Against Non Nuclear Weapon State.

It is questioned on the ground that there is a possibility of nuclear attack by non state actors
(terrorists) from the soil of non nuclear weapon states.

Nuclear deterrence of chemical and biological weapon (CBW) use.

India’s nuclear doctrine envisages that India can retaliate against CBW use with nuclear
weapons. According to Manpreet sethi, CBW are outlawed and if Nonstate actors use these
weapons, then India’s nuclear deterrent couldn’t be effective because it is not designed to
counter such actors.

Massive Retaliation In Case Of Any Nuclear Attack.

Demands have been made to substitute ‘punitive’ for ‘massive’ in the doctrine.

According to Basrur, massive retaliation is a threat that the enemy will always expect and is
thus unnecessary. Others suggest that Pakistan’s first use might be a very limited attack
calibrated to avoid massive retaliation by India. Another problem is that it violates the principle
of proportionality and is incompatible with minimum deterrence. Thus it is proposed to
incorporate ‘flexible response’ rather than simply relying on ‘massive retaliation’.

Command And Control.


India’s nuclear doctrine maintains that political leadership will command and control the
nuclear weapons and their use. It has been contended that India’s nuclear deterrence will not be
effective unless potential adversaries accept that India has the operational capacity to employ
its nuclear weapons. India’s nuclear operational capacity is questioned because the doctrine
leaves the military out of the decision making loop. However creation of the post of Chief of
Defence Staff (CDS) has been an important step.

After the 1998 nuclear test when India declared itself as a nuclear weapon state, it also
enunciated a doctrine of No-first- Use (NFU), which categorically rejected the idea of initiating
the use of nuclear weapons in any conflict scenario and thus India’s nuclear doctrine is purely
retaliatory in nature.

However, despite the fact that India’s self-proclaimed restraint in the form of NFU has formed
the basis for its claims to belong to the nuclear mainstream, the sanctity of NFU has been called
into question many a times when political leaders have tried to insert an element of
ambivalence into India’s nuclear doctrine. The case to revoke the NFU pledge has been based on
two grounds:

a)NFU allows Pakistan to take the initiative while restricting India’s options and thus put India in
a disadvantageous position militarily.

b)The increasing asymmetry of conventional military power between china and India.

Thus by revoking NFU, India could leverage its nuclear capability where it fails to deter china
conventionally.

8(c) The form and the thrust of the negotiations are changing at WTO and India is watching
from the sidelines. Comment.

The World Trade Organization (WTO) is the only global international organization dealing with
the rules of trade between nations. At its heart are the WTO agreements, negotiated and signed
by the bulk of the world’s trading nations and ratified in their parliaments. The WTO has 164
members (including European Union) and 23 observer governments (like Iran, Iraq, Bhutan,
Libya etc).

World Trade Organization, as an institution was established in 1995. It replaced General


Agreement on Trade and Tariffs (GATT) which was in place since 1946. After World War II,
western countries came out with their version of development, which was moored in promotion
of free trade and homogenization of world economy on western lines. This version claims that
development will take place only if there is seamless trade among all the countries and there are
minimal tariff and non- tariff barriers. Along with two Bretton wood institutions – IMF and
World Bank, an International Trade Organization (ITO) was conceived.

ITO was successfully negotiated and agreed upon by almost all countries. It was supposed to
work as a specialized arm of United Nation, towards promotion of free trade. However, United
States along with many other major countries failed to get this treaty ratified in their respective
legislatures and hence it became a dead letter. Consequently, GATT became de-facto platform
for issues related to international trade. It has to its credit some major successes in reduction of
tariffs (custom duty) among the member countries.

Measures against dumping of goods like imposition of Anti-Dumping Duty in victim countries,
had also been agreed upon. It was signed in Geneva by only 23 countries and by 1986, when
Uruguay round started (which was concluded in 1995 and led to creation of WTO in Marrakesh,
Morocco), 123 countries were already its member. India has been member of GATT since 1948;
hence it was party to Uruguay Round and a founding member of WTO. China joined WTO only in
2001 and Russia had to wait till 2012. While WTO came in existence in 1995, GATT didn’t cease
to exist. It continues as WTO’s umbrella treaty for trade in goods.

WTO’s Appellate Body:

The Appellate Body, set up in 1995, is a standing committee of seven members that presides
over appeals against judgments passed in trade-related disputes brought by WTO members.
Countries involved in a dispute over measures purported to break a WTO agreement or
obligation can approach the Appellate Body if they feel the report of the panel set up to
examine the issue needs to be reviewed on points of law. Existing evidence is not re-examined;
legal interpretations are reviewed.

The Appellate Body can uphold, modify, or reverse the legal findings of the panel that heard the
dispute. Countries on either or both sides of the dispute can appeal. The WTO’s dispute
settlement procedure is seen as being vital to ensuring smooth international trade flows. The
Appellate Body has so far issued 152 reports. The reports, once adopted by the WTO’s disputes
settlement body, are final and binding on the parties.

WTO Appellate Body on Shaky Ground:

Over the last two years, the membership of the body has dwindled to just three persons
instead of the required seven. The understaffed appeals body has been unable to stick to its 2-
3 month deadline for appeals filed in the last few years, and the backlog of cases has prevented
it from initiating proceedings in appeals that have been filed in the last year.
With the Appellate Body unable to review new applications, there is already great uncertainty
over the WTO’s dispute settlement process. If the body is declared non-functional, countries
may be compelled to implement rulings by the panel even if they feel that gross errors have
been committed.

Doha Development Agenda 2001

A significant casualty at WTO has been the lack of any meaningful outcome of the negotiations
on rules of trade that members set for themselves under the Doha Development Agenda in
2001. Adopted by consensus, the Nairobi Declaration had then noted that while “many”
members reaffirm the Doha mandate, others do not, and that while “many Members want to
carry out the work on the basis of the Doha structure”, “some want to explore new
architectures”, as well as “identify and discuss other issues for negotiation”. India had then
expressed its deep disappointment at the Declaration.

The spark lit by the “some” in Nairobi for “new architectures” and “new issues”, resulted in
further fragmentation at the 11th ministerial conference in Buenos Aires in 2017. This saw the
birth of joint statement initiatives (JSIs) in four areas: E-commerce, investment facilitation,
services domestic regulation (DR) and micro, small and medium enterprises (MSMEs). The JSI
participants claimed that the objective was to have focused discussions among interested
members to achieve concrete outcomes. India and South Africa pointed out the risks that
fragmented JSIs pose to the foundations of multilateralism.

JSIs have, however, grown. In the build-up to WTO’s 12th ministerial conference scheduled for
November, concrete outcomes are envisaged on the services DR and ecommerce, and in
investment facilitation. India has remained outside all these negotiations. There are now 64
participants for the JSI.

But staying out of the room only deprives India of an opportunity to influence the outcomes. As
an original multilateralist, India needs to play a more constructive role.
PSIR Crash Course 2021
Model Answers - Test 7

For any issue related to PSIR Test Series, write only at psirtestseries@shubhraranjan.com.

Section A

1(a) Post Behaviouralism is not the negation of behaviouralism rather its corrective.
Comment.

Behaviouralism seeks to study politics as an aspects of human behaviour and prescribes the use
of empirical research, mathematical-statistical-quantification techniques of data collection and
analysis with the purpose of building a scientific theory political behavior.

Almond and Powell defines the behavioural approach as the study of the actual behaviour of the
incumbents or political roles, rather than of the content of legal rules or ideological patterns. For
Robert Dahl, it was an attempt to make empirical component of political science more scientific.
It aims at stating all the phenomena of government in terms of observed and observable
behaviour of men.

Post-behaviouralism, on the other hand emerged as a reform movement to oppose the efforts of
the behavioral approach to make political science a ‘value-free’ science. It is a future oriented
approach aimed at solving the social problems of both present and future.

While Behaviouralism started as a methodological revolution to address the decline of


traditionalism , post-behaviouralism was based on the synthesis between traditionalism and
behaviouralism . Post-behaviouralism gives priority to purpose or substance over technique.
Post-behaviouralism views political science as an ‘applied’ science rather than a ‘pure’ science.

Therefore, post-behaviouralism is not a rejection of behaviouralism. In fact, the rise of post-


behaviouralism ended the contradiction between ‘fact and value’ as it was based on normative-
empirical synthesis.

1(b) Write short note on the neo-liberal perspective of state.


In the past few decades, the theory of laissez-faire liberalism was revived with a new vigour. It
sought new grounds of keeping the state away from interference in the market forces. It is
variously described as neo-liberalism, neo-classical liberalism, libertarianism.

The chief exponents of neoliberalism include F.A. Hayek , Isaiah Berlin , Milton Friedman and
Robert Nozick (Nozick’s book "Ananrchy, state and Utopia" ) and along with politicians and
policy-makers such as Margaret Thatcher, Ronald Reagan and Alan Greenspan. These thinkers
in the liberal tradition found the theory of 'welfare state' to be inimical to individual liberty, and
sought to revive the original concern of this tradition with laissez faire philosophy. It particularly
defends the right to acquire and hold property and freedom of contract. These rights are by no
means the product of the state itself, hence the state cannot be allowed to intervene for any
artificial balancing of rights.

Neoliberal theory contends that free markets encourage economic efficiency, economic growth,
and technological innovation. State intervention, even if aimed at encouraging these
phenomena, is generally believed to worsen economic performance.

Critics of neoliberalism like Joseph Stiglitz, Amartya Sen , Noam Chomsky, especially after the
Great Recession in 2008 have argued about the negative aspects of market fundamentalism ,
which is a major theme in Neoliberalism.

Some scholars contend that neoliberal focuses may even undermine the basic elements of
democracy. The economic inequality engendered by neoliberalism creates inequality of political
power, undermining democracy and the citizen's ability to meaningfully participate.

1(c) Write short note on the concept of original position

Justice is one of the most important concerns of the field of political philosophy since the time
Plato. Campbell considers justice as the central concept of current mainstream normative
political philosophy.

Rawls book, ‘A Theory of Justice’, presents a very strong defense of the idea of justice. Rawls
calls his conception “justice as fairness.” His aim in designing the original position is to describe
an agreement situation that is fair among all the parties to the hypothetical social contract. He
assumes that if the parties to the social contract are fairly situated and take all relevant
information into account, then the principles they would agree to are also fair. Rawls’s original
position is an initial agreement situation wherein the parties are without information that
enables them to tailor principles of justice favorable to their personal circumstances. Among the
essential features of the original position is that no one knows his or her place in society, class
position or social status, nor does any one know his or her race or gender, fortune in the
distribution of natural assets and abilities, level of intelligence, strength, education, and the like.
The principles of justice are chosen behind a “veil of ignorance”.

However, the communitarians criticize Rawls’ ‘original position’. Michael Sandel in his book
‘Liberalism and limits of justice’ argues that Rawls theory rests on a flawed understanding of
self or individual. Feminist scholars such as Carole Pateman questioned Rawls whether women
are active decision makers in original position.

1(d) End of ideology thesis

The word ‘ideology’ was coined during the French Revolution by Antoine Destutt de Tracy
(1754–1836), and was first used in public in 1796. For de Tracy, idéologie referred to a new
‘science of ideas’, literally an idea-ology.

With a rationalist zeal typical of the Enlightenment, he believed that it was possible to uncover
the origins of ideas objectively, and proclaimed that this new science would come to enjoy the
same status as established sciences such as biology and zoology. More boldly, since all forms of
inquiry are based on ideas, de Tracy suggested that ideology would eventually come to be
recognized as the queen of the sciences. However, despite these high expectations, this original
meaning of the term has had little impact on later usage, which has been influenced by both
Marxist and non-Marxist thinking.

The notion of the ‘end of ideology’ was particularly fashionable in the 1950s and 1960s. The
most influential statement of this position was advanced by Daniel Bell.

Bell was impressed by the fact that, after World War II, politics in the West was characterized
by broad agreement among major political parties and the absence of ideological division or
debate. Fascism and communism had both lost their appeal, while the remaining parties
disagreed only about which ideologies could best be relied on to deliver economic growth and
material prosperity. Daniel Bell observed “Today ideologies are exhausted...”

However, the process to which Bell drew attention was not the ‘end of ideology’ so much as the
emergence of a broad ideological consensus among major parties, and therefore the suspension
of ideological debate.

However, C. Wright Mills, C.B. Macpherson and Alasdair Maclntyre severely criticized the end of
ideology thesis. C. Wright Mills dubbed the upholders of end of ideology thesis the advocates of
status quo. In his view, it is an ideology of political complacency which appears to be the only
way now available for many social scientists to acquiesce in or to justify the established social
structure.

1(e) Saptanga Theory

Kautilya’s saptanga (seven organs) theory of state illustrates a novel dimension of state power
that dialectically engages political rationality and normativity; artha and dharma respectively.

His theory borrows from the Ayurveda medical treatise which speaks of seven elements of the
body. It echoes the ideas of health, disease and cure in the context of ‘body politic’.

Drawing from a complex political life, Kautilya hand picks a set of seven key structural elements
(prakritis) as ‘state factors’ which together constitute state power. The seven prakritis are
constitutive of swamin, amatya, janapada,,durga, kosa, danda and mitra.

With this cluster of seven prakritis of the saptanga theory, the state is theorised.

The prakritis are ordered according to the weight and importance Kautilya assigns to them, the
ruler (swamin) being the most important state element, and the ally (mitra) being the least.

Kautilya’s innovative conception of state power stands out on many counts. One that it is more
comprehensive as compared to power understood in narrow, military terms. Two, that power as
an aggregate of the prakritis can be reasonably estimated. Three, and most important of all, it
weaves the politically realist objective of enhancing state capacity together with the idealist aim
of bringing about welfare of the people. The policy of optimising the prakritis, an act driven by
‘pure power politics’, inevitably leads to happiness and welfare of the people.

2(a) Define the concept of Rights. Elaborate on the liberal individualistic perspective of Rights
with reference to Entitlement Theory of Nozick.

A right is an entitlement to act or to be treated in a particular way. As such, rights are social
claims given to individuals, as members of the society, necessary for the development of human
personality. Discourses on rights are the fundamental aspects of modern political philosophy.
Rights are seen as claims essential for the full realization of the personality of the individual.
According to Laski, “Rights are those conditions of life without which no man can seek in general
to be at his best.” T.H. Green defines rights as powers necessary for the fulfilment of man’s
vocations as a moral being. Human rights are seen as entitlement to which people are entitled
by the virtue of being human. They’re a non-negotiable ingredient of human identity. According
to James Nickel, human rights are basic moral guarantees possessed by the virtue of being
human.

Central to the theory of individualism is its conception or understanding of the self. Individualism
builds its understanding of relations between the state and the citizen as well as the proper
scope of state activities on the basis of its conception of the self. In the individualist view, people
are free, rational and capable of self-determination. People are rational in that they are the best
judges of their interest. They are capable of self-determination; that is, they are capable of
determining their own conception of good life. The liberal individualist perspective talks about
right to equality, which means equality before law. For them, state’s intervention undermines
equality .

Nozick, in his Anarchy State And Utopia, talks about the minimal state and maximization of
individual freedom and rights . In his Entitlement theory, he criticises Rawls for compromising
with the autonomy of individuals. He explicitly talks about the right to property . He argues that
if a person has acquired the property through fair means and labour, then state cannot deny
him the entitlement of his property. Nozick entitlement theory is a good example of procedural
theory. According to Nozick, outcome should not be predetermined or determined by the state.

However, social-liberal scholars like Ronald Dworkin has questioned Nozick’s theory. He
emphasizes on circumstances and fair initial distribution. Similarly, Amartya Sen has
emphasized on the ‘Capability’ approach’.

2(b) Compare Buddhist political thought with Ancient Hindu political thought. Do you think
that there is a greater element of continuity than change between the two?

According to Bhikhu Parekh, although ancient Indian political thought displays remarkable
continuity, it also undergoes important changes in response to new theoretical problems posed
by the rise of new religious traditions, especially Buddhism, which is considered to be the ‘Rebel
child of Hinduism’. According to Bhikhu Parekh, the Hindu tradition of political thought met its
most radical critique at the hands of Buddhism.

On origin of state, While Hindu political thought associate origin of state with ARAJAKTA
(‘anarchy’), Buddhist political thought associate it with ‘fall of man’ / ‘corruption’ emerging
after origin of private property/materialism. Buddhism subscribes to the contractual theory of
state. According to it, the origin of the private property led to the environment of social conflict,
rule and subordination. There’s an eternal law to govern the human behaviour, but int the
absence of a ruler to enforce it, it is not followed. At this point, people decide to enter into
contract with each other and entrust power to a king who governs in accordance with the
dharma.

On the ruler, unlike Manu smriti, the divine basis of the royal power is rejected in Buddhism.
According to Buddhist political philosophy, a person, who as a monk has destroyed all moral
taint and has achieved equilibrium between good (right) and bad (wrong) actions can rule as he
knows what is ‘Dharma’. In addition, the association of Buddhism with the republic of the period
suggest that the Buddhist political philosophy adopted republican tradition.

On duties of the ruler (Mahasammat), Buddhism believes that the king must rule in the most
righteous manner for the people’s welfare. Thus, Buddhism advices the ruler to be liberal,
selfless, noble and act according to the dharma.

On form of the government, unlike Hinduism, which supports monarchical form of government,
Buddhism considers republican government as the best practice in which the King
(Mahasammat) is elected by the people. Moreover, the Buddhist tradition promotes deliberative
democracy for the determination of public issues. Buddhist ‘Sanghas’ were the earliest examples
of deliberative democracy. In addition, unlike Hinduism which is ‘Patriarchal’, Buddhism gives
‘Equality of status to women by admitting them into sanghas’.

However, apart from these differences, there’re certain similarities also like Hinduism, which
revolves around the concept of ‘DHARMA’, Buddhism also revolves around the concept of
‘DHAMMA’ (‘Duty’). It also continued to share some basic Hindu beliefs such previous life
determines his destiny in this life and the idea of a knowledge person to be the king.

It is in this context of similarities with the Hindu political thought that, despite being its most
radical critique, Buddhist challenge didn’t lead to radical transformation or reformulation of
Hindu tradition.

2(c) Critically examine the relevance of Hannah Arendt’s views on Totalitarianism.

While totalitarianism did not first emerge in the twentieth century, the totalitarian states of
Nazi Germany under Adolf Hitler (1933-1945) and the Soviet Union under Joseph Stalin (1924-
1953) were distinct. In The Origins of Totalitarianism Hannah Arendt (1906-1975), one of the
most influential political philosophers of the twentieth century, seeks to explain why European
populations were amenable to totalitarianism in the twentieth century and to identify what
factors distinguish modern totalitarian regimes. Arendt was born into a German-Jewish family in
Hanover, Germany in 1906 and in 1933, fearing Nazi persecution, she left Germany. The Origins
of Totalitarianism is Arendt’s attempt to better understand the tragic events of her time.
In The Origins of Totalitarianism Arendt explores the histories of anti-semitism and imperialism
and their influence on the development of modern totalitarian regimes. Arendt argues that anti-
semitism, race-thinking, and the age of new imperialism from 1884-1914 laid the foundation for
totalitarianism in the twentieth century. Arendt traces how racism and anti-semitism were used
as instruments of imperialism and nationalism in nineteenth-century Europe. Arendt shows that
imperialism and its notion of unlimited expansion promoted annexation regardless of how
incompatible a country may have been. Nationalism developed along with imperialism, and
foreign peoples who did not fit in with the nation were oppressed. Modern totalitarian regimes,
aware of the efficacy of these instruments, used them in pursuit of their singular goals.

Arendt argues that the origins of totalitarianism in the twentieth century have been too
simplistically attributed to nationalism, and totalitarianism has been too easily defined as a
government characterized by authoritative single-party rule. Arendt also argues that scholars
and leaders have mistakenly equated nationalism and imperialism. Arendt rejects the notion
that a dictatorship is necessarily totalitarian. Dictatorships can be totalitarian, but they are not
inherently totalitarian. Totalitarian governments are characterized by their replacement of all
prior traditions and political institutions with new ones that serve the specific and singular goal
of the totalitarian state. Totalitarian governments strive for global rule and are distinguished by
their successful organization of the masses. In fact, Arendt argues that totalitarianism is
significantly less likely to originate in locations with small populations.

Arendt also argues that modern totalitarian regimes are defined by their use of
terror. Arendt’s The Origins of Totalitarianism is an influential work that takes on the difficult
task of trying to understand the devastating rise of Nazi Germany and Soviet Stalinism.

3(a) Compare the Normative approach with the empirical approach. Which approach is more
suitable for political science? Give reasons.

The nature, scope and methodology of political science as a discipline remains contested. There
hasn’t yet been developed a single well-defined and all encompassing method of studying
political phenomena.

The contemporary political science gives prominence to empirical mode of analysis, traditional
study was dominated by the normative approach which emerged with Plato, who is called as
‘father of Political philosophy’.

The normative approach seeks to determine and prescribe ‘values’, whereas, the empirical
analysis involves determining and describing ‘facts. Proponents of empirical approach criticize
the normative approach on the ground that there is no ‘scientifically valid’ or reliable method of
determining values. The ‘reliability’ based criticism of normative approach is based on the fact
that a normative statement of ‘what ought to be’ expresses preference for a particular type of
order as dictated by moral principle or ideals. It seeks to serve an ‘intrinsic’ value, which is an
end in itself, such as truth, justice and good. On the contrary the empirical approach primarily
relies on the observation through sense-experience to describe ‘what is’.

In this sense, the empirical mode of analysis can be verified and falsified. The ‘scientific’
affiliation of empirical approach has been criticized for its indifference towards ‘values’,
particularly in its ignorance of discrimination between higher values and lower values.

According to Leo Strauss, by teaching the equality of values, the empirical method contributes to
the victory of the gutter. The descriptive nature of the approach based on the paradigm of ‘true
or false’ limits the relevance of empirical approach.

In recent times, scholars like Rawls and MacPherson have tried to connect normative arguments
with the problems of real world. However, the proponents of empirical approach, while
describing its objective nature, repudiates normative approach i.e. to its subjectivity. T.D.
Weldon, in his work ‘vocabulary of politics’ (1953), maintains that a political value is like a
matter of taste which can only be stated and not debated.

However, the debate between normative and empirical approach is based on the ‘fact-value
dichotomy’ which presents existence of a state of estrangement between the two approaches.
Critics of this dichotomy call for a reconciliation between the approaches. It is in this context,
Oakeshott argued that, as political science is bottomless and borderless sea, no single approach
can comprehend the complexities of political world. Hence multiple approaches to be used in
combinations. It is in this context, post-behaviouralism emerged as an attempt to develop an
integrated study of political theory based on normative-empirical synthesis.

3(b) Fascism is the destruction of Liberal ideas in the interest of those who own the
instruments of economic power- Laski. Comment.

Fascism is considered as a child of 20th century, specifically of the period between the two world
wars. The origins of fascism have provoked significant historical interests and often fierce
disagreements. Although, no single factor can, on its own, account for the rise of fascism, it is
seen as the interest of those who wanted to own the economic power. Fascism is considered to
be an ‘ill-assorted hodge-podge of ideas’ because there is no single unifying idea that guided the
fascist movement. It lacks a rational and coherent core idea to be described or categorized as an
ideology

Fascism is often seen as an attack on values reflected in French Revolution like Liberty, equality
and fraternity. Benito Mussolini , very clearly held that 1789, the year of the French Revolution ,
is abolished .Similarly, Hitler talked about new man and new society and the need to go for the
destruction of the order that was established after French Revolution. So, if Mussolini replaced
the slogans of liberty, equality and fraternity with duty, discipline and sacrifice for the state .
Hitler talked about volk or the race. Hitler gave the policy of Lebensraum against Immanuel
kant's policy of Perpetual Peace. Fascist rulers have rejected that man is rational and
autonomous . For Hitler man is a member of a race . For Mussolini nothing is more important
than the state.

Fascism has been seen as the destruction of liberal ideas and institutions in the interest of those
who own the instruments of economic power. What it has done, wherever it has gained power
is, above all, to destroy the characteristic defenses of the working class, their political parties,
their trade unions, their cooperative societies. Fascism, in short, emerges as the institutional
technique of capitalism in its phase of contraction.

However, exploring the exact nature of fascism is in itself a very challenging task, because it has
been a hodge-podge of ideologies . In present times, the parties which are seen as far right
parties are being regarded as neo-fascist parties. Thus , fascism is multidimensional ideology.

3(c) Draw parallels between the Arthshastra tradition of Kautilya and the realist tradition of
Machiavelli.

There has been a tradition to compare Kautilya and Machiavelli. J.L. Nehru in his book
‘Discovery of India’ has called Kautilya as the ‘Indian Machiavelli’. Scholars such as G. Bottazi,
Winternitz and Max Weber has compared Kautilya’s Arthashastra with realist thinkers of west
such as Thucydides and Machiavelli.

Both Kautilya and Machiavelli represented their ages in a true setting. Kautilya had become the
chief adviser of Chandra Gupta Maurya whom he had placed on the throne of Magadh after
ending the rule of the Nanda dynasty. He wrote his Arthashastra while he practically guided the
destinies of Magadh, for the benefit of his king. It is an undisputed fact, supported by
overwhelming evidence, that most of Kautilya's views had been put into practice in the
administration of Magadh. Machiavelli's famous works, The Discourses and The Prince, were
written after he had been deprived of all power in the Republic of Florence and had been
compelled to live in virtual banishment.

Both thinkers have relied on history and have frequently quoted from previous rulers. Their
object was to suggest practical solutions to political problems as they found them. They had no
taste for idealism or speculative thinking. Both separate ethics from politics. But they differ in
their views regarding religions and morality in life. Kautilya respects religion, values morality in
rulers and state officials, as essential qualities. Machiavelli, on the other hand, divorces religion
from politics to the extent of almost completely disregarding the value of religious principles
and dogmas or moral values.

Both Kautilya and Machiavelli hold almost similar views regarding the conception of the state.
Both belief in the strong monarchy as the best form of government. Both attach importance to
force and its use in keeping order and hence the necessity of a powerful king as head of the
state. The law of punishment must remain ever vigilant to prevent the people of the four castes
and orders from swerving from their respective duties and avocations. He looked upon Royalty
as the most vital factor in the body-politics.

However, Max Weber, in his work ‘Politics as Vocation’, has argued that despite these
similarities in their perspective, Kautilya was more realist than Machiavelli and truly radical
Machiavellianism is classically expressed in Kautilya’s ‘Arthashastra’.

4(a) Make a comparative assessment of Ambedkar’s Theory of Social Justice with that of John
Rawls.

Social justice means availability of equal social opportunities for the development of personality
to all the people in the society, without any discrimination on the basis of any social factors like
caste, colour, religion, gender etc.

The concept of social justice emerged out a process of evolution of social norms, order, law and
morality. It laid emphasis upon the just action and creates intervention in the society by
enforcing the rules and regulations based on the principles of social equality.

Being driven by his concern for the upliftment of the Dalits in the society, Ambedkar’s thoughts
were aimed at providing the probable solution to the social problems prevalent in the Indian
society. To quote B R Ambedkar, ‘ a just society is that society in which ascending sense of
reverence and descending sense of contempt is dissolved into the creation of a compassionate
society. Thus social justice for Ambedkar meant a life of dignity to the people belonging to the
community of untouchables/Dalits. Caste and consequent graded structure of Indian society has
been a dominated issue in Ambedkar’s quest and vision of social justice. He believed that
untouchability was an expression of caste system, which leads to artificially dividing of
population and preventing them from fusion into one another. He thus held that out-caste is a
by product of caste system, and it cannot be removed without overall 'Annihilation of caste'.
According to Ambedkar, as caste is the basic structure of Hindu society, annihilation of caste
also means rejection of Hinduism.

John Rawls has propounded the liberal - egalitarian theory of social justice inspired by Kantian
moral idea of freedom & equality Rawls gave centrality to the moral principle of freedom &
equality of every person (liberal egalitarianism). He argues that the distribution of 'Primary
social goods' such as liberties, powers, opportunities etc. is just and fair if that distribution is
made in accordance with the principles. John Rawls conceptualizes justice fairness where there
is desirability of advantage for the marginalized groups in some respect.

4(b) Explain Gandhian views on State and their relationship with Modern democracy and
Principles of anarchism.

Gandhi has been considered as moral anarchist. He believed in swaraj, which stresses on gover-
nance not by a hierarchical government but self-governance through individuals and
community-building. The focus is on political decentralisation. Hind swaraj constitutes Gandhi’s
first blueprint on swaraj and conveys a comprehensive notion of the idealistic aspect of his
thought. Although the term swaraj means “self-rule”, Gandhi gave it the content of an integral
revolution that encompasses all spheres of life. Swaraj is a kind of individualist anarchism. It
warrants a Stateless society. According to Gandhi, the overall impact of the State on the people
is harmful; he viewed the State as a “soulless machine” which ultimately does the greatest harm
to mankind. A non-violent State is like an “ordered anarchy”. Adopting swaraj means
implementing a system whereby the State machinery is virtually nil, and the real power directly
resides in the hands of the people.

Gandhi believed in grassroot democracy. He talked about Sarvodaya as the ultimate aim of
democracy . He visioned the post independent Indian democracy as a simple, non-violent, and
decentralised model designed on horizontal distribution of power. Fir him , democracy becomes
meaningful and assumes significance only when its structure is reared on the foundation of
village Panchayats.

4(c) Plato as an enemy of Open Society. Comment


Karl Popper in his book “Open Society and its Enemies.” calls Plato, Hegel and Karl Marx as
enemies of open society. In the Vol 1 of his book, he has shown that Plato’s theories have the
dangerous tendencies towards totalitarianism. He believes that Plato’s readers have been under
‘Spell of Plato’.

Plato has been called as the “Enemy of Open Society” by Popper, on the basis of three
arguments – Essentialism, Historicism and Holism.

Essentialism is linked to Plato’s ‘theory of ideas.’ Plato has established that idea/essence form
absolute reality. Plato does not give importance to the individual, sacrifices individual in the
name of collectivity. Plato has presented history in the manner which serves political purpose.All
the totalitarian leaders reflect above ideas. e.g. they establish the primacy of state over
individual.

However, while karl Popper considers Plato's political program far from being morally superior
and in fact similar to fascism. Levenson has called such allegations by Popper as “superficial
comparison” and like “comparing the uncomparables”.
Section B

5(a) Radical humanist perspective of National Movement

M.N. Roy was one of the most vibrant political thinkers of the modern Indian political tradition.
Roy has given the radical humanist perspective of national movement . Radical humanism
philosophy keeps individual at the centre. It accepts individual in his radical state (that is free
from any form of collective identity)

Radical humanism talks about the cosmopolitan union of truly liberated person. MN Roy, in his
work ‘India in transition’, has referred to congress as a bourgeoise party and Gandhi as a
bourgeoise leader. Instead of working for the masses , used masses to advance the interest of
the bourgeoisie class.

Roy believed that true freedom of men emerges from Man’s struggle for self-protection, survival
and reproduction. Freedom means freedom from all scars of fear and insecurities.

Therefore , the Radical Humanist perspective of national movement can be a regarded as a


broader philosophy of freedom and human emancipation, going beyond political independence .

5(b) Compare the strategies of Extremists with Moderates.

The nationalist response was articulated differently in different phases of India’s freedom
struggle.

During the period 1885-1907, the early nationalist response to the colonial rule was ‘moderate’
response to the colonial rule was ‘moderate’ in nature based on their peaceful and
constitutional methods.

On the contrary, the extremist phase began from 1907 and continued until the Gandhian phase
started. The basic difference between the two nationalist responses lay in their perception of
British rule and its articulation in concrete programmes.

While the moderates considered British rule as a ‘blessing-in-disguise’ and had faith in
Britisher’s sense of justice, the extremists had no faith in the ‘benevolence’ of the British rulers
and wanted to establish complete ‘Swaraj’ or ‘Self-rule’.They differed substantially in the form
of the strategies adopted. The early nationalist response in India was based on
‘Constitutionalism’, which got transformed into the following two-pronged methodology -
Creation of a strong public opinion to arouse public consciousness and then educate and unite
people on common political questions .
Secondly they focused on persuading the British government in India and British public opinion
to introduce reforms in India to attain political rights and self-government stage by stage.

On the contrary to the strategy adopted by moderates, the extremists favoured a strategy of
direct actions, such as passive resistance, boycott, swadeshi, national education etc, to
overthrow the colonial rule.

However, despite constituting contrasting viewpoints, the two groups supplemented each other
with their response. According to Bidyut Chakraborty, extremists were an offshoot of the
process emerged due to moderate campaign.

5 (c) Write short note on the Relevance of Rajya Sabha

The Constitution provides for a bicameral legislature consisting of an Upper House (Rajya
Sabha) and a Lower House (Lok Sabha). The Rajya Sabha was designed for the purpose of
representing the interests of States. The Rajya Sabha represents the states of Indian Federation,
while the Lok Sabha represents the people of India as a whole. The Rajya Sabha is required to
maintain the federal equilibrium by protecting the interests of the states against the undue
interference of the Centre. The Parliament is empowered to legislate on any subject of the State
List if Rajya Sabha passes a resolution to that effect in the national interest.

However, scholars like Pratap Bhanu Mehta point out the decline in the significance of Rajya
Sabha. Rajya Sabha tends to block the legislations and most of the time not on the basis of
principles. It results into the attempts by government to bypass Rajya Sabha either through
ordinances or presenting even ordinary bills as money bills. Though Rajya Sabha is supposed to
be the house of elders, however in India,it is argued that, there is hardly any difference in the
composition of the two houses. In fact, the Rajya Sabha has been criticized as the rehabilitation
chamber or the ‘house on sale’. Only some members have the necessary qualifications that will
do justice to the role of the Rajya Sabha. . The problem has been exacerbated by the Kuldip
Nayar judgment which removed the requirement of domicile. It has now been misinterpreted for
political expediency.

For the Rajya Sabha to be more effective, certain steps need to be taken. All States must be
given an equal number of seats. Most importantly, it needs to be seen as a House brimming
with talented policymakers.

4(d) Write about the relationship between Judicial review and the Doctrine of Basic Structure
Doctrine of judicial review means that the courts have to expound and perform the role of
protector of the constitution by ensuring that any law or administrative action which is
inconsistent with the constitution is declared void. The above powers of judiciary are based on
the assumption that without authoritative, independent and impartial arbiter of the
constitutional issues, the purpose of having a constitution is lost. Judicial review has two prime
functions

1. Legitimizing governmental actions


2. Protecting the constitution against undue encroachment by the government

The doctrine of basic structure is a judicial innovation to ensure that the power of amendment is
not misused by Parliament. The idea is that the basic features of the Constitution of India should
not be altered to an extent that the identity of the Constitution is lost in the process. The
concept of the basic structure of the constitution evolved over time from the judgement in
Golaknath case (1967) to Kesavananda Bharati case (1973), which explicitly defined the concept
.

In India , Judicial review is a “basic structure ” of the constitution as ruled in Keshavanand


Bharati case , 1973, and Minerva Mills Case 1980

5(e) Comment on Fundamental Duties under Article 51A of the Indian Constitution

In Chandra Bhavan Boarding And Lodging v. State of Mysore,1969, the Supreme Court held that
it is not possible for our constitution to protect all the rights of citizens without assigning certain
duties to its citizens.

The original constitution did not provide for the Fundamental Duties of the citizens. These were
added during the operation of internal emergency (1975–77) by the 42nd Constitutional
Amendment Act of 1976 on the recommendation of the Swaran Singh Committee. Article 51(A)
talks about these Fundamental Duties and has 11 fundamental duties that are expected to be
performed by the citizens . The 11th duty was added by the 86th amendment.

The main objective behind fundamental duties is to maintain the Sovereign nature of State,
maintain Unity and Integrity of the nation, help judiciary in the interpretation of different
statues which are made by the legislature. FDs create a balance between the claims and
commitment of the individual citizen towards the nation.

However, the Fundamental Duties have been criticised on the following grounds that they are
not exhaustive and do not cover important duties like paying taxes, as was recommended by the
Swaran Singh Committee. FDs are vague, ambiguous phrases like ‘noble ideals’, ‘composite
culture’, ‘scientific temper’ have been used. The critics have opined that the FDs should have
been added in Part III so as to make them enforceable.

6(a) What do you understand by the phenomenon of the Decline of Parliament? What are the
reasons behind the decline of Parliament debate in India? Suggest a way forward.

The general decline of Parliament that set in the post-Nehru era has not yet been reversed.
Major indicators of this decline are shortening sessions, time lost due to disruptions in
proceedings by the opposition and violation of norms and precedents by the government,
rampant absenteeism, actual hours of sitting as a percentage of available hours, phenomenon
of weak legislative federalism via Rajya Sabha in the overall setting of parliamentary-
federalism, challenges of new extra-parliamentary mass movements, and judicial activism.
Although the Indian Parliament has witnessed progressive democratization in terms of
representation of various sections of society, it has declined as an effective institution of
accountability. Unlike in Western democracies, the decline of Indian parliament is not due to
strengthening of the executive. Ironically both the executive and the parliament in India have
remained weak during the ‘democratic upsurge’ era, while some non-parliamentary institutions
have succeeded in asserting their autonomy. The emergence of new players representing
sectional interests though increased representation of various sections of society, yet
adversarial politics among these parties led to parliament’s decline.

Three reasons for the decline of the parliament

In the 1950-60s, Parliament, the law-making organ of the State, used to meet, on an average,
125-130 days in a year. This has come down to just 65-70 days in the last decade. In fact, 2017
was the year of the lowest number of sittings 57.

The second indicator is the social inclusion of Parliament. This means, to what extent Parliament
has been able to accommodate the different social communities and groups, making it a
representative of Indian society.

The institutional decay of parliamentary efficacy. As per data from the Association of
Democratic Reforms, 186 MPs out of 543 are facing criminal charges in the current Lok Sabha.
That is, one in every three of our MPs is charged with criminal cases. We, as a society, have
collectively failed to elect honest candidates.

One way is to say that any motion or discussion has to be taken up if a certain number of MPs
gives a written notice. The no-confidence motion requires just 50 MPs (slightly less than 10% of
the strength of the House) to be admitted. The threshold can be increased, and suitable
thresholds fixed for discussions without a vote and voting motions.

Another approach is to guarantee some time for the opposition througha calendar of sittings at
the beginning of each year for limited flexibility. Second, the rules should be amended to ensure
that House is summoned if a significant minority (say 25% or 33%) of members gives a written
notice.

Review of Anti-Defection law: In order to revive debate and deliberations in the Parliament, the
use of whip can be restricted to no-confidence motion only. The adjudicating power of speaker
vis-à-vis anti-defection law can be transferred to Election Commission of India.

6(b) The frequent interventions by judiciary which sometimes border on judicial adventurism
tend to weaken the functioning of the other two branches of government. Comment.

Judicial adventurism is an extreme form of judicial activism. It denotes a situation where


judiciary goes beyond its territory and enters into that of the legislature or the executive,
thereby disrupting the balance of powers between the three organs of the State. Judicial
activism is the oxygen of the Rule of Law and the Rule of Life. It is obscurantist retreat from
constitutional obligation for a Court to blink or wink at social justice issues brought to its notice
by any citizen, be he the humblest or the highest, basing his title to rights conferred by the
Constitution of India, but deprived by the executive. Judicial activism comes in for criticism at
the time on the ground that judicial populism is leading the Judges to usurp the power, which
legitimately belonged to Parliament. It is true the Judges do not legislate but their judgements
serve to give meaning and content to the fundamental rights enshrined in our Constitution. It
remains the sacred duty of the Courts to secure the fundamental liberties of the citizens against
arbitrary exercise of power but Judges must know their jurisdictional limits and activism should
not mean invading the exclusive legislative and executive domains. Judicial activism does not
become judicial adventurism.

Writ of Habeas Corpus is the fundamental instrument for safeguarding individual freedom
against the arbitrary and lawless State actions. As is rightly quoted by Pascal in Pensees Justice
without force is impotent force without justice is tyranny. Has the Honble Supreme Court in
these cases done complete justice to its constitutionally assigned role of protector of
fundamental rights? It needs to be said that in these cases the Honble Supreme Court failed in
its role as the sentinel on the qui vive of civil liberties.
The judiciary, as one of the three powers of the state, is accountable to the society it serves.
Accordingly, the judiciary, like the other powers of the state, must always have the best interest
of the public as its fundamental concern. This requires that the judiciary must recognise the
social and economic conditions in which the other two powers of the state have to work.
Moreover, the judiciary must be aware that there are limits to judicial and legal intervention in
relation to political decisions that have to be made by the legislative and executive powers.
Therefore, all courts within the judicial power must take care not to step outside the legitimate
area for the exercise of judicial power. In its dealings with the other two powers of state, the
judiciary must seek to avoid being seen as guarding only its own interests and so overstating its
particular concerns. Rather, the judiciary must take responsibility for the society it serves.
Judiciaries must also take care not to oppose all proposed changes in the judicial system by
labelling it an attack on judicial independence. But, if judicial independence or the ability of the
judicial power to exercise its constitutional role are threatened, or attacked, the judiciary must
defend its position fearlessly. Examples of decisions which might come into those categories are
massive reductions in legal aid or the closure of courts for economic or political reasons.

6(c) Explain the linkages between caste system and OBC politics in India. What is the current
trend of the role of caste in Indian election?

Linkages between caste system and OBC politics

Emergence of electoral democracy in India created a very fertile ground. Caste system is a
closed system but it’s still evolving. Myron Weiner’s concept of “political co-optation” became
very relevant. The policy of political mobilization followed by Congress and other parties also
resulted in co-optation many lower castes into the party. Thus according to Rajni Kothari,
politicisation of caste in India played a very important role in developing party politics. MN
Srinivas also used the concept of dominant caste. A dominant caste is a caste which dominates
numerically, due to its numeric preponderance it enjoys political power.

The Mandal Commission, or the Socially and Educationally Backward Classes Commission(SEBC),
was established in 1979 to “identify the socially or educationally backward classes” of India. It
basically defined backward classes in terms of caste. The caste membership rather than
individual class characteristics became the matter of importance. Thus low social ranking in the
class rather than average per capita income became the criteria for inclusion in the OBC list.
Thus caste and class became cross cutting identities.

Also with time as stated by Christophe Jaffrelot, class started transforming caste relations in
India. Thus class and caste started coexisting in Indian society. Reservation helped certain jatis
within dalits to catch up with OBC’s and other dominant castes in terms of acquiring revenues.
Thus caste continues to be a ‘lived in social reality’ as stated by Myron Weiner. Caste became an
instrument of social change which resulted in rise of ‘casteism’. Mandal framework led
‘egregious consequences’ (Marc Galanter) because it identified Other Backward Classes in terms
of caste. It created a heterogeneous vast diverse category of OBC’s. Thus caste became the basis
of a scheme of permanent protection.

Current trend of the role of caste in Indian election?

The first trend that one sees is that traditional vote bank support towards the so-called caste
based parties they are associated with is gradually eroding. In recent years, in Bihar, more
Yadavs vote for non-RJD candidates than in the past.

The second misconception that one sees frequently is the idea that caste operates on its own,
irrespective of other dimensions. One has to identify the right candidate within the targeted
groups. In that regard, caste is only one aspect of a candidate's "winnability", other are the
capacity to garner support across caste barriers and the resources they can mobilise.

7(a) “The Indian federal structure is largely symmetric albeit with some asymmetric features.”
Comment. Do you think federalism has a critical role in democratizing the political process
and strengthening the union in India?

An attempt is made to locate union territories as a distinct unit of the Indian federation in the
context of asymmetrical federalism, and their status is examined through the principle of
weighted and differentiated equality in India. There has been an attempt to accommodate
diversity in India through asymmetrical provision of power sharing between states. The
establishment of union territories has also followed this principle of asymmetry justified by their
unique context and origin as seen in the distinctive powers of the union territories of Delhi,
Chandigarh, Puducherry, and Jammu and Kashmir.

India has tried to accommodate diversity through an asymmetrical provision for power sharing,
whereby some states are more equal than others. Similar to many federations, India has certain
de facto asymmetrical federal features with regard to differences of size, population, wealth,
and influence between the federating units. India also has all sorts of de jure or constitutional
asymmetries in terms of special position, powers, and protection enjoyed by the states of
Nagaland (Article 371A of the Constitution), Sikkim (Article 371F), and Mizoram (Article 371G)
as well as the former state of Jammu and Kashmir (Article 370 of the Constitution abrogated in
August 2019). Further, the Fifth and the Sixth Schedules of the Constitution comprise numerous
distinctive provisions for administration and protection of Scheduled Areas and Scheduled Tribes
in any state (Fifth Schedule), and in the states of Assam, Meghalaya, Tripura, and Mizoram
(Sixth Schedule). Comparably, the union territories are less autonomous than the states and are
coordinated in varying degrees by the union government. Some union territories are entirely
administrated by the union without any local representation.

The Indian model is characterised by weighted and differentiated equality. Within Indian
federalism, while all states are considered “equal,” some states are more equal or unequal than
others, which is analogous to the concept of “asymmetrical federalism” in comparative theory.

7(b) “Slowly but steadily, women elected to Panchayats are asserting themselves and giving a
new thrust to the grass root governance in India.” Substantiate.

Decentralising power at the grassroots; level was on the national agenda for many years. It
became a reality through the 73rd and 74th amendments to the Constitution in April 1993. The
73rd amendment, among other things, handed over the reins of power to the people at the
panchayat level with a 33 per cent reservation of certain seats and key positions within the
panchayat to women. The amendment also made it mandatory for all states to hold gram
panchayat and municipal elections and empowered these bodies to undertake development
activities at the local level.

While the 73rd amendment has opened the way for women to exercise their right to be involved
in village governance, it does not let them define programme priorities and develop and
manage natural resources. Nor is there an integrated approach to environmental planing at a
microlevel because each department has its own plan. Women constitute only 6.7 per cent of
the total seats in Parliament. The emerging feedback from women-ruled panchayats should
hopefully help evolve a consensus in favour of reserving seats for women in Parliament.

"By and large, women sarpanchs or members attend panchayat or block level meetings by
themselves and are not represented by their male relatives," says Chandan Dutta of the Delhi-
based non-governmental organisation (NGO), Participatory Research in Asia. There has also
been a perceptible improvement in administration at the gram panchayat level mainly because
women sarpanchs hold meetings during the day and no dubious deals are struck late at night.
They are also more accessible than men who are often not around. Due to these factors there is
much more honesty and transparency in local administrative matters.
The obstacles

Yet, like any major social transformation, women who have shown the courage to take their
destiny into their own hands have bad to surmount hindrances at every stage. Fatima Suhra, a
primary school teacher who was elected as president of the Puthigepanchayat of Kasargode
district in Kerala, was not allowed to function by the local unit of Communist Party of India-
Marxist (cpm) which in fact had sponsored her as their candidate. The party put up a sub-
committee to oversee pan chayat affairs and started dictating terms to Subra who rightly
refused. The local unit of the cpm ordered social ostracism against her, a lethal strategy in the
Muslim-dominated region of Malabar in north Kerala. Suhra finally resigned. "I was on the side
of justice," she said after she was forced to resign.

If it is not ostracism imposed by political parties it could be caste-based oppression that women
have had to bear. Muktiben Patel, a backward caste woman who became sarpanch of Nitaya
village of Hoshangabad district, had to face several no-confidence motions against her by the
thakurs (a higher caste) of the village but continues her work like repair- ing school buildings
and getting a pukka road for her village.

7(c) Critically assess the circumstances under which ethnicity can become a source of conflict
with reference to any two case studies from India.

Since partition in 1947, the status and treatment of the remaining Muslim citizens in India’s new
multiethnic nation alongside the majority Hindu citizenry has been a bone of contention. India’s
nominally secular society, as it was under Nehru’s brand of socialism, has come under
significant strain, particularly in the latter stages of the twentieth and the turn of the twenty-
first century. The arrival of identity politics during the 1980s, whereby people mobilise politically
around particularistic identities, be it religion, caste, jati or language, has promoted the political
expediency of organisations and political parties such as the BharatiyaJanata Party (BJP). The
communal violence intensified the tension between Hindu and Muslim in Gujarat and Mumbai.

Poverty plays a key role as a foundational backdrop that can lead to ethnic violence, but
ambiguity that exists between the fate of peaceful and violence parts of the cities’ poorer areas
can be explained by the methods in which communities access their resources. The ideology of
Hindutva therefore only spreads to areas where Hindu-nationalist parties controlled the access
to community resources.

In Gujarat and Mumbai, there is compelling evidence that implicates relative impoverishment
with its social implications in the fostering of ethnic violence. It is widely agreed that the slums
in both Mumbai and Eastern Ahmedabad oversaw the worst of the violence including mob
killings, rape and looting. Hindu and Muslims mobs swelled to numbers of around five thousand
in the Isanpur. Meanwhile, in Mumbai, Dharavi slum saw some of the most prolonged and
violent scenes in 1992 and 1993. The Shrikrishna Commission, a Judicial Inquiry, was set up in
1998 to investigate the circumstances of the violence in Mumbai and found that the ‘relative
deprivation’ of the urban poor is an ‘immediate cause…facilitating ethnic violence’. The claim
however that this became a source of conflict via a ‘frustration-aggression syndrome’ is a lofty
interpretation of the role of poverty in ethnic conflict.

The strong civic links between Hindus and Muslims in these impoverished areas have become
weakened through a history of deindustrialization. The capital of Gujarat, Ahmedabad, for
example, began the deindustrialisation of its textile mills in the 1980s. The existence of peace
and resistance of ethnic conflict within poor deindustrialised areas of Mumbai and Gujarat
would suggest that the relative socio-economic circumstance is not a prerequisite of ethnic
violence. In order to build a fuller conceptual picture of the circumstances that lead to ethnic
violence, we need to add an understanding of the forces that can actively provoke ethnic hatred
in poor areas. The ambiguity between poverty as a condition and violence as an outcome can be
best explained by looking deeper into resource networks. Communities that have suffered from
impoverishment, in this case via deindustrialisation, do not automatically provide a
circumstance for ethnic unrest.

8(a) “Political defections are one of the most ubiquitous phenomena of democratic politics in
India.” Substantiate. Do you think it is high time to shift towards presidential form of
government?

The anathema of defections has been a prominent feature of Indian political discourse, like in
many other democracies, since the commencement of Indian democratic politics. Rampant
instances of defections across parties became a modus operandi of Indian politics resulting
in incessant dislodging of democratically elected governments by rival parties through
orchestrated defection, especially at the state level in India.

The proliferation of such practices resulted in the enactment of the anti-defection law in India
in 1985. Thereafter, the subsequent unfolding of political developments revealed that the
anti-defection legislation has largely failed to stall the menace of defections and
it continues to take place unabated even today. With the rise of BharatiyaJanata Party as the
dominant political force in Indian politics since 2014, a plethora of defections from the
politically weaker Congress party and other regional parties who joined the BJP at various
points of times in the last six years, have continued the trend.
As, transparent decision-making and political mobility rules are absent, the members feel
insecure about their political career within the party and hence resort to defections to other
parties or float new political outfits. Hence, the paper emphasises that the higher degree of
inner-party democracy and organisational coherence disincentives the party members from
defecting as they feel secured about their political career in a more democratic and organised
party structure. On the other hand, parties which are run on the sole arbitrary diktats of the
highest leader, factionalism and personality clashes within the lower rungs of leadership
might more likely trigger defection for considerations of better political career of the
disgruntled leaders.

8(b) Examine the debates on the appointment procedure of judges to the higher judiciary in
India.

It is the system of appointment and transfer of judges that has evolved through judgments of
the Supreme Court, and not by an Act of Parliament or by a provision of the Constitution. The
Supreme Court collegium is headed by the Chief Justice of India and comprises four other
seniormost judges of the court. A High Court collegium is led by its Chief Justice and four other
seniormost judges of that court. Names recommended for appointment by a High Court
collegium reaches the government only after approval by the CJI and the Supreme Court
collegium. Judges of the higher judiciary are appointed only through the collegium system —
and the government has a role only after names have been decided by the collegium. The
government’s role is limited to getting an inquiry conducted by the Intelligence Bureau (IB) if a
lawyer is to be elevated as a judge in a High Court or the Supreme Court. It can also raise
objections and seek clarifications regarding the collegium’s choices, but if the collegium
reiterates the same names, the government is bound, under Constitution Bench judgments, to
appoint them as judges.

Judges of the Supreme Court and High Courts are appointed by the President under Articles
124(2) and 217 of the Constitution. The President is required to hold consultations with such of
the judges of the Supreme Court and of the High Courts as he may deem necessary. So how did
the collegium system evolve when the Constitution is silent on it?The collegium system has its
genesis in a series of judgments called “Judges Cases”. The collegium came into being through
interpretations of pertinent constitutional provisions by the Supreme Court in the Judges Cases,
First Judge Case 1981 and Second Judge Case and Third Judge Case.
Critics argue that the system is non-transparent, since it does not involve any official mechanism
or secretariat. It is seen as a closed-door affair with no prescribed norms regarding eligibility
criteria or even the selection procedure. There is no public knowledge of how and when a
collegium meets, and how it takes its decisions. Lawyers too are usually in the dark on whether
their names have been considered for elevation as a judge.

8(c) Examine the relation between DPSP and FR.

Fundamental rights and DPSP as cherished in the Constitution of India together comprises the
human rights of an individual. The Constitution expresses fundamental rights as an idea which
appeared in India in 1928 itself. The Motilal Committee Report of 1928 clearly shows inalienable
rights derived from the Bill of Rights enshrined in the American Constitution to be given to the
individual. These rights were preserved in Part III of the Indian Constitution. of India.

The Relationship between Part III and Part IV is the one that is not a novice one and was
discussed by the Constitutional Advisor Sir B.N. Rau who advocated the idea that the right of an
individual on the basis of their nature can be divided into:

1. Justiciable Rights
2. Non-justiciable Rights

The list of Justiciable Rights was engulfed in Part–III while the non-justiciable one became the
member of Part–IV of the Constitution. At times and again these Directive Principles are used by
the Judiciary to determine the constitutional validity of any legislation when they are found to
be in conflict with the Fundamental Rights or Part–III of the Constitution.

The first case we are going to discuss in this light is of Sajjan Singh V. State of Rajasthan of 1964
where the Obiter Dicta laid down by Justice Madhukar becomes apposite, even the fundamental
rights enshrined in Part III were taken as unalterable, the much-needed dynamism may be
according to him achieved by a proper interpretation of the Fundamental Rights in light of the
Directive Principles. Further, he observed that the Part IV is fundamental in the governance of
the country and the provision relating to Part III must be interpreted harmoniously with these
principles”. As discussed above in the case of Champakam Dorairajan (Supra) it was held by the
Supreme Court that the Fundamental Rights would be reduced to a “Mere rope of sand” if they
were to be override or superseded by the Directive Principles of State Policy.

Also, as we discussed earlier in this article while deliberating on the case of I.C. Golaknath
(Supra), Hon’ble Justice SubbaRao of the Apex Court accentuated that the Fundamental Rights
and the Directive Principles of State Policy together form an integrated scheme which is elastic
enough to respond to the changing needs of the society. On a similar note in Bijoya Cotton
Mills V. State of West Bengal, the supreme court has two folded view regarding the same:

In a case of conflict between the rights of an Individual and a law that particularly aims at the
implementation of socio-economic policies in furtherance of the Directive Principles, the weight
would be accorded to the latter.

Every Act or Legislation enacted in fulfilment of the Directive Principles should be construed as
the one professing in the public interest or as a reasonable restriction to Part III of the
Constitution.
PSIR Crash Course 2021
Model Answers - Test 8

For any issue related to PSIR Test Series, write only at psirtestseries@shubhraranjan.com.

Section A

1(a) Highlight the relevance of Political Sociology approach to the study of Comparative
Politics

In order to compare the major political similarities and differences between countries, scholars
of comparative politics employ 2 major approaches- Traditional and Modern.

While the traditional approaches such as institutional, historical, legal approaches are value-
laden and are mainly associated with traditional outlook underlying study of state and
government. Modern approaches on the other hand, call for replacement of normativism by
empiricism, compartmentalization of disciplines by inter-disciplinary study, and speculative by
scientific rigour.

Political sociology:

As a discipline, it aims to examine the interaction between politics and society. Based on the
idea that political processes and structures are shaped by social order, organisations, and
cultural values.

As such by focusing on two way interaction between state and society, it assumes huge
importance in study of political phenomena in 'Third world' countries because in these countries
political actors, institutions and processes operate within a wider social context.

Relevance:

According to Maurice Duverger, it ends the isolated study of politics and locates it in a broad
spectrum of social phenomena, and this emphasizes on essential unity of social sciences.

Criticism:

a) Fear that independent identity of political science will be lost. As such scholars like Theda
Skocpol calls for “bringing back the state in ”.
b)Makes politics a dependent variable. In this context, Lipset in his book 'Political man' mentions
social determinants of political behaviour.

1(b) Comment on the concept of Overdeveloped State.

The concept of ‘overdeveloped state’ was propounded by Hamza Alvi, a neo-Marxist scholar,
who has applied the structural approach to study the nature of the post-colonial states,
especially Pakistan & Bangladesh. However, the theory is useful in understanding the nature of
state in other third world countries also.

Hamza Alvi developed his theory in rejection of the instrumentalist approach to the study of
third world countries. The instrumentalist approach applied by scholars like A G Frank, Sameer
Amin etc., consider post-colonial state as ‘instruments’ used by metropolitan bourgeoisie
resulting in development of core countries and under-developed of peripheries.

However according to Hamza Alvi, the instrumentalist approach is relevant only where the
capitalism is fully developed i.e. western countries.

According to Hamza Alvi’s over-developed theory of state, there’s a structural mismatch


between the ‘political’ and ‘social’ evolution in a post-colonial state, resulting in fully-developed
state machinery while the socio-economic structure is still in transition from primitive to modern
(capitalist). It is in this, context of comparison, the political structure (state) is over-developed
than socioeconomic structure of the country. On the contrary, in western countries, there’s a
perfect sync between political and socio-economic structure. Once the economic system became
modern capitalist, elements of superstructure also changed accordingly i.e. modern nation state
came into existence.

According to Hamza Alvi, this mismatch exists in the third world countries owing to their colonial
legacy. Unlike western countries where capitalist state developed naturally with the rise of
indigenous bourgeoisie class, state in postcolonial societies is an imported institution imposed
by colonial masters. A strong police state was devised to catch to the needs of colonial rulers.

Hamza Alvi argues that the inheritance of over-developed state machinery, in form of ‘military-
bureaucracy oligarchy’ and its cooperation with three major elites -landed feudal, indigenous
bourgeoisie and metropolitan bourgeoisie – resulted in a complex post-colonial state. in such a
condition, state becomes an ‘equilibrium- maker’ among the three classes, and thus, acquire a
relatively autonomous role rather than the instrument of any class.

Although Alvi’s theory propagated the notion of Pakistan’s post-colonial symptoms in the early
70’s; it is still relevant in today’s state including India. This is reflected when Pakistan’s military
& bureaucracy (permanent executive) still holds more power compared to elected
representatives. inability on the part of government of Pakistan to take any serious action
against Hafeez Saeed (despite sufficient proofs given by GoI) can be attributed to its more
powerful military & ISI. This also explains why major hurdles still exist for Pakistan to accomplish
a full western democracy model.

Thus, Alvi’s ideas helps, even in the contemporary discourse, to understand & analyse Pakistan’s
& other ‘third world countries’ political and administrative problems.

1(c) With Taliban in power, global feminist movement is at crossroads. Comment.

The Global Feminist Movement refers to a series of Social movements and Political campaigns
for reforms on women's issues created by the inequality between men and women.Such issues
are women's liberation, reproductive rights, domestic violence, maternity leave, equal pay,
women's suffrage, sexual harassment, and sexual violence.

During its inception, feminism has gone through a series of four high moments termed Waves.
The First-wave feminism was oriented around the station of middle- or upper-class white
women and involved suffrage and political equality, education, right to property, organizational
leadership, and marital freedoms.The Second-wave feminism attempted to further combat
social and cultural inequalities. Although the first wave of feminism involved mainly middle class
white women, the second wave brought in women of different social classes, women of color,
and women from other developing nations that were seeking solidarity. The Third-wave
feminism continued to address the financial, social, and cultural inequalities of women in
business and in their home lives, and included renewed campaigning for greater influence of
women in politics and media. In reaction to political activism, feminists have also had to
maintain focus on women's reproductive rights, such as the right to abortion. The Fourth-wave
feminism examines the interlocking systems of power that contribute to the social stratification
of traditionally marginalized groups, as well as the world around them. It is in this wave, that
the rights of women in Afghanistan is studied and analysed by Feminists.

During the 1990s, the Taliban not only brutally imposed social restrictions on women such as
mandatory burqa coverings, but, more fundamentally and deleteriously, restricted their access
to health care, education, and jobs. It prohibited women from appearing in public spaces
without a male chaperon, de facto sentencing widows and their children to starvation. The
Taliban regime destroyed Afghan institutions and the economy, which was already devastated
by decades of fighting and the Soviet scorched-earth counterinsurgency strategy. This critically
affected women and children. And, with the exception of poppy cultivation and opium
harvesting, the Taliban prohibited women from holding jobs, including working as doctors for
other women.

The post-Taliban constitution in 2004 gave Afghan women all kinds of rights, and the post-
Taliban political dispensation brought social and economic growth that significantly improved
their socio-economic condition. In 2003, fewer than 10 percent of girls were enrolled in primary
schools; by 2017, that number had grown to 33 percent. Women’s life expectancy grew from 56
years in 2001 to 66 in 2017. By 2020, 21 percent of Afghan civil servants were women.

However, with the Taliban back in power from September 2021, the fate of Afghan women have
again come back into the spotlight. It is far too early to discuss the reversal of women rights.
However, as per the statements made by the Interim Government, they have firmly stated that
the Taliban will protect women’s rights under Sharia—a rubric, that can cover a range of
policies and behavior. By stating that they will “protect” women’s rights under sharia, the
Taliban give themselves a wide berth of options. Very likely, however, the Taliban’s inclinations
will be to weaken women’s rights, further tighten cultural restrictions on women, and shrink
socio-economic opportunities for them. In short, it is very likely that the Taliban in power will
seek to restrict the formal rights that Afghan women have today, worsening these women’s
social, economic, and political conditions. This brings a deep setback to the global feminist
movement.

1(d) Highlight the significance of the 2021 G20 agenda- Planet, People and Prosperity.

In the 16th G20 Summit held in October 2021, the agenda was set on three priority pillars-
People, Planet and Prosperity in the backdrop of Covid 19 pandemic and the economic, financial,
employment and social devastation it has brought, especially for the health, well-being,
equality, education, and safety of women, girls and youth.

The G20, bringing together a diverse grouping of 19 countries and the European Union (EU), has
worked briskly since 1999 to build international consensus on tackling major issues relating to
global economics, international financial stability, climate change mitigation and sustainable
development. The intergovernmental forum comprises the world’s largest developed and
emerging economies, with the members representing more than 80% of the world’s GDP, 75%
of global trade and 60% of the total global population. The sheer size and strategic importance
of the economies involved give the “Group of 20” a crucial responsibility to lay the foundation
for long-term global economic growth.

The year 2021 presented the world with unique challenges unencountered in the past. The
incoming shock of a global health crisis in the name of the coronavirus (COVID-19) pandemic
upended the daily functioning of people and economies worldwide. A result of these profound
changes is the magnification of other systemic burdens, such as climate change and inequality,
in every corner of the world. It is now recognized that local and regional challenges have
assumed an urgency that is global in reach, and the process towards building back a better
world requires decision-making and implementation on a scale unhindered by borders.

The 2021 G20, under Italian Presidency, focused on sharing experiences and learnings from the
past year to develop plans that would set the tone for long-term growth that the G20 says
should be ‘resilient, sustainable and green’. The three interconnected pillars of action that arise
from these objectives are: People, Planet, Prosperity. Here, multilateralism has found a new
meaning in being indispensable to ensure a swift pandemic response in line with guaranteeing
equity and access to healthcare, building up resilience to future shocks and promoting overall
prosperity.

In this context, India’s standing as one of the influential emerging powers is only going to gain
more firm ground. For the first time, India will hold the G20 presidency from December 2022
and will convene the leaders’ summit in 2023. This event is a testament to the country’s ever-
increasing efforts at spearheading growth and attracting global investor interest. It has
positioned India on an attractive platform to be considered as a strategic and compelling
partner in the post-Covid-19 global recovery.

1(e) Relevance of nuclear deterrence as a security strategy in 21st Century.

The term ‘deterrence’ comes from Latin word ' De terre', which means to ‘frighten’ the potential
adversity from attacking. Thus, deterrence refers to a strategy intended to frighten an adversary
from taking an action, not yet started. The concept of deterrence is developed by ‘Game
theorists’ based on the idea to convince the potential aggressor that the benefit of aggression
will be outweighed by the loss. Therefore, it is a 'psychological concept.

Deterrence’ is a realist approach to the ‘security’ . From the realist’s point of view, war remains
eminently possible in the international context. Thus, it is best kept at bay through the threat of
punishing force. Thus, deterrence works only when the adversary believes in the credibility of
deterrence. For this, it is necessary to communicate to adversary, that if he prefers to attack, the
victory will come at a cost and thus preventing the adversary from taking action.

Although, traditionally the concept of deterrence has been used for conventional weapons, in
the contemporary context from the cold war era, it is being used or associated with nuclear
weapons. According to Kenneth Waltz, the horizontal expansion of the nuclear weapons have
helped in maintaining peace and order in world at large.
Realists argue about three main issues which calls our nuclear deterrence as the most relevant
security strategy.

Absence of nuclear war. The most remarkable thing about nuclear weapons is how rarely they
have been used. Nuclear weapons have only been used as an instrument of war in 1945, to
hasten the end of war in the Pacific by bringing about the surrender of Japan (even if the USA
was also concerned to send a message to the Soviet Union). The fact that they have not been
used subsequently, and that conventional war has never broken out between two nuclear
powers, suggests that nuclear weapons are weapons of a very particular kind. They are almost
entirely of symbolic, not practical, importance.

Effective deterrence. The primary motive for acquiring nuclear weapons is deterrence, the
prevention of war through the massive devastation that would befall an aggressor. Nuclear
weapons are particularly well-suited to this role, both because of their enormous destructive
capability and because they are relatively ineffective as defensive weapons. This means that
there is a low possibility of a state achieving a first-strike nuclear knockout, since nuclear powers
invariably seek to develop a second-strike capability. This makes a nuclear war, fought between
two nuclear powers, virtually unthinkable.

International stability. The vertical proliferation of nuclear arms has not destabilized
international politics because it has tended to preserve the balance of power, albeit through a
‘balance of terror’. Horizontal proliferation has been gradual (with the ‘nuclear club’ growing
from five in 1964 to eight by 2005, although Israel and possibly Iran are widely seen as ‘opaque’
nuclear states). Arguably, the gradual spread of nuclear weapons preserves international
stability better than either no spread or a rapid spread would.

Nuclear statesmanship. The possession of nuclear weapons may engender a sense of


responsibility and a strong bias in favour of caution, even in states that had previously been
inclined towards adventurism or aggression. For, example, regional tensions between India and
Pakistan are much less likely to lead to war now that both powers possess nuclear weapons.

On the other hand, critics of Nuclear deterrence as a security strategy argue,

Fallibility of deterrent systems. The theory of nuclear deterrence is naive and dangerous. A
world in which there are nuclear weapons will always carry the threat of nuclear war.
Deterrence may always fail due to miscalculations and accidents. For instances, states may
make miscalculations about whether other states possess an invulnerable second-strike
capability or, for that matter, whether they possess nuclear weapons at all.
Danger of nuclear imbalances. There is no guarantee that vertical or horizontal nuclear
proliferation will preserve the balance of power. Indeed, proliferation inevitably creates
temporary imbalances which may then be exploited by aggressive states. After all, the
Hiroshima and Nagasaki bombs were dropped to take advantage of precisely such a military
imbalance.

Useable nuclear weapons. Developments in recent years have focused increasingly on the
production of nuclear weapons that have a more precise and contained impact, making them
‘useable’. These ‘tactical’ or ‘battlefield’ nuclear weapons are no longer of symbolic importance
alone.

The greatest concern is that nuclear weapons may fall into the hands of military-based
dictatorial regimes, or even terrorist organizations, which may have fewer scruples about using
them.

Different views of Scholars:

Scott D. Sagan- He argues that India-Pakistan historical rivalry, protracted ideological and
territorial disputes may escalate a crisis between them to a nuclear level. This may happen
either wilfully, accidentally or by miscalculations. According to him, the nuclear rhetoric
employed by both India and Pakistan during ‘Operation Parakram' demonstrated the fragility of
escalation control mechanism. According to Sagan, geographical proximity, inadequate warning
systems, short flight times, terrorist bases within Pakistan etc. could trigger a nuclear reaction

Kenneth Waltz- He contends that the alarmist views about the South Asian nuclear situation are
‘imperialist’ and tend to looks South Asian decision-makers as possessing lower level of rational
conduct. In his assessment, both India and Pakistan are likely to contain their nuclear arsenal to
the requirements of a ‘credible second strike’. He also argues that, the chief purpose of
Pakistan’s nuclear strategy is to deter India’s superior conventional capabilities, not their use as
weapons of coercion.

Rajesh Rajagopalan (‘Second Strike’ Arguments about Nuclear war in South Asia)- Given the fact
that nuclear weapons signify catastrophic consequences, the possibility of conventional conflict
escalating to nuclear level is ‘zero.

2(a) Discuss the various notions of security with special reference to the Feminist approach to
security.

At its most basic, security implies freedom from threats. Human existence and the life of a
country are full of threats. There are two ways to understand various notions of security. First,
traditional security (External), in the traditional conception of security, the greatest danger to a
country is from military threats. The source of this danger is another country which by
threatening military action endangers the core values of sovereignty, independence and
territorial integrity. Military action also endangers the lives of ordinary citizens. It is unlikely that
in a war only soldiers will be hurt or killed. Quite often, ordinary men and women are made
targets of war, to break their support of the war. Security policy is concerned with preventing
war, which is called deterrence, and with limiting or ending war, which is called defence.
Traditional security policy has a third component called balance of power. When countries look
around them, they see that some countries are bigger and stronger. A fourth and related
component of traditional security policy is alliance building.

Second, Non-Traditional Security. Non-traditional notions of security go beyond military threats


to include a wide range of threats and dangers affecting the conditions of human existence.
Non-traditional views of security have been called ‘human security’ or ‘global security’

Feminist analysis has placed particular emphasis on developing a gendered conception of


security and war. Conventional approaches to security present it as ‘the highest end’ of
international politics. In this view, states have prime responsibility for maintaining security, as
reflected in the notion of ‘national security’. The major threats to security are therefore external,
coming in particular from other states. In this way, the threat of violence and other forms of
physical coercion are intrinsically linked to the prospect of interstate war. National security is
thus closely linked to the prevention of such wars, usually through a build-up of military capacity
to deter potential aggressors. Feminists, for their part, have criticized this view of security on
two grounds.

First, it is premised on masculinist assumptions about rivalry, competition and inevitable


conflict, arising from a tendency to see the world in terms of interactions among a series of
power-seeking, autonomous actors. Second, the conventional idea of national security tends to
be self-defeating as a result of the security paradox. This creates what has been called the
‘insecurity of security’.

Feminist theorists, by contrast, have embraced alternative conceptions of security, most


commonly the notion of ‘human security’. From a gender perspective, therefore, the apparently
clear distinction between ‘war’ and ‘peace’, which arises from a primary concern with the threat
of inter-state war, is quite bogus and merely serves to conceal the wide range of other threats
from which women suffer. They define security more broadly, as the diminution of all forms of
violence – including domestic violence, rape, poverty, gender subordination, economic, and
ecological destruction. Cynthia Enloe argues that “all kinds of health professionals, all kinds of
educators and environmentalists, climate change, sea level rise experts and so on are providing
security.”
One of the alternative ways of considering insecurity using a gender-sensitive approach is
considering economic security. Ann Tickner states that women are disproportionately located at
the bottom of the socio-economic scale in all societies due to the gendered division of labor. In
combination to women’s relegation to certain types of jobs that are considered ‘natural’ to
them, this makes women especially vulnerable economically. Especially in economic terms,
regular security analysis approaches fail to uncover the vulnerability in such situations.

Feminist theorists, have therefore, embraced alternative conceptions of security, most


commonly the notion of ‘human security’. Nevertheless, the parameters of human security are
sometimes unclear. While some argue that it should be confined to ‘freedom from fear’ (in
which case the key threats to security would be armed conflict and human-made physical
violence), others extend it to include ‘freedom from want’ (in which case poverty, inequality and
structural violence become key threats).

2(b) To what extent Mortan Kaplan’s Systems Approach is a qualitative improvement in the
field of theorization of International Politics.

Systems Approach can be regarded as a useful approach to the study of International Politics. It
can be used for an overall view of the relations among nations. It can be used for both
macroscopic as well as microscopic studies of regional sub-systems which form parts of the
international system. The concept of system can be used for achieving the objective of theory-
building in international politics.

The emergence of Systems Approach to the study of International Politics can be described as
one of the most significant developments of the 20th century. Immediately after its birth,
Systems Approach captured the interest of a large number of modern political scientists. They
began using it as a convenient tool for a large scale macro-cosmic analysis of the political
phenomena in the world.

Morton Kaplan has been the chief exponent of the Systems Approach in international Relations.
He advocates that international politics offers the best sphere for the application of the concept
of ‘system’ as a tool for investigating all its phenomena. International System is most inclusive
as it is constituted by those interactions among international actors which are neither fully
cooperative nor totally conflictual. It has its sub-systems and a set of actors, both international
and supra-national actors. It is constituted by interactions among the international actors.

He lays greatest stress on the pattern of the behaviour of states. But as the character of state
has been changing since its birth (in its size, objectives, leader-ship, population, role and so on),
so has the International System. International System is thus never static. The shape system at a
particular time reflects the conditions prevailing at that time. Kaplan believes that International
System is the most important of all the systems. He does not regard International System as a
political system because, in his view, the role of decision makers in the international system is
subordinate to their role in the national domestic system affirms that the behaviour of the
national actors in the field of international affairs is invariably governed and guided by the basic
consecration of national interest.

He divides international actors into two categories. The first category is that of the national
actors while the second is that of supranational actors. The U.S.A., India, China etc. are the
examples of national actors while the NATO is an example supranational actors. According to
Kaplan, International action takes places between international actors. It is the interaction
between these two types of actors that ultimately gives birth to the International system.

On the basis of such a conceptual framework, Morton Kaplan discusses six models of
international system:

(1) The Balance of Power System: It is constituted by 5 or 7 major powers/actors. Each actor
seeks to increase its capabilities through negotiations and not through resort to war. Each actor
is prepared to fight rather than pass on an opportunity to increase capabilities for protecting its
national interest. The actors maintain a balance in their power positions, and no actor is
permitted to become unduly powerful. An actor or a group of actors in such a system can resort
to war for maintaining the balance of power in their relations.

No actor is to be eliminated from the system. The actor terminates the war before the opponent
is eliminated. An actor or a group of actors acts for opposing any other group or single actor
that tends to assume unduly powerful position and dominance with respect to the rest of the
system. Attempts are made to check the actors who try to follow supranational organizing
principles. The defeated or constrained essential actors are permitted to re-enter the system as
acceptable role partners. Actors act to bring some previously inessential actor within the
essential actor classification and treat all essential actors as acceptable role partners.

(2) The Loose Bi-polar System: The Loose Bi-polar System comes into existence when two
powerful nations are successful in organizing the other nations into their two respective
competing blocs or groups. However, the organisation of the blocs is loose and internal
differences prevail among the members of each bloc. In it, blocs try to increase their relative
capabilities as well as to limit or weaken their rival bloc.

(3) The Tight Bi-polar System: The Loose Bi-polar System easily gets transformed into a Tight Bi-
polar System. It is a bi-polar system in which the two major powers lead their respective blocs of
allied powers. Each bloc is dominated by a major power.
(4) The Universal System: In this system, the nations get organised in a federal system. It is a
hypothetical model in which the world gets transformed into a Federal World State based upon
the principle of mutual toleration and universal rule of law. It works through a universal actor—
an international organisation. The universal actor is powerful enough to check war and preserve
peace or a balance in international relations.

(5) The Hierarchical System: Such a model can come into existence when a single powerful super
power may bring, either through a conquest or a treaty, all other nations under its control. The
states as territorial units are then transformed into functional units.

(6) The Unit Veto System: It involves the conception of a situation of multi-polarity in which each
state is equally powerful. Each possesses such weapons (nuclear weapons) as can be used by it
for destroying any other state. It becomes stable when each state can resist and retaliate
threats from every other state.

In the field of theorization of International Politics, Systems Approach can be regarded as a


qualitative improvement as it can be used for both macroscopic as well as microscopic studies of
regional sub-systems which form parts of the international system. It can improve the ability to
describe, explain, predict and even control the nature and course of international politics.

2(c) Give reasons behind South Asia- one of the poorest regions of the world. What should be
done to address the poverty in the region?

“Poverty entails more than lack of income and productive resources to ensure sustainable
livelihoods. Its manifestations include hunger and malnutrition, limited access to education and
other basic services, social discrimination and exclusion as well as lack of participation in
decision-making”.- UN Department of Economic & Social Affairs

South Asia has been accommodating a significantly large share of global income and
multidimensional poor compared to other regions. The multidimensional poor refers to
individuals who are deprived of more than one dimension such as health, education and living
standards. During the last decade, the share of income poor in South Asia has increased, despite
the share of multidimensionally poor showing only a marginal decline.

The global poverty rate dramatically dropped from 94% (1820) to 10.7% (2013) during the last
two centuries. However, South Asia’s share of global poor has increased from 27.3% to 33.4%
during the period of 1990-2013, leaving behind only to Sub-Saharan Africa which accounts for
the largest share (50.7%) of global poor.

According to the latest available data by World Bank (2018), 21.2% of the Indian population is
living below the international poverty line of 1.90 US$. In fact, 270 million are considered poor,
making India the poorest country in the region followed by Bangladesh where 18.5% of
population is below the poverty line. The headcount indices of Nepal, Pakistan and Maldives are
15.0%, 6.1% and 7.3% respectively. In contrast, Sri Lanka has the lowest headcount index (1.9%)
while Bhutan reported the second lowest (2.2%) in South Asian region.

The reasons responsible for the low level of development of the South Asia is socio-economic
factors like Unemployment, Low level of literacy, Gender-based exclusion, Capital inadequacy,
Poor health status, Overpopulation, Low rate of labour-force participation and High rate of
socio-economic inequality.

There are three ways to combat poverty in the region. Firstly, by promoting sustainable and
inclusive economic growth especially through boosting private investment in infrastructure.
Secondly by investing in human capital starting from a child’s early years, because people with
better skills, education, health and training which will make the biggest difference to countries’
ability to grow and compete. Lastly, by fostering resilience to global shocks, including forced
displacement, climate change and pandemics, which threaten to roll back our hard-fought
development gains. South Asia, for example, is particularly vulnerable to natural disasters, as
2015’s earthquake in Nepal illustrated with its tragic loss of life. From Afghanistan to
Bangladesh, much of South Asia is located in one of the highest seismically active regions in the
world. More than 600 million people live along the fault-line across the Himalayan belt that runs
through Afghanistan, Pakistan, India, Nepal and Bhutan. Disaster preparedness is a must as
these disasters fall hardest on the poor.

3(a) Do you agree that the global war on terror can only be won if the West promotes human
rights more vigorously? Substantiate

The human cost of terrorism has been felt in virtually every corner of the globe. Terrorism clearly
has a very real and direct impact on human rights, with devastating consequences for the
enjoyment of the right to life, liberty and physical integrity of victims. In addition to these
individual costs, terrorism can destabilize Governments, undermine civil society, jeopardize
peace and security, and threaten social and economic development. All of these also have a real
impact on the enjoyment of human rights.

In recent years, however, the measures adopted by Western States to counter terrorism have
themselves often posed serious challenges to human rights and the rule of law. Some States
have engaged in torture and other ill-treatment to counter terrorism, while the legal and
practical safeguards available to prevent torture, such as regular and independent monitoring
of detention centres, have often been disregarded. Other States have returned persons
suspected of engaging in terrorist activities to countries where they face a real risk of torture or
other serious human rights abuse. These practices, particularly when taken together, have a
corrosive effect on the rule of law, good governance and human rights. They are also
counterproductive to national and international efforts to combat terrorism.

September 11 has sometimes been described as ‘the day the world changed’. This certainly
applied in terms of its consequences, notably the unfolding ‘war on terror’ and the invasions of
Afghanistan and Iraq and their ramifications. The controversial wars in Afghanistan and Iraq
were both justified, in part, on humanitarian grounds. In the case of Afghanistan, the Taliban
was seen to have established a brutal and repressive regime that, in particular, violated the
rights of women, who were entirely excluded from education, careers and public life. In the case
of Iraq, the Saddam regime was viewed as an ongoing threat to the Kurds in the north and the
majority Shia population, both of whom had been subject to political exclusion and physical
attack. ‘Regime change’ through the overthrow of the Taliban and Saddam Hussein therefore
promised to bring about respect for human rights, greater toleration and the establishment of
democratic government.

As used in Afghanistan and Iraq, however, the idea of humanitarian intervention was drawn into
a larger project of liberal interventionism. Liberal interventionism is based on two assumptions.
First, liberal values and institutions, notably market-based economies and liberal democracy,
are universally applicable and superior to alternative values and institutions. Second, in
circumstances where the advance of liberalism is being blocked by obstacles that the domestic
population finds impossible to remove, notably a dictatorial and repressive government,
established liberal states have a right, and maybe even a duty, to provide support. This support
may take the form of diplomatic pressure, economic sanctions or, when basic human rights are
being flagrantly violated, possibly military intervention.

However, the lack of success in both Afghanistan and Iraq to stabilize the situation has resulted
many scholars to argue that the promotion of Human rights by the West is mere window-
dressing, rather than a true attempt to bring change. The ‘war on terror’ raised serious
questions about the universalist assumptions that underpin liberal interventionism. Not only
have doubts surfaced about the viability of imposing western-style democracy ‘from above’, but
the wars in Afghanistan and Iraq also in many ways deepened tensions between the Islamic
world and the West. If liberal values such as human rights and multi-party democracy are not
universally applicable, it is difficult to see how consistent standards can be established for
interventions that have a humanitarian or moral basis.

3(b) Do you agree that the ‘Asian values’ debate was generated by the desire of authoritarian
regimes in the region to protect themselves from criticism? Elaborate
Cultural determinism argues that cultural values condition modes of social and economic
organization, including patterns of political relationships, political participation, citizenship and
government. Cultural approaches in the social sciences are not new, especially in comparative
research.

There are two broad arguments. With the decline of global ideological conflict and with it the
polarizing effect this had upon international politics, economic and political regionalism has
become more prominent. This has been on the basis of a process of development over a number
of decades. In parallel to this, the ideological debate has given way to a cultural one. This has
been tied to various political and economic indicators, but it has also been argued to underpin
economic and political friction. Famously, Huntington wrote that "the fundamental source of
conflict in this new world will not be primarily ideological or primarily economic. The great
divisions among humankind and the dominating source of conflict will be cultural... The principal
conflicts of global politics will occur between nations and groups of different civilizations. The
clash of civilizations will dominate global politics." This has been heightened by the resurgence
of traditional values resulting from the uncertainties of socio-political change in some societies.

The debate over "Asian values" is at the heart of this controversy. The extraordinary economic
growth experienced within East Asian countries - often achieved under different modalities than
that of the Western countries- has put the political and social arrangements of these countries
under the spotlight. The success of these countries, in relation to the downturn of Western
economies, and the friction which has arisen over trade protectionism, economic conditionality,
democracy and human rights have made the "Asian values" debate more than culture
differences.

The idea that Asian culture and beliefs may constitute an alternative to western ones gained
momentum during the 1980s and 1990s, fuelled by the emergence of Japan as an economic
superpower and the success of the so-called Asian ‘tiger’ economies – Hong Kong, South Korea,
Thailand and Singapore. This position was outlined most clearly by the Bangkok Declaration of
1993, when Asian state representatives from Iran to Mongolia, meeting in preparation for the
World Conference on Human Rights in Vienna, issued a bold statement in favour of what they
called ‘Asian values’. While not rejecting the idea of universal human rights, Asian values drew
attention to supposed differences between western and Asian value systems as part of an
argument in favour of taking culture difference into account in formulating human rights.
Particularly keen advocates of this view included Mahathir Mohamad and Lee Kuan Yew, at that
time the prime ministers, respectively, of Malaysia and Singapore

The social problems in the West have compounded the contrast of fortunes and a number of
political leaders - both in the East and West - have suggested that the "Asian way" is the way
forward. The principal representatives of the "Asian values" thesis in Asia - whilst challenging
the universalization of liberal social ideas and reveling in higher growth levels than their former
colonial overlords - have been happy to proclaim the ascendancy of "Asian values".

The idea of Asian values was dealt a damaging blow by the Asian financial crisis of 1997–98.
This occurred not only because it cast doubt over the image of ‘rising Asia’, but also, and more
seriously, because so-called Asian values were sometimes held to be responsible for the crisis in
the first place. For some Asian countries, it simply served as an excuse for the survival of
authoritarian rule and absence of liberal-democratic. The key Asian value, from this perspective,
is political passivity, an unwillingness to question authority based on a trade-off between
economic well-being and political freedom.

In a number of the Asian countries there has not been complete freedom for opposition parties,
freedom of speech, a separation of powers, or civil and political rights as conceived in Western
political thought. In societies where the emphasis is upon consensus and harmony, especially
when pursuing economic growth, it has proved possible to deem opposition as subversive.
Cultural values have been a tool to control dissent. Governments emphasize the need for "an
environment of social and political order", but this conception of the "common good" is always
in the interest of particular groups. However, this perennial democratic paradox is complicated
and enlivened in East Asia by rapid change and the controversial relationship between economic
development and political liberalization. "Asian values" have a role in this debate but it would
be wrong to suggest that they are the determining factor in the outcomes.

3(c) Explain the difference between analyzing international relations as a single international
system and as the global politics of many different policy domains.

As a separate discipline, international relations is a rich interdisciplinary field of study that


draws direct connections to other disciplines such as geography, psychology, demography,
history, economics, and feminism, among others. However, it all boils down to a set of
directions revolving around state action. Take, for example, the issue of nuclear weapons. An
academic approach to IR may objectively determine the number of nuclear weapons in the
world and which countries possess them. From a policy perspective, however, international
relations provides a field from which a number of policy proposals are presented—how nuclear
weapons proliferation should be curtailed, how to keep certain rogue actors from obtaining
nuclear weapons, or how to limit the threat of nuclear weapons in the international
environment.

The subject of International Relations originally covered simply the relations between states.
Economic bodies and social groups, such as banks, industrial companies, students,
environmentalists and women organisations were given secondary status as non-state actors.
This two-tier approach has been challenged, particularly by the effects of globalization. First,
ambiguities in the meaning given to a state and its mismatch with the contemporary world
results in state not being an effective concept. Second, we can recognize governments are losing
sovereignity when faced with economic activities of transnational companies, and the violent
threat from criminals and terrorists. Third, NGOs engage in such a web of global relations,
including participation in diplomacy, that governments have lost their political independence.
Therefore, it is useful to analyse global politics in terms of a variety of policy domains rather
than a single international system.

Traditionally Realism and Liberalism view the International system as one single system
revolving around the state. Realist theory stresses on the absence of authority that governs the
international system. In short, the international environment is one in which anarchy is the
norm—states are sovereign in themselves and seek either coercion or consent to realize their
interests in the international system. There is no governing international authority to determine
state behavior in a global context, nor can there be any true governing international authority
so long as states maintain sovereignty and autonomy from one another. The overriding principle
in a realist environment is state survival and preservation, and because of this, power is the only
means through which preservation is maintained in an anarchic environment. Indeed, realist
theory posits that international organizations—such as the United Nations—are merely an
institutionalized means through which states project power and bring about coercion over other
states.

Where the international arena is characterized by conflict and anarchy in the realist perspective,
the liberal perspective is characterized by cooperation and the motivation of peace. Liberalism
also emphasizes that, while states are sovereign and often act in their own self-interest,
individuals, private parties, international organizations, and multinational corporations are
important mediating entities in the international arena too often overlooked by realists. The
United Nations is not simply a crude reflection of self-interested states, but an international
ideology predicated on cooperation and peace.

To challenge these state-centric unitary system, Critical theorists point out that IR is made of
various policy domains which need to be emphasized like the role of women in the domain. A
crucial assumption lies behind this theoretical framework: states are not simply institutions,
certain sets of preferences, or abstract individuals that we can isolate and understanding
through positivist and empirical analysis. States are rather constructions of values and
ideologies that reflect normative positions on the human experience and its relation to power.
Because of this, we must be attentive to cultural values and how the state is constructed to
reflect those values.
The move from a state-centric to a Pluralistic model depends on rejecting a static
unidimensional concept of power. Though as a discipline international relations dominated by
Realists tend to emphasize the system as unitary system but recent trends of critical theory
including Feminism has challenged this approach and provide an alternative account.

4(a) How does the ‘tragedy of the commons’ analogy help to illustrate the need for
governance of the global commons?

Tragedy of the commons is a concept highlighting the conflict between individual and collective
rationality. The idea of the tragedy of the commons was made popular by the American
ecologist Garrett Hardin, who used the analogy of ranchers grazing their animals on a common
field. When the field is not over capacity, ranchers may graze their animals with few limitations.
However, the rational rancher will seek to add livestock, thereby increasing profits. Thinking
logically but not collectively, the benefits of adding animals adhere to the rancher alone, while
the costs are shared. The tragedy is that ultimately no rancher will be able to graze the field,
due to overconsumption. This scenario is played out on a daily basis in numerous instances,
having grave consequences for the world’s resources.

As Hardin puts it, ‘Freedom in a commons brings ruin to all’. The idea of the ‘tragedy of the
commons’ draws attention to the importance of the ‘global commons’, sometimes seen as
‘common pool resources’, and of threats posed to these by overpopulation, pollution, resource
depletion, habitat destruction and over-fishing.

Hardin argued in favour of strengthened political control, especially to restrict population


growth. Liberals, nevertheless, argue that the solution is, in effect, to abolish the commons by
extending property rights, allowing the disciplines of the market control resource usage.
Although, as capitalism expanded, common land gradually became privately owned. Elinor
Ostrom (American Economist) argued that some societies in Africa have succeeded in managing
common pool resources through developing diverse, and institutional arrangements. Robert Cox
points out that historical evidence suggest that common land was usually successfully managed
by communities, as is borne out by examples such as the Aboriginal peoples of Australia, but this
then goes against the assumption that human nature is selfish and unchanging.

It is commonly recognized that one of the primary roles of government at the local, state,
national, and international levels is to define and manage shared resources. However, there are
a number of practical problems associated with this. Management inside clear political
boundaries is a relatively straightforward task, but more problematic are resources shared
across jurisdictions. For example, neighbouring cities may seek to maximize their benefits by
competing for industry, but they may minimize their costs by pushing residents outside their
jurisdictions. Another dimension is added at the international level when states are not bound
by a common authority and may view restrictions on resource extraction as a threat to
their sovereignty. Additional difficulties arise when resources cannot be divided or are
interrelated, such as in whale hunting treaties when the fishing of the whales’ food source is
separately regulated.

That brings to light the governance of global concerns through an International effort. Some
argue that top-down governance with binding agreements is the only effective solution for
problems of a global scale. Multilateral negotiations on climate change and other global
commons over decades have had limited success. Others have emphasised a more decentralised
multi-level or polycentric approach that builds on the observed successes of local solutions.

To manage our global commons, we need to facilitate and accommodate the self-governance of
local commons, but provide safeguards at different levels to avoid exploitation and manage
risks. Governing the global commons is the defining challenge for current and future
generations.

4(b) How would you define Polarity of contemporary International system? Is Global
governance a significant element of today’s order?

Polarity in contemporary international relations can be defined through different ways in which
power is distributed within the international system. There are three types of systems:
unipolarity, bipolarity, and multipolarity for three or more centers of power. The type of system
is completely dependent on the distribution of power and influence of states in a region or
globally.

World order, in the modern period, is being shaped by a number of multipolar trends. The most
significant of these is the rise of so-called ‘emerging powers’. These are the new, or the would-
be, great powers of the twenty-first century. Some states already have a significant measure of
regional influence – Brazil and, possibly, Argentina, Chile, Mexico and Venezuela in Latin
America; South Africa and Nigeria in Africa; Israel, Egypt, Saudi Arabia and Iran in the Middle
East; and South Korea, Indonesia, Pakistan and Australia in Asia and Oceania.

However, a range of other powers have acquired, or are acquiring, wider, and possibly global,
significance. These include, most obviously, China, Russia and India, but also Japan and the
European Union. Between them, and together with the USA, these powers account for over half
the world’s population, about 75 per cent of global GDP and around 80 per cent of global
defence spending. Nevertheless, the rise of China is often seen as part of a larger shift in the
balance of global power from West to East, and specifically to Asia, and maybe from the USA to
the BRICs countries, sometimes dubbed ‘the Rest’ or the Asian Century.
Three broader developments have supported the fragmentation and pluralization of global
power. The first of these developments is unfolding globalization. As all great powers are
embedded to a greater or lesser extent in global economic arrangements and participate within
an interlocking capitalist system, the pursuit of national self-interest can only mean, globalists
argue, increased integration and cooperation. The second development is the growing trend
towards global and sometimes regional governance. This stems from the fact that the principal
challenges confronting states – climate change, crime, migration, disease and so on – are
increasingly transnational in character and so can only be tackled through transnational
cooperation, emphasizing that power is as much about collaboration as it is about conflict.
Finally, the trends towards globalization and in favour of regional and global governance have
both had the effect of strengthening the role of non-state actors in world affairs. These non-
state actors are many and various, ranging from transnational corporations (TNCs).

There are two quite different models of a multipolar world order. The first highlights the
pessimistic implications of a wider diffusion of power amongst global actors. Neorealists have
been particularly prominent in warning against the dangers of multipolarity, seeing a tendency
towards instability and chaos as the key feature of its structural dynamic. Mearsheimer
lamented the end of Cold War bipolarity, warning that Europe’s future in particular would be
characterized by a ‘back to the future’ scenario. By this, he was referring to the multipolar world
orders that, arguably, gave rise to WWI and WWII by allowing ambitious powers to pursue
expansionist goals precisely because power balances within the international system remained
fluid. In this view, multipolarity is inherently unstable, certainly by comparison with bipolarity.

In addition to concerns about the structural implications of multipolarity, a number of emerging


fault-lines and tensions have been identified. The most common of these has been the possibility
of growing enmity, and possibly war, between the USA, the old hegemon, and China, the new
hegemon.

However, there is an alternative, and more optimistic, model of multipolarity. In the first place,
this suggests that the emergence of new powers and the relative decline of the USA may be
managed in a way that preserves peace and keeps rivalry under control. Such an approach
tends to encourage emerging powers to ‘bandwagon’ rather than ‘balance. It encourages
Global Governance.

Global governance has been described as the ‘collection of governance-related activities, rules
and mechanisms, formal and informal, existing at a variety of levels in the world today’. As such,
it refers to a wide variety of cooperative problem-solving arrangements whose common
characteristic is that they facilitate ‘governance’. The growth of international organizations
provides both evidence of a greater willingness amongst states to cooperate and engage in
collective action, and fosters further cooperation by strengthening trust amongst states,
accustoming them to rule-governed behaviour.

Therefore, global governance is closely linked to multi-polarity as it provides the emerging


powers to co-operate rather than increased conflict. Trans-national issues ranging from
international migration and global terrorism to transnational criminal organizations and global
pandemics have made global governance a reality .

4(c) In present times, do you think that Global South is more divided than united with respect
to the negotiating positions on climate change? Elaborate.

Climate change is not only the most prominent global environmental issue, but it is also, some
argue, the most urgent and important challenge currently confronting the international
community. The issue of climate change can be seen as a classic example of the ‘tragedy of the
commons’. Clean air and a healthy atmosphere are collective goods, key elements of the ‘global
commons’. However, tackling global warming imposes costs on individual states in terms of
investment in sometimes expensive mitigation and adaptation strategies as well as accepting
lower levels of economic growth. . Climate change, in other words, serves to widen the North–
South divide.

The mainsource of tension is that current emissions levels arguably provide an unfair guide for
setting targets. The transfer of much of manufacturing industry to the developing world means
that over a third of carbon dioxide emissions associated with the consumption of goods and
services in many developed countries are actually emitted outside their borders From a Southern
perspective, the developed world has a historic responsibility for the accumulated stock of
carbon emitted since the beginning of the industrial age. In effect, developed countries have
used up a large part of the safe carbon-absorbing capacity of the atmosphere, and made
substantial gains in terms of economic growth and prosperity as a result. Developing countries,
by contrast, are both disproportionately badly affected by climate change and have the fewest
capabilities to tackle it, whether through mitigation or adaptation. This implies either that
emissions targets should not be imposed on developing countries (as at Kyoto), or that any such
targets should take account of historic responsibilities and be structured accordingly, imposing
significantly heavier burdens on developed countries than on developing ones.

From a Northern perspective, however, countries cannot be held responsible for actions whose
consequences were quite unknown at the time they were carried out. In this view, targets
should be set in line with current emission levels alone, in which case Global North and South
countries would be treated alike. Apart from anything else, the growing importance of emerging
economies such as China, India and Brazil means that unless the developing world plays a
significant role in cutting emissions global targets will be impossible to meet.

Even among the Global South countries, there are several fault lines that have led to the
emergence of divided approach towards climate change. In the recently concluded Glasgow
conference, India and China argued "common but differentiated responsibilities and respective
capabilities" or CBDR - it means countries that have signed the UN climate deal have a common
responsibility to fight climate change, but have different capacities to do so given that they are
in different stages of economic development. China said that various countries' efforts to meet
the 1.5C target should be seen in the context of their efforts to eradicate poverty.India's
Environment Minister, Bhupender Yadav pointed out "How can anyone expect that developing
countries make promises about phasing out coal and fossil fuel subsidies? Developing countries
still have to do deal with their poverty reduction agenda."

Other Global South countries like Brazil, Venezuela and others called for ‘phase out’ of coal
consumption but the demand was diluted by India and China when they called for ‘phase down’
of coal consumption. India argued that they are being put under pressure to move from fossil
fuels to renewables, while developed countries are not helping them financially and with
technology. Thus, complete ‘phasing out’ was difficult to comply. This went against the
demands of climate change vulnerable countries who cried foul over the change.

The divided Global South approach gave leeway for the developed countries to not be
accountable for drastic cutting down of carbon emissions. India’s intervention on coal was not
supported by the groups of G77 countries and the LDMC, but it was supported by China. These
emerging tensions between the Global South countries have made it more difficult to find a
common path to negotiate terms for climate change.
Section B

5(a) Though India’s developmental co-operation is valued in Africa, it lacks clear strategy.
Comment.

India has a long history of partnership with Africa, with solidarity and political affinity going
back to the early 1920s when both regions were fighting against colonial rule and oppression.
After India gained independence, it became a leading voice in support of African decolonisation
at the United Nations. Independent India strived to share its limited resources with African
countries under the banner of South-South cooperation. In 1964, India launched the Indian
Technical and Economic Cooperation (ITEC) programme to provide technical assistance through
human resource development to other developing countries, with African countries the greatest
beneficiaries of it and the Special Commonwealth African Assistance Programme (SCAAP).

India’s economic engagement with Africa, on the other hand, only began intensifying in the
early 2000s. India’s total trade with Africa grew from US$ 6.8 billion in 2003 to US$ 76.9 billion
in 2018, and India is now Africa’s third-largest trade partner. Indian investments in Africa have
also grown rapidly in the last decade and the country is currently the seventh-largest investor in
Africa. The scale of India’s development cooperation with Africa has also grown rapidly. From
2003 onwards, India began to use concessional lines of credit (LoC) as one of its key
development partnership instruments to fund the construction of railway lines, electrification
and irrigation projects, farm mechanisation projects, among others. So far, India has sanctioned
182 LoC projects in Africa through the Export Import (EXIM) Bank of India, with a total credit
commitment of about US$ 10.5 billion.

India and Africa have often held common positions in global platforms and worked together to
guard the interests of other developing countries. They have moved joint proposals, such as the
Agricultural Framework Proposal and Protection of Geographical Indications, at the World
Trade Organisation (WTO) and World Intellectual Property Organisation, and have worked
towards protecting the food and livelihood concerns of farmers at the Doha Development Round
of WTO negotiations. The ‘Framework for Strategic Cooperation,’ the outcome document of the
Third India-Africa Forum Summit, also mentions that India and Africa will “enhance cooperation
through training and collective negotiations on global trade issues, including at the WTO to
protect and promote the legitimate interests of developing countries, especially the LDCs [least
developed countries]”. India and South Africa are also currently pressing for a waiver of certain
provisions of the Trade Related Intellectual Property Rights for COVID-19 treatment and
vaccines.

India has played an important role in building African capacity, with several notable ongoing
initiatives. Additionally, the values that steer India’s development cooperation — demand
driven, conditionality free and based on the principle of partnership among equals — are
appreciated in Africa. But India’s model of development cooperation in Africa lacks a clear
strategy.

There are two main flaws in India’s development strategy in Africa. Firstly, India is not actively
pursuing any specific development goals. An assessment of India’s development cooperation
instruments (LoCs, grants, and capacity building projects like ITEC) reflects the absence of a plan
for Africa. Indian LoCs have not been designed to achieve a larger development goal such as
food security, health security, clean energy or education for all.

Secondly, there is no synchronisation between different development instruments. LoCs, grants


and capacity building initiatives operate as standalone instruments of development
cooperation, with almost no links with each other. As a result, the overall development impact
of India’s development cooperation is small and difficult to measure.

India must chart out a roadmap for its development cooperation programme in Africa that
outlines a long-term strategy and delineates how it will deploy state capacity to pursue common
development goals.

“India-Africa partnership is yet to achieve its full


[development cooperation] potential. What is needed is an
infusion of energy, of something new and concrete, and with
a specific focus and direction”----- M. Vishwathan

5(b) “India-Russia friendship has stood the test of time.” Elaborate.

In a virtual address at the plenary session of the Eastern Economic Forum (EEF) being held in the
Russian city of Vladivostok, Prime Minister Narendra Modi said the India-Russia friendship has
stood the test of time and asserted that the two countries together can help bring stability to
the global energy market. He noted the “robust” cooperation between the two countries during
the Covid-19 pandemic, including in the vaccination programme.

India–Russia relations are the bilateral relations between India and Russia. During the Cold War,
India and the Soviet Union (USSR) had a strong strategic, military, economic and diplomatic
relationship. After the Dissolution of the Soviet Union, Russia inherited its close relationship with
India which resulted in both nations sharing a Special Relationship. Russia and India both term
this relationship as a "special and privileged strategic partnership". Owing to the bonhomie
shared by the countries' respective leaders, Prime Minister Narendra Modi and President
Vladimir Putin, the bilateral relationship has seen further growth and development. An informal
meeting between them in 2018 at Sochi helped accelerate the partnership, displaying the role of
interaction and cooperation between India and Russia.

On 6th December 2021, President Putin came to India to attend the 21st Bilateral Annual
Summit. The outcome of the brief visit was momentous. Twenty-eight agreements were signed
between the two sides, both at the government as well as at the private sector level. They
clearly demonstrate the extensive gamut of the bilateral engagement. The first 2+2 Dialogue
between the defence and foreign ministers of the two countries took place. It provided an
opportunity to share views about the current security, political and economic situation in the
world.

Military-technical cooperation continues to be a lynch-pin of the bilateral time-tested


partnership. The Bilateral Defence Agreement was extended for a further period of 10 years up
to 2031. Some of the new areas identified for cooperation include Central Asia, the Far East
Region of Russia and the Arctic. All these regions face a threat of growing dominance by China
which would decrease Russia’s influence in these regions.

Traditionally, the Indo-Russian strategic partnership has been built on five major components:
politics, defence, civil nuclear energy, anti-terrorism co-operation and space. These five major
components were highlighted in a speech given by former Indian Foreign Secretary Ranjan
Mathai in Russia. However, in recent years a sixth, economic component has grown in
importance. Trade between the two sides had declined by 17 percent last year but has grown by
38 percent in the first nine months of the year. The two countries have fixed a target of bilateral
trade of $30 billion and investment of $50 billion by 2025.

Both countries are members of many international bodies where they collaborate closely on
matters of shared national interest. Important examples include the UN, BRICS, G20 and SCO.
Russia has stated publicly that it supports India receiving a permanent seat on the United
Nations Security Council. In addition, Russia has expressed interest in joining SAARC with
observer status in which India is a founding member.

Prime Minister Narendra Modi has underlined the bilateral relationship as a “special and
privileged strategic partnership [that] continues to become stronger,” while Putin couched India
“as a great power, a friendly nation and a time-tested friend.”

5(c) Though India has historically paid little attention to Latin America, the recent
developments show India’s quest to become a major player in the region. Discuss.
Despite LAC’s (Latin America and Caribbean) geographical significance, large population, and
economic potential, the region has received little consideration in Indian foreign policy. India’s
outreach to the LAC region was long hindered by a geographical disconnect and late realization
of the opportunities the region offers. While trade between the two has steadily risen since the
first decade of the 21st century, existing literature on India-LAC relations underlines the slow
pace of growth in other areas.

Efforts by successive governments in New Delhi to strengthen ties with LAC, it seems, have
fixated around augmenting trade and investments in the region. Interest at the highest levels
continues to be motivated by an understanding of LAC as an alternative “resource-rich” region
that could meet the demands of India’s future energy needs. In the absence of any effective
framework and activism to transform its engagement with the region, India’s position in LAC
remains low key, in contrast to the other extraregional power, China.

As of 2019, India has established a diplomatic presence in only 15 LAC countries; there are more
resident missions from the region in New Delhi. In 2010, India’s then-Minister of State for
External Affairs Shashi Tharoor announced the opening of embassies in Ecuador, Uruguay, and
the Dominican Republic – that proposal, however, remains unrealized. While under the Modi
government’s foreign policy activism India has engaged other regions such as West Asia from a
strategic perspective, there seems to be little concern in reviewing the current state of relations
with LAC.

Despite this inconsistent interest, a Inter-American Development Bank (IDB) report released in
2011 outlined India as “the next big thing” in Latin America. Similarly, a 2016 report by the
United Nations Economic Commission for Latin America and the Caribbean (ECLAC) suggests
that the LAC countries could enhance trade and economic relations with India by undertaking
structural adjustments in their economies. It is this optimism and expectation that sustains the
region’s interest in India. From a wider perspective, the strengthening of India’s profile in the
region provides LAC an opportunity to diversify away from China – which many view as pushing
LAC into a “raw materials” corner apart from turning it into a dumping ground for cheap
consumer goods.

Trade volumes with Argentina, Venezuela and to a lesser extent with Peru, have grown
significantly, largely on account of fuel and commodity imports viz. crude oil, edible oil and
copper. India’s export basket is a robust and versatile mix of engineering goods,
pharmaceuticals, textiles and IT services. Indian companies, mainly Oil and Natural Gas
Corporation (ONGC), and also private sector organizations like Reliance, Videocon and Gammon
India, had invested in producing oil and gas fields in LAC.
When Prime Minister Narendra Modi travelled to Brazil for the BRICS (Brazil, Russia, India, China
and South America) Summit soon after the 2014 general elections, his address hinted at a
promise of greater engagement: “I have also had the privilege of sowing the seeds of personal
relationships with each of the leaders. I look forward to seeing them blossom into deep and
strong personal bonds in the days to come.”

With two-way commerce slightly below the US$ 40 billion-mark, current trade is indeed still the
business of just a handful of products. However, in the last decade, trade has grown twenty-fold
and bilateral investment is on the rise. Trade between LAC and India has room to grow through
diversification. LAC exports to India are heavily concentrated in extractive products, accounting
for 72% of exports in 2017. Agricultural products followed with 19% and industrial manufactures
only accounted for 9%.

Both regions rely on cooperation programs in different areas, but opportunities are still
growing. Agriculture, aerospace cooperation and renewable energy are some examples of the
sectors in which effective cooperation could benefit all participants. Both regions will need to
explore new ways of relating to each other and new international cooperation agreements.
They will need to discuss rapprochement through trade and investment agreements that
facilitate trade relations. For both regions, there is enormous potential for growth.

India and Latin America are emerging as major growth engines of the world economy and must
reap the mutual benefits of a growing partnership with each other. With ever-increasing
challenges in the world economy, diversification of trade partners and access to new markets
has become a priority for both sides. Development finance institutions may emerge as strategic
partners that may help and support a renewed commitment of LAC and India to foster ties
between the two subcontinents and to facilitate a joint step forward toward deeper South-South
cooperation for a brighter future.

5(d) “Japan is India’s most trustworthy friend”. Enumerate.

India–Japan relations have traditionally been strong. The people of India and Japan have
engaged in cultural exchanges, primarily as a result of Buddhism, which spread indirectly from
India to Japan via China and Korea. The people of India and Japan are guided by common
cultural traditions including the shared heritage of Buddhism and share a strong commitment to
the ideals of democracy, tolerance, pluralism, and open societies. India and Japan, two of the
largest and oldest democracies in Asia, having a high degree of congruence of political,
economic, and strategic interests, view each other as partners that have responsibility for, and
are capable of, responding to global and regional challenges. India is the largest recipient of
Japanese aid and both countries have a special relationship official development assistance
(ODA). As of 2017, bilateral trade between India and Japan stood at US$17.63 billion.
The two nations (with India being a British colony) were enemies during the Second World War,
but political relations between the two nations have remained warm since India's independence.
Japanese companies, such as Yamaha, Sony, Toyota, and Honda have manufacturing facilities in
India. With the growth of the Indian economy, India is a big market for Japanese firms. Japanese
firms were some of the first firms to invest in India. The most prominent Japanese company to
have an investment in India is automobiles multinational Suzuki, which is in partnership with
Indian automobiles company Maruti Suzuki, the largest car manufacturer in the Indian market,
and a subsidiary of the Japanese company.

In December 2006, Indian Prime Minister Manmohan Singh's visit to Japan culminated in the
signing of the "Joint Statement Towards Japan-India Strategic and Global Partnership". The year
2007 was declared "India-Japan Friendship Year.”

Areas of commonality

First, a continuation of the balancing security policy against China that began with Prime
Minister Narendra Modi and Shinzo Abe in 2014 continues till date. During a phone call with the
Indian Prime Minister, Shinzo Abe expressed concern over China’s “unilateral” actions in the
East and South China Seas, Xinjiang and Hong Kong. Crucially, India’s clashes with China in
Galwan have turned public opinion in favour of a more confrontational China policy.

In just a decade, New Delhi and Tokyo have expanded high-level ministerial and bureaucratic
contacts, conducted joint military exercises and concluded military pacts such as the Acquisition
and Cross-Servicing Agreement (ACSA) logistics agreement. Further, there is a strong
affirmation by both countries India and Tokyo towards supporting a Free and Open Indo-Pacific
and continued willingness to work with the Quad, which is fast emerging as a central pillar of
the security strategies of both nations.

Second, the two powers look to expand cooperation in sectors such as cybersecurity and
emerging technologies. During the Shinzo Abe years, New Delhi and Tokyo put together a digital
research and innovation partnership that ran the gamut of technologies from AI and 5G to the
Internet of Things and space research.

Third, economic ties and infrastructure development are top drawer items on the agendas of
India and Tokyo. While Japan has poured in around $34 billion in investments into the Indian
economy over the course of the last two decades, Japan is only India’s 12th largest trading
partner and trade volumes between the two stand at just a fifth of the value of India-China
bilateral trade.
In years past, New Delhi and Tokyo have collaborated to build infrastructure in Iran and Africa,
provide vital aid to Myanmar and Sri Lanka and hammer out a common Association of
Southeast Asian Nations outreach policy in an attempt to counter China’s growing influence in
these corners of the globe. However, the time has come for India and Japan to take a hard look
at the bilateral relations and think of avenues to boost the same into becoming a strategic
partnership.

Writing in 2006, Shinzo Abe, in his book, Utsukushii Kuni E (Toward a Beautiful
Country), expressed his hope that “it would not be a surprise if in another 10 years,
Japan-India relations overtake Japan-U.S. and Japan-China relations”.

5(e) Afghan chaos is an opportunity for India and Iran to strengthen their relations. Elaborate.

With the withdrawal of the US forces from Afghanistan and the Taliban’s takeover in Kabul, the
diplomatic situation is complicated for both India and Iran. The Taliban’s violent means to
consolidate power in Afghanistan have resulted in a huge wave of refugees fleeing Afghanistan
to the neighbouring countries like India and Iran. This huge Afghan refugee population is also
looking for asylum in India, which could strain the Indian economy. In such a scenario, India
once again looked forward to reframe ties with Iran with a ray of hope as a new president in the
White House raised hopes of a prospective breakthrough between Iran and the US related to
talks on the nuclear deal and removal of sanctions. The increased pace of activities vis-à-vis
Chabahar port’s cargo-handling capacity to enhance its trade with Afghanistan has been
jeopardised with Taliban forces running Afghanistan. The fact is that the expeditious actions of
the Taliban to consolidate power in Kabul affect both India and Iran and have, in fact, become a
matter of concern for both the nations.

Furthermore, with both China and Pakistan supporting the Taliban regime in Afghanistan, the
situation leaves India to partner with Iran in its goals vis-à-vis Afghanistan. However, it has
become more challenging for India to convince Iran on its actions related to Afghanistan.

There is indeed a strong convergence of interests between the two countries. Both India and
Iran desire a stable Afghanistan to be run by a democratically elected government that is
neither dominated by the Taliban nor under the thumb of Pakistan. The Taliban gaining power
in Kabul has made Indian fears of a repeat of the 1990s situation come true. For India, this
would mean an Afghanistan that was the haven for anti-India terror groups supported by the
Pakistani military establishment; whereas for Iran, it would mean a threat to the Shias in the
region as well as extensive drug trade through its borders. Despite Iran’s continuous
engagement with the Taliban, the former fears that the Sunni extremist Taliban might support
other like-minded terror outfits that could use Afghanistan as a base for terror attacks in the
region, especially against the Shias.

Iran and India had signed the New Delhi Declaration in 2003 known as the ‘Tehran Declaration’,
which defined the framework of the strategic defence pact between the two nations.
Consequently, India and Iran can facilitate the additional dimension of security collaboration in
their bilateral relations. However, Iran’s growing bonhomie with China affects Indo-Iranian ties
to some extent in the similar manner as India’s growing relations with Israel and Saudi Arabia in
the West Asian region. But both India and Iran visualise Taliban-controlled Afghanistan as a
threat to their national unity and security, which could become a mutual concern, eventually
leading Iran and India to come together and strengthen their bilateral cooperation particularly
on security matters.

The fall of the democratically elected Afghan government under Ashraf Ghani and US ending its
20-year-long War on Terror against Afghanistan has made countries like India and Iran to weigh
their options in order to deal with the emerging situation in Afghanistan.

The prevailing situation in Afghanistan will indirectly affect India’s Chabahar venture with Iran,
which was purposely developed to enhance trade with Afghanistan and Central Asia. However,
the aim of New Delhi and Tehran is to salvage the connectivity route created via Chabahar port.
The bilateral relations between India and Iran suffered a major setback when former US
president Donald Trump imposed stringent sanctions on Iran, which also forced India to cut back
its oil trade with Iran to almost negligible levels. However, India’s trade and development
related to Chabahar port were spared, even though there had been concerns in India and Iran
related to the pace of work being done at Chabahar port by India. At that time, Iran was among
the top three energy suppliers. This situation is further aggravated by the Iranian move of
developing a railway line—the approximately 628km-long Chabahar-Zahedan railway—as well
as the Farzad B gas field sans Indian involvement. This has further deteriorated the relations
between the two nations.

President Ebrahim Raisi has indicated the desire of the Iranian government to develop ties with
the neighbouring nations in the region, especially India. He further elaborated that India and
Iran would collaborate on various sectors in the economic and commercial fields as well as in
the fields of new and developing technologies. Though both the sides have indicated strong
desire to collaborate on various issues and aspects, there are some factors that also could
hamper progress in developing the bilateral relations between India and Iran.

At the foremost level are the divergent views that both the nations hold regarding the role of
the US in the West Asian region. In all of this, with the increasing camaraderie between India
and the US in the past decade, arguably, leading to a strategic partnership, it has become a
tightrope walk for India during the escalation of tensions between Iran and the US. This
eventually resulted in the negligible trade between India and Iran. Furthermore, India’s relations
with the Arab nations, especially Saudi Arabia and the United Arab Emirates (UAE), have also
deepened in the past few years. On the other hand, Iran’s relations with the Gulf Arab nations
have deteriorated especially with Saudi Arabia, which resulted in huge debates regarding
sectarianism in the region. In addition, India’s strong strategic and defence relations with Israel
could create troubles in India-Iran relations. In future, these bilateral relations of India with the
West Asian region could test New Delhi’s balancing skills. In such a scenario, it becomes
necessary and pertinent for India to carefully tread in its relations with Iran so as not to
antagonise this relationship further.

It cannot be ruled out that the two recent visits by Jaishankar have provided a platform to India
to give fresh impetus to ties, especially at a time when both India and Iran are concerned about
the emerging situation in Afghanistan. In the contemporary scenario, what draws both India
and Iran closer are the concerns related to Taliban’s Sunni extremism and its control of Kabul,
which could be detrimental to the security of both India and Iran. Therefore, it is necessary for
both India and Iran to converge and collaborate in order to prevent the Taliban’s emerging
hegemony over Afghanistan, which can intensely alter the geopolitics of the South Asian region
as well as have serious repercussions for West Asian region as well.

This is significant in the light of India being the major stakeholder in the peace and stability of
Afghanistan due to its major investments as part of its aid and reconstruction activities in war-
ravaged Afghanistan. In addition, India has been continuously supporting a national peace and
reconciliation process for Afghanistan that is Afghan-led, Afghan-owned and Afghan-
controlled—an idea also favoured and supported by Iran.

The challenges on the Afghanistan front could also have a broader impact on India-Iran bilateral
ties along with some points for convergence between the two nations. India had hoped that
Jaishankar’s recent diplomatic outreach to Iran would help reset bilateral ties that have been
strained over the past few years. New Delhi was desirous of this also because of the emerging
situation in Afghanistan, which made it visualise Iran as its ally in its actions vis-à-vis
Afghanistan. However, in the recent past, India was doing the precarious balancing act between
the US and Iran. Plus, Iran signing a 25-year strategic agreement with China in March 2021 has
raised apprehensions in India. China is India’s main strategic adversary and while New Delhi has
strengthened its strategic ties with Israel, Saudi Arabia and the UAE, the pact can become an
issue of concern for India. In all of this, Afghanistan can become the most viable avenue for
bilateral cooperation between India and Iran. But whether this can happen and evolve into an
effective partnership while concurrently helping improve India-Iran ties remains to be seen in
future.

6(a) Explore the geo-political dimensions, economic ties and the aspect of Diaspora in India’s
links with the West Asia.

Geopolitically West Asia occupies an important position for India due to its geographical
location and proximity to European continent. The region is strategically significant due to its
enormous energy resources, trade route links to different parts of the world and the fact that it
is a place of origin for the Abrahamic religions. It is the world largest oil-producing region
accounting for 34% of world production, 45% of crude oil exports and 48% of oil proven
reserves.

India’s commercial and cultural relations with the region have ancient roots. People to people
contacts were established between the two great civilizations in those early days when the
merchants of the Kulli culture in Southern Baluchistan and the early Sumer dynasties were in
existence. Later the period between the rise of Islam in the 7th century A.D. to about the 10th
century A.D. may be termed as the golden age of trade relations between India and the Arab
world.

On the Economic front, GCC member states, along with the countries of the West Asian and
North African region are India’s largest trading partner. Two-way trade between India and the
region was over 180 billion U.S. dollars in 2014-15.A ccording to Indian Ministry of Commerce
data for July, 2018, trade with the Gulf in 2017- 2018 was over 123 billion U.S. dollars, the UAE
alone accounting for U.S. dollars 50 billion. Saudi Arabia is India’s fourth largest trading partner,
trade being 28 billion U.S. dollars, after peaking at 39.26 billion U.S. dollars in 2014-15.

West Asian region meets a major share of India’s energy needs – contributing over 60 per cent
of India’s total imports of crude oil and over 85 per cent of India’s LNG requirements. Saudi
Arabia is India’s largest supplier of crude oil, meeting 20 per cent of India’s needs, valued at US
$ 21.8 billion in 2014- 2015.

An important factor influencing India’s foreign policy is the socio-cultural affinity of Indian
Muslims owing to – Mecca and Medina located in this region. Every year more than a lakh
Indian Muslims go for Hajj, providing a binding force between two regions. For the past four
decades trade, energy and human resource have been the principal drivers of India’s economic
relations with the Gulf Cooperation Council (G.C.C.). India has been heavily dependent on energy
supplies from the region, while Indian expatriates have constituted a substantial share of the
regional labor market. Remittances from the region were last estimated to be fifty percent of
the total of 80 million USD coming to India.

The presence of Indian diaspora has also become an important element of spreading soft power
prerogatives. They are looked upon as an unofficial ambassador to foreign policy and an
important bridging link between two regions. The role played by the Gulf diaspora is profound in
India’s development. It is only recently that India accorded importance to the Gulf region as is
clearly evident in the diplomatic visits paid by Indian PM to the countries of the West Asia in
general and to the Gulf States in particular. His visits to Saudi Arabia, UAE, Qatar, Iran, and
Israel have yielded results, like UAE allocating land for the construction of a temple. The change
instance of India’s foreign policy has been due to the large influx of labor from India to the
economies of the GGC, which account for nearly 9 million contributing significantly for both the
economies of India and the Gulf.

For India, the Arab countries are an important source of investments, especially in the post-
COVID economic recovery period. For the Arab countries, India offers an opportunity to diversify
their economy thereby reducing dependence on oil trade. The existing geopolitical and more
importantly economic circumstances have led India to pursue an energetic diplomatic outreach
to the West Asian region.

b) India and USA can have overlapping consensus but lack overarching consensus on geo-
political issues. Comment

India-USA relations have travelled a long road since the time when India and USA were
termed as two ‘estranged democracies’ by Dennis Kux to being ‘Strategic partners’,
embodying the most defining partnership of the 21 century. The United states designation
of India as a ‘Major Defence Partner’ is indicative of the elevated status of India-US
relationship.

When one assesses the present and the future of the Indo-US relationship, one needs to factor in
history as well. One needs to understand that the two countries were practically on opposite
sides of the fence during the Cold War. The Non-Aligned movement (NAM) was conceived
during the Cold War with India as a founding member so that newly independent countries like
India could stay out of the global race for supremacy between the US and Soviet Union.

Historically, even though the trajectory of India-United States ties have witnessed chequered
momentum from time to time, yet the relationship between the two countries has gradually
evolved into a more robust and matured partnership based on mutual trust especially after the
end of Cold War with India embarking into an outward looking economic developmental
roadmap. In recent years, especially since 2014, with the Narendra Modi government coming to
office in India one can notice that the relationship between New Delhi and Washington has
entered an altogether revamped phase of warming up scaling newer heights of mutual
commitment and trust.

This is not just a recent change, with the two countries in fact signing four major security
pacts since 2002, including on logistics to allow each country’s military to use bases for repair
and replenishment, communications, and for the sharing and sale of sophisticated American
military technology to India. The Bush administration in the 2000s also moved to normalise civil
nuclear cooperation with India.

US-India ties are not without tensions. India’s decision in 2018 to buy Russia’s S-400 missile has
created tension between New Delhi and Washington. US Defense Secretary Lloyd Austin in his
recent visit to India highlighted America’s wish that its allies and partners move away from
Russian acquisitions which, as he put it, would “trigger sanctions on our behalf”. The
US Countering America’s Adversaries Through Sanctions Act (CAATSA) restricts defence
purchases from Russia, Iran and North Korea. Yet the issue of sanctions was not
discussed during Austin’s visit, because India has yet to acquire the S-400. CAATSA remains an
elephant in the room, and the Biden administration – as with the Trump administration before it
– has not ruled out imposing sanctions on India if it goes ahead with the deal.

India counters that the acquisition of the S-400 missiles was on its agenda before the US
introduced CAATSA, and that its military has a diversified portfolio. Russia is its top arms
supplier, but it also buys weapons from the US, Israel and France.

Another area of tension is the issue of America’s constant criticism of India’s internal issues like
the CAA in North East. US Defense Secretary Lloyd Austin said that he had discussed human
rights issues, particularly about citizenship for Muslim minorities in Assam, with Indian
ministers. New Delhi maintained a public silence on this aspect of the talks, along with Austin’s
remark that the US values its partnership with India and partners “need to be able” to discuss
such issues. During their election campaign, Joe Biden and his running mate, now Vice
President Kamala Harris, criticised the Modi government’s suppression of human rights in the
Muslim-majority region of Jammu and Kashmir and the Citizenship Amendment Act.

Another aspect of antagonism are the current trade relations between US and India. Two
interconnected transformations are particularly relevant to friction in the trade relationship:
first, the digitalisation of the global economy and second, the politics of ‘techno-nationalism’.
Around the world, there is disagreement over the governance of data and the role of technology
in development and security. Countries are used to conceding sovereign power over the course
of decades, through long and hardened negotiations on say, trade and migration.
At issue is a fundamental disagreement about how digital technologies will shape our societies.
In the United States, there is a firm belief that Silicon Valley can create value in developing
economies like India while protecting human rights and cybersecurity. The Indian perspective is
dramatically different. There is a sense that Indian industry and governance propositions must
manage the development and deployment of emerging technologies.

Fourthly, Iran is another geopolitical issue where both the countries lack overarching consensus.
In failing to exempt India from its sanctions regime, the United States is, unfortunately, pursuing
short-term interests over long-term gain. It is essential to understand that energy trade
between India and Iran is only one aspect of a far more crucial trend: the gradual merger of the
Asian and European continents that is underway and is largely being effected by one player:
China. At the moment, Eurasia is being defined primarily by China’s geo-economic thrust
through its Belt and Road Initiative and the attendant politics riding on Beijing’s tracks and
roads.

Way Ahead:

While both states have done well in keeping these tensions from affecting the broader bilateral
relationship, such questions on geopolitical issues will only continue to get larger and more
consequential. It is incumbent on the United States to acknowledge that India as a growing
power will assert its independence on issues that it considers crucial to its sovereignty. India,
meanwhile, must articulate its policy preferences clearly and quickly. It must continue with
keeping its national interests at the forefront.

6(c) Though the relationship between the two Asian giants is painted in Apocalyptic terms yet
the two need a new equilibrium in their relations. Discuss.

With China, India's relations are often seen in “zhongyong” dialectic style, that means adopting
'a middle course' strategy.. The conflictual attitudes are quietly embedded in the Chinese moves.
It is embroiled in land and sea territorial disputes with 13 neighbouring and eight other
countries despite its signing 8 various agreements with all of them. The conflicts are kept
simmering within its political insight and in dealings with countries of the world. In today's
world of nuclear arms and advanced weaponry, a country, howsoever mighty, it cannot fight
with 21 others, if it goes to war. If China today chooses to go for war with any country like Japan
or India, chances are that there would be polarisation of many others countries against it and
the Third World War would be enacted in the Asian theatre.

Given recent developments, few will deny that the increasingly adversarial relationship between
India and China is one of the most important factors in the emerging geopolitics of the
21st century. The founding fathers of strategic theory: Alfred Thayer Mahan (1840-1914), who
believed that strategic power depended on the control of the oceans; and Halford Mackinder
(1861-1947) who saw the Eurasian heartland as the ‘geopolitical pivot of history’, on which
strategic dominance depended have shaped the understanding of geo-strategic interaction.
‘India’s geostrategy,’ writes Daulet Singh, ‘is being contested by two somewhat competing
images: the Mackinder image – or the idea that continental spaces are what really matter in
power politics – and the Mahanian image – or the idea that maritime spaces in the seas are
what ultimately account for the power of states.’

In terms of global geopolitics, what is truly epochal about the rise of China is that it has begun
to move the locus of strategic power away from the oceans and back to the continental heart of
Eurasia. This is indeed the aim of the Belt and Road Initiative, which inaugurates, for the first
time in 500 years, a strategic shift initiated by a non-Western power. In opposing this initiative,
Daulet Singh suggests, India has more to lose than to gain: ‘India’s main concern has been that
the BRI is designed to stamp China’s geopolitical dominance. Some argue that such a fear might
be misplaced for “all great infrastructure and connectivity ventures – throughout history – have
altered the prevailing geoeconomics matrix, and hence the resulting geopolitical balance. While
the geopolitical effects are short-lived, the geo-economic benefits survive over time”. Or put
simply, new trading linkages eventually trump geopolitics.’

In cutting itself off from a major geo-political and geo-economic shift, Daulet Singh suggests, it
is India that risks isolation within its region: ‘In an ideal world, a South Asia in splendid isolation,
and, an India with the economic and institutional capacities might have produced an order for
the entire region. But India’s internal preoccupations, a lop-sided growth model with a weak
state, and, China’s rise has exposed the idea of South Asia as an exclusive sphere of influence. It
is no longer a viable venture.’

A better alternative is for India and China to ‘learn to sensibly manage their complex
relationship and evolve their existing modus vivendi to incorporate a framework where a gentler
rivalry or competition is handled maturely and at the same time does not inhibit the pursuit of
their common or overlapping interests.’

7(a) As the great game in Afghanistan is entering into a new phase, strategic patience and
nuanced actions would serve India’s interest the best in the region. Comment.

Due to the imbroglio in Afghanistan where India has limited leverage, a one-size fits all
approach will not be fruitful. India, in recent times, has been confronted with some hard choices
in Afghanistan. A decade-long policy of providing huge humanitarian and developmental
assistance, which has accrued tremendous goodwill among the Afghans, is perceived to be in
imminent danger of being disrupted and overwhelmed by the withdrawal of the United States.
This is in addition to the recently shifting discourse of negotiating with the Taliban, which is
interpreted as an upsurge of Pakistani influence in Kabul. The choice for India was never
whether it should stay engaged in Afghanistan or not. Even in the face of repeated onslaughts
on its personnel and mission, India was committed to staying the course.

India’s role in post-9/11 Afghanistan has been a subject of intense policy deliberations, at times
bordering on speculations. On one hand, India is seen to play a critical role in the reconstruction
of the war-ravaged country, and on the other, India’s growing influence in Afghanistan as a
result of its aid diplomacy is said to have increased Pakistan’s real or imagined concerns of
encirclement in what it perceives to be its ‘strategic backyard’. This ‘zero-sum’ geopolitical
rivalry between India and Pakistan, dubbed by Western analysts as the ‘new great game’, is
seen as a source of further instability. India’s interests in post-9/11 Afghanistan have centred on
three broad objectives – security concerns, economic interests and regional aspirations.

In understanding India’s strategic thinking on Afghanistan, it would be useful to locate India‟s


strategic mind within the broad parameters of three schools of thought/paradigms: (1) Realist,
(2) Neo-liberal, and (3) Nehruvian-Marxist. The Realist perspective envisions India’s aid and
development assistance as a useful tool directed at supporting the nascent democratic regime,
the strengthening of which denies the space for return of the Taliban to the seat of power. The
Neo-liberals prod India to take a more proactive role as a major regional and economic power
with ambitions of extending its influence beyond its immediate neighbourhood for tapping
enormous energy and trade potential in the region. India has worked towards this objective by
reviving the role of Afghanistan as a land bridge, which connects South Asia with energy rich
Central Asia for economic and energy interests. By linking stability with greater economic
integration, India has been actively promoting greater trade and economic integration of
Afghanistan with South Asia through the regional mechanism of South Asian Association of
Regional Cooperation (SAARC). The thinkers in the Nehruvian tradition, while arguing against
interference in internal affairs, emphasise the need for regional cooperation and restoring
‘peace and neutrality’ in Afghanistan, preferably under the aegis of the United Nations (UN).

To achieve these objectives, as indicated by various strands of Indian thinking, India adopted the
‘soft power’, reviving its historical, traditional, socio-cultural and civilisational linkages with the
war-ravaged country. This ideational and enabling role which has been well received by the
Afghans, has important lessons for the international community for the long-term stabilisation
of Afghanistan.

India’s interests in Afghanistan need to be viewed in the security paradigm, in the context of
India’s concerns of terror emanating from the extremely volatile Pakistan-Afghanistan border
spilling into India. A strong, stable and democratic Afghanistan would reduce the dangers of
extremist violence and terrorism destabilising the region. The Realist line of thinking is driven by
‘fear’ of security considerations. This could be real or perceived fear of Pakistani military and
intelligence agencies regaining their ‘strategic depth’ by reinstalling a pliant Taliban regime in
Afghanistan much to the detriment of India’s security imperatives. Thus, this stream of thinking
emphasises on the need for India to use military and diplomatic tools to secure its ‘outer
periphery’ or ‘extended neighbourhood’.

Since 9/11, New Delhi’s policy has broadly been in congruence with the US objectives of
decimating the Taliban-Al Qaeda combine and instituting a democratic regime in Kabul.
Alongside the United States, India has spent the past 20 years trying to foster a democratic
system in Afghanistan. It invested $3 billion into building Afghan roads, bridges, schools and
clinics. In 2015, Indian Prime Minister Narendra Modi traveled to Kabul to inaugurate the $90
million parliament building, made with marble quarried from Rajasthan. He and then-Afghan
President Ashraf Ghani spoke about their "special friendship" that was "bound by a thousand
ties."

However, today, the Taliban has been able to regroup, resurge and regain power, in addition to
further intensification of its linkages with Pakistan based anti-India groups. Diplomats and
analysts say that for India, the power shift in Kabul almost certainly means the painful loss of a
fellow democracy — albeit a beleaguered one that rested on U.S. support — in an otherwise
largely hostile region. It could also mean a loss of safety and security for India, if militants from
its neighbor and archrival, Pakistan, expand training bases into Afghanistan.

The Third Regional Security Dialogue on Afghanistan, at the level of National Security Advisers
(NSAs), was hosted by Delhi on 10 September 2021. An Iranian initiative, the first two meetings
of this forum were held in Tehran in 2018 and 2019. NSAs from Iran, Kazakhstan, Kyrgyzstan,
Russia, Tajikistan and Turkmenistan responded positively to Indian NSA Ajit Doval’s invitation.
There were two significant no-shows. Pakistani NSA Moeed Yusuf had publicly rejected the
invitation, stating that “a spoiler (India) can’t be a peacemaker”.

The Delhi Declaration, issued at the conclusion of the NSAs’ meeting, was along predictable
lines. It “condemned” the recent terrorist attacks in Kunduz, Kandahar and Kabul, but
reaffirmed a shared commitment to ensuring that “Afghanistan would never become a safe
haven for global terrorism”. It stressed the need for “an open and truly inclusive government”
and ensuring the “rights of women, children and minority communities”. While emphasising a
central role for the United Nations and underlining the need for urgent humanitarian assistance,
it reiterated that this should be distributed across the country “in a non-discriminatory manner”
through “unimpeded, direct and assured access”. The declaration sought collective cooperation
in tackling extremism and drug trafficking in the region.
The welcomed the Delhi Declaration though they were not invited and have repeatedly
suggested that they would welcome the reopening of the embassy by India. However, the Indian
government is unwilling to take any chances, given the influence of the Haqqanis, and by
extension, the ISI. Nevertheless, unlike the West, India is part of the region and cannot
disengage. It has responded to the humanitarian call by offering 50,000 metric tonnes of wheat
and urgent medical supplies.

Leaving the reconciliation process primarily to an unstable administration of Taliban in Kabul


will do little for India’s long-term interests in Afghanistan. Repositioning Indian imperatives
means also remaining ever-connected with the deep ties that India has nurtured with
Afghanistan over the ages and India must be willing to pivot to the changed realities in
Afghanistan and being much more involved in conversations on and around reconciliation than
ever before.

7(b) India should see Quad as an avenue to accelerate its internal balancing and enhance its
military capabilities. Comment. Suggest the way forward for India.

The Quadrilateral Security Dialogue (or Quad, of India, the United States, Australia and Japan) b
egan as a coordination mechanism to respond to the humanitarian crisis in the aftermath of the
2004 Indian Oceanthat killed 227,898 people and displaced nearly two million others.

The rise of the Quad signals the acceptance, both within and beyond the member countries, of t
he “IndoPacific” as a strategic concept. Starting with the Japanese Prime Minister Shinzo Abe’s “
Confluence of
the Two Seas” speech before the Indian parliament in 2007, the traditional understanding of wh
at const-tuted the Asia-Pacific region started to change.

The consolidation of the Quad is an important political pivot in reconfiguring the Asian order.
For the United States, it signals a commitment to remain engaged with the Indo-Pacific and lead
a more coordinated effort to contain China. For India, the big political signal is that it is willing
to be part of a group that will deny China the opportunity to upend what this group thinks is a
world order based on principles of democracy, and a free, open and inclusive Indo-Pacific. For
Australia, it is a clear signal that the die has been cast. China’s assertiveness has left these
countries with no other option. The Quad is part of a churning of the security architecture in
Asia. It will be more confrontational, with all the attendant risks.

India’s views of the Quad are a product of its unique position on the geopolitical map. India is
the only member of the Quad that borders China, has been invaded by China, has an active land
border dispute with China, is wedged between two nuclear-armed rivals, and lacks treaty
alliance commitments from the other Quad members. As a result of this relative strategic
vulnerability, India is more sensitive to the risks of aggravating a security dilemma with China
without a comparable improvement in its strategic position.

As a result, India seeks to minimize perceptions of the Quad as a U.S.-led containment coalition,
both to limit the damage to China-India relations and to enhance the Quad’s broader
regionwide appeal. India has long preferred to cast the Quad in a more open, multilateral, and
inclusive light. India endeavors to acquire high-end defense platforms and foster greater
cooperation in intelligence- and technology-sharing with its Quad partners. It seeks joint
development of defense platforms and greater investments in its indigenous defense sector,
which, to date, have been largely stifled by arcane regulations and red tape.

The Quad’s innovative form is also supposed to reflect that churning. The value of most
groupings is often not specific policies, but the socialisation it produces. The Quad is not an
alliance. It aims to secure an open order in the Indo-Pacific, but it is not a full-fledged security
pact. It is supposed to pursue broad goals: Humanitarian relief, climate change, vaccines,
coordination on technology, securing supply chains, trade, and a general outlook on a
democratic and inclusive world order. And India needs to ensure that the ‘Indo’ part of the ‘Indo-
Pacific’ is not ignored in the larger game of checkmating China by other Quad members.

7(c) The efforts of regional integration in South Asia are hijacked by the dilemma of
comprehending India’s role as a stepping stone or a stumbling block for regional efforts.
Critically evaluate.

South region comprises eight independent countries and is a largest geographical reality of the
Indian Ocean and occupies an important strategic, commercial and natural position. Although
the region is located in a small area of world map, it consist nearly one fifth of worlds total
population. There are a number of infrastructural linkages between the countries of this region
like common history, a compact geographical area, almost similar economic systems, cultural
and social commonalities. These states belong to non-aligned movement, have a shared view of
the evils of colonialism, racialism and economic exploitation, all opposed to foreign bases and
foreign intervention.
SAARC marked the first regional effort to rise above residual prejudices and mistrust in order to
evolve a positive framework of cooperative economic development to promote the welfare of
the peoples of south Asia. The significance of Asian regional cooperation lies in the fact that it
represents an effort to develop Asian solution to the Asian problems in a cooperative
arrangement.

India's size, resources and power potential make its predominance in the South Asian power
structure. India as a largest, strongest and the richest country of the region has a crucial role to
shape the future and cooperation of these nations. India's strategic location provides it a pivotal
position in Asia and world politics. As Nehru stated that India is a “bridge between the East and
the West.”

Constantino Xavier of the think-tank Centre for Social and Economic Progress (CESP) argued that
it is “impossible to understand India’s regional policies in South Asia without looking at Delhi
from the perspective” of its neighbours. Professor Selim Raihan of the University of Dhaka brings
out India’s overwhelming ‘size imbalance’ in South Asia: “The shares of India in the total land
area, population, and real GDP of South Asia in 2016 are 62%, 75%, and 83%, respectively. The
two other big countries in South Asia are Pakistan and Bangladesh with shares in regional GDP
of only 7.6% and 5.6%, respectively.”

Not only the dominance of India, there are other stumbling blocks for regional integration in the
region. The inability of the two largest economies in the region, India and Pakistan, to resolve
their political tensions has also been a major stumbling block in achieving effective regional
cooperation. Another major stumbling block is that Institutions remain weak and not able to
push regional integration forward to a level commensurate with potential. Regional agreements
like SAFTA are beset with problems and have the danger of lagging behind ground realities that
can change quite rapidly.

However, the necessary condition for pushing forward regional integration in South Asia is for
India to become a stepping stone for regional integration.

India has a certain positive role in the region to accelerate the pace of development because
stability of the region is in benefit of India also. If India wants to be a strongest regional power it
has to keep all the states of the region with her and if this century is to be an "Asian century"
than the regional integration is a must. The responsibility of the development lies on India's
shoulders. The eventual success of cooperation will be significantly determined by the role
played by India. India has taken the initiative to forge South- South Cooperation to promote the
solidarity among the South Asian countries.
It has always demanded the restructuring of the world order according to the requirements of
the weaker states. In the changed international scenario of the new century the necessity of
South Asian regional cooperation is also multiplying because US is taking major interest in the
area to get the benefits of sufficient natural resources, huge markets, big consumer society,
developing economies and above all to get the support of the nations of this area to counter
growing Chinese impact in the region and to counter international terrorism.

India wants that South Asian countries should choose an independent policy free from the
pressure of superpowers because only than they can bloom and be independent in real sense
and for that they have to develop a common outlook-strategically and politically. If Indian
Ocean is a key to the seven seas and if south Asia is the key to Indian Ocean than the concept of
South Asian cooperation is the key of India's destiny and the key to the destiny of all South Asian
Countries. Keeping this in mind India has been trying to develop an effective cooperation.

8(a) Discuss the components of India’s repeated calls at UN for “reformed multilateralism.”

The call for “reformed multilateralism” has become an integral part of India’s foreign policy,
seeking to maximise the country’s participation in the multilateral system to accelerate its
transformation into one of the major powers of the 21st century. India has contributed
significantly to creating the contemporary multilateral system. As a country with one-sixth of
the world’s population and a thriving democracy, India is a major stakeholder in a functional
multilateral system.

Following his call for a multilateral approach to achieve sustainable peace and prosperity at the
High-Level meeting of the UN’s Economic and Social Council (ECOSOC) on 17 July 2020, Prime
Minister Modi made a strong pitch for reformed multilateralism “that reflects today’s realities,
gives voice to all the stakeholders, addresses contemporary challenges, and focuses on human
welfare at the UN’s 75th-anniversary summit on September 21, 2020”. The Prime Minister’s
speech at the general debate of the UN General Assembly (UNGA) on September 26, 2020,
provided the components of India’s vision of “reformed multilateralism”.

At the core of this vision is India’s belief that UN member-states “cannot fight today’s challenges
with outdated structures. Without comprehensive reforms, the UN faces a crisis of confidence.”
The only UN structure that obstructs the democratic principle of taking decisions by consensus
or majority voting is the UNSC, where the veto powers of its five self-selected permanent
members overrides democratic decision-making. Prime Minister Modi castigated the UNSC for
failing to prevent conflicts, including civil wars, and terrorist attacks resulting in the deaths of
many ordinary human beings. Millions of people uprooted by conflicts have become refugees.
He said that this has happened despite the significant contributions made by many UN member-
states to the UNSC to help maintain peace and security.
In June 2020, India obtained 184 out of 193 votes in the UNGA to be elected to a two-year term
in the UNSC for 2021-22. This marked the eighth time since 1949 that over two-thirds of the
UNGA has endorsed India’s credentials to be elected to the UNSC. The Prime Minister pointed
out that “the people of India have been waiting for a long time for the completion of the
reforms of the United Nations”. He asked: “Today, the people of India are concerned whether
this reform-process will ever reach its logical conclusion. For how long will India be kept out of
the decision-making structures of the United Nations?” In his remarks at the 75th anniversary
summit, the Prime Minister referred to the “far-reaching” declaration adopted by world leaders
and cautioned that unless the UN was comprehensively reformed, the vision of the declaration
“in preventing conflict, in ensuring development, in addressing climate change, in reducing
inequalities, and in leveraging digital technologies” would remain unfulfilled.

India’s call for reformed multilateralism focuses on the inter-linkage between peace, security,
and development. An ineffective UNSC jeopardises India’s national efforts to achieve Agenda
2030’s Sustainable Development Goals (SDGs). Showing how the UN member-states can
“reform-perform-transform”, the Prime Minister listed the “transformational changes” in India
over the past five years that had enabled hundreds of millions of people to enter the formal
financial sector; become free from open defecation, and get access to free healthcare services.
The empowerment of women through promotion of entrepreneurship and leadership, access to
micro-financing and paid maternity leave were integral to India’s non-discriminatory
development policies.

The Prime Minister committed India to “sharing experiences of our development” as a practical
way to implement the principle of international cooperation on which multilateralism depends.
This included a commitment to build on the supply of essential medicines by India to more than
150 countries to respond to the Covid-19 pandemic by leveraging India’s vaccine production and
delivery capacity to “help all humanity”. Based on its experience of implementing the SDGs,
India advocated “a multi-stakeholder ground-based” approach to achieve the global goals. This
required engaging “state and local governments, civil society, communities and people.” India’s
international experience in implementing various global initiatives for a holistic approach to
peace, security and development had similarly been sustained by a multi-stakeholder approach.
The UN has already adopted a multi-stakeholder approach for its activities under the Tunis
Agenda to respond to the emerging digital order, and Agenda 2030 to achieve the SDGs. The
call for “reformed multilateralism” based on democratic decision-making, prioritising
development, and including all stakeholders comes at a critical time for India’s foreign policy.
India’s role as an elected non-permanent member of the UNSC and the incoming Chair of the
G20 during 2021-2022 provides a window of opportunity for the country to play a leadership
role to reform and transform the UN.
8(b) India’s continental grand strategy is facing existential crisis. A revitalized Indian maritime
grand strategy creates more space for maneuvering in the region. Discuss.

For a country that has been traditionally obsessed with a continental approach to war and
peace, India’s continental ‘grand’ strategy is facing an existential crisis today. Given that
reconciliation with its key adversaries, China and Pakistan, is unlikely at this point and pursuing
its ambitious territorial claims on the ground is almost impossible, India’s continental options
seem restricted to holding operations to prevent further territorial loss. Put differently, New
Delhi’s grand strategic plans in the continental space may have reached a dead end.

In recent years, China has begun to push the boundaries with India and it is neither keen on
ending the ongoing border stalemate nor reinstating the status quo with India despite series of
diplomatic talks. The peaceful India-China Line of Actual Control in the Northeast is now a thing
of the past with China pushing back New Delhi’s claims on Aksai Chin and New Delhi defending
against Beijing’s expansive territorial claims.

In the Northwest, the Pakistan front has also been heating up. Ceasefire violations on the Line of
Control (LoC) have spiked since last year as has the infiltration of terrorists across the LoC. With
the change of the status of Jammu and Kashmir (J&K) by New Delhi in 2019, and Pakistan
altering its political map a few months ago to include all of J&K, the India-Pakistan contestation
over Kashmir has become fiercer. Equally important is the geopolitical collusion between
Islamabad and Beijing to contain and pressure New Delhi from both sides. While this is not a
new phenomenon, the intensity of the China-Pakistan containment strategy against India today
is unprecedented.

The withdrawal of the United States from Afghanistan and the return of the Taliban, has turned
the geopolitical tide against India — similar to the situation in the early 1990s. But unlike in the
1990s, tables have turned in Afghanistan: the Taliban is no more an outcaste, and with the
withdrawal of forces of the North Atlantic Treaty Organization from Afghanistan, the
geopolitical interests of Pakistan, China and Russia has broadly converged in the region. The
change of the geopolitical landscape in Afghanistan and the frictions in Iran-India relations on
energy imports has further dampened India’s ‘Mission Central Asia’.

It appears abundantly clear now that New Delhi’s excessive focus on the continental sphere
since Independence has not yielded great returns in terms of secure borders, healthy relations
with its neighbours or deterrence stability vis-à-vis adversaries. If so, it is time for India to
change its grand strategic approach — by shifting its almost exclusive focus from the
continental sphere to the maritime sphere. Clearly, New Delhi has already begun to think in this
direction with the Ministry of External Affairs (MEA) establishing a new division to deal with the
Indo-Pacific in April 2019. The work in this direction, both ideational and practical, needs to be
fast-tracked to keep pace with the emerging realities and to make use of new opportunities.

There are several reasons why a maritime grand strategy would work to India’s advantage
while still struggling with a continental dilemma. To begin with, unlike in the continental sphere
where India seems to be hemmed in by China-Pakistan collusion, the maritime sphere is wide
open to India to undertake coalition building, rule setting, and other forms of strategic
exploration.

Second, unlike in the continental sphere, there is a growing great power interest in the maritime
sphere, especially with the arrival of the concept of ‘Indo-Pacific’. The Euro-American interest in
India’s land borders with Pakistan and China is negligible, and more so, there is little any
country can do to help India in its continental contestations. The situation in the maritime
sphere is the exact opposite: great powers remain ever more interested in the maritime sphere
and this interest has grown substantially since the coinage of Indo-Pacific. For instance,
Germany recently released its Indo-Pacific guidelines following the example of France which
brought out its Indo-Pacific strategy last year.

Third, Beijing’s bullying behaviour in the South China Sea in particular and the region in general
has generated a great deal of willingness among the Euro-American powers and the countries
of the region, including Australia and Japan, to push back Chinese unilateralism. This provides
India with a unique opportunity to enhance its influence and potentially checkmate the Chinese
ambitions in the region.

Finally, the maritime space is a lot more important to China than engaging in expansionist
attempts in the Himalayas, due to the massive Chinese trade that takes place through the
Oceanic routes and the complex geopolitics around the maritime chokepoints which can
potentially disrupt that trade.

Therefore, it is high time New Delhi shifted its almost exclusive focus from the continental space
to the maritime space, stitching together a maritime grand strategy. The MEA’s Indo-Pacific
Division is a good beginning; so is the decision in 2019 to elevate the Quad meetings among
India, Japan, the United States and Australia to the ministerial level. New Delhi would do well to
ideate on the current and future maritime challenges, consolidate its military and non-military
tools and engage its strategic partners.

India’s obsession with continental strategies has yielded unflattering results — no


secure borders or deterrence stability-Happymon Jacob
8(c) After a long journey as a love-less arranged marriage, India-EU relations is set to begin a
new era to realize the full potential. Substantiate.

For long, the EU and India partnership had been slow-moving and fragmented, struggling to
maintain momentum. Unfairly large emphasis was laid on foreign policy cooperation, but on
many issues divisions far exceeded commonalities, leading to disappointment and an overall
delusion in the potential of the partnership. Today, not only do the EU and India have a thriving
commercial relationship and growing security collaboration, but both partners have developed
greater depth in political cooperation internationally while exploring new and innovative areas
for further broadening their partnership.

India-EU relations date to the early 1960s, with India being amongst the first countries to
establish diplomatic relations with the European Economic Community. A cooperation
agreement signed in 1994 took the bilateral relationship beyond trade and economic
cooperation. At the 5th India-EU Summit at The Hague in 2004, the relationship was upgraded
to a ‘Strategic Partnership’. The two sides adopted a Joint Action Plan in 2005 (which was
reviewed in 2008) that provided for strengthening dialogue and consultation mechanisms in the
political and economic spheres, enhancing trade and investment, and bringing peoples and
cultures together.

The EU-India Strategic Partnership for more than half a decade was seen as slow-moving and
fragmented. Much of the potential was untapped. Similarities between the EU and India made
them natural partners but the relationship remained too focused on set-piece summits rather
than fostering dynamic everyday linkages. Even the annual EU-India summits failed to push
forward long pending issues like free trade, maritime cooperation or a nuclear agreement.
Collaboration on security and counter-terrorism remained negligible. The EU-India Strategic
Partnership was considered as one of the most static and disappointing of such accords. Well
matched but with no spark of chemistry, the EU and India appeared to be tied together in a
loveless arranged marriage.

However, in May 2021, a virtual India-EU leaders meeting was held between Indian Prime
Minister and 27 EU leaders. The meeting reflected the EU’s changing perception of India due to
changing geo-political circumstances.

Highlights of the meeting:


Resumption of FTA Talks: The most significant outcome of the summit was that after eight
years, India and the EU have decided to resume negotiations for a comprehensive trade
agreement.

These talks were suspended in 2013 after the two sides failed to bridge their differences on
some key issues such as tariff reductions, patent protection, data security and the right of Indian
professionals to work in Europe.

Resumption of BIT Talks: The two sides have also agreed to commence talks for a standalone
investment protection pact and an accord on geographical indications.

Connectivity Partnership: The virtual summit saw India and the EU launching an ambitious
“connectivity partnership” in digital, energy, transport, and people-to-people sectors, enabling
the two to pursue sustainable joint projects in regions spanning from Africa, Central Asia to the
wider Indo-Pacific.

The need for both EU and India to realize the full potential of their relations:

Firstly, there is an urgent need for EU to Pivot Away from China. EU recently signed a
Comprehensive Agreement on Investment with China, which has drawn a lot of criticism and its
ratification has now been suspended because of diplomatic tensions. The European Parliament
overwhelmingly opposed to this deal after China imposed sanctions on some of its members, in
response to the EU imposing sanctions against China for its treatment of the Uyghur Muslim
minority in the Xinjiang region.

Secondly, with the EU being India’s largest trading partner and the second-largest export
destination, the economic logic of strong India-EU economic relations is self-evident. Further,
India wants to showcase its commitment to open trade at a time of renewed focus on
developing a domestic manufacturing base.

Thirdly, given the current situation, health cooperation assumed a new salience. EU member-
states have rallied to support India by sending critical medical supplies in the last few months in
recognition for the role India had played in helping others over the last year. As the two sides
commit themselves to working together on global health, the need to focus on resilient medical
supply chains is all the more evident.

Fourthly, the EU is being forced to reckon with the geopolitical implications of its foreign policy
imperatives and India is looking for substantive partnerships with like-minded nations to bring
stability to the Indo-Pacific theatre. Further, India is looking beyond the bipolar geopolitical
competition between the US and China and works towards the establishment of a Multi-polar
world.
Fifthly, India can learn from a new industrial strategy called the Green Deal of EU to render its
carbon-emission neutral by 2050. The EU and India could endeavour transforming into carbon-
neutral economies by 2050 by investing in clean energies. In India’s efforts to increase the use of
renewable energy in India, the investment and technology of Europe is of paramount
importance.

Today’s changed circumstances provide the two sides with new sets of opportunities to move
forward on their long stalled relationship. As strategic realities evolve rapidly in the post
pandemic world order, India and EU have to re-evaluate the fundamentals of their engagement.
The EU is being forced to reckon with the geopolitical implications of its foreign policy
imperatives and India is looking for substantive partnerships.

From New Delhi’s perspective, this is also an inflection point in the


way it is thinking about its foreign policy priorities; a renewed
momentum in India-EU ties is the need of the hour. - Harsh V. Pant

You might also like